Tuyen Tap de Thi Vao Lop 10 Chuyen Khoa Hoc Tu Nhien

You might also like

Download as pdf or txt
Download as pdf or txt
You are on page 1of 322

Tailieumontoan.

com


Điện thoại (Zalo) 039.373.2038

TUYỂN TẬP 65 ĐỀ TOÁN VÀO 10


CHUYÊN KHOA HỌC TỰ NHIÊN
(Liệu hệ tài liệu word môn toán SĐT (zalo) : 039.373.2038

Tài liệu sưu tầm, ngày 10 tháng 10 năm 2022


Website:tailieumontoan.com

TUYỂN TẬP ĐỀ THI TOÁN


VÀO LỚP 10 CHUYÊN KHOA HỌC TỰ NHIÊN HÀ NỘI

LỜI NÓI ĐẦU

Nhằm đáp ứng nhu cầu về của giáo viên toán THCS và học sinh luyện thi vào lớp 10 môn toán,
website tailieumontoan.com giới thiệu đến thầy cô và các em bộ đề thi vào lớp 10 chuyên Đại học khoa học
tự nhiên Hà Nội. Đây là bộ đề thi mang tính chất thực tiễn cao, giúp các thầy cô và các em học sinh luyện
thi vào lớp 10 có một tài liệu bám sát đề thi để đạt được thành tích cao, mang lại vinh dự cho bản thân, gia
đình và nhà trường. Bộ đề gồm nhiều Câu toán hay được các thầy cô trên cả nước sưu tầm và sáng tác, ôn
luyện qua sẽ giúp các em phát triển tư duy môn toán từ đó thêm yêu thích và học giỏi môn học này, tạo
được nền tảng để có những kiến thức nền tốt đáp ứng cho việc tiếp nhận kiến thức ở các lớp, cấp học trên
được nhẹ nhàng và hiệu quả hơn.
Các vị phụ huynh và các thầy cô dạy toán có thể dùng có thể dùng tuyển tập đề toán này để giúp
con em mình học tập. Hy vọng Tuyển tập đề thi toán vào lớp 10 chuyên Đại học khoa học tự nhiên này sẽ
có thể giúp ích nhiều cho học sinh phát huy nội lực giải toán nói riêng và học toán nói chung.
Mặc dù đã có sự đầu tư lớn về thời gian, trí tuệ song không thể tránh khỏi những hạn chế, sai sót.
Mong được sự góp ý của các thầy, cô giáo và các em học!
Chúc các thầy, cô giáo và các em học sinh thu được kết quả cao nhất từ bộ đề này!

Liên hệ tài liệu word môn toán: 039.373.2038 TÀI LIỆU TOÁN HỌC
Website:tailieumontoan.com

MỤC LỤC
PHẦN 1: ĐỀ THI

Đề số Đề thi Trang
1. Đề thi vào lớp 10 chuyên Đại học KHTN Hà Nội năm 2022 (vòng 1)
2. Đề thi vào lớp 10 chuyên Đại học KHTN Hà Nội năm 2022 (vòng 2)
3. Đề thi vào lớp 10 chuyên Đại học KHTN Hà Nội năm 2021 (vòng 1)
4. Đề thi vào lớp 10 chuyên Đại học KHTN Hà Nội năm 2021 (vòng 2)
5. Đề thi vào lớp 10 chuyên Đại học KHTN Hà Nội năm 2020 (vòng 1)
6. Đề thi vào lớp 10 chuyên Đại học KHTN Hà Nội năm 2020 (vòng 2)
7. Đề thi vào lớp 10 chuyên Đại học KHTN Hà Nội năm 2019 (vòng 1)
8. Đề thi vào lớp 10 chuyên Đại học KHTN Hà Nội năm 2019 (vòng 2)
9. Đề thi vào lớp 10 chuyên Đại học KHTN Hà Nội năm 2018 (vòng 1)
10. Đề thi vào lớp 10 chuyên Đại học KHTN Hà Nội năm 2018 (vòng 2)
11. Đề thi vào lớp 10 chuyên Đại học KHTN Hà Nội năm 2017 (vòng 1)
12. Đề thi vào lớp 10 chuyên Đại học KHTN Hà Nội năm 2017 (vòng 2)
13. Đề thi vào lớp 10 chuyên Đại học KHTN Hà Nội năm 2016 (vòng 1)
14. Đề thi vào lớp 10 chuyên Đại học KHTN Hà Nội năm 2016 (vòng 2)
15. Đề thi vào lớp 10 chuyên Đại học KHTN Hà Nội năm 2015 (vòng 1)
16. Đề thi vào lớp 10 chuyên Đại học KHTN Hà Nội năm 2015 (vòng 2)
17. Đề thi vào lớp 10 chuyên Đại học KHTN Hà Nội năm 2014 (vòng 1)
18. Đề thi vào lớp 10 chuyên Đại học KHTN Hà Nội năm 2014 (vòng 2)
19. Đề thi vào lớp 10 chuyên Đại học KHTN Hà Nội năm 2013 (vòng 1)
20. Đề thi vào lớp 10 chuyên Đại học KHTN Hà Nội năm 2013 (vòng 2)
21. Đề thi vào lớp 10 chuyên Đại học KHTN Hà Nội năm 2012 (vòng 1)
22. Đề thi vào lớp 10 chuyên Đại học KHTN Hà Nội năm 2012 (vòng 2)
23. Đề thi vào lớp 10 chuyên Đại học KHTN Hà Nội năm 2011 (vòng 1)
24. Đề thi vào lớp 10 chuyên Đại học KHTN Hà Nội năm 2011 (vòng 2)
25. Đề thi vào lớp 10 chuyên Đại học KHTN Hà Nội năm 2010 (vòng 1)
26. Đề thi vào lớp 10 chuyên Đại học KHTN Hà Nội năm 2010 (vòng 2)
27. Đề thi vào lớp 10 chuyên Đại học KHTN Hà Nội năm 2009 (vòng 1)
28. Đề thi vào lớp 10 chuyên Đại học KHTN Hà Nội năm 2009 (vòng 2)

PHẦN 2: HƯỚNG DẪN GIẢI

Liên hệ tài liệu word môn toán: 039.373.2038 TÀI LIỆU TOÁN HỌC
Website:tailieumontoan.com

SỞ GIÁO DỤC VÀ ĐÀO TẠO KỲ THI VÀO LỚP 10 CHUYÊN KHOA HỌC TỰ NHIÊN
HÀ NỘI NĂM HỌC 2022 – 2023
Môn: TOÁN (VÒNG 1)
ĐỀ CHÍNH THỨC Thời gian làm bài: 120 phút
(Không kể thời gian giao đề)
Đề thi gồm 01 trang

Đề số 1

Câu I. (4 điểm)
6 ( xy + 5 ) + x3 y + 5 x 2 =
42
1) Giải hệ phương trình 
 x + 5 x y + 6 x + 30 y =
3 2
42
2) Giải phương trình :
( 3
)( )
x + 6 + 3 3 − x 2 + 3 3 ( x + 6 )( 3 − x ) =24
Câu II. (2 điểm)
1) Tìm tất cả các cặp số nguyên ( x, y ) thỏa mãn đẳng thức :
25 y 2 + 354 x + 60= 36 x 2 + 305 y + ( 5 y − 6 x )
2022

2) Trên bàn có 8 hộp rỗng (trong các hộp không có viên bi nào). Người ta thực hiện các lần thêm bi
vào các hộp theo quy tắc sau : mỗi lần ta chọn ra 4 hộp bất kỳ và bỏ vào hộp 1 viên, một hộp 2
viên, hai hộp còn lại mỗi hộp 3 viên. Hỏi số lần thêm bi ít nhất có thể để nhận được số bi ở 8 hộp
trên là số tự nhiên liên tiếp ?
Câu III. (3 điểm)
Cho hình chữ nhật ABCD ( AB < AD ) nội tiếp đường tròn ( O ) . Trên cạnh AD lấy hai điểm E và F ( E , F
1
không trùng với A, D) sao cho E nằm giữa A và F, đồng thời ∠ABE + ∠DCF = ∠BOC
2
1) Chứng minh rằng BE và CF cắt nhau tại một điểm nằm trên đường tròn ( O )
2) Đường thẳng qua O song song với BC cắt BE , CF theo thứ tự tại M , N . Chứng minh rằng
1
∠DAM + ∠ADN + ∠AOD = 180°
2
3) Dựng hình chữ nhật MNPQ sao cho NQ song song với BD, đồng thời MP song song với AC.
Chứng minh rằng đường tròn ngoại tiếp hình chữ nhật MNPQ tiếp xúc với đường tròn ( O )
Câu IV. (1 điểm)
Cho a, b, c là những số thực dương. Chứng minh rằng :
2a a + b 6a + 2c 4a + 3b + c 32a
+ + + ≥
a + b a + c 3b + c b+c 2a + b + c

Liên hệ tài liệu word môn toán: 039.373.2038 TÀI LIỆU TOÁN HỌC
Website:tailieumontoan.com
SỞ GIÁO DỤC VÀ ĐÀO TẠO KỲ THI VÀO LỚP 10 CHUYÊN KHOA HỌC TỰ NHIÊN
HÀ NỘI NĂM HỌC 2022 – 2023
Môn: TOÁN (VÒNG 2)
ĐỀ CHÍNH THỨC Thời gian làm bài: 120 phút
(Không kể thời gian giao đề)
Đề thi gồm 01 trang

Đề số 2

Câu I. (3,5 điểm)


1 1 1
1) Với a, b, c là những số thực dương thỏa mãn điều kiện + + =1. Chứng minh rằng :
a b c
1 1 1 1  abc
 + + =
2  a + bc b + ca c + ab  ( a + bc )( b + ca )( c + ab )
2 x 2 + 3 xy + y 2 =
6
2) Giải hệ phương trình 
3 x + 2 y =
+1 2 2x + y + 6
Câu II. (2,5 điểm)
1) Tìm tất cả các cặp số nguyên dương ( x; y ) thỏa mãn đẳng thức :
( x + y )( 5 x + y ) + xy 3 = ( 5 x + y ) + x 2 y 3 + xy 4
3 3

2) Với a, b, c là những số thực dương thỏa mãn các điều kiện sau
c ≤ b < a ≤ 3; b 2 + 2a ≤ 10; b 2 + 2a + 2c ≤ 14
 2 . Tìm giá trị lớn nhất của biểu thức:
( a + 1)( b 2
+ 1) + 4 ab ≤ 2 a 3
+ 2b 3
+ 2 a + 2b
P = 4a 4 + b 4 + 2b 2 + 4c 2
Câu III. (3 điểm)
Cho tam giác ABC nhọn, không cân, nội tiếp đường tròn (O). Điểm P nằm trong tam giác ABC .
Gọi E , F lần lượt là hình chiếu vuông góc của P trên các cạnh CA, CB . Giả sử tứ giác BCEF nội tiếp
trong đường tròn ( K )
1) Chứng minh rằng AP vuông góc với BC
2) Chứng minh rằng AP = 2OK
3) Đường thẳng qua P vuông góc với AP cắt đường tròn tại hai điểm Q, R . Chứng minh rằng đường
tròn tâm A bán kính AP tiếp xúc với đường tròn ngoại tiếp ∆KQR
Câu IV. (1 điểm)
Cho các điểm A1 , A2 ,....., A30 theo thứ tự nằm trên một đường thẳng sao cho độ dài các đoạn Ak Ak +1
bằng k (đơn vị dài), với k = 1, 2,...., 29 . Ta tô màu mỗi đoạn thẳng A1 A2 ,....., A29 A30 bởi 1 trong 3 màu (mỗi
đoạn được tô bởi đúng 1 màu). Chứng minh rằng với mọi cách tô màu,, ta luôn chọn được hai số nguyên
dương 1 ≤ j ≤ i ≤ 29 sao cho hai đoạn Ai Ai +1 và Aj Aj +1 được tô cùng màu và i − j là bình phương của số
nguyên dương.

Liên hệ tài liệu word môn toán: 039.373.2038 TÀI LIỆU TOÁN HỌC
Website:tailieumontoan.com

SỞ GIÁO DỤC VÀ ĐÀO TẠO KỲ THI VÀO LỚP 10 CHUYÊN KHOA HỌC TỰ NHIÊN
HÀ NỘI NĂM HỌC 2021 – 2022
Môn: TOÁN (VÒNG 1)
ĐỀ CHÍNH THỨC Thời gian làm bài: 120 phút
(Không kể thời gian giao đề)
Đề thi gồm 01 trang

Đề số 3

Bài 1. Giải phương trình 13 5 − x + 18 x + 8 = 61 + x + 3 ( 5 − x )( x + 8) .


 x 4 + y 4 + 6 x 2 y 2 = 1 (1)
Bài 2. Giải hệ phương trình:  .
 ( + ) =− ( )
4
x x y x y 2
Bài 3. Tìm số nguyên dương n nhỏ nhất, biết rằng khi chia n cho 7, 9, 11, 13 được các số dư tương
ứng là 3, 4, 5, 6 .
Bài 4. Cho tam giác nhọn ABC có điểm P nằm trong tam giác ( P không nằm trên các cạnh). Gọi
J , K , L lần lượt là tâm đường tròn nội tiếp các tam giác PBC , PCA , PAB .
 + CKA
1. Chứng minh rằng BJC + ALB =450° .
2. Giả sử PB = PC và PC < PA . Gọi X , Y , Z lần lượt là hình chiếu vuông góc của J , K , L trên
.
các cạnh BC , CA, AB . Dựng hình bình hành XYWZ . Chứng minh W nằm trên phân giác BAC
Bài 5. Cho tập A = {1; 2; 3;...; 2021} . Tìm số nguyên dương k lớn nhất ( k > 2 ) sao cho ta có thể chọn
được k số phân biệt từ tập A mà tổng của hai số phân biệt bất kỳ trong k số được chọn không
chia hết cho hiệu của chúng.

 HẾT 

Liên hệ tài liệu word môn toán: 039.373.2038 TÀI LIỆU TOÁN HỌC
Website:tailieumontoan.com

SỞ GIÁO DỤC VÀ ĐÀO TẠO KỲ THI VÀO LỚP 10 CHUYÊN KHOA HỌC TỰ NHIÊN
HÀ NỘI NĂM HỌC 2021 – 2022
Môn: TOÁN (VÒNG 2)
ĐỀ CHÍNH THỨC Thời gian làm bài: 120 phút
(Không kể thời gian giao đề)
Đề thi gồm 01 trang

Đề số 4

Câu 1.
1) Với a, b, c là các số thực thỏa mãn a + b + c ≠ 0 và (a + b)(b + c)(c + a ) =
1.
a b 1 + abc + ab(a + b + c)
Chứng minh rằng + 2 = ⋅
a (a + b + c) + 1 + abc b (a + b + c) + 1 + abc
2
( a + b + c) 2
 x 2 + 4 y 2 + 4 xy + 2 x 2 y 2 =
 11
2) Giải hệ phương trình:  .
3 xy ( x + 2 y ) + 31 = 9 x + 18 y + 13 xy

Câu 2.
1) Tìm x, y nguyên dương thỏa mãn 3x + 29 =
2 y.
2) Với a, b, c là các số thực dương thỏa mãn điều kiện 2 ( a + b + c ) + ab + bc + ca =
9.
a +1 b +1 c +1
Tìm giá trị lớn nhất của biểu thức M = + 2 + 2 ⋅
a + 10a + 21 b + 10b + 21 c + 10c + 21
2

 nhọn có đường tròn nội tiếp ( O ) . Các điểm M , N lần lượt thuộc
Câu 3. Cho hình thoi ABCD có BAD
các cạnh CB, CD sao cho MN tiếp xúc (O) tại P và tam giác CMN không cân. MN lần lượt cắt
AB, AD tại E , F . Gọi K , L lần lượt là trực tâm các ∆BME , ∆DNF .
1) Chứng minh OP đi qua trung điểm I của KL .
OI EF 1
2) Gọi H là trực tâm tam giác CMN . Chứng minh − =
− .
CH 2 MN 2
3) Gọi EK , FL lần lượt cắt BD tại S , T . NS cắt MT tại Q . Đường tròn nội tiếp tam giác CMN
tiếp xúc MN với tại G . Chứng minh PQ song song với GH .
a1 a a
Câu 4. Giả sử a1 , a2 ,....., a2021 là những số thực thỏa mãn + 2 2 + ...... + 2021 =
0.
1 + a1 1 + a2
2
1 + a2021
2

a1 2a2 kak 2k + 1
Chứng minh rằng tồn tại số nguyên k (1 ≤ k ≤ 2021) sao cho + + ...... + ≤ ⋅
1 + a1 1 + a2
2 2
1 + ak
2
8
……………………….HẾT………………………

Liên hệ tài liệu word môn toán: 039.373.2038 TÀI LIỆU TOÁN HỌC
Website:tailieumontoan.com

SỞ GIÁO DỤC VÀ ĐÀO TẠO KỲ THI VÀO LỚP 10 CHUYÊN KHOA HỌC TỰ NHIÊN
HÀ NỘI NĂM HỌC 2020 – 2021
Môn: TOÁN (VÒNG 1)
ĐỀ CHÍNH THỨC Thời gian làm bài: 120 phút
(Không kể thời gian giao đề)
Đề thi gồm 01 trang

Đề số 5

Câu I. (4 điểm)

 x 2 + y 2 + xy =
7
1) Giải hệ phương trình :  3
9 x =xy + 70 ( x − y )
2

2) Giải phương trình: 11 5 − x + 8 2 x − 1 = 24 + 3 ( 5 − x )( 2 x − 1)

Câu II. (2 điểm)

1) Tìm x, y nguyên dương thỏa mãn x 2 y 2 − 16 xy + 99 = 9 x 2 + 36 y 2 + 13 x + 26 y


2) Với a, b là những số thực dương thỏa mãn
2 ≤ 2a + 3b ≤ 5 ;8a + 12b ≤ 2a 2 + 3b 2 + 5ab + 10
Chứng minh rằng: 3a 2 + 8b 2 + 10ab ≤ 21
Câu III. (3 điểm)
 là góc nhỏ nhất trong ba góc của tam giác và nội tiếp đường tròn (O). Điểm
Cho tam giác ABC có BAC
 . Lấy các điểm M , N thuộc (O) sao cho đường thẳng
D thuộc cạnh BC sao cho AD là phân giác BAC
CM , BN cùng song song với đường thẳng AD
1) Chứng minh rằng AM = AN
2) Gọi giao điểm của đường thẳng MN với các đường thẳng AC , AB lần lượt là E , F . Chứng minh
rằng bốn điểm B, C , E , F cùng thuộc một đường tròn
3) Gọi P, Q theo thứ tự là trung điểm của các đoạn thẳng AM , AN . Chứng minh rằng các đường
thẳng EQ, FP, AD đồng quy.

Câu IV. (1 điểm) Với a, b, c là những số thực dương thỏa mãn a + b + c =3. Chứng minh rằng:

a ( a + bc ) b ( b + ca ) c ( c + ab )
2 2 2

+ + ≥4
b ( ab + 2c 2 ) c ( bc + 2a 2 ) a ( ca + 2b 2 )

Liên hệ tài liệu word môn toán: 039.373.2038 TÀI LIỆU TOÁN HỌC
Website:tailieumontoan.com

SỞ GIÁO DỤC VÀ ĐÀO TẠO KỲ THI VÀO LỚP 10 CHUYÊN KHOA HỌC TỰ NHIÊN
HÀ NỘI NĂM HỌC 2020 – 2021
Môn: TOÁN (VÒNG 1)
ĐỀ CHÍNH THỨC Thời gian làm bài: 120 phút
(Không kể thời gian giao đề)
Đề thi gồm 01 trang

Đề số 6

Câu I. (4 điểm)
( x + y )( x + 1) =
4
1) Giải hệ phương trình:  2
( )(
 y + xy + x + y + 5 x + y + 12 y + 13 =
3 3
)
243

2) Giải phương trình: ( x − 12)7 + (2 x − 12)7 + (24 − 3 x)7 =


0

Câu II. (2 điểm)


1) Tìm tất cả các số nguyên dương a, b, c sao cho cả ba số 4a 2 + 5b; 4b 2 + 5c; 4c 2 + 5a đều là bình phương
của số nguyên dương.
2) Từ một bộ bốn số thực (a, b, c, d ) ta xây dựng bộ số mới (a + b, b + c, c + d , d + a ) và liên tiếp xây dựng
các bộ số mới theo quy tắc trên. Chứng minh rằng nếu ở hai thời điểm khác nhau, ta thu được cùng một bộ
số (có thể khác thứ tự) thì bộ số ban đầu phải có dạng ( a, −a, a, −a )

Câu III. (3 điểm) Cho tam giác ABC cân tại A với ∠BAC < 90o . Gọi P là giao điểm của BE với trung
trực của BC. Gọi K là hình chiếu vuông góc của P lên AB. Gọi Q là hình chiếu vuông góc của E lên
AP. Gọi giao điểm của EQ và PK là F .
1) Chứng minh rằng bốn điểm A, E, P, F cùng thuộc một đường tròn.
2) Gọi giao điểm của KQ và PE là L. Chứng minh LA vuông góc LE.
3) Gọi giao điểm của FL và AB là S . Gọi giao điểm của KE và AL là T . Lấy R là điểm đối xứng với A
qua L . Chứng minh rằng đường tròn ngoại tiếp tam giác AST và đường tròn ngoại tiếp tam giác BPR tiếp
xúc với nhau.
Câu IV. (1 điểm) Với a, b, c là những số thực dương thỏa mãn a + b + c =3 . Chứng minh rằng
2
1 1 1  4  a b c 
3  + + − 1 + 1 ≥ + 3 + + 
a b c  abc  bc ca ab 

Liên hệ tài liệu word môn toán: 039.373.2038 TÀI LIỆU TOÁN HỌC
Website:tailieumontoan.com

SỞ GIÁO DỤC VÀ ĐÀO TẠO KỲ THI VÀO LỚP 10 CHUYÊN KHOA HỌC TỰ NHIÊN
HÀ NỘI NĂM HỌC 2019 – 2020
Môn: TOÁN (VÒNG 1)
ĐỀ CHÍNH THỨC Thời gian làm bài: 150 phút
(Không kể thời gian giao đề)
Đề thi gồm 01 trang

Đề số 7

Bài 1.

26 x + 5
a, Giải phương trình: + 2 26 x +=
5 3 x 2 + 30
x + 30
2

 x 2 + y 2 = 2
b, Giải hệ phương trình: 
( x + 2 y )(2 + 3 y + 4 xy ) =
2
27

Bài 2.

a, Tìm tất cả các cặp số nguyên thỏa mãn: ( x 2 − x + 1)( y 2 + xy ) = 3 x − 1

b, Với x,y là các số thực thay đổi thỏa mãn 1≤ y ≤ 2 và xy + 2 ≥ 2y, tìm giá trị nhỏ nhất của biểu
x2 + 4
thức M = 2
y +1

Bài 3. Cho hình vuông ABCD, đường tròn (O) nội tiếp hình vuông tiếp xúc với các cạnh AB, AD
tại hai điểm E,F. Gọi G là giao điểm các đường thẳng CE và BF.

a, Chứng minh rằng năm điểm A,F,O,G,E cùng nằm trên một đường tròn

b, Gọi giao điểm của đường thẳng FB và đường tròn là M(M ≠ F). CMR M là trung điểm của
đoạn thẳng BG.

c, CMR trực tâm của tam giác GAF nằm trên đường tròn (O)

Bài 4. Cho x, y, z là các số thực dương thỏa mãn xy + yz + xz = 1. Chứng minh rằng:

3
1 1 1 2 x y z 
+ + ≥  + + 
1+ x 1+ y 1+ z
2 2 2
3  1+ x 2
1+ y 2
1 + z2 
 

Liên hệ tài liệu word môn toán: 039.373.2038 TÀI LIỆU TOÁN HỌC
Website:tailieumontoan.com

SỞ GIÁO DỤC VÀ ĐÀO TẠO KỲ THI VÀO LỚP 10 CHUYÊN KHOA HỌC TỰ NHIÊN
HÀ NỘI NĂM HỌC 2019 – 2020
Môn: TOÁN (VÒNG 2)
ĐỀ CHÍNH THỨC Thời gian làm bài: 150 phút
(Không kể thời gian giao đề)
Đề thi gồm 01 trang

Đề số 8

Bài 1.
 3 x 2 + y 2 + 4 xy = 8
a. Giải hệ phương trình:  .
( x + y ) ( x + xy + 2 ) =
2
8

27 + x 2 + x 27 + 2 x
b. Giải phương trình: =
2 + 5 − ( x2 + x ) 2 + 5 − 2x

Bài 2.
a. Chứng minh rằng với mọi số nguyên dương n , ta luôn có
7 7 7
( 27 n + 5 )7 + 10  + (10n + 27 )7 + 5 + ( 5n + 10 )7 + 27 
     
chia hết cho 42 .
b. Với x, y là các số thực dương thay đổi thỏa mãn điều kiện
4 x 2 + 4 y 2 + 17 xy + 5 x + 5 y ≥ 1 .
Tìm giá trị nhỏ nhất của biểu thức: P = 17 x 2 + 17 y 2 + 16 xy .
Bài 3. Cho tam giác ABC cân tại A , có đường tròn nội tiếp ( I ) . Các điểm E , F theo thứ tự
thuộc các cạnh CA, AB ( E khác C và A ; F khác B và A ) sao cho EF tiếp xúc với đường tròn
(I ) tại điểm P . Gọi K , L lần lượt là hình chiếu vuông góc của E , F trên BC . Giả sử FK cắt EL
tại điểm J . Gọi H là hình chiếu vuông góc của J trên BC .
a) Chứng minh rằng HJ là phân giác của góc EHF .
S1 BF 2
b) Kí hiệu S1 , S 2 lần lượt là diện tích của các tứ giác BFJL và CEJK . Chứng minh rằng =
S 2 CE 2
.
c) Gọi D là trung điểm của cạnh BC . Chứng minh rằng ba điểm P, ,  J D thẳng hàng.
Bài 4. Cho M là tập tất cả 4039 số nguyên liên tiếp từ −2019 đến 2019 . Chứng minh rằng trong
2021 số đôi một phân biệt được chọn bất kì từ M luôn tồn tại ba số phân biệt có tổng bằng 0 .

Liên hệ tài liệu word môn toán: 039.373.2038 TÀI LIỆU TOÁN HỌC
Website:tailieumontoan.com

SỞ GIÁO DỤC VÀ ĐÀO TẠO KỲ THI VÀO LỚP 10 CHUYÊN KHOA HỌC TỰ NHIÊN
HÀ NỘI NĂM HỌC 2018 – 2019
Môn: TOÁN (VÒNG 1)
ĐỀ CHÍNH THỨC Thời gian làm bài: 120 phút
(Không kể thời gian giao đề)
Đề thi gồm 01 trang

Đề số 9

Bài 1.
a) Giải phương trình x 2 − x + 2 x 3 + 1= 2 x + 1 .
 2
xy + y =1 + y
b) Giải hệ phương trình  2 2
.
x + 2y + 2xy =+
 4 x
Bài 2.
a) Tìm tất cả các cặp số nguyên ( x; y ) thỏa mãn ( x + y )( 3x + 2y ) = 2x + y − 1 .
2

b
b) Với a, b là các số thực dương thay đổi thỏa mãn a + 2b =2 + . Tìm giá trị nhỏ nhất
3
a b
của biểu thức
= M + .
a + 2b b + 2a
Bài 3. Cho tam giác ABC có đường tròn nội tiếp ( I ) tiếp xúc với các cạnh BC, CA, AB lần lượt tại
các điểm D, E, F. Gọi K là hình chiếu vuông góc của B trên đường thẳng DE và M là trung điểm
của đoạn thẳng DF.
a) Chứng minh rằng hai tam giác BKM và DEF đồng dạng với nhau.
b) Gọi L là hình chiếu của vuông góc của C trên đường thẳng DF và N là trung điểm của
đoạn thẳng DE. Chứng minh rằng hai đường thẳng MK và NL song song với nhau.
c) Gọi J, X lần lượt là trung điểm của các đoạn thẳng KL và ID. Chứng minh rằng đường
thẳng JX vuông góc với đường thẳng EF.
Bài 4. Trên mặt phẳng cho hai điểm P và Q phân biệt. Xét 10 đường thẳng nằm trong mặt
phẳng trên thỏa mãn các tính chất sau:
i) Không có hai đường thẳng nào song song hoặc trùng nhau.
ii) Mỗi đường thẳng đi qua P hoặc Q, không có đường thẳng nào đi qua cả P và Q.
Hỏi 10 đường thẳng trên có thể chia mặt phẳng thành tối đa bao nhiêu miền? Hãy giải thích.

Liên hệ tài liệu word môn toán: 039.373.2038 TÀI LIỆU TOÁN HỌC
Website:tailieumontoan.com

SỞ GIÁO DỤC VÀ ĐÀO TẠO KỲ THI VÀO LỚP 10 CHUYÊN KHOA HỌC TỰ NHIÊN
HÀ NỘI NĂM HỌC 2018 – 2019
Môn: TOÁN (VÒNG 2)
ĐỀ CHÍNH THỨC Thời gian làm bài: 150 phút
(Không kể thời gian giao đề)
Đề thi gồm 01 trang

Đề số 10

Câu 1.

 xy ( x + y ) =2
a) Giải hệ phương trình :  3
 x + y + x y + 7 ( x + 1)( y + 1) =
3 3 3
31

b) Giải phương trình: 9 + 3 x ( 3 − 2 x ) = 7 x + 5 3 − 2 x

Câu 2.
a) Cho x, y là các số nguyên sao cho x 2 − 2 xy − y 2 ; xy − 2 y 2 − x đều chia hết cho 5. Chứng minh
2 x 2 + y 2 + 2 x + y cũng chia hết cho 5
b) Cho a1 , a2 ,......, a50 là các số nguyên thỏa mãn: 1 ≤ a1 ≤ a2 ...... ≤ a50 ≤ 50 , a1 + a2 + ..... + a50 =
100 .
Chứng minh rằng từ các số đã cho có thể chọn được một vài số có tổng là 50
Câu 3. Cho ngũ giác lồi ABCDE nội tiếp (O) có CD / / BE . Hai đường chéo CE và BD cắt nhau tại
 = PAE
P. Điểm M thuộc BE sao cho MAB  . Điểm K thuộc AC sao cho MK song song AD, điểm L

thuộc đường thẳng AD sao cho ML // AC. Đường tròn ngoại tiếp tam giác KBC cắt BD, CE tại Q
và S (Q khác B, S khác C)
a) Chứng minh 3 điểm K, M, Q thẳng hàng
b) Đường tròn ngoại tiếp tam giác LDE cắt BD, CE tai T và R (T khác D, R khác E). Chứng
minh M, S, Q, R,T cùng thuộc một đường tròn
c) Chứng minh đường tròn ngoại tiếp tam giác PQR tiếp xúc (O)
Câu 4. Cho a, b, c là các số thực dương. Chứng minh rằng
 ab bc   1 1 
 +   + ≤2
 a+b b + c  a + b b+c 

Liên hệ tài liệu word môn toán: 039.373.2038 TÀI LIỆU TOÁN HỌC
Website:tailieumontoan.com

SỞ GIÁO DỤC VÀ ĐÀO TẠO KỲ THI VÀO LỚP 10 CHUYÊN KHOA HỌC TỰ NHIÊN
HÀ NỘI NĂM HỌC 2017 – 2018
Môn: TOÁN (VÒNG 1)
ĐỀ CHÍNH THỨC Thời gian làm bài: 120 phút
(Không kể thời gian giao đề)
Đề thi gồm 01 trang

Đề số 11

Câu 1. (3.5 điểm)


x 2  y 2  xy  1
a) Giải hệ phương trình 
x  x 2y  2y 3


b) Giải phương trình 2 x  1 x  1   


x  1  1 x 2  1 x2 
Câu 2 (2.5 điểm)

a) Chứng minh rằng không tồn tại các số nguyên x, y thỏa mãn đẳng thức

12x 2  26xy  15y 2  4617

b) Với a, b là các số thực dương. Tìm giá trị lớn nhát của biểu thức
 1 1  1
M  a  b  2  2 
a  b b  a  ab

Câu 3 (3.0 điểm)



Cho hình thoi ABCD có BAD  900 . Đường tròn tâm I nội tiếp tam giác ABD tiếp xúc với BD

và BA lần lượt tại J và L. Trên đường thẳng IJ lấy điểm K sao cho BK song song ID.
  ABI
a) Chứng minh rằng CBK .

b) Chứng minh rằng KC  KB .

c) Chứng minh rằng bốn điểm C, K, I ,L cùng nằm trên một đường tròn.

Câu 4. (1.0 điểm)

Tìm tập hợp số nguyên dương n sao cho tồn tại một cách sắp xếp các số 1;2; 3;...;n thành

a1; a2 ; a 3 ;...; an mà khi chia các số a1; a1a2 ; a1a2a 3 ;...; a1a2 ...a n cho n ta được các số dư đôi một khác

nhau.
Liên hệ tài liệu word môn toán: 039.373.2038 TÀI LIỆU TOÁN HỌC
Website:tailieumontoan.com

SỞ GIÁO DỤC VÀ ĐÀO TẠO KỲ THI VÀO LỚP 10 CHUYÊN KHOA HỌC TỰ NHIÊN
HÀ NỘI NĂM HỌC 2017 – 2018
ĐỀ CHÍNH THỨC Môn: TOÁN (VÒNG 2)
Thời gian làm bài: 150 phút
Đề thi gồm 01 trang (Không kể thời gian giao đề)
Đề số 12

Câu 1 (3.5 điểm).




1. Giải hệ phương trình  x  y  x  3y

 2 2
x  y  xy  3


2. Với a, b là các số thực dương thỏa mãn ab  a  b  1 . Chứng minh rằng:
a b 1  ab
 
  
2
1a 1  b2 2 1  a 2 1  b2

Câu 2 (2.5 điểm). 1. Giả sử p và q là các số nguyên tố thỏa mãn đẳng thức p p  1  q q 2  1 .  
a) Chứng minh rằng tồn tại số nguyên dương k sao cho p  1  kq, q 2  1  kp .

b) Tìm tất cả các số nguyên tố p, q thỏa mãn đẳng thức p p  1  q q 2  1 . 
2. Với a, b, c là các số thực dương thỏa mãn ab  bc  ca  abc  2 . Tìm giá trị lớn nhất của
a 1 b 1 c 1
biểu thức M  2
 2  2
a  2a  2 b  2b  2 c  2c  2
Câu 3 (3.0 điểm). Cho tam giác ABC nhọn với AB  AC . Gọi E, F lần lượt là trung điểm của
 và nằm ngoài tam
CA, AB. Đường trung trực của EF cắt BC tại D. Giả sử P nằm trong góc EAF
 D
giác AEF sao cho PEC    DFE
EF và PEB  . Đường thẳng PA cắt đường tròn ngoại tiếp tam
giác PEF tại Q khác P.
  BAC
a) Chứng minh rằng EQF   EDF
.

b) Tiếp tuyến tại P của đường tròn ngoại tiếp tam giác PEF cắt CA, AB lần lượt tại M, N.
Chứng minh rằng bốn điểm C, M, B, N cùng nằ trên một đường tròn. Gọi đường tròn này là
đường tròn K  .
c) Chứng minh rằng đường tròn K  tiếp xúc với đường tròn ngại tiếp tam giác AEF.
Câu 4 (1.0 điểm). Cho n là số nguyên dương với n  5 . Xét đa giác lồi n cạnh. Người ta muốn
kẻ một số đường chéo của đa giác mà các đường chéo này chia đa giác thành đúng k miền, mõi
miền là mọt ngũ giác lồi (hai miền bất kì không có điểm chung trong).
a) Chứng minh rằng ta có thể thực hiện được với n  2018, k  672
b) Với n  2017, k  672 ta có thể thực hiện được không? Hãy giải thích.

Liên hệ tài liệu word môn toán: 039.373.2038 TÀI LIỆU TOÁN HỌC
Website:tailieumontoan.com

SỞ GIÁO DỤC VÀ ĐÀO TẠO KỲ THI VÀO LỚP 10 CHUYÊN KHOA HỌC TỰ NHIÊN
HÀ NỘI NĂM HỌC 2016 – 2017
Môn: TOÁN (VÒNG 1)
ĐỀ CHÍNH THỨC Thời gian làm bài: 120 phút
(Không kể thời gian giao đề)
Đề thi gồm 01 trang

Đề số 13

Câu 1 (3.5 điểm).

x + y + xy x + y = 4

 3 3

a) Giải hệ phương trình 





 
xy + 1 x2 + y 2 = 4 
8x  3
b) Giải phương trình 7x  2  5  x 
5
Câu 2 (2.5 điểm)
a)Tìm tất cả các giá tri của m sao cho tồn tại cặp số nguyên x ; y  thỏa mãn hệ phương trình :
2  mxy 2  3m


 
2  m x 2  y 2  6m

b) Với x, y là những số thực thỏa mãn các điều kiện 0  x  y  2;2x  y  2xy .

 
Tìm giá trị lớn nhất của biểu thức P  x 2 x 2  1  y2 y2  1  
Câu 3 (3.0 điểm). Cho tam giác ABC nhọn nội tiếp đường tròn O  với AB  AC . Phân giác của
 cắt BC tại D và cắt đường tròn
góc BAC O  tại E khác A. M là trung điểm của đoạn thẳng AD.
Đường thẳng BM cắt đường tròn O  tại P khác B. Giả sử các đường thẳng EP và AC cắt nhau tại
N.
a) chứng minh rằng tứ giác APNM nội tiếp và N là trung điểm của đoạn thẳng AC.
b) Giả sử đường tròn K  ngoại tiếp tam giác EMN cắt đường thẳng AC tại Q khác N. Chứng
minh rằng B và Q đối xứng nhau qua AE.
c) Giả sử đường tròn K  cắt đường thẳng BM tại M. Chứng minh rằng RA vuông góc RC.
Câu 4 (1.0 điểm).
Số nguyên a được gọi là số “đẹp” nếu với mọi cách sắp xếp theo thứ tự tùy ý của 100 số 1,
2, 3,…, 100 luôn tồn tại 10 số hạng liên tiếp có tổng lớn hơn hoặc bằng a. Tìm số “đẹp” lớn nhất

Liên hệ tài liệu word môn toán: 039.373.2038 TÀI LIỆU TOÁN HỌC
Website:tailieumontoan.com

SỞ GIÁO DỤC VÀ ĐÀO TẠO KỲ THI VÀO LỚP 10 CHUYÊN KHOA HỌC TỰ NHIÊN
HÀ NỘI NĂM HỌC 2016 – 2017
Môn: TOÁN (VÒNG 2)
ĐỀ CHÍNH THỨC Thời gian làm bài: 150 phút
(Không kể thời gian giao đề)
Đề thi gồm 01 trang

Đề số 14

Câu 1(3.5 điểm).


x 2  4y 2  5

a) Giải hệ phương trình  2
4x  8xy 2  5x  10y  1

64x 3  4x
b) giải phương trình 5x 2  6x  5 
5x 2  6x  6
Câu 2 (2.5 điểm).
x 2  1 y2  1
a) Với x, y là những số nguyên thỏa mãn đẳng thức  . Chứng minh
2 3
x 2  y 2  40 .
b) Tìm tất cả các cặp số nguyên x ; y  thỏa mãn đẳng thức sau x 4  2x 2  y 3 .
Câu 3 (3.0 điểm) Cho hình vuông ABCD nội tiếp đường tròn O  . P là điểm thuộc cung nhỏ AD
của đường tròn O  và P khác A, D. Các đường thẳng PB, PC lần lượt cắt AD tại AD tại M, N.
Đường trung trực của AM cắt đường thẳng AC, PB lần lượt tại E, K. Đường trung trực DN cắt
các đường thẳng BD, PC lần lượt tại F, L.
a) Chứng minh rằng ba điểm K, O, L thẳng hàng.
b) Chứng minh đường thẳng PO đi qua trung điểm của EF
c) Giả sử đường thảng EK cắt đường thẳng FL và AC cắt nhau tại T. Đường thẳng ST cắt
các đường thẳng PB, PC lần lượt tại U và V. Chứng minh rằng bốn điểm K, L, V, U cùng thuộc
một đương tròn.
Câu 4(1.0 điểm) Chứng minh rằng với mọi số tự nhiên n  3 luôn tồn tại cách xếp bộ n số
xi  xk
1, 2, 3,...,n thành bộ số x 1, x 2 , x 3 ,..., x n sao cho x j  với mọi bộ chỉ số i; j ; k  mà
2
1i  j k n .

Liên hệ tài liệu word môn toán: 039.373.2038 TÀI LIỆU TOÁN HỌC
Website:tailieumontoan.com

SỞ GIÁO DỤC VÀ ĐÀO TẠO KỲ THI VÀO LỚP 10 CHUYÊN KHOA HỌC TỰ NHIÊN
HÀ NỘI NĂM HỌC 2015 – 2016
Môn: TOÁN (VÒNG 1)
ĐỀ CHÍNH THỨC Thời gian làm bài: 120 phút
(Không kể thời gian giao đề)
Đề thi gồm 01 trang
Đề số 15
Câu 1. (3,0 điểm).
1). Giả sử a; b là hai số thực phân biệt thỏa mãn a 2  3a  b2  3b  2 .
a). Chứng minh rằng a  b  3 .
b). Chứng minh rằng a 3  b3  45 .

2 x  3 y  5 xy
2). Giải hệ phương trình  2 .
 2 2
4 x  y  5 xy

Câu 2. (3,0 điểm).
1). Tìm các số nguyên  x; y không nhỏ hơn 2 sao cho xy  1 chia hết cho  x  1 y  1 .
2). Với x; y là những số thực thỏa mãn đẳng thức x 2 y 2  2 y  1  0. Tìm giá trị lớn nhất và nhỏ
xy
nhất của biểu thức P  .
3y  1
Câu 3. (3,0 điểm). Cho tam giác nhọn ABC không cân có tâm đường tròn nội tiếp là điểm I .
Đường thẳng AI cắt BC tại D . Gọi E; F lần lượt là các điểm đối xứng của D qua IC ; IB .
1). Chứng minh rằng EF song song với BC .
2). Gọi M ; N ; J lần lượt là trung điểm các đoạn thẳng DE; DF ; EF . Đường tròn ngoại tiếp tam
giác AEM cắt đường tròn ngoại tiếp tam giác AFN tại P khác A . Chứng minh rằng bốn điểm
M ; N ; P; J cùng nằm trên một đường tròn.
3). Chứng minh rằng ba điểm A; J ; P thẳng hàng.
Câu 4. (1,0 điểm).
1). Cho bảng ô vuông 2015 x2015. Kí hiệu ô (i ; j ) là ô ở hang thứ i ,
cột thứ j . Ta viết các số nguyên dương từ 1 đến 2015 vào các ô của
bảng theo quy tắc sau:
i). Số 1 được viết vào ô (1,1)
ii). Nếu số k được viết vào ô (i ; j ) ( i  1 ) thì số k  1 được viết vào
ô (i  1; j  1) .
iii). Nếu số k được viết vào ô (1; j ) thì số k  1 được viết vào ô ( j  1; 1) (xem hình 1).
Khi đó số 2015 được viết vào ô m; n .
Hãy xác định m và n.
2). Giả sử a; b; c là các số thực dương thỏa mãn ab  bc  ac  abc  4.
Chứng minh rằng a 2  b2  c 2  a  b  c  2 ab  bc  ac .

Liên hệ tài liệu word môn toán: 039.373.2038 TÀI LIỆU TOÁN HỌC
Website:tailieumontoan.com

SỞ GIÁO DỤC VÀ ĐÀO TẠO KỲ THI VÀO LỚP 10 CHUYÊN KHOA HỌC TỰ NHIÊN
HÀ NỘI NĂM HỌC 2015 – 2016
Môn: TOÁN (VÒNG 2)
ĐỀ CHÍNH THỨC Thời gian làm bài: 150 phút
(Không kể thời gian giao đề)
Đề thi gồm 01 trang

Đề số 16

Câu 1. (3,0 điểm).


1) Với a; b; c là các số thỏa mãn
3 3 3 3
3a  3b  3c  24  3a  b  c  3b  c  a  3c  a  b .

Chứng minh rằng a  2bb  2cc  2 a  1 .



2 x  2 y  xy  5
2) Giải hệ phương trình  .
27  x  y  y  7  26 x 3  27 x 2  9 x
3

Câu 2. (3,0 điểm).
1) Tìm số tự nhiên n để n  5 và n  30 là số chính phương (số chính phương là bình phương
của một số nguyên)
2) Tìm  x; y nguyên thỏa mãn đẳng thức 1  x  y  3  x  y .
3) Giả sử x; y; z là các số thực lớn hơn 2. Tìm giá trị nhỏ nhất của biểu thức:
x y z
P   .
y  z4 z x4 x y4

Câu 3. (3,0 điểm). Cho tam giác ABC nhọn không cân với AB  AC . Gọi M là trung điểm của
đoạn thẳng BC . Gọi H là hình chiếu vuông góc của B trên đoạn AM . Trên tia đối của tia AM
lấy điểm N sao cho AN  2 MH .
1) Chứng minh rằng BN  AC .
2) Gọi Q là điểm đối xứng với A qua N . Đường thẳng AC cắt BQ tại D .Chunwgs minh rằng
bốn điểm B; D; N ; C cùng thuộc một đường tròn,gọi đường tròn này là (O) .
3) Đường tròn ngoại tiếp tam giác AQD cắt (O) tại G và D . Chứng minh rằng NG song song
với BC .
Câu 4. (1,0 điểm). Ký hiệu S là tập hợp gồm 2015 diểm phân biệt trên một mặt phẳng. Giả sử tất
cả các điểm của S không cùng nằm trên một đường thẳng. Chứng minh rằng có ít nhất 2015
đường thẳng phân biệt mà mỗi đường thẳng đi qua ít nhất hai điểm của S .
Liên hệ tài liệu word môn toán: 039.373.2038 TÀI LIỆU TOÁN HỌC
Website:tailieumontoan.com

SỞ GIÁO DỤC VÀ ĐÀO TẠO KỲ THI VÀO LỚP 10 CHUYÊN KHOA HỌC TỰ NHIÊN
HÀ NỘI NĂM HỌC 2014 – 2015
Môn: TOÁN (VÒNG 1)
ĐỀ CHÍNH THỨC Thời gian làm bài: 120 phút
(Không kể thời gian giao đề)
Đề thi gồm 01 trang

Đề số 17

Câu 1. 1) Giải phương trình ( 1+ x + 1− x )(2 + 2 )


1 − x2 =
8.

 x − xy + y =
2 2
1
2) Giải hệ phương trình  2
 x + xy + 2 y =
2
4

Câu 2. 1) Giả sử x, y, z là ba số dương thỏa mãn điều kiện x + y + z = xyz. Chứng minh rằng:
x 2y 3z xyz ( 5 x + 4 y + 3 z )
+ + = .
1+ x 1+ y 1+ z
2 2 2
( x + y )( y + z )( z + x )
2) Tìm nghiệm nguyên của phương trình: x 2 y 2 ( x + y ) + x + y + 3 + xy.

Câu 3. Cho tam giác ABC nhọn với AB < BC. Gọi D là điểm thuộc cạnh BC sao cho AD là phân
giác của ∠BAC . Đường thẳng qua C song song với AD cắt trung trực của AB tại F.
1) Chứng minh tam giác ABF đồng dạng với tam giác ACE.
2) Chứng minh rằng các đường thẳng BE, CF, AD đồng quy tại một điểm, gọi điểm đó là G.
3) Đường thẳng qua G song song với AE cắt đường thẳng BF tại Q. Đường thẳng QE cắt đường
tròn ngoại tiếp tam giác GEC tại P khác E. Chứng minh rằng các điểm A, P, G, Q, F cùng thuộc
một đường tròn.
Câu 4. Giả sử a, b, c là các số thực dương thỏa mãn đẳng thức ab + bc + ca = 1 . Chứng minh rằng
5
2abc ( a + b + c ) ≤ + a 4b2 + b4 c2 + c4 a 2 .
9

Liên hệ tài liệu word môn toán: 039.373.2038 TÀI LIỆU TOÁN HỌC
Website:tailieumontoan.com

SỞ GIÁO DỤC VÀ ĐÀO TẠO KỲ THI VÀO LỚP 10 CHUYÊN KHOA HỌC TỰ NHIÊN
HÀ NỘI NĂM HỌC 2014 – 2015
Môn: TOÁN (VÒNG 2)
ĐỀ CHÍNH THỨC Thời gian làm bài: 150 phút
(Không kể thời gian giao đề)
Đề thi gồm 01 trang

Đề số 18

Câu 1.
y 2 y2 4 y4 8 y4
1. Giả sử x, y là những số thực dương thỏa mãn: + 2 + + =
4
x + y x + y 2 x 4 + y 4 x8 − y 4
Chứng minh rằng: 5y = 4x

2 x − 3 y + xy =
2 2
12
2. Giải hệ phương trình: 
6 x + x y =12 + 6 y + y x

2 2

Câu 2.
1. Cho x, y là những số nguyên lớn hơn 1 sao cho 4x2y2 – 7x + 7y là số chính phương.
Chứng minh rằng: x = y.
2. Giả sử x, y là những số thực không âm thỏa mãn: x3 + y3 + xy = x2 + y2 Tìm giá trị lớn
1+ x 2 + x
nhất và nhỏ nhất của biểu thức:
= P +
2 + y 1+ y
Câu 3. Cho tam giác ABC nội tiếp đường tròn (O) và điểm P nằm trong tam giác thỏa mãn PB =
PC. D là điểm thuộc cạnh BC (D khác B và D khác C) sao cho P nằm trong đường tròn ngoại tiếp
tam giác DAC và đường tròn ngoại tiếp tam giác DAC. Đường thẳng PB cắt đường tròn ngoại
tiếp tam giác DAB tại E khác B. Đường thẳng PC cắt đường tròn ngoại tiếp tam giác DAC tại F
khác C.
1. Chứng minh rằng bốn điểm A, E, B, F cùng thuộc một đường tròn.
2. Giả sử đường thẳng AD cắt đường tròn (O) tại Q khác A, đường thẳng AF cắt đường thẳng
QC tại L. Chứng minh rằng tam giác ABE đồng dạng với tam giác CLF.
3. Gọi K là giao điểm của đường thẳng AE và đường thẳng QB. Chứng minh rằng:
 + PAB
QKL  = QLK + PAC
.
Câu 4. Cho tập hợp A gồm 31 phần tử và dãy gồm m tập hợp của A thỏa mãn đồng thời các
điều kiện sau:
i) Mỗi tập hợp thuộc dãy có ít nhất hai phần tử.
ii) Nếu hai tập hợp thuộc dãy có chung nhau ít nhất hai phần tử thì số phần tử của
hai tập hợp này khác nhau.
Chứng minh rằng: m ≤ 900

Liên hệ tài liệu word môn toán: 039.373.2038 TÀI LIỆU TOÁN HỌC
Website:tailieumontoan.com

SỞ GIÁO DỤC VÀ ĐÀO TẠO KỲ THI VÀO LỚP 10 CHUYÊN KHOA HỌC TỰ NHIÊN
HÀ NỘI NĂM HỌC 2013 – 2014
Môn: TOÁN (VÒNG 1)
ĐỀ CHÍNH THỨC Thời gian làm bài: 120 phút
(Không kể thời gian giao đề)
Đề thi gồm 01 trang

Đề số 19

Câu 1. (3,0 điểm).


1). Giải phương trình
3x  1  2  x  3 .
2). Giải hệ phương trình

 1 1 9

 xy  

 x y 2
 .
 1 3 1  1

   x    xy 
 y  xy
4 2 
 

Câu 2. (3,0 điểm).
1). Cho các số thực a; b; c  0 thỏa mãn a  bb  cc  a  8 abc . Chứng minh rằng

a b c 3 ab bc ca
      .
a  b b  c c  a 4 a  bb  c b  cc  a c  aa  b

2). Có bao nhiêu số nguyên dương có 5 chữ số abcde sao cho abc  10d  e chia hết cho 101 ?

Câu 3. (3,0 điểm). Cho tam giác nhọn ABC nội tiếp đường tròn (O) với AB  AC. Đường phân
 cắt (O) tại điểm D khác A. Gọi M là trung điểm của AD và E là điểm đối
giác của góc BAC
xứng với D qua tâm O . Giả sử đường tròn ngoại tiếp tam giác ABM cắt đoạn thẳng AC tại
điểm F khác A.
1). Chứng minh rằng tam giác BDM và tam giác BCF đồng dạng.
2). Chứng minh rằng EF vuông góc với AC.
Câu 4. (1,0 điểm).
Giả sử a; b; c; d là các số thực dương thỏa mãn điều kiện abc  bcd  cda  dab  1. Tìm giá
trị nhỏ nhất của biểu thức P  4 a 3  b3  c 3   9d 3 .

Liên hệ tài liệu word môn toán: 039.373.2038 TÀI LIỆU TOÁN HỌC
Website:tailieumontoan.com

SỞ GIÁO DỤC VÀ ĐÀO TẠO KỲ THI VÀO LỚP 10 CHUYÊN KHOA HỌC TỰ NHIÊN
HÀ NỘI NĂM HỌC 2013 – 2014
Môn: TOÁN (VÒNG 2)
ĐỀ CHÍNH THỨC Thời gian làm bài: 150 phút
(Không kể thời gian giao đề)
Đề thi gồm 01 trang

Đề số 20
Câu 1. (3,0 điểm).
1). Giải hệ phương trình
x 3  y 3  1  y  x  xy

 .

7 xy  y  x  7
2). Giải phương trình

x  3  1  x2  3 x  1  1  x .
Câu 2. (3,0 điểm).
1). Tìm các cặp số nguyên  x; y thỏa mãn

5 x 2  8 y 2  20412 .

2). Với  x; y là các số thực dương thỏa mãn x  y  1, tìm giá trị nhỏ nhất của biểu thức
1 1
P     1  x 2 y 2 .
 x y 

Câu 3. (3,0 điểm). Cho tam giác nhọn ABC nội tiếp đường tròn (O) có trực tâm H . Gọi P là
điểm nằm trên đường tròn ngoại tiếp tam giác HBC ( P khác B, C và H ) và nằm trong tam
giác ABC . PB cắt (O) tại M khác B, PC cắt (O) tại N khác C . BM cắt AC tại E, CN cắt AB
tại F . Đường tròn ngoại tiếp tam giác AME và đường tròn ngoại tiếp tam giác ANF cắt nhau
tại Q khác A.
1). Chứng minh rằng ba điểm M ; N ; Q thẳng hàng.
 . Chứng minh rằng khi đó PQ đi qua trung điểm của BC.
2). Giả sử AP là phân giác góc MAN
Câu 4. (1,0 điểm). Giả sử dãy số thực có thứ tự x1  x2    x192 thỏa mãn các điều kiện
x1  x2    x192  0 và x1  x2    x192  2013.

Chứng minh rằng


2013
x192  x1  .
96

Liên hệ tài liệu word môn toán: 039.373.2038 TÀI LIỆU TOÁN HỌC
Website:tailieumontoan.com

SỞ GIÁO DỤC VÀ ĐÀO TẠO KỲ THI VÀO LỚP 10 CHUYÊN KHOA HỌC TỰ NHIÊN
HÀ NỘI NĂM HỌC 2012 – 2013
Môn: TOÁN (VÒNG 1)
ĐỀ CHÍNH THỨC Thời gian làm bài: 120 phút
(Không kể thời gian giao đề)
Đề thi gồm 01 trang

Đề số 21

Câu 1. (3,0 điểm).

1). Giải phương trình: x  9  2012 x  6  2012  x  9x  6 .



x 2  y 2  2 y  4
2). Giải hệ phương trình  .

2 x  y  xy  4

Câu 2. (3,0 điểm).

1). Tìm tất cả các cặp số nguyên  x; y thỏa mãn đẳng thức

x  y  1xy  x  y  5  2 x  y .
2). Giả sử  x; y là các số thực dương thỏa mãn điều kiện  x 1 
y  1  4 . Tìm giá trị nhỏ nhất

của biểu thức

x2 y 2
P  .
y x

Câu 3. (3,0 điểm). Cho tam giác nhọn ABC nội tiếp đường tròn tâm O . Gọi M là một điểm trên
 ( M khác B; C và AM không đi qua O ). Giả sử P là một điểm thuộc đoạn thẳng AM
cung nhỏ BC

 tại điểm N khác M .


sao cho đường tròn đường kính MP cắt cung nhỏ BC

1). Gọi D là điểm đối xứng với điểm M qua O . Chứng minh rằng ba điểm N ; P; D thẳng hàng.

2). Đường tròn đường kính MP cắt MD tại điểm Q khác M . Chứng minh rằng P là tâm đường

tròn nội tiếp tam giác AQN .

Câu 4. (1,0 điểm). Giả sử a; b; c là các số thực dương thỏa mãn: a  b  3  c ; c  b  1 ; a  b  c .

2 ab  a  b  c ab  1
Tìm giá trị nhỏ nhất của biểu thức: Q  .
a  1b  1c  1
Liên hệ tài liệu word môn toán: 039.373.2038 TÀI LIỆU TOÁN HỌC
Website:tailieumontoan.com

SỞ GIÁO DỤC VÀ ĐÀO TẠO KỲ THI VÀO LỚP 10 CHUYÊN KHOA HỌC TỰ NHIÊN
HÀ NỘI NĂM HỌC 2012 – 2013
Môn: TOÁN (VÒNG 2)
ĐỀ CHÍNH THỨC Thời gian làm bài: 150 phút
(Không kể thời gian giao đề)
Đề thi gồm 01 trang

Đề số 22

Câu 1. (3,0 điểm).



xy  x  y  2
1). Giải hệ phương trình  .
9 xy 3 x  y  6  26 x 3  2 y 3


2). Giải phương trình  x4 2  


4  x  2  2x .

Câu 2. (3,0 điểm).

1). Tìm hai chữ số cuối cùng của số A  41106  57 2012 .

1 5
2). Tìm giá trị lớn nhất của hàm số y  3 2 x  1  x 5  4 x 2 , với x .
2 2

Câu 3. (3,0 điểm). Cho tam giác nhọn ABC ( AB  AC ) nội tiếp đường tròn (O) . Giả sử M ; N là

 sao cho MN song song với BC và tia AN nằm giữa hai tia
hai điểm thuộc cung nhỏ BC

AM ; AB . Gọi P là hình chiếu vuông góc của điểm C trên AN và Q là hình chiếu vuông góc của

điểm M trên AB .

1). Giả sử CP cắt QM tại điểm T . Chứng minh T nằm trên đường tròn (O) .

2). Gọi giao điểm của NQ và (O) là R khác N . Giả sử AM cắt PQ tại S . Chứng minh rằng bốn

điểm A; R; Q; S cùng thuộc một đường tròn.

Câu 4. (1,0 điểm). Với mỗi số n nguyên lớn hơn hoặc bằng 2 cố định, xét các tập n số thực đôi

một khác nhau X  x1 ; x2 ;...; xn  . Kí hiệu C X  là số các giá trị khác nhau của tổng xi  x j

( 1  i  j  n ). Tìm giá trị lớn nhất và giá trị nhỏ nhất của C X  .

Liên hệ tài liệu word môn toán: 039.373.2038 TÀI LIỆU TOÁN HỌC
Website:tailieumontoan.com

SỞ GIÁO DỤC VÀ ĐÀO TẠO KỲ THI VÀO LỚP 10 CHUYÊN KHOA HỌC TỰ NHIÊN
HÀ NỘI NĂM HỌC 2011 – 2012
Môn: TOÁN (VÒNG 1)
ĐỀ CHÍNH THỨC Thời gian làm bài: 120 phút
(Không kể thời gian giao đề)
Đề thi gồm 01 trang

Đề số 23

Câu 1 (3,0 điểm).



  x  1 y  x  y  3
 2
1). Giải hệ phương trình  .
 2
 ( y  2)x  y  x  1

3 x2  7
2). Giải phương trình x  .
x 2  x  1

Câu 2. (3,0 điểm).

1). Chứng minh rằng không tồn tại các bộ ba số nguyên  x; y; z thỏa mãn đẳng thức:

x4  y 4  7 z4  5 .

2). Tìm tất cả các cặp số nguyên  x; y thỏa mãn đẳng thức
4 4
 x  1   x  1   y 3 .
  90 . Đường phân giác của góc BCD
Câu 3. (3,0 điểm). Cho hình bình hành ABCD với BAD  cắt

đường tròn ngoại tiếp tam giác BCD tại O khác C . Kẻ đường thẳng d đi qua A và vuông góc
với CO . Đường thẳng d lần lượt cắt các đường thẳng CB; CD tại E; F .

1). Chứng minh rằng OBE  ODC .


2). Chứng minh rằng O là tâm đường tròn ngoại tiếp tam giác CEF .
3). Gọi giao điểm của OC và BD là I , chứng minh rằng
IB.BE.EI  ID.DF.FI .
Câu 4. (1,0 điểm). Với x; y là những số thực dương, tìm giá trị nhỏ nhất của biểu thức

x3 4y3
P  .
x3  8 y 3 y 3   x  y
3

Liên hệ tài liệu word môn toán: 039.373.2038 TÀI LIỆU TOÁN HỌC
Website:tailieumontoan.com

SỞ GIÁO DỤC VÀ ĐÀO TẠO KỲ THI VÀO LỚP 10 CHUYÊN KHOA HỌC TỰ NHIÊN
HÀ NỘI NĂM HỌC 2011 – 2012
Môn: TOÁN (VÒNG 2)
ĐỀ CHÍNH THỨC Thời gian làm bài: 150 phút
(Không kể thời gian giao đề)
Đề thi gồm 01 trang

Đề số 24

Câu 1. (3,0 điểm).


1). Giải phương trình :  x3 x  
1 x  1  1 .
 2 2 2 2
x  y  2 x y

2). Giải hệ phương trình:  .



 x  y1  xy  4 x 2 y 2

Câu 2. (3,0 điểm).


1). Với mỗi số thực a ta gọi phần nguyên của a là số nguyên lớn nhất không vượt quá a và ký
2
 1 1 
  
hiệu là  a . Chứng minh rằng với mọi số nguyên dương n , biểu thức n   n    không
3

 27 3 
biểu diễn được dưới dạng lập phương của một số nguyên dương.
2). Với x; y; z là các số thực dương thỏa mãn đẳng thức xy  yz  zx  5 , Tìm giá trị nhỏ nhất của
3x  3 y  2 z
biểu thức : P  .
6  x 2  5  6  y 2  5  6  z 2  5

Câu 3. (3,0 điểm). Cho hình thang ABCD với BC song song AD . Các góc BAD  và CDA là các góc
nhọn. Hai đường chéo AC và BD cắt nhau tại I . P là điểm bất kỳ trên đoạn thẳng BC ( P không
trùng với B; C ). Giả sử đường tròn ngoại tiếp tam giác BIP cắt đoạn thẳng PA tại M khác P và
đường tròn ngoại tiếp tam giác CIP cắt đoạn thẳng PD tại N khác P
1). Chứng minh rằng năm điểm A; M ; I ; N ; D cùng nằm trên một đường tròn. Gọi đường tròn
này là ( K ) .
2). Giả sử các đường thẳng BM và CN cắt nhau tại Q , chứng minh rằng Q cũng nằm trên
đường tròn ( K ) .
PB BD
3). Trong trường hợp P; I ; Q thẳng hàng, chứng minh rằng  .
PC CA
Câu 4. (1,0 điểm). Giả sử A là một tập con của tập các số tự nhiên  . Tập A có phần tử nhỏ nhất
là 1, phần tử lớn nhất là 100 và mỗi x thuộc A ( x  1 ) luôn tồn tại a; b cũng thuộc A sao cho
x  a  b ( a có thể bằng b ). Hãy tìm một tập A có số phần tử nhỏ nhất.

Liên hệ tài liệu word môn toán: 039.373.2038 TÀI LIỆU TOÁN HỌC
Website:tailieumontoan.com

SỞ GIÁO DỤC VÀ ĐÀO TẠO KỲ THI VÀO LỚP 10 CHUYÊN KHOA HỌC TỰ NHIÊN
HÀ NỘI NĂM HỌC 2010 – 2011
Môn: TOÁN (VÒNG 1)
ĐỀ CHÍNH THỨC Thời gian làm bài: 120 phút
(Không kể thời gian giao đề)
Đề thi gồm 01 trang

Đề số 25

Câu I.
1) Giải hệ phương trình

3 x + 8 y + 12 xy = 23
2 2

 2

 x + y = 2.
2

2) Giải phương trình


2 x + 1 + 3 4 x 2 − 2 x + 1 = 3 + 8 x 3 + 1.
Câu II.
1) Tìm tất cả các số nguyên không âm (x, y) thoả mãn đẳng thức
( )( )
1 + x 2 1 + y 2 + 4 xy + 2(x + y )(1 + xy ) = 25.
2) Với mỗi số thực a, ta gọi phần nguyên của số a là số nguyên lớn nhất không vượt quá
a và ký hiệu là [a]. Chứng minh rằng với mọi n nguyên dương ta luôn có.
 3 7 n 2 + n + 1
 + + ... =n
1.2 2.3 n(n + 1) 
Câu III.
Cho đường tròn (O) với đường kính AB = 2R. Trên đường thẳng tiếp xúc với đương tròn
(O) tại A ta lấy điểm C sao cho góc ACB = 30 0 . Gọi H là giao điểm thứ hai của đường thăng BC
với đường tròn (O).
1) Tính độ dài đương thẳng AC, BC và khoảng cách từ A đến đương thẳng BC theo R.
2) Với mỗi điểm M trên đoạn thẳng AC, đường thẳng BM cắt đường tròn (O tại điểm N
(khác B). Chứng minh rằng bốn điểm C, M, N, H nằm trên cùng một đường tròn và
tâm đường tròn đó luôn chạy trên một đường thẳng cố định khi M thay đổi trên đoạn
thẳng AC.
9
Câu IV. Với a,b là các số thực thoả mãn đẳng thức (1 + a )(1 + b) = , hãy tìm giá trị nhỏ nhất của
4
biểu thức P = 1 + a 4 + 1 + b 4 .

Liên hệ tài liệu word môn toán: 039.373.2038 TÀI LIỆU TOÁN HỌC
Website:tailieumontoan.com

SỞ GIÁO DỤC VÀ ĐÀO TẠO KỲ THI VÀO LỚP 10 CHUYÊN KHOA HỌC TỰ NHIÊN
HÀ NỘI NĂM HỌC 2010 – 2011
Môn: TOÁN (VÒNG 2)
ĐỀ CHÍNH THỨC Thời gian làm bài: 150 phút
(Không kể thời gian giao đề)
Đề thi gồm 01 trang

Đề số 26

Câu I.

1) Giải phương trình x + 3 + 3x + 1 =4


 5x2 + 2y2 + 2xy = 26
2) Giải hệ phương trình 
(
3x + 2x + y x − y = )(
11. )
Câu II.

1) Tìm tất cả các số nguyên dương n để n2 + 391 là số chính phương.


2) Giả sử x, y, z là những số thực dương thỏa mãn điều kiện x + y + z = 1. Chứng minh rằng:

xy + z + 2x2 + 2y2
≥ 1.
1 + xy
Câu III.
Cho tam giác ABC có ba góc nhọn và M là điểm nằm trong tam giác. Kí hiệu H là hình
chiếu của M trên cạnh BC và P, Q, E, F lần lượt là hình chiếu của H trên các đường thẳng MB,
MC, AB, AC. Giả sử bốn điểm P, Q, E, F thẳng hàng.
1) Chứng minh rằng M là trực tâm của tam giác ABC.
2) Chứng minh rằng BEFC là tứ giác nội tiếp.
Câu IV. Trong dãy số gồm 2010 số thực khác 0 được sắp xếp theo thứ tự a1 , a 2 , a 3 ,..., a 2010 ta
đánh dấu tất cả các số dương và tất cả các số mà tổng của nó với một số liên tiếp liền ngay sau
nó là một số dương.
Chứng minh rằng nếu trong dãy số đã cho có ít nhất một số dương thì tổng của tất cả các số
được đánh dấu là một số dương.

Liên hệ tài liệu word môn toán: 039.373.2038 TÀI LIỆU TOÁN HỌC
Website:tailieumontoan.com

SỞ GIÁO DỤC VÀ ĐÀO TẠO KỲ THI VÀO LỚP 10 CHUYÊN KHOA HỌC TỰ NHIÊN
HÀ NỘI NĂM HỌC 2009 – 2010
Môn: TOÁN (VÒNG 1)
ĐỀ CHÍNH THỨC Thời gian làm bài: 120 phút
(Không kể thời gian giao đề)
Đề thi gồm 01 trang

Đề số 27

Câu I. 1) Giải phương trình

x2 − x + 2 = 2 x2 − x +1
2) Giải hệ phương trình
 x 2 − y 2 + xy = 1



3 x + y = y + 3
2

Câu II. 1) Tìm chữ số tận cùng của số 1313 + 6 6 + 2009 2009
2) Với a, b là những chữ số thực dương, tìm giá trị nhỏ nhất của biểu thức
a+b
P=
a (4a + 5b) + b(4b + 5a )
Câu III. Cho hình thoi ABCD. Gọi H là giao điểm của hai đường chéo AC và BD. Biết rằng bán
kính đường tròn ngoại tiếp tam giác ABC bằng a và bán kính đường tròn ngoại tiếp tam giác
ABD bằng b.
AH a
1) Chứng minh rằng =
BH b
2) Tính diện tích hình thoi ABCD theo các bán kính a, b
Câu IV. Với a, b, c là những số thực dương, chứng minh rằng
a2 b2 c2 a+b+c
+ + ≥
3a 2 + 8b 2 + 14ab 3b 2 + 8c 2 + 14bc 3c 2 + 8a 2 + 14ca 5

Liên hệ tài liệu word môn toán: 039.373.2038 TÀI LIỆU TOÁN HỌC
Website:tailieumontoan.com

SỞ GIÁO DỤC VÀ ĐÀO TẠO KỲ THI VÀO LỚP 10 CHUYÊN KHOA HỌC TỰ NHIÊN
HÀ NỘI NĂM HỌC 2009 – 2010
Môn: TOÁN (VÒNG 2)
ĐỀ CHÍNH THỨC Thời gian làm bài: 150 phút
(Không kể thời gian giao đề)
Đề thi gồm 01 trang

Đề số 28

Câu I. 1) Giải phương trình 14 x + 35 + 6 x + 1 = 84 + x 2 + 36 x + 35


1 3 2n − 1 n2
2) Chứng minh rằng + + ... + =
4 + 14 4 + 34 4 + (2n − 1) 4 4n 2 + 1
Với mọi n nguyên dương
Câu II. 1) Tìm số nguyên dương n sao cho tất cả các số
n + 1, n + 5, n + 7, n + 13, n + 17, n + 25, n + 37 đều là nguyên tố
2) Mỗi lần cho phép thay thế cặp số (a,b) thuộc tập hợp
M = {(16,2), (4,32), (6,62), (78,8)} bằng cặp số (a + c, b + d) trong đó cặp
số (c, d) cũng thuộc M.
Hỏi sau một số hữu hạn lần thay thế ta có thể nhận được tập hợp các cặp
số M 1 = {(2018,702), (844,2104), (1056,2176), (2240,912)} hay không?
Câu III. Cho đường tròn (O) và (O’) cắt nhau tại hai điểm A và B. Trên đường thẳng AB ta lấy
một điểm M bất kỳ sao cho điểm A nằm trong đoạn BM (M ≠ A) .Từ điểm M kẻ tới đường tròn
(O’) các tiếp tuyến MC và MD (C và D là các tiếp điểm, C nằm ngoài (O)). Đường thẳng AC cắt
lần thứ hai đường tròn (O) tại điểm P và đường thẳng AD cắt lần thứ hai đường tròn (O) tại Q.
Đường thẳng CD cắt PQ tại K.
1) Chứng minh rằng hai tam giác BCD và BPQ đồng dạng
2) Chứng minh rằng khi M thay đổi thì đường tròn ngoại tiếp tam giác KCP luôn đi qua điểm cố
định.
Câu IV. Giả sử x,y,z là những số thực thoả mãn điều kiện
0 ≤ x, y, z ≤ 2 và x+ y + z = 3
Tìm giá trị nhỏ nhất và lớn nhất của biểu thức :
M = x 4 + y 4 + z 4 + 12(1 − x )(1 − y )(1 − z )

Liên hệ tài liệu word môn toán: 039.373.2038 TÀI LIỆU TOÁN HỌC
Website:tailieumontoan.com

ĐẠI HỌC QUỐC GIA HÀ NỘI ĐỀ THI TUYỂN SINH LỚP 10


TRƯỜNG ĐẠI HỌC KHOA HỌC TỰ NHIÊN HỆ THPT CHUYÊN NĂM 2008

Đề số 29- MÔN : TOÁN (Vòng 1)


Thời gian làm bài: 150 phút (không kể thời gian phát đề)

Câu 1.
2 x 2 y − y 2 x =1
a) Giải hệ phương trình:  3 .
8 x − y =
3
7

x + 2 (1 − x ) .
b) Cho 0 ≤ x ≤ 1 . Tìm giá trị lớn nhất của biểu thức: y =
Câu 2.
a) Tìm tất cả các số nguyên x, y thỏa mãn: 2 x 2 + y 2 + 3 xy + 3 x + 2 y + 2 =0.
( ab − 1)( bc − 1)( ca − 1)
b) Tìm số nguyên dương a, b, c sao cho là một số nguyên.
abc
Câu 3.
a) Tìm nghiệm nguyên của phương trình: 8 x 2 y 2 + x 2 + y 2 =
10 xy .
b) Ký hiệu [ x ] là phần nguyên của số x (số nguyên lớn nhất không vượt quá x ). Chứng minh rằng với
mọi số tự nhiên n , ta luôn có:
 3 72n + 1=]
[ 3 9n + 3 9n + 1=] [ 3 72n + 7 

Câu 4. Cho tam giác ABC nội tiếp đường tròn ( O ) . Giả sử các tiếp tuyến với đường tròn ( O ) tại B và
C cắt nhau tại P nằm khác phía với A so với BC . Trên cung BC không chứa A lấy điểm K ( K ≠ B, C )
. Đường thẳng PK cắt ( O ) tại điểm thứ hai Q khác A .
a) Chứng minh rằng các đường phân giác của các góc ∠ KBQ, ∠ KCQ đi qua cùng một điểm trên đường
thẳng PQ .
b) Giả sử đường thẳng AK đi qua trung điểm M của cạnh BC . Chứng minh rằng AQ  BC .
Câu 5.
Cho phương trình: a0 x n + a1 x n −1 + a2 x n − 2 + … + an −1 x + an = 0 thỏa mãn các hệ số a0 , a1 , a2 , …, an chỉ nhận
một trong ba giá trị: 0 , hoặc 1 , hoặc −1 và a0 ≠ 0 .
Chứng minh rằng nếu x0 là nghiệm của phương trình thì x0 < 2

Liên hệ tài liệu word môn toán: 039.373.2038 TÀI LIỆU TOÁN HỌC
32
Website:tailieumontoan.com

ĐẠI HỌC QUỐC GIA HÀ NỘI ĐỀ THI TUYỂN SINH LỚP 10


TRƯỜNG ĐẠI HỌC KHOA HỌC TỰ NHIÊN HỆ THPT CHUYÊN NĂM 2007

Đề số 30- MÔN : TOÁN (Vòng 1)


Thời gian làm bài: 150 phút (không kể thời gian phát đề)

Câu 1. (3 điểm) Giải phương trình và hệ phương trình:


a) 4x2 − 1 + = x 2x2 − x + 2x + 1
 xy ( x + y ) = 2
b)  3
x + y + x + y =
3
4
Câu 2. (3 điểm)
a) Giả sử x1 , x2 là hai nghiệm của phương trình x 2 − 4 x + 1 =0. Chứng minh rằng x15 + x25 là một số
nguyên.
b) Cho a, b là các số nguyên dương thỏa mãn a + 1 và b + 2007 đều chia hết cho 6.
Chứng minh rằng 4a + a + b chia hết cho 6.
Câu 3. (3 điểm) Cho M là trung điểm của cung nhỏ AB của đường tròn tâm O (AB không phải là
đường kính). C và D là 2 điểm phân biệt, thay đổi nằm giữa A và B. Các đường thẳng MC, MD cắt
(O) tương ứng tại E, F khác M.
a) Chứng minh các điểm C, D, E, F nằm trên một đường tròn.
b) Gọi O1 và O2 lần lượt là tâm các đường tròn ngoại tiếp các tam giác ACE và BDF.
Chứng minh rằng khi C và D thay đổi trên đoạn AB thì giao điểm của hai đường thẳng AO1
và BO2 là một điểm cố định.

Câu 4. (1 điểm) Cho a, b, c là các số thực dương thỏa mãn abc = 1. Chứng minh rằng:
a b c 1
+ + ≥
( ab + a + 1) ( bc + b + 1) ( ca + c + 1) a + b + c
2 2 2

Tài liệu sưu tầm và tổng hợp bản word đầy đủ liên hệ 0393732038 TÀI LIỆU TOÁN HỌC
33
Website:tailieumontoan.com
ĐẠI HỌC QUỐC GIA HÀ NỘI ĐỀ THI TUYỂN SINH LỚP 10
TRƯỜNG ĐẠI HỌC KHOA HỌC TỰ NHIÊN HỆ THPT CHUYÊN NĂM 2007

Đề số 31- MÔN : TOÁN (Vòng 2)


Thời gian làm bài: 150 phút (không kể thời gian phát đề)

Câu 1.
 x2 + 4 y 2 = 5
a) Giải hệ phương trình:  .
 x + 2 y + 4 xy =
7
b) Cho a, b, c là các số thực khác 0 . Tìm giá trị nhỏ nhất của biểu thức:

 1  b +c
2 2
1
P= a 2  2 + 2  + .
b c  a2
Câu 2.
a) Tìm cặp số nguyên ( x, y ) thỏa mãn 5 x 2 + y 2 =17 + 2 xy .
b) Tìm tất cả các số nguyên tố p sao cho p 4 + 2 là số nguyên tố.
Câu 3.
Cho hai đường thẳng d1 và d 2 vuông góc tại O . Đường tròn ( O1 ) tiếp xúc với d1 , d 2 lần lượt tại
A, B . Đường tròn ( O2 ) tiếp xúc với d1 , d 2 lần lượt tại C , D .
a) Chứng minh rằng B là trực tâm tam giác ACD .
b) Giả sử CB cắt ( O1 ) tại E , AD cắt ( O2 ) tại F . Chứng minh rằng ACEF là hình thang cân.
Câu 4.
Trong các tứ giác có ba cạnh đều bằng a cho trước. Tìm tứ giác diện tích lớn nhất.
Câu 5 .
Cho dãy số a0 , a1 , …, an , … được xác định bởi như sau: a0 = 0 và
a=
n +1 2 an + 3 (1 + an )∀n ∈  .
1
( )
2
Chứng minh rằng: a= (2 + 3) n − 2 − 3  .
4 
n

Tài liệu sưu tầm và tổng hợp bản word đầy đủ liên hệ 0393732038 TÀI LIỆU TOÁN HỌC
34
Website:tailieumontoan.com
ĐẠI HỌC QUỐC GIA HÀ NỘI ĐỀ THI TUYỂN SINH LỚP 10
TRƯỜNG ĐẠI HỌC KHOA HỌC TỰ NHIÊN HỆ THPT CHUYÊN NĂM 2006

Đề số 32- MÔN : TOÁN (Vòng 1)


Thời gian làm bài: 150 phút (không kể thời gian phát đề)

Câu I (2,0 điểm)


x 2 + xy + x + y = 4
Giải hệ phương trình 
(x + y )(1 + xy) = 4
Câu II (2,0 điểm)
1
Với những giá trị của x thỏa mãn điều kiện x ≥ − , hãy tìm giá trị lớn nhất của biểu thức:
2
f ( x ) = 2 x 2 + 5x + 2 + 2 x + 3 − 2 x
Câu III (2,0 điểm)
Tìm số tự nhiên gồm bốn chữ số thỏa mãn đồng thời hai tính chất:
(i) Khi chia số đó cho 100 ta được số dư là 6.
(ii) Khi chia số đó cho 51 ta được số dư là 17.
Câu IV (3,0 điểm)
Cho hình vuông ABCD có cạnh AB = α . Trên các cạnh AB, BC, CD, DA lấy lần lượt các
0
điểm M, N, P, Q sao cho: MN // AC, PQ // AC và ∠AMQ = 30 .
1) Gọi A ′ là điểm đối xứng với A qua đường thẳng MQ, C ′ là điểm đối xứng với C qua
đường thẳng NP. Giả sử đường thẳng QA′ cắt đoạn thẳng NP tại E, đường thẳng PC ′ cắt
đoạn thẳng MQ tại F. chứng minh rằng năm điểm E, F, Q, D, P nằm trên cùng một đường
tròn.
2) Biết AC = 3MN, tính diện tích của hình thang MNPQ theo a.
Câu V (1,0 điểm)
4 2
Chứng minh rằng với mỗi số dương a cho trước, đa thức f ( x ) = x + ax + 2 luôn là tổng
bình phương của hai đa thức bậc hai.

Tài liệu sưu tầm và tổng hợp bản word đầy đủ liên hệ 0393732038 TÀI LIỆU TOÁN HỌC
35
Website:tailieumontoan.com
ĐẠI HỌC QUỐC GIA HÀ NỘI ĐỀ THI TUYỂN SINH LỚP 10
TRƯỜNG ĐẠI HỌC KHOA HỌC TỰ NHIÊN HỆ THPT CHUYÊN NĂM 2006

Đề số 33- MÔN : TOÁN (Vòng 2)


Thời gian làm bài: 150 phút (không kể thời gian phát đề)

Câu 1.
84 3 84
Chứng minh rằng: A = 3 1 + + 1− là một số nguyên.
9 9
Câu 2.
 x2 − y 2 = 4x − 2 y − 3
Giải hệ phương trình:  2 .
x + y =
2
5
Câu 3.
a) Tìm nghiệm nguyên của phương trình: 8 x 2 y 2 + x 2 + y 2 =
10 xy .
b) Ký hiệu [ x ] là phần nguyên của số x (số nguyên lớn nhất không vượt quá x ). Chứng minh
rằng với mọi số tự nhiên n , ta luôn có:

 3 72n + 1=] [ 3 9n + 3 9n + 1=] [ 3 72n + 7 



Câu 4.
Cho tam giác ABC nội tiếp đường tròn ( O ) và I là điểm nằm trong tam giác ABC . Các đường
thẳng AI , BI , CI cắt ( O ) lần lượt tại A′, B ′, C ′ (khác A, B, C ). Dây cung B ′C ′ cắt các cạnh
AB, AC tương ứng tại các điểm M , N . Dây cung C ′A′ cắt các cạnh AB, BC tương ứng tại các
điểm Q, P . Dây cung A′B ′ cắt các cạnh BC , CA tương ứng tại các điểm F , E .
=
a) Giả sử AM AN = , BP BQ= , CE CF xảy ra đồng thời. Chứng minh rằng I là tâm đường tròn
nội tiếp tam giác ABC .
b) Giả sử AM= AN = BP = BQ = CE = CF . Chứng minh rằng sáu điểm M , N , P, Q, E , F cùng
thuộc một đường tròn.
Câu 5.
Chứng minh rằng đa giác lồi 2n cạnh ( n ∈ , n ≥ 2 ) luôn có ít nhất n đường chéo không không
song song với bất kỳ cạnh nào của đa giác đó.

Tài liệu sưu tầm và tổng hợp bản word đầy đủ liên hệ 0393732038 TÀI LIỆU TOÁN HỌC
36
Website:tailieumontoan.com
ĐẠI HỌC QUỐC GIA HÀ NỘI ĐỀ THI TUYỂN SINH LỚP 10
TRƯỜNG ĐẠI HỌC KHOA HỌC TỰ NHIÊN HỆ THPT CHUYÊN NĂM 2005

Đề số 34- MÔN : TOÁN (Vòng 1)


Thời gian làm bài: 150 phút (không kể thời gian phát đề)

Câu 1: Giải hệ phương trình


x + y + xy = 3
 2
x + y = 2
2

Câu 2: Giải phương trình


x + 4 x + 3 + 2 3 − 2 x = 11
Câu 3: Tìm nghiệm nguyên của phương trình
x 2 + 17y 2 + 34xy + 51(x + y ) = 1740
Câu 4:
Cho hai đường tròn (O), (O′) nằm ngoài nhau có tâm tương ứng là O và O′ . Một tiếp
tuyến chung ngoài của hai đường tròn tiếp xúc với (O) tại A và (O′) tại B. Một tiếp tuyến chung
trong của hai đường tròn cắt AB tại I, tiếp xúc với (O) tại C và (O′) tại D. Biết rằng C nằm giữa I
và D.
1) Hai đường thẳng OC, O′B cắt nhau tại M. Chứng minh rằng OM > O′M .
2) Ký hiệu (S) là đường tròn đi qua A, C, B và (S ′) là đường tròn đi qua A, D, B. Đường
thẳng CD cắt (S) tại E khác C và cắt (S ′) tại F khác D. Chứng minh rằng AF vuông góc với BE
Câu 5:
Giả sử x, y, z là các số dương thay đổi và thỏa mãn điều kiện:
xy 2 z 2 + x 2 z + y = 3z 2 .
z4
Hãy tìm giá trị lớn nhất của biểu thức: P =
(
1 + z4 x4 + y4 )

Tài liệu sưu tầm và tổng hợp bản word đầy đủ liên hệ 0393732038 TÀI LIỆU TOÁN HỌC
37
Website:tailieumontoan.com
ĐẠI HỌC QUỐC GIA HÀ NỘI ĐỀ THI TUYỂN SINH LỚP 10
TRƯỜNG ĐẠI HỌC KHOA HỌC TỰ NHIÊN HỆ THPT CHUYÊN NĂM 2005

Đề số 35- MÔN : TOÁN (Vòng 2)


Thời gian làm bài: 150 phút (không kể thời gian phát đề)

Bài 1: Giải phương trình 2 − x + 2 + x + 4 − x2 =


2

 x 3 + y 3 − xy 2 =
1
Bài 2: Giải hệ phương trình  4
4 x + y = 4 x + y
4

Bài 3: Giả sử x, y là số không âm thỏa mãn x 2 + y 2 =


1
a) Chứng minh rằng 1 ≤ x + y ≤ 2
b)Tìm giá trị lớn nhất và nhỏ nhất của biểu thức P = 1 + 2 x + 1 + 2 y
Bài 4: Cho hình vuông ABCD và điểm P nằm trong tam giác ABC
 = 135o . Chứng minh rằng: 2PB 2 + PC 2 =
a) Giả sử BPC PA2
b) Các đường thẳng AP và CP cắt các cạnh BC và BA tại M, N. Gọi Q là điểm đối xứng với B
qua trung điểm của đoạn MN. Chứng minh rằng khi P thay đổi trong tam giác ABC, đường thẳng
PQ luôn đi qua D.
Bài 5:
a) Cho đa giác đều (H) có 14 đỉnh. Chứng minh rằng trong 6 đỉnh bất kỳ của (H) luôn có 4 đỉnh là
các đỉnh của 1 hình thang
m
b) Có bao nhiêu phân số tối giản > 1 (m, n là các số nguyên dương ) thỏa mãn m.n = 13860.
n

Tài liệu sưu tầm và tổng hợp bản word đầy đủ liên hệ 0393732038 TÀI LIỆU TOÁN HỌC
38
Website:tailieumontoan.com
ĐẠI HỌC QUỐC GIA HÀ NỘI ĐỀ THI TUYỂN SINH LỚP 10
TRƯỜNG ĐẠI HỌC KHOA HỌC TỰ NHIÊN HỆ THPT CHUYÊN NĂM 2004

Đề số 36- MÔN : TOÁN (Vòng 1)


Thời gian làm bài: 150 phút (không kể thời gian phát đề)

Câu 1:
1) Giải phương trình
x + 1 + x − 1 = 1 + x2 − 1
2) Tìm nghiệm nguyên của hệ
2y 2 − x 2 − xy + 2y − 2x = 7
 3
x + y + x − y =8
3

Câu 2: Cho các số thực dương a và b thỏa mãn


a100 + b100 = a101 + b101 = a102 + b102
Hãy tính giá trị của biểu thức P = a +b 2004 2004

Câu 3: Cho tam giác ABC có AB = 3 cm, BC = 4 cm, CA = 5 cm. Đường cao, đường phân giác,
đường trung tuyến của tam giác kẻ từ đỉnh B chia tam giác thành bốn phần. Hãy tính diện tích mỗi
phần.
Câu 4: Cho tứ giác ABCD nội tiếp trong đường tròn có hai đường chéo AC và BD vuông góc với
nhau tại H (H không trùng với tâm của đường tròn). Gọi M và N lần lượt là chân các đường vuông
góc hạ từ H xuống các đường thẳng AB và BC; P và Q lần lượt là giao điểm của đường thẳng MH
và NH với các đường thẳng CD và DA. Chứng minh rằng đường thẳng PQ song song với đường
thẳng AC và bốn điểm M, N, P, Q nằm trên cùng một đường tròn.
Câu 5: Tìm giá trị nhỏ nhất của biểu thức
1  x 10 y 10  1 16
(
Q =  2 + 2  + x + y 16 − 1 + x 2 y 2
2 y
) ( ) 2

x  4

Tài liệu sưu tầm và tổng hợp bản word đầy đủ liên hệ 0393732038 TÀI LIỆU TOÁN HỌC
39
Website:tailieumontoan.com
ĐẠI HỌC QUỐC GIA HÀ NỘI ĐỀ THI TUYỂN SINH LỚP 10
TRƯỜNG ĐẠI HỌC KHOA HỌC TỰ NHIÊN HỆ THPT CHUYÊN NĂM 2004

Đề số 37- MÔN : TOÁN (Vòng 2)


Thời gian làm bài: 150 phút (không kể thời gian phát đề)

Bài 1. Giải phương trình x + 3 + x −1 =2

( ) (
Bài 2. Giải hệ phương trình ( x + y ) x 2 + y 2 = 15( x − y ) x 2 − y 2 = 3 )

Bài 3. Tìm giá trị nhỏ nhất của biểu thức P =


(x 3
) (
+ y3 − x2 + y 2 )
( x − 1)( y − 1)

trong đó, x, y là những số thực lớn hơn 1 .


Bài 4. Cho hình vuông ABCD và điểm M nằm trong hình vuông.

1. Tìm tất cả các vị trí của điểm M sao cho MAB 
= MBC 
= MCD 
= MDA
2. Xét điểm M nằm trên đường chéo AC. Gọi N là chân đường vuông góc hạ từ điểm M xuống
OB
A B và O là trung điểm của đoạn AM. Chứng minh rầng tỷ số có giá trị không đổi khi M di
CN
chuyển trên đường chéo A C.

3. Với giả thiết M nằm trên đường chéo AC, xét các đường tròn ( S1 ) và ( S 2 ) có đường kính
tương ứng là AM và CN. Hai tiếp tuyến chung của ( S1 ) và ( S 2 ) tiếp xúc với ( S 2 ) tại P và Q .
Chứng minh rằng đường thẳng PQ tiếp xúc với ( S1 )

Bài 5. Với số thực a , ta định nghĩa phần nguyên của số a là số nguyên lớn nhất không vượt quá a
và ký hiệu là [a]. Dãy các số x0 , x1 , x2 , …, xn , …. được xác định bởi công thức

 n + 1  n 
=xn  − 
 2   2

Hỏi trong 200 số { x0 , x1 , …, x199 } có bao nhiêu số khác 0? (Cho biết 1, 41 < 2 < 1, 42 ).

Tài liệu sưu tầm và tổng hợp bản word đầy đủ liên hệ 0393732038 TÀI LIỆU TOÁN HỌC
40
Website:tailieumontoan.com
ĐẠI HỌC QUỐC GIA HÀ NỘI ĐỀ THI TUYỂN SINH LỚP 10
TRƯỜNG ĐẠI HỌC KHOA HỌC TỰ NHIÊN HỆ THPT CHUYÊN NĂM 2003

Đề số 38- MÔN : TOÁN (Vòng 1)


Thời gian làm bài: 150 phút (không kể thời gian phát đề)

Câu 1 (2 điểm): Giải phương trình:


( x + 5 − x + 2)(1 + x 2 + 7 x + 10) =3
Câu 2 (2 điểm): Giải hệ phương trình:
2x 3 + 3x 2 y = 5
 3
y + 6xy 2 = 7
Câu 3 (2 điểm): Tìm các số nguyên x, y thỏa mãn đẳng thức:
2y 2 x + x + y + 1 = x 2 + 2y 2 + xy
Câu 4 (3 điểm) Cho nửa đường tròn (O) đường kính AB = 2R (R là một độ dài cho trước). M, N là
hai điểm trên nửa đường tròn (O) sao cho M thuộc cung AN và tổng các khoảng cách từ A, B đến
đường thẳng MN bằng 3.
1. Tính độ dài đoạn MN theo R.
2. Gọi giao điểm của hai dây AN và BM là I, giao điểm của các đường thẳng AM và BN là K.
Chứng minh rằng bốn điểm M, N, I, K cùng nằm trên một đường tròn. Tính bán kính của đường
tròn đó theo R.
3. Tìm giá trị lớn nhất của diện tích tam giác KAB theo R khi M, N thay đổi nhưng vẫn thỏa
mãn giả thiết của bài toán.
Câu 5 (1 điểm): x, y, z là các số thực thỏa mãn điều kiện:
x + y + z + xy + yz + zx = 6
2 2 2
Chứng minh rằng: x + y + z ≥ 3

Tài liệu sưu tầm và tổng hợp bản word đầy đủ liên hệ 0393732038 TÀI LIỆU TOÁN HỌC
41
Website:tailieumontoan.com
ĐẠI HỌC QUỐC GIA HÀ NỘI ĐỀ THI TUYỂN SINH LỚP 10
TRƯỜNG ĐẠI HỌC KHOA HỌC TỰ NHIÊN HỆ THPT CHUYÊN NĂM 2003

Đề số 39- MÔN : TOÁN (Vòng 2)


Thời gian làm bài: 150 phút (không kể thời gian phát đề)

Bài 1. Cho phương trình x 4 + 2mx 2 + 4 =0


Tìm giá trị của tham số m để phương trình có 4 nghiệm phân biệt x1 , x2 , x3 , x4 thoả mãn

x14 + x24 + x34 + x44 =


32

2 x 2 + xy − y 2 − 5 x + y + 2 =
0
Bài 2. Giải hệ phương trình  2
x + y + x + y − 4 =
2
0

Bài 3. Tìm các số nguyên x, y thoả mãn đẳng thức x 2 + xy + y 2 =


x2 y 2

Bài 4. Cho đường tròn tâm O nội tiếp tam giác ABC tiếp xúc với các cạnh BC, CA, AB tương ứng
 của tam giác ABC tiêp xúc với
tại các điểm D, E, F. Đường tròn tâm O′ bàng tiếp trong góc BAC
cạnh BC và phần kéo dài của các cạnh AB, AC tương ứng tại các điểm $P, M, N$.
1. Chứng minh rằng: BP = CD .
2. Trên đường thẳng MN ta lấy các điểm I và K sao cho CK / / AB, BI / / AC . Chứng minh rằng
các tứ giác BICE và BKCF là các hình binh hành.
3. Gọi ( S ) là đường tròn đi qua ba điểm I, K, P. Chứng minh rằng ( S ) tiếp xúc với các đường
thẳng BC, BI, CK.

Bài 5. Số thực x thay đổi và thoả mãn điều kiện x 2 + (3 − x) 2 5 . Tìm giá trị nhỏ nhất của biểu
thức p = x 4 + (3 − x) 4 + 6 x 2 (3 − x) 2

Tài liệu sưu tầm và tổng hợp bản word đầy đủ liên hệ 0393732038 TÀI LIỆU TOÁN HỌC
42
Website:tailieumontoan.com
ĐẠI HỌC QUỐC GIA HÀ NỘI ĐỀ THI TUYỂN SINH LỚP 10
TRƯỜNG ĐẠI HỌC KHOA HỌC TỰ NHIÊN HỆ THPT CHUYÊN NĂM 2002

Đề số 40- MÔN : TOÁN (Vòng 1)


Thời gian làm bài: 150 phút (không kể thời gian phát đề)
Bài 1.

1. Giải phương trình: 8+ x + 5− x =


5

( x + 1)( y + 1) = 8
2. Giải hệ phương trình 
 x( x + 1) + y ( y + 1) + xy =
17

Bài 2. Cho a, b, c là dộ dài ba cạnh của một tam giác. Chứng minh rằng phương trình
x 2 + (a + b + c) x + ab + bc + ca =
0 vô nghiệm.

Bài 3. Tìm tất cả các số nguyên n sao cho n 2 + 2002 là một số chính phương.
1 1 1
Bài 4. Tìm giá trị nhỏ nhất của biểu thức P = + +
1 + xy 1 + yz 1 + zx

trong đó x, y, z là các số dương thay đổi thoả mãn điều kiện x 2 + y 2 + z 23 .

Bài 5. Cho hình vuông ABCD, M là điểm thay đổi trên cạnh BC ( M không trùng với B ) và N là

điểm thay đổi trên cạnh CD( N không trùng với D ) sao cho: MAN
= MAB + NAD

1. BD cắt AN và AM tương ứng tại P và Q . Chứng minh rằng năm điểm P, Q, M, C, N cùng nằm
trên một đường tròn.
2. Chứng minh rằng đường thẳng MN luôn tiếp xúc với một đường tròn cố định khi M và N thay
đổi.
3. Ký hiệu diện tích của tam giác APQ là S1 là diện tích của tứ giác PQMN là S 2 . Chứng minh rằng
S1
tỉ số không đổi khi M và N thay đổi.
S2

Tài liệu sưu tầm và tổng hợp bản word đầy đủ liên hệ 0393732038 TÀI LIỆU TOÁN HỌC
43
Website:tailieumontoan.com
ĐẠI HỌC QUỐC GIA HÀ NỘI ĐỀ THI TUYỂN SINH LỚP 10
TRƯỜNG ĐẠI HỌC KHOA HỌC TỰ NHIÊN HỆ THPT CHUYÊN NĂM 2002

Đề số 41- MÔN : TOÁN (Vòng 2)


Thời gian làm bài: 150 phút (không kể thời gian phát đề)

Bài 1.

1. Giải phương trình: x 2 − 3x + 2 + x + 3 = x − 2 + x2 + 2 x − 3

2. Tìm nghiệm nguyên của phương trình x + xy + y =


9

 x 2 + y 2 + xy =
1
Bài 2. Giải hệ phương trình  3
 x + y =x + 3 y
3

Bài 3. Cho mười số nguyên dương 1, 2, …,10 . Sắp xếp mười số đó một cách tuỳ ý thành một hàng.
Cộng mỗi số với số thứ tự của nó trong hàng, ta được mười tổng. Chứng minh rằng trong mười tổng
đó tồn tại ít nhất hai tổng có chữ số tận cùng giống nhau.
4a 9b 16c
Bài 4. Tìm giá trị nhỏ nhất của biểu thức P = + + trong đó a, b, c là độ
b+c−a a+c−b a+b−c
dài ba cạnh của một tam giác.
Bài 5. Đường tròn (C ) tâm I nội tiếp tam giác ABC tiếp xúc với các cạnh BC, CA, AB tương ứng
tại các điểm A′ , B′ , C ′ .

1. Gọi các giao điểm của đường tròn (C ) với các đoạn IA, IB, IC lần lượt là M, N, P. Chứng minh
rằng các đường thẳng A′ M , B′ N , C ′ P đồng quy.

2. Kéo dài đoạn AI cắt đường tròn ngoại tiếp tam giác ABC tại D (khác A ). Chứng minh rằng
IB.IC
= 2r trong đó r là bán kính đường tròn (C ) .
ID

Tài liệu sưu tầm và tổng hợp bản word đầy đủ liên hệ 0393732038 TÀI LIỆU TOÁN HỌC
44
Website:tailieumontoan.com
ĐẠI HỌC QUỐC GIA HÀ NỘI ĐỀ THI TUYỂN SINH LỚP 10
TRƯỜNG ĐẠI HỌC KHOA HỌC TỰ NHIÊN HỆ THPT CHUYÊN NĂM 2001

Đề số 42-- MÔN : TOÁN (Vòng 1)


Thời gian làm bài: 150 phút (không kể thời gian phát đề)

Bài 1. Tìm các giá trị nguyên x, y thoả mãn đẳng thức ( y + 2) x 2 + 1 =y 2

Bài 2.

1. Giải phương trình x(3 x + 1) − x( x − 1) =


2 x2

 x 2 + xy + 2 = 3 x + y
2. Giải hệ phương trình  2
x + y =
2
2

Bài 3. Cho nửa vòng tròn đường kính AB = 2a . Trên đoạn AB lấy điểm M . Trong nửa mặt phẳng
bờ AB chứa nửa vòng tròn, ta kẻ hai tia Mx và My sao cho 
= BMx
AMx = 30° . Tia Mx cắt nửa
vòng tròn ở E , tia My cất nửa vòng tròn ở F . Kẻ EE ′ , FF ′ vuông góc xuống AB.

a
1. Cho AM = , tính diện tích hình thang vuông EE ' F ' F theo a .
2
2. Khi điểm M di động trên AB, chứng minh rằng đường thẳng EF luôn tiếp xúc với một vòng tròn
cố định.
Bài 4. Giả sử x, y, z là các số thực khác không thoả mãn hệ đẳng thức:

 1 1 1 1 1 1
x  +  + y  +  + z  +  =
−2
  y z z x x y
 x3 + y 3 + z 3 =
 1

1 1 1
Hãy tính giá trị của biểu thức P = + +
x y z

Bài 5. Với x, y, z là những số thực dương, hãy tìm giá trị lớn nhất của biểu thức
xyz
M =
( x + y )( y + z )( z + x)

Tài liệu sưu tầm và tổng hợp bản word đầy đủ liên hệ 0393732038 TÀI LIỆU TOÁN HỌC
45
Website:tailieumontoan.com
ĐẠI HỌC QUỐC GIA HÀ NỘI ĐỀ THI TUYỂN SINH LỚP 10
TRƯỜNG ĐẠI HỌC KHOA HỌC TỰ NHIÊN HỆ THPT CHUYÊN NĂM 2001

Đề số 43- MÔN : TOÁN (Vòng 2)


Thời gian làm bài: 150 phút (không kể thời gian phát đề)

Bài 1.

1. Cho f ( x) = ax 2 + bx + c có tính chất f ( x) nhận giá trị nguyên khi x là số nguyên.


Hỏi các hệ số a, b, c có nhất thiết phải là các số nguyên hay không? Tại sao?
2. Tìm các số nguyên không âm x, y thoả mãn đẳng thức: x 2 = y 2 + y + 1
Bài 2. Giải phương trình 4 x + 1 = x 2 − 5 x + 14

ax + by = 3
ax 2 + by 2 =
 5
Bài 3. Cho các số thực a, b, x, y thoả mãn hệ  3
ax + by =
3
9
ax + by =
4 4
17

Hãy tính giá trị của biểu thức A =


ax5 + by 5 ; B =+
ax 2001 by 2001
Bài 4. Cho đoạn thẳng AB có trung điểm là O . Gọi d1 , d 2 là các đường thẳng vuông góc với AB
tương ứng tại A và B . Một góc vuông đỉnh O có một cạnh cắt d1 ở M , còn cạnh kia cắt d 2 ở N .
Kẻ OH vuông góc xuống MN. Vòng tròn ngoại tiếp tam giác MHB cắt d1 ở điểm thứ hai E khác
M, MB cắt NA ở I , đường thẳng HI cắt EB ở K . Chứng minh rằng K nằm trên một vòng tròn cố
định khi góc vuông quay xung quanh đỉnh O .
Bài 5. Cho 2001 đồng tiền, mổi đồng tiền được sơn một mặt bằng màu đỏ và mặt kia bằng màu
xanh. Xếp 2001 đồng tiền đó theo một vòng tròn sao cho tất cả các đồng tiền đều có mặt xanh ngửa
lên phía trên. Cho phép mổi lần đổi mặt đồng thời 5 đồng tiền liên tiếp cạnh nhau. Hỏi với cách làm
như thế, sau một số hữu hạn lần ta có thể làm cho tất cả các đồng tiền đều có mặt đổ ngửa lên phía
trên được hay không? Tại sao?

Tài liệu sưu tầm và tổng hợp bản word đầy đủ liên hệ 0393732038 TÀI LIỆU TOÁN HỌC
46
Website:tailieumontoan.com
ĐẠI HỌC QUỐC GIA HÀ NỘI ĐỀ THI TUYỂN SINH LỚP 10
TRƯỜNG ĐẠI HỌC KHOA HỌC TỰ NHIÊN HỆ THPT CHUYÊN NĂM 2000

Đề số 44- MÔN : TOÁN (Vòng 1)


Thời gian làm bài: 150 phút (không kể thời gian phát đề)
Bài 1.
1 1 1
1. Tính S = + + +
1.2 2.3 1999.2000

 2 1 x
x + y2 + y = 3

2. Giải hệ phương trình 
x + 1 + x =3
 y y

Bài 2.

1. Giải phương trình x − 1 + x3 + x 2 + x + 1 = 1 + x 4 − 1

 11 
2. Tìm tất cả các giá trị của a ( a là số thực) để phương trình 2 x 2 −  4a +  x + 4a 2 + 7 =0
 2

có ít nhất một nghiệm nguyên.


Bài 3. Cho đường tròn tâm O nội tiếp trong hình thang ABCD( AB / / CD) , tiếp xúc với cạnh AB tại
E và với cạnh CD tại F (như hình vẽ)
BE DF
1. Chứng minh rằng =
AE CF
2. Cho biết AB =a, CB =b, (a < b), BE =2 AE . Tính diện tích hình thang ABCD.

4x2 y 2 x2 y 2
Bài 4. Cho x, y là hai số thực bất kỳ khác không. Chứng minh rằng + + 3
(x )
2
2
+ y2 y 2 x2

Dấu đẳng thức xảy ra khi nào?

Tài liệu sưu tầm và tổng hợp bản word đầy đủ liên hệ 0393732038 TÀI LIỆU TOÁN HỌC
47
Website:tailieumontoan.com
ĐẠI HỌC QUỐC GIA HÀ NỘI ĐỀ THI TUYỂN SINH LỚP 10
TRƯỜNG ĐẠI HỌC KHOA HỌC TỰ NHIÊN HỆ THPT CHUYÊN NĂM 2000

Đề số 45- MÔN : TOÁN (Vòng 2)


Thời gian làm bài: 150 phút (không kể thời gian phát đề)

Bài 1.

1. Tìm tất cả các cặp số nguyên ( x, y ) thoả mãn đẳng thức y ( x − 1) =x 2 + 2

2. Cho cặp số ( x, y ) thoả mãn các điều kiện −1x + y1, − 1xy + x + y1

Chứng minh rằng | x | 2,| y | 2

Bài 2.

1 1 5
1. Giải phương trình + x − =x + 2x −
x x x

2. Cho f ( x) = ax 2 + bx + c có tính chất f (1), f (4) và f (9) là các số hữu tỷ. Chứng minh rằng khi
đó a, b, c là các số hữu tỷ.
Bài 3.
1. Cho tứ giác lồi ABCD. Chứng minh rằng nếu các góc B và D của tứ giác là vuông hoặc tù thì
ACBD

2. Cho đoạn thẳng AC cố định và điểm B di động. Hãy tìm tập hợp tất cả các điểm B để tam giác
 . là góc bé nhất của tam giác ABC.
$A B C$ là tam giác không tù và góc BAC
Bài 4. Trên mặt phẳng cho 6 điểm sao cho không có ba điểm nào thẳng hàng và khoảng cách giữa
các cặp điểm là các số khác nhau. Ta nối mổi cặp điểm bởi một đoạn thẳng. Chứng minh rằng trong
các đoạn thẳng thu được có một đoạn thẳng là cạnh bé nhất của một tam giác có 3 đỉnh là 3 trong 6
điểm đã cho đồng thời là cạnh lớn nhất của một tam giác khác cũng có 3 đỉnh là 3 trong 6 điểm đã
cho.

Tài liệu sưu tầm và tổng hợp bản word đầy đủ liên hệ 0393732038 TÀI LIỆU TOÁN HỌC
48
Website:tailieumontoan.com
ĐẠI HỌC QUỐC GIA HÀ NỘI ĐỀ THI TUYỂN SINH LỚP 10
TRƯỜNG ĐẠI HỌC KHOA HỌC TỰ NHIÊN HỆ THPT CHUYÊN NĂM 1999

Đề số 46- MÔN : TOÁN (Vòng 1)


Thời gian làm bài: 150 phút (không kể thời gian phát đề)

a + b + c =0
Bài 1. Cho các số a, b, c thoả mãn điều kiện  2
a + b + c =
2 2
14

Hãy tính giá trị của biểu thức: P =1 + a 4 + b 4 + c 4


Bài 2.

1. Giải phương trình x + 3 − 7 − x= 2x − 8

 1 1 9
x + y + x + y =
 2
2. Giải hệ phương trình 
 xy + 1 = 5
 xy 2

Bài 3. Tìm tất cả các số nguyên dương n sao cho: n 2 + 9n − 2 chia hết ho n + 11 .
Bài 4. Cho vòng tròn () và điểm I ở trong vòng tròn. Dựng qua I hai lây cung bất kỳ MIN và E
IF. Gọi M ', N ', E ', F ' là các trung điểm của IM, IN, IE, IF

1. Chứng minh rằng tứ giác M ' E ' N ' F ' là tứ giác nội tiếp.
2. Giả sử I thay đổi, các dây cung MIN, EIF thay đổi. Chứng minh rằng vòng tròn ngoại tiếp tứ
giác M ' E ' N ' F ' có bán kính không đổi.
3. Giả sử I cố định, các dây cung MIN, EIF thay đổi nhưng luôn luôn vuông góc với nhau. Tìm vị
trí của các dây cung MIN và EIF sao cho tứ giác M ' E ' N ' F ' có diện tích lớn nhất.
Bài 5. Các số dương x và y thay đổi thoả mãn điều kiện: x + y =
1 . Hãy tìm giá trị nhỏ nhất của
 2 1  2 1 
biểu thức: P =+
x  y + 2 
 y2   x 

Các thí sinh chuyên Sinh không phải làm bài 5

Tài liệu sưu tầm và tổng hợp bản word đầy đủ liên hệ 0393732038 TÀI LIỆU TOÁN HỌC
49
Website:tailieumontoan.com
ĐẠI HỌC QUỐC GIA HÀ NỘI ĐỀ THI TUYỂN SINH LỚP 10
TRƯỜNG ĐẠI HỌC KHOA HỌC TỰ NHIÊN HỆ THPT CHUYÊN NĂM 1999

Đề số 47- MÔN : TOÁN (Vòng 2)


Thời gian làm bài: 150 phút (không kể thời gian phát đề)

x+7
Bài 1. Giải phương trình + 8= 2 x 2 + 2 x − 1
x +1

3k 2 + 3k + 1
Bài 2. Các số a1 , a2 , … được xác định bởi công thức ak = với mọi k1
( )
3
k2 + k

Hãy tính giá trị của tổng: 1 + a1 + a2 +  + a9 .

Bài 3. Chứng minh rằng tồn tại một số chia hết cho 1999 và tổng các chữ số của số đó bằng1999.
Bài 4. Cho vòng tròn tâm O bán kính R . Giả sử A và B là hai điểm cố định trên vòng tròn với
AB = R 3 .

1. Giả sử M là một điểm thay đổi trên cung lớn AB của đường tròn. Vòng tròn nội tiếp ∆MAB tiếp
xúc với MA tại E và tiếp xúc với MB tại F . Chứng minh rằng đường thẳng EF luôn tiếp xúc với
một đường tròn cố định khi M thay đổi.
2. Tìm tập hợp tất cả các điểm P sao cho đường thẳng ∆ vuông góc với OP tại P cắt đoạn thẳng
AB.
Bài 5. Cho hình tròn (C ) bán kính bằng 1 . Giả sử A1 , A2 , …, A8 là 8 điểm bất kỳ nằm tròn hình tròn
(kể cả biên). Chứng minh rằng trong các điểm đã cho luôn tồn tại hai điểm ma khoảng cách giữa
chúng nhỏ hơn 1.

Tài liệu sưu tầm và tổng hợp bản word đầy đủ liên hệ 0393732038 TÀI LIỆU TOÁN HỌC
50
Website:tailieumontoan.com
ĐẠI HỌC QUỐC GIA HÀ NỘI ĐỀ THI TUYỂN SINH LỚP 10
TRƯỜNG ĐẠI HỌC KHOA HỌC TỰ NHIÊN HỆ THPT CHUYÊN NĂM 1998

Đề số 48- MÔN : TOÁN (Vòng 1)


Thời gian làm bài: 150 phút (không kể thời gian phát đề)

Bài 1.

1. Giải phương trình 2 − x2 + x2 + 8 =4

 x 2 + xy + y 2 = 7
2. Giải hệ phương trình  4
x + x y + y =
2 2 4
21

a 3 − 3ab 2 =
19
Bài 2. Các số a, b thoả mãn điều kiện:  3
b − 3a b =
2
98

Hãy tính giá trị của biểu thức sau: =


P a 2 + b2 .

Bài 3. Cho các số a, b, c ∈ [0,1] . Chứng minh rằng a + b 2 + c3 − ab − bc − ca1

Bài 4. Cho đường tròn (ε ) bán kính R . A và B là hai điểm cố định trên đường tròn, ( AB < 2 R) .
Giả sử M là một điểm thay đổi trên cung lớn AB của đường tròn.
1. Kẻ từ B đường thẳng vuông góc với AM, đường thẳng này cắt AM tại I và cắt đường tròn (ε )
tại N . Gọi J là trung điểm của MN. Chứng minh rằng khi M thay đổi trên đường tròn thì mổi
điểm I, J đều nằm trên một đường tròn cố định.
2. Xác định vị trí của điểm M để chu vi của ∆AMB là lớn nhất.
Bài 5.
1. Tìm tất cả các số nguyên dương n sao cho mỗi số n + 26 và n − 11 đều là lập phương của một
số nguyên dương.

2. Cho các số x, y, z thay đổi thoả mãn điều kiện: x 2 + y 2 + z 2 = 1 . Hãy tìm giá trị lớn nhất của biểu
1
thức: P = xy + yz + zx +  x 2 ( y − z ) 2 + y 2 ( z − x) 2 + z 2 ( x − y ) 2 
2

Tài liệu sưu tầm và tổng hợp bản word đầy đủ liên hệ 0393732038 TÀI LIỆU TOÁN HỌC
51
Website:tailieumontoan.com
ĐẠI HỌC QUỐC GIA HÀ NỘI ĐỀ THI TUYỂN SINH LỚP 10
TRƯỜNG ĐẠI HỌC KHOA HỌC TỰ NHIÊN HỆ THPT CHUYÊN NĂM 1998

Đề số 49- MÔN : TOÁN (Vòng 2)


Thời gian làm bài: 150 phút (không kể thời gian phát đề)

Bài 1.
 x + x 2 + x3 + x 4 =y + y 2 + y 3 + y 4
1. Giải hệ phương trình  2
x + y =
2
1
2. Với những giá trị nào của a thì phương trình sau đây có nghiệm
1 − x + 1 + x =| 1 − a | + | 1 + a |
Bài 2. Tìm nghiệm nguyên của phương trình 19 x 3 − 98 y 2 = 1998
Bài 3.
1. Cho a, b, c là các số thỏa mãn hai điều kiện sau:
i) 0 < a < b .
a+b+c
ii) Phương trình ax 2 + bx + c = 0 vô nghiệm. Chứng minh rằng >3
b−a
2. Cho x, y, z > 0 . Hãy tìm giá trị nhỏ nhất của biểu thức
x2 y2 z2
P= 2 + +
x + 2 yz y 2 + 2 zx z 2 + 2 xy
Bài 4. Cho bảng ô vuông kích thước 1998 × 2000 (bảng gồm 1998 hàng và 2000 cột)
Ký hiệu (m, n) là ô vuông nằm ở giao của hàng thứ m (tính từ trên xuống dưới)và cột thứ n (tính
từ trái qua phải).
Cho các số nguyên p, q với 1p1993 và 1q1995 ;
Tồ màu các ô vuông con của bảng theo quy tắc: Lần thứ nhất tô màu năm ô:
( p, q );( p + 1, q + 1);( p + 2, q + 2);( p + 3, q + 3);( p + 4, q + 4) . Lần thứ hai trở đi, mổi lần tô năm ô
chưa có màu nằm liên tiếp trong cùng một hàng hoặc cùng một cột.
Hỏi bằng cách đó ta có thể tô màu hết tất cả các ô vuông con của bảng hay không? Vì sao?
Bài 5. Cho tam giác đều ABC.
Trong ∆ABC , vẽ ba vòng tròn ε1 , ε 2 , ε 3 có bán kính bằng nhau, tiếp xúc ngoài lâ̄n nhau và mô̄i
vòng tròn đều tiếp xúc với hai cạnh của tam giác.
Gọi ε là vòng tròn tiếp xúc ngoài với cả ba vòng tròn ε1 , ε 2 , ε 3 . Biết bán kính của vòng tròn ε là r
, hãy tính độ dài cạnh của ∆ABC .

Tài liệu sưu tầm và tổng hợp bản word đầy đủ liên hệ 0393732038 TÀI LIỆU TOÁN HỌC
52
Website:tailieumontoan.com
ĐẠI HỌC QUỐC GIA HÀ NỘI ĐỀ THI TUYỂN SINH LỚP 10
TRƯỜNG ĐẠI HỌC KHOA HỌC TỰ NHIÊN HỆ THPT CHUYÊN NĂM 1997

Đề số 50- MÔN : TOÁN (Vòng 1)


Thời gian làm bài: 150 phút (không kể thời gian phát đề)

10 + 6 3 ( 3 − 1)
(x )
3
1997
Bài 1. Cho x = . Tính P = 3
− 4x + 1
6+2 5 − 5

Bài 2. Giải phương trình x+3 + x+8 =5 x

2 xy = x + y + 1

Bài 3. Giải hệ phương trình 2 yz = y + z + 7
2 xz = z + x + 2

Bài 4. Tìm tất cả các số tự nhiên n để 2n + 15 là số chính phương.

Bài 5. Cho tam giác đều ABC cạnh l . Bên trong tam giác ta đặt 2 đường tròn (O, R) và ( O ', R ')
tiếp xúc ngoài với nhau, sao cho một trong hai đường tròn tiếp xúc với các cạnh BC và BA, đường
tròn kia tiếp xúc với các cạnh BC và CA.

3 −1
1. Chứng minh rằng R + R ' .
2
2. Các bán kính R và R ' bằng bao nhiêu để tổng diện tích các hình tròn (O, R) và O ', R ' nhỏ nhất
và tính giá trị nhỏ nhất đó.

Tài liệu sưu tầm và tổng hợp bản word đầy đủ liên hệ 0393732038 TÀI LIỆU TOÁN HỌC
53
Website:tailieumontoan.com
ĐẠI HỌC QUỐC GIA HÀ NỘI ĐỀ THI TUYỂN SINH LỚP 10
TRƯỜNG ĐẠI HỌC KHOA HỌC TỰ NHIÊN HỆ THPT CHUYÊN NĂM 1997

Đề số 51- MÔN : TOÁN (Vòng 2)


Thời gian làm bài: 150 phút (không kể thời gian phát đề)

 y 3 + y 2 x + 3x − 6 y =
0
Bài 1. Giải hệ phương trình  2
 x + xy = 3

Bài 2. Có tồn tại hay không các số nguyên x, y thoả mãn điều kiện 1992 x1993 + 1993 y1994 =
1995

Bài 3. Số 1997 viết được dưới dạng tổng n hợp số, nhưng không viết được dưới dạng tổng n + 1
hợp số. Hỏi n bằng bao nhiêu?

Bài 4. Cho các tam giác ABC ngoại tiếp vòng tròn có bán kính bằng 1. Gọi ha , hb , hc lần lượt là độ
dài các đường cao hạ từ đỉnh A, B, C tới các cạnh đối diện. Hãy tìm giá trị lớn nhất của biểu thức.
1 1 1
M = + +
ha + 2hb hb + 2hc hc + 2ha

Bài 5. Trên đường tròn cho 16 điểm và dùng 3 màu: xanh, đỏ, vàng để tô các điểm này (mô̄i điểm
tô bằng một màu. Giữa mô̄i cặp điểm nối bằng một đoạn thẳng được tô bằng màu tím hoặc màu
nâu.
Chứng minh rằng với mọi cách tô màu trên các điểm (chỉ dùng 3 màu: xanh, đỏ, vàng) và mọi cách
tô màu trên các đoạn thẳng nối giữa các cặp điểm (chỉ dùng hai màu: tím hoặc nâu) ta đều tìm được
trên hình vẽ một tam giác có đỉnh là các điểm đã cho, mà các đỉnh được tô bằng cùng một màu và
các cạnh cũng được tô bằng cùng một màu (dĩ nhiên khác màu tô trên đỉnh).

Tài liệu sưu tầm và tổng hợp bản word đầy đủ liên hệ 0393732038 TÀI LIỆU TOÁN HỌC
54
Website:tailieumontoan.com
ĐẠI HỌC QUỐC GIA HÀ NỘI ĐỀ THI TUYỂN SINH LỚP 10
TRƯỜNG ĐẠI HỌC KHOA HỌC TỰ NHIÊN HỆ THPT CHUYÊN NĂM 1996

Đề số 52- MÔN : TOÁN (Vòng 1)


Thời gian làm bài: 150 phút (không kể thời gian phát đề)

6
 1  6 1 
x+  −x + 6 −2
Bài 1. Cho x > 0 , hãy tìm giá trị nhỏ nhắt của biểu thức P = 
x  x 
3
 1 1
x+  + x + 3
3

 x x

 1 1
 + 2− =
2
 x y
Bài 2. Giải hệ phương trình 
 1
+ 2−
1
=
2
 x
 y

Bài 3. Chứng minh rằng với mọi n nguyên dương ta có n3 + 5n  6

a 3 b3 c 3
Bài 4. Cho a, b, c > 0 , chứng minh rằng + + ab + bc + ca
b c a
Bài 5. Cho hình vuông ABCD cạnh bầng a . Gọi M,N,P,Q là các điểm bắt kỳ lằn lượt nằm trên các
cạnh AB, BC, CD, DA.

1. Chứng minh rà̀ ng 2a 2MN 2 + NP 2 + PQ 2 + QM 24a 2

2. Giả sử M là một điểm cố định cho trước trên cạnh AB. Hãy xác định vị trí của các điểm N, P, Q
lằn lượt trên các cạnh BC, CD, DA sao cho MNPQ là một hình vuông.

Tài liệu sưu tầm và tổng hợp bản word đầy đủ liên hệ 0393732038 TÀI LIỆU TOÁN HỌC
55
Website:tailieumontoan.com
ĐẠI HỌC QUỐC GIA HÀ NỘI ĐỀ THI TUYỂN SINH LỚP 10
TRƯỜNG ĐẠI HỌC KHOA HỌC TỰ NHIÊN HỆ THPT CHUYÊN NĂM 1996

Đề số 53- MÔN : TOÁN (Vòng 2)


Thời gian làm bài: 150 phút (không kể thời gian phát đề)

Phần chung cho chuyên toán và chuyên tin


Bài 1. Giải phương trình ( x − 1 + 1)3 + 2 x − 1 = 2 − x
x − y = 1

Bài 2. Giải hệ phương trình  y − z = 1

 z − x =
1
Bài 3. Cho x, y là những số nguyên dương thay đổi thoả mãn điều kiện x + y =201
( ) (
Hãy tìm giá trị lớn nhất và nhỏ nhất của biểu thức: P= x x 2 + y + y y 2 + x . )
Bài 4. Cho đoạn thẳng BC và đường thẳng (d ) song song với BC. Biết rầng khoảng cách giữa
BC
đường thẳng (d ) và đường thẳng đi qua BC nhỏ hơn . Giả sử A là một điểm thay đổi trên
2
đường thẳng (d ) .
1. Hãy xác định vị trí của điểm A để bán kính vòng tròn ngoại tiếp ∆ABC nhỏ nhát
2. Gọi ha , hb , hc là độ dài các đường cao của ∆ABC . Hãy xác định vị trí của điểm A để tích
ha ⋅ hb ⋅ hc là lớn nhất.
Phần dành cho chuyên toán
3 1 1 1 3
Bài 5. Cho x, y, z > 0 và x + y + z . Chứng minh: x 2 + 2 + y 2 + 2 + z 2 + 2  17
2 x y z 2
Phần dành cho chuyên tin
Câu 5. Chia một hình tròn thành 14 hình quạt bằng nhau. Trong mỗi hình quạt đặt một viên bi. Gọi
T là một phép biến đổi: Lấy hai hình quạt bắt kỳ có bi và chuyển từ mổi hình quạt đó một viên bi
sang hình quạt liền kề nhưng theo hai chiều ngược nhau (ví dụ, nếu viên bi ở một hình quạt được
chuyển theo chiều kim đồng hồ thì viên bi ở hình quạt kia được chuyển theo chiều ngược lại). Hỏi
bằng việc thực hiện phép biến đổi trên, sau một số hữu hạn bước ta có thể chuyển được tất cả các
viên bi vào một hình quạt được không. Nếu có, hãy chỉ ra quá trình biến đổi.Nếu không, hãy giải
thích tại sao?

Tài liệu sưu tầm và tổng hợp bản word đầy đủ liên hệ 0393732038 TÀI LIỆU TOÁN HỌC
56
Website:tailieumontoan.com
ĐẠI HỌC QUỐC GIA HÀ NỘI ĐỀ THI TUYỂN SINH LỚP 10
TRƯỜNG ĐẠI HỌC KHOA HỌC TỰ NHIÊN HỆ THPT CHUYÊN NĂM 1995

Đề số 54- MÔN : TOÁN (Vòng 1)


Thời gian làm bài: 150 phút (không kể thời gian phát đề)

2 x 2 − y 2 =1
Bài 1. Giải hệ phương trình 
 xy + x =
2
2

Bài 2. Giải phương trình 1− x + 4 + x =3

a +1 b +1
Bài 3. Giả sử a, b là các số nguyên dương sao cho: + là một số nguyên. Gọi d là ước số
b a
của a và b . Chứng minh rằng: d  a + b .

Bài 4. Cho hai hình chữ nhật có cùng diện tích. Hình chữ nhật thứ nhất có các kích thước a và
b(a > b) . Hình chữ nhật thứ hai có các kích thước c và d (c > d ) . Chứng minh rầng: nếu a > c thì
chu vi của hình chữ nhật thứ nhắt lớn hơn chu vi của hình chữ nhật thứ hai.
Bài 5. Cho ba điểm cố định A, B, C thẳng hàng theo thứ tự ấy. Gọi (Ω) là một vòng tròn qua B và
C . Kẻ từ A các tiếp tuyến AE và AF đến vòng tròn (Ω) . ( E và F là các tiếp điểm). Gọi O là
tâm của vòng tròn (Ω), I là trung điểm của BC, N là trung điểm của EF.

1. Chứng minh rà̀ ng: E và F nầm trên một vòng tròn cố định khi vòng tròn (Ω) thay đổi.

2. Đường thẳng FI cất vòng tròn (Ω) tại E ' . Chứng minh rằng EE ' song song với AB.

3. Chứng minh ràng tâm vòng tròn ngoại tiếp tam giác ONI nằm trên một đường thẳng cố định khi
vòng tròn (Ω) thay đổi.

Tài liệu sưu tầm và tổng hợp bản word đầy đủ liên hệ 0393732038 TÀI LIỆU TOÁN HỌC
57
Website:tailieumontoan.com
ĐẠI HỌC QUỐC GIA HÀ NỘI ĐỀ THI TUYỂN SINH LỚP 10
TRƯỜNG ĐẠI HỌC KHOA HỌC TỰ NHIÊN HỆ THPT CHUYÊN NĂM 1995

Đề số 55- MÔN : TOÁN (Vòng 2)


Thời gian làm bài: 150 phút (không kể thời gian phát đề)

(
Bài 1. Cho x + x 2 + 3 )( y + y2 + 3 =
3)
Hãy tính giá trị của biểu thức E= x + y

 x + xy + y =1

Bài 2. Giải hệ phương trình  y + yz + z =3
 z + zx + x =
 1

1
Bài 3. Cho x, y0 và x 2 + y 2 =
1 . Chứng minh rà̀ ng x3 + y 31
2

Bài 4. Tìm số nguyên có chín chữ số A = a1 a2 a3 b1b2 b3 a1 a2 a3 , trong đó a1 ≠ 0 và b1b2 b3 = 2a1 a2 a3


đồng thời A có thể viết được dưới dạng A = p12 p22 p32 p42 với p1 , p2 , p3 , p4 là bốn số nguyên khác nhau.

Bài 5. Cho vòng tròn (Ω) , vẽ hai dây cung AB và CD cắt nhau ở I ( I nầm trong vòng tròn). Gọi
AN AI 2
M là trung điểm của BD, MI kéo dài cất AC ở N . Chứng minh rằng =
NC CI 2

Tài liệu sưu tầm và tổng hợp bản word đầy đủ liên hệ 0393732038 TÀI LIỆU TOÁN HỌC
58
Website:tailieumontoan.com
ĐẠI HỌC QUỐC GIA HÀ NỘI ĐỀ THI TUYỂN SINH LỚP 10
TRƯỜNG ĐẠI HỌC KHOA HỌC TỰ NHIÊN HỆ THPT CHUYÊN NĂM 1994

Đề số 56- MÔN : TOÁN (Vòng 1)


Thời gian làm bài: 150 phút (không kể thời gian phát đề)

Bài 1. Giải các phương trình sau:

1. x 4 − 2 x3 − 6 x 2 + 16 x − 8 =0

2. x 2 + 2 x +=
4 3 x3 + 4 x

Bài 2. Xét các số x, y, z , t > 0 thoả mãn hệ thức xy + 4 zt + 2 yz + 2 xt =


9

Tìm giá trị lớn nhấtt của biểu thức=


A xy + 2 zt

 xy − 3 zt =1
Bài 3. Tìm tất cả các số nguyên x, y, z, t thoả mãn hệ phương trình 
 xz + yt = 2

Bài 4. Cho tam giác cân ABC có AB = AC và H là trung điểm của cạnh BC. Một đường tròn đi
qua A và tiếp xúc với cạnh BC tại B cắt AC, AH lằn lượt tại D và E . Biết rằng D là trung điểm
của AC và bán kính đường tròn bằng R . Tính độ dài các dãy cung AE, AD theo R .
Bài 5. Cho tam giác ABC có BC > AC . Một đường thẳng song song với cạnh AB cắt các cạnh BC
và AC lần lượt tại các điểm M và N . Chứng minh rằng BN > AM .

Tài liệu sưu tầm và tổng hợp bản word đầy đủ liên hệ 0393732038 TÀI LIỆU TOÁN HỌC
59
Website:tailieumontoan.com
ĐẠI HỌC QUỐC GIA HÀ NỘI ĐỀ THI TUYỂN SINH LỚP 10
TRƯỜNG ĐẠI HỌC KHOA HỌC TỰ NHIÊN HỆ THPT CHUYÊN NĂM 1994

Đề số 57- MÔN : TOÁN (Vòng 2)


Thời gian làm bài: 150 phút (không kể thời gian phát đề)

( x + y )( y + z ) =
4 xy 2 z

Bài 1. Giải hệ phương trình ( y + z )( z + x) =4 yz 2 x

( z + x)( x + y ) =4 zx 2 y

Bài 2. Tìm tất cả các cặp số nguyên ( x, y ) thoả mãn phương trình

12 x 2 + 6 xy + 3 y 2 = 28( x + y )

Bài 3. Xác định các giá trị nguyên dương n(n3) sao cho số A = 1, 2,3… n (tích của n số nguyên
dương đầu tiên) chia hết cho số B = 1 + 2 + 3 +  + n .
1 1 1 1 1 1
Bài 4. Cho a, b, c1 . Chứng minh rằng + +  + +
1 + a 1 + b 1 + c 1 + ab
4 3
1 + bc
4 3
1 + ca 3
4

Bài 5. Cho ∆ABC có AB = AC .

20° thì luôn tìm được các điểm D và K trên các cạnh AB và AC
1. Chứng minh rằng nếu ∠BAC =
= DK
sao cho AD = KC = CB .
2. Ngược lại, chứng minh rằng nếu tồn tại các điểm D và K trên các cạnh AB và AC sao cho
= DK
AD = KC = CB thì ∠BAC = 20° .

Tài liệu sưu tầm và tổng hợp bản word đầy đủ liên hệ 0393732038 TÀI LIỆU TOÁN HỌC
60
Website:tailieumontoan.com
ĐẠI HỌC QUỐC GIA HÀ NỘI ĐỀ THI TUYỂN SINH LỚP 10
TRƯỜNG ĐẠI HỌC KHOA HỌC TỰ NHIÊN HỆ THPT CHUYÊN NĂM 1993

Đề số 58- MÔN : TOÁN


Thời gian làm bài: 150 phút (không kể thời gian phát đề)

Bài 1.

1 1
1. Giải phương trình x + x + + x+ =2
2 4

 x 3 + 2 xy 2 + 12 y =
0
2. Giải hệ phương trình  2
8 y + x =
2
12

Bài 2. Tìm giá trị lớn nhất và bé nhất của biểu thức=
A x 2 y (4 − x − y )

khi x và y thay đổi thoả mãn điều kiện: x0, y0, x + y6

Bài 3. Cho hình thoi ABCD. Gọi R, r lần lượt là bán kính các đường tròn ngọị tiếp các tam giác
1 1 4
ABD, ABC và a là độ dài cạnh hình thoi. Chứng minh rằng: 2 + 2 =
R r a2

Bài 4. Cho tam giác đều ABC nội tiếp đường tròn tâm O bán kính R . Quay ∆ABC một góc 90o
quanh tâm O ta được ∆A1 B1C1 . Tính diện tích phần chung của hai hình tam giác ABC và A1 B1C1
theo R .
Bài 5. Tìm tất cả các số nguyēn dương a, b, c đôi một khác nhau sao cho biểu thức
1 1 1 1 1 1
A= + + + + + nhận giá trị nguyên dương.
a b c ab ac bc

Tài liệu sưu tầm và tổng hợp bản word đầy đủ liên hệ 0393732038 TÀI LIỆU TOÁN HỌC
61
Website:tailieumontoan.com
ĐẠI HỌC QUỐC GIA HÀ NỘI ĐỀ THI TUYỂN SINH LỚP 10
TRƯỜNG ĐẠI HỌC KHOA HỌC TỰ NHIÊN HỆ THPT CHUYÊN NĂM 1992

Đề số 59- MÔN : TOÁN (Vòng 1)


Thời gian làm bài: 150 phút (không kể thời gian phát đề)

Bài 1. 1. Giải phương trình x + 2 + 3 2x − 5 + x − 2 − 3 2x − 5 =2 2

 xy 2 − 2 y + 3 x 2 =0
2. Giải hệ phương trình  2
 y + x y + 2x =
2
0

Bài 2. Tìm tất cả các cặp số nguyên không âm (m, n) để phương trình x 2 − mnx + m + n =0

có nghiệm nguyên.

Bài 3. Cho tam giác ABC có diện tích S . Trên các cạnh AB, BC, CA lần lượt lấy C ′ , A′ , B′ tương
BA′ 1 CB′ 1
ứng, sao cho= AC ′ C ′ B=, =, . Giả sử AA′ cắt BB′ tại M , BB′ cắt CC ′ tại
A′ C 2 B′ A 3
N , CC ′ cắt AA′ tại P . Tính diện tích tam giác MNP theo S .

Bài 4. Cho tam giác ABC nội tiếp trong một đường tròn. Lấy một điểm D trên cung BC (không
chứa A) của đường tròn đó. Hạ DH vuông góc với BC, DI vuông góc với CA và DK vuông góc với
BC AC AB
AB. Chứng minh rằng = +
DH DI DK
Bài 5. Tìm tất cả các cặp số nguyên dương (m, n) sao cho 2m + 1 chia hết cho n và 2n + 1 chia hết
cho m .

Tài liệu sưu tầm và tổng hợp bản word đầy đủ liên hệ 0393732038 TÀI LIỆU TOÁN HỌC
62
Website:tailieumontoan.com
ĐẠI HỌC QUỐC GIA HÀ NỘI ĐỀ THI TUYỂN SINH LỚP 10
TRƯỜNG ĐẠI HỌC KHOA HỌC TỰ NHIÊN HỆ THPT CHUYÊN NĂM 1992

Đề số 60- MÔN : TOÁN (Vòng 2)


Thời gian làm bài: 150 phút (không kể thời gian phát đề)

( )
1945
Bài 2. Cho a là tổng các chữ số của 29 ,b là tổng các chữ số của số a . Tìm tổng các chữ số
của b .
Bài 3. Cho tam giác ABC. Giả sử đường phân giác trong và ngoài của góc A cất đường thẳng BC
tại D, K tương ứng. Chứng minh rầng nếu AD = AK thì AB 2 + AC 2 = 4 R 2 , trong đó R là bán kính
đường tròn ngoại tiếp tam giác ABC
Bài 4. Trong mặt phẳng kẻ 1992 đường thẳng sao cho không có 2 đường nào song song và không có ba
đường nào đồng quy. Tam giác tạo bởi ba đường thẳng trong số các đường thẳng đã cho gọi là "tam giác
xanh" nếu nó không bị đường thẩng nào trong số các đường thẳng còn lại cắt.
1. Chứng minh rằng số tam giác xanh không ít hơn 664 .
2. Chứng minh kết luận mạnh hơn: Số tam giác xanh không ít hơn 1328 .
Bài 5. Có 41 thành phố được nối với nhau bầng các đường chỉ đi được một chiều. Biết ràng từ mổi
thành phố có đúng 16 đường đến các thành phố khác và đúng 16 đường từ các thành phố khác đến
nó. Giữa hai thành phố bất kỳ không có quá một con đường của mạng đường nói trên. Chứng minh
rầng từ một thành phố bất kỳ A đều có thể đi đến một thành phố bắt kỳ B mà chỉ đi qua nhiều nhất
hai thành phố trung gian.

Tài liệu sưu tầm và tổng hợp bản word đầy đủ liên hệ 0393732038 TÀI LIỆU TOÁN HỌC
63
Website:tailieumontoan.com
ĐẠI HỌC QUỐC GIA HÀ NỘI ĐỀ THI TUYỂN SINH LỚP 10
TRƯỜNG ĐẠI HỌC KHOA HỌC TỰ NHIÊN HỆ THPT CHUYÊN NĂM 1991

Đề số 61- MÔN : TOÁN (Vòng 1)


Thời gian làm bài: 150 phút (không kể thời gian phát đề)

Bài 1.
a+x + a−x
1. Giải và biện luận phương trình = b
a+x − a−x
Trong đó a, b là các số dương đã cho.
2. Cho phương trình x 2 + ax + b + 1 =0 . Trong đó a, b ∈  và b ≠ −1 . Chứng minh rằng nếu
phương trình có hai nghiệm đều là những số nguyên thì a 2 + b 2 là hợp số.
a 3 x + a 2 y + az =
1
 3
Bài 2. Cho a, b, c là các số đôi một khác nhau và khác 0. Giải hệ b x + b y + bz =
2
1
c3 x + c 2 y + cz =1

Bài 3.Tìm nghiệm nguyên, dương của phương trình=
7 x 3.2 y + 1 .
Bài 4.
1. Cho hình thang ABCD( AB / / CD) . Gọi giao điểm của AD và BC là E , giao điểm của AC và BD
là F . Chứng minh rằng đường thẳng EF đi qua giao điểm của hai đáy AB, CD.
2. Cho tam giác ABC, M, N, P lần lượt là các điểm trên các cạnh BC, CA, AB. Nối AM, BN, CP.
Chứng minh rằng nếu diện tích của bốn tam giác gạch chéo bằng nhau thì các diện tích của ba tứ
giác không gạch chéo cũng bằng nhau. (Xem hình vẽ)

Bài 5. Tồn tại hay không 1991 điểm trên mặt phẳng sao cho ba điểm bất kỳ trong chúng là ba đỉnh
của một tam giác có một góc tù?

Tài liệu sưu tầm và tổng hợp bản word đầy đủ liên hệ 0393732038 TÀI LIỆU TOÁN HỌC
64
Website:tailieumontoan.com
ĐẠI HỌC QUỐC GIA HÀ NỘI ĐỀ THI TUYỂN SINH LỚP 10
TRƯỜNG ĐẠI HỌC KHOA HỌC TỰ NHIÊN HỆ THPT CHUYÊN NĂM 1991

Đề số 62- MÔN : TOÁN (Vòng 2)


Thời gian làm bài: 150 phút (không kể thời gian phát đề)

Bài 1.
1. Rút gọn biểu thức A= 3
2 3 − 4 2 ⋅ 6 44 + 16 6
2. Phân tích biểu thức sau thành nhân tử P = ( x − y )5 + ( y − z )5 + ( z − x)5
Bài 2.

a + b + c =0

1. Cho các số a, b, c, α , β , γ thoả mãn các điều kiện α + β + γ = 0
α β γ
 + + = 0
a b c
Hãy tính giá trị của biểu thức A = α a 2 + β b 2 + γ c 2
2. Cho bốn số a, b, c, d mỗi số đều không âm và nhỏ hơn hoặc bằng 1. Chứng minh rằng
0 ≤ a + b + c + d − ab − bc − cd − da ≤ 2
Khi nào thì dấu đẳng thức xảy ra?
Bài 3. Cho trước a và d là những số nguyên dương. Xét tất cả các số có dạng
a, a + d , a + 2d , …, a + nd , …
Chứng minh rằng trong các số đó có ít nhất một số mà 4 chữ số đầu tiên của nó là 1991.
Bài 4. Trong một cuộc hội thảo khoa học có 100 người tham dự. Giả sử mỗi người đều quen biết
với ít nhất 67 người. Chứng minh rằng có thể tìm được một nhóm 4 người mà bất kỳ 2 người trong
nhóm đó đều quen biết nhau.
Bài 5.
1. Cho hình vuông ABCD. Lấy diểm M nằm trong hình vuông sao cho MAB = MBA 
= 15° .
Chứng minh rằng tam giác MCD là tam giác đều.
2. Hãy xây dựng một tập hợp gồm 8 diểm có tính chất: Đường trung trực của đoạn nối hai điểm bất
kỳ luôn đi qua ít nhất hai điểm của tập hợp điểm đó.

Tài liệu sưu tầm và tổng hợp bản word đầy đủ liên hệ 0393732038 TÀI LIỆU TOÁN HỌC
65
Website:tailieumontoan.com
ĐẠI HỌC QUỐC GIA HÀ NỘI ĐỀ THI TUYỂN SINH LỚP 10
TRƯỜNG ĐẠI HỌC KHOA HỌC TỰ NHIÊN HỆ THPT CHUYÊN NĂM 1989

Đề số 63- MÔN : TOÁN (Vòng 1)


Thời gian làm bài: 150 phút (không kể thời gian phát đề)

Bài 1. Cho đa thức P( x) = ax 2 + bx + c .


Biết rằng với mọi giá trị nguyên của x , giá trị của đa thức P( x) đều là những số chính phương
(nghĩa là bằng bình phương của một số nguyên). Chứng minh rằng các hệ số $a, b, c$ dều là những
số nguyên, và b là một số chẵn.
Bài 2. Tìm giá trị bé nhất của biểu thức a 2 + ab + b 2 − 3a − 3b + 1989
Giá trị bé nhất đó đạt được tại giá trị nào của a và b ?
Bài 3. Chứng minh rằng trong 52 số nguyên dương bất kỳ luôn luôn có thể tìm được 2 số sao cho
tổng hoặc hiệu của 2 số đó chia hết cho 100 .
 = CAy
Bài 4. Cho tam giác ABC. Về phía ngoài tam giác vẽ các góc BAx  = 21o . Hạ BE vuông góc
với Ax ( E nằm trên Ax ) , CF vuông góc với Ay ( F nằm trên Ay). M là trung điểm của BC.
1. Chứng minh rằng tam giác MEF là tam giác cân
2. Tính các góc của tam giác MEF.
Bài 5. Có 9 học sinh vừa lớp A vừa lớp B sắp thành một hàng dọc, đứng cách đều. Chứng minh
rằng có ít nhất 1 học sinh đứng cách hai em cùng lớp với mình một khoảng cách như nhau.

Tài liệu sưu tầm và tổng hợp bản word đầy đủ liên hệ 0393732038 TÀI LIỆU TOÁN HỌC
66
Website:tailieumontoan.com
ĐẠI HỌC QUỐC GIA HÀ NỘI ĐỀ THI TUYỂN SINH LỚP 10
TRƯỜNG ĐẠI HỌC KHOA HỌC TỰ NHIÊN HỆ THPT CHUYÊN NĂM 1989

Đề số 64- MÔN : TOÁN (Chuyên lý)


Thời gian làm bài: 150 phút (không kể thời gian phát đề)

−2 x 2 + x + 36
Bài 1. Tìm tất cả những giá trị nguyên của x để biểu thức sau là số nguyên
2x + 3
Bài 2. Tìm giá trị bé nhất của biểu thức a 2 + ab + b 2 − 3a − 3b + 3
Giá trị bé nhất đó đạt được tại giá trị nào của a và b ?
Bài 3.
1. Chứng minh rằng với mọi m nguyên dương, biểu thức m 2 + m + 1 không phải là số chính
phương (nghĩa là không thể bằng bình phương của số nguyên).
2. Chứng minh rằng với mọi m nguyên dương, m(m + 1) không thể bằng tích của bốn số nguyên
liên tiếp.
Bài 4. Cho tam giác ABC vuông cân, góc A = 90°.CM là trung tuyến (M nằm trên AB) . Từ A vẽ
BH
đường vuông góc với MC cắt BC ở H . Tính tỷ số .
HC
Bài 5. Có 6 thành phố, trong đó cứ 3 thành phố bất kỳ thì có ít nhất 2 thành phố liên lạc được với
nhau. Chứng minh rằng trong 6 thành phố nói trên tồn tại 3 thành phố liên lạc được với nhau.

Tài liệu sưu tầm và tổng hợp bản word đầy đủ liên hệ 0393732038 TÀI LIỆU TOÁN HỌC
67
Website:tailieumontoan.com

ĐẠI HỌC QUỐC GIA HÀ NỘI ĐỀ THI TUYỂN SINH LỚP 10


TRƯỜNG ĐẠI HỌC KHOA HỌC TỰ NHIÊN HỆ THPT CHUYÊN NĂM 1989

Đề số 65- MÔN : TOÁN (Chuyên toán)


Thời gian làm bài: 150 phút (không kể thời gian phát đề)

Bài 1. Phân tích biểu thức sau thành nhân tử


$ a 4 + b 4 + c 4 − 2a 2 b 2 − ab 2 c 2 − 2c 2 a 2 $
Bài 2.
x 2 x2
1. Cho biết 2 = − . Hãy tính giá trị của biểu thức 4
x + x +1 3 x + x2 + 1
x2
2. Tìm giá trị lớn nhất của biểu thức
x4 + x2 + 1
Giá trị lớn nhất đó đạt được tại giá trị nào của x
Bài 3. Cho biểu thức P(n) = a n + bn + c , trong đó a, b, c là những số nguyên dương. Chứng minh
rằng nếu với mọi giá trị nguyên dương của n, P(n) luôn chia hết cho m ( m là số nguyên dương cố
định), thì b 2 phải chia hết cho m . Với ví dụ sau đây hãy chứng tỏ rằng không thể suy ra b chia hết
cho m
P(n) = 3n + 2n + 3 ( xét khi m = 4)
Bài 4. Cho đa giác lồi sáu cạnh ABCDEF . M,I,L,K,N,H lần lượt là trung điểm của các cạnh AB, BC,
CD, DE, EF, FA. Chứng minh rằng các trọng tâm của hai tam giác MNL và HIK trùng nhau.
Bài 5. Giả sử trong một trường có n lớp ta ký hiệu am là số học sinh của lớp thứ m, d k là số lớp
trong đó mỗi lớp có ít nhất k học sinh, M là số học sinh của lớp đông nhất. Chứng minh rằng:
1. a1 + a2 +  + an = d1 + d 2 +  + d M
2. a12 + a22 +  + an2 = d1 + 3d 2 + 5d3 +  + (2k − 1)d k +  + (2 M − 1)d M

Tài liệu sưu tầm và tổng hợp bản word đầy đủ liên hệ 0393732038 TÀI LIỆU TOÁN HỌC
1
Website:tailieumontoan.com

HƯỚNG DẪN GIẢI


Đề số 1

Câu I. (4 điểm)

6 ( xy + 5 ) + x 3 y + 5 x 2 = 42
1) Giải hệ phương trình 
 x + 5 x y + 6 x + 30 y =
3 2
42
Trừ vế với vế của 2 phương trình ta được :
( x 2 y − x 2 ) + ( 6 xy − 6 x ) − ( 5 x 2 y − 5 x 2 ) − ( 30 x − 30 ) =
0
Tương đương với ( y − 1)( x − 5 ) ( x 2 + 6 ) =
0
Từ đây suy ra y = 1 hoặc x = 5
−113
Với y =1 ⇒ x =1, với x = 5 thì y =
155
 −113 
Vậy hệ phương trình có nghiệm ( x; y ) là (1;1) ,  5; 
 155 
2) Giải phương trình :
( 3
)( )
x + 6 + 3 3 − x 2 + 3 3 ( x + 6 )( 3 − x ) =24
Đặt 3
x += x b . Khi đó, ta có hệ phương trình :
6 a, 3 3 − =

( a + b )( 2 + 3ab ) =
24
 2 . Cộng hai phương trình lại ta suy ra
a + b =
2
9

(a + b) + 2 ( a + b ) − 33 =0 ⇔ ( a + b − 3) ( a + b ) + 3 ( a + b ) + 11 =0
2 2
 

Lưu ý ( a + b ) + 3 ( a + b ) + 11 > 0 nên ta phải có a + b =3 , kéo theo ab = 2.


2

( a, b ) = (1, 2 )
Giải ta tìm được  . Xét hai trường hợp:
( a, b ) = ( 2,1)

+)a =1, b =2⇒ x= −5


+)a =2, b =1 ⇒ x =2

Vậy =
S {2; −5}
Câu II. (2 điểm)
1) Tìm tất cả các cặp số nguyên ( x, y ) thỏa mãn đẳng thức :
25 y 2 + 354 x + 60= 36 x 2 + 305 y + ( 5 y − 6 x )
2022

Phương trình đã cho có thể viết lại dưới dạng


( 5 y − 6 x − 1)( 5 y + 6 x − 60 ) = ( 5 y − 6 x )
2022

Rõ ràng ( 5 y − 6 x − 1,5 y − 6 x ) = (
1 nên 5 y − 6 x − 1, ( 5 y − 6 x )
2022
)=
1, từ đó ta phải có
5 y − 6 x − 1 =±1 . Xét hai trường hợp

Tài liệu sưu tầm và tổng hợp bản word đầy đủ liên hệ 0393732038 TÀI LIỆU TOÁN HỌC
2
Website:tailieumontoan.com
+) Nếu 5 y − 6 x − 1 =1 thì 5 y + 6 x − 60 ≡ 22022 . Cộng lại, ta có :
=
10 y ≡ 22022 + 62. Tuy nhiên 22022 (=
2 ) .2
4 505 2
16505.4 có tận cùng là 4 nên 22022 + 62 có tận cùng là
8 không chia hết cho 10. Trường hợp này vô nghiệm
+)Nếu 5 y − 6 x − 1 =−1 thì 5 y + 6 x − 60 ≡ 0 . Từ đây tìm được=
y 6,=
x 5
Vậy phương trình đã cho có đúng một nghiệm là ( 5;6 )
2) Trên bàn có 8 hộp rỗng (trong các hộp không có viên bi nào). Người ta thực hiện các
lần thêm bi vào các hộp theo quy tắc sau : mỗi lần ta chọn ra 4 hộp bất kỳ và bỏ vào
hộp 1 viên, một hộp 2 viên, hai hộp còn lại mỗi hộp 3 viên. Hỏi số lần thêm bi ít nhất có
thể để nhận được số bi ở 8 hộp trên là số tự nhiên liên tiếp ?
Gọi số lần thêm ít nhất là n thì các hộp bi tương ứng là a, a + 1, a + 2,..., a + 7

Mỗi lần thêm tất cả là 9 bi nên ta có :


a + ( a + 1) + ( a + 2 ) + ( a + 7 ) = 9n ⇔ 8a + 28 = 9n

Suy ra 4 ( 2n + 7 ) chia hết cho 9, kéo theo 2a + 7 chia hết cho 9.

Từ đó ta phải có 2a + 7 ≥ 9 ⇒ n ≥ 4 . Ta chứng minh n = 4 thỏa mãn


Gọi các hộp tương ứng là 1, 2,3, 4,5, 6, 7,8 và thực hiện thêm bi ở các lần như sau :

- Lần 1: 1, 2, 0, 0, 0, 0,3,3
- Lần 2: 0, 0,1, 0, 2,3, 0,3
- Lần 3: 0, 0, 2,1,3, 0,3, 0
- Lần 4: 0, 0, 0,3, 0,3,1, 2

Câu III. (3 điểm) Cho hình chữ nhật ABCD ( AB < AD ) nội tiếp đường tròn ( O ) . Trên cạnh
AD lấy hai điểm E và F ( E , F không trùng với A, D) sao cho E nằm giữa A và F, đồng thời
1
∠ABE + ∠DCF = ∠BOC
2

A E F D

O
M N

J
B X C
Q P
T

1) Chứng minh rằng BE và CF cắt nhau tại một điểm nằm trên đường tròn ( O )
Đặt BE ∩ CF = K
Gọi H là hình chiếu của K trên AD.
Để ý rằng KH / / AB và KH / / CD nên ta có :

Tài liệu sưu tầm và tổng hợp bản word đầy đủ liên hệ 0393732038 TÀI LIỆU TOÁN HỌC
3
Website:tailieumontoan.com

∠BOC . Từ đó suy ra K ∈ ( O )
1
∠BKC = ∠BKH + ∠CKH = ∠ABE + ∠DCF =
2
2) Đường thẳng qua O song song với BC cắt BE , CF theo thứ tự tại M , N . Chứng minh
1
rằng ∠DAM + ∠ADN + ∠AOD = 180°
2
Trước hết để ý rằng ∠KMO = ∠KBC =∠KAC = ∠KAO ta tìm được tứ giác KAMO là tứ giác nội
tiếp
Chứng minh tương tự, ta cũng có KDNO là tứ giác nội tiếp .Đặt AM ∩ DN = T
Khi đó ta có ∠TAC = ∠MAO = ∠MKO = ∠BKC − ∠NKO = ∠BDC − ∠NDO = ∠TDC
Từ đó suy ra T ∈ ( O ) . Vì vậy,
1 1
∠DAM + ∠ADN + ∠AOD = 180° − ∠ATD + ∠AOD = 180°
2 2
3) Dựng hình chữ nhật MNPQ sao cho NQ song song với BD, đồng thời MP song song với
AC. Chứng minh rằng đường tròn ngoại tiếp hình chữ nhật MNPQ tiếp xúc với đường
tròn ( O )

J . Để ý rằng ∠JMN =
Gọi MP ∩ NQ = ∠OCB =
∠OBC =
∠JNM nên JM = JN .

Như vậy, ta quy về chứng minh đường tròn ( J ; JM ) tiếp xúc với ( O ) . Thật vậy, ta cần chứng minh
JM + OJ =
R với R là bán kính của (O)
OM XB
X . Khi đó ta có
Đặt KO ∩ BC = = . Ta thấy
ON XC
∆JMN  ∆OBC . Do đó, ta được :
KO OM MN JO JM JM JM + JO JM + JO
= = = = = = =
KX BX BC OX OB OK OK + OX KX

Suy ra , JM + JO = OK = R. Như vậy ( MNPQ ) ≡ ( J , JM ) tiếp xúc trong với (O)

Câu IV. (1 điểm)


Cho a, b, c là những số thực dương. Chứng minh rằng :

2a a + b 6a + 2c 4a + 3b + c 32a
+ + + ≥
a + b a + c 3b + c b+c 2a + b + c
Ta viết lại vế trái dưới dạng :

2a  2a b + c   2 ( a + c ) 4a   4a 3b + c 3b + c 
=
P + + − 1 +  +  +  + + 
a + b  a + c a + c   3b + c 3b + c   b + c 2 ( b + c ) 2 ( b + c ) 
2a 2a 4a 4a 3b + c  b + c 2 ( a + c ) 3b + c 
= + + + + + + +  −1
a + b a + c 3b + c b + c 2 ( b + c )  a + c 3b + c 2 ( b + c ) 

b + c 2 ( a + c ) 3b + c
Áp dụng bất đẳng thức AM − GM thì + + ≥ 3 nên :
a+c 3b + c 2 (b + c )

2a 2a 4a 4a 3b + c
P≥ + + + + + 2 . Chú ý rằng :
a + b a + c 3b + c b + c 2 ( b + c )
Tài liệu sưu tầm và tổng hợp bản word đầy đủ liên hệ 0393732038 TÀI LIỆU TOÁN HỌC
4
Website:tailieumontoan.com

2a 2a 8a 4a 3b + c 2a 4a 2a
+ ≥ . + ≥2 và +2≥4 nên :
a + b a + c 2a + b + c 3b + c 2 ( b + c ) b+c b+c b+c

8a 2a 8a 12a 8a 24a 32a


P≥ +6 = + ≥ + =
2a + b + c b + c 2a + b + c 2a ( b + c ) 2a + b + c 2a + b + c 2a + b + c

Điều phải chứng minh

Đề số 2

Câu I. (3,5 điểm)


1 1 1
1) Với a, b, c là những số thực dương thỏa mãn điều kiện + + =1. Chứng minh
a b c
1 1 1 1  abc
rằng :  + + =
2  a + bc b + ca c + ab  ( a + bc )( b + ca )( c + ab )
Từ giả thiết suy ra ab + bc + ca =
abc. Ta có :
1 a2 a
= =
a + bc a 2 + abc ( a + b )( a + c )
1 b 1 c
Tương
= tự, ta có: =; . Từ đó suy ra
b + ca ( b + c )( b + a ) c + ab ( c + a )( c + b )
a (b + c ) + b (c + a ) abc
VT = (1) , Và :
2 ( a + b )( b + c )( c + a ) ( a + b )( b + c )( c + a )
( abc )
2
abc
VP = ( do a, b, c > 0 )( 2 )
( a + b) 2 ( b + c ) ( c + a )
2 2
( a + b )( b + c )( c + a )
Từ (1), (2) suy ra điều phải chứng minh
2 x 2 + 3 xy + y 2 =
6
2) Giải hệ phương trình 
3 x + 2 y =
+1 2 2x + y + 6
Điều kiện : 2 x + y + 6 ≥ 0 . Nhân 4 vào phương trình thứ nhất của hệ ta có :
4 ( 2 x 2 + 3 xy + y 2 ) − 24 =
0
Phương trình thứ hai của hệ tương đương với :
3 x + 2 y + 1 ≥ 0
 . Ta viết lại thành hệ mới :
( 3 x + 2 y + 1) − 4 ( 2 x + y + 6 ) =
2
0
3 x + 2 y + 1 ≥ 0 (1)

4 ( 2 x + 3 xy + y ) − 24 = 0 ( 2)
2 2


( 3 x + 2 y + 1) − 4 ( 2 x + y + 6 ) =0 ( 3)
2

Lấy phương trình (3) trừ đi phương trình (2), vế với vế, ta thu được :
( x − 1) = 0 ⇔ x =1
2

y =1
⇒ 2 x 2 + 3 xy + y 2 = 6 ⇔ y 2 + 3 y − 4 = 0 ⇔ 
 y = −4(ktm)

Tài liệu sưu tầm và tổng hợp bản word đầy đủ liên hệ 0393732038 TÀI LIỆU TOÁN HỌC
5
Website:tailieumontoan.com
Vậy ( x; y ) = (1;1)
Câu II. (2,5 điểm)
1) Tìm tất cả các cặp số nguyên dương ( x; y ) thỏa mãn đẳng thức :
( x + y )( 5 x + y ) + xy 3 = ( 5 x + y ) + x 2 y 3 + xy 4
3 3

Ta biến đổi như sau :


( x + y )( 5 x + y ) + xy 3 = ( 5 x + y ) + x 2 y 3 + xy 4
3 3

⇔ ( x + y − 1)( 5 x + y= ) xy 3 ( x + y − 1)
3

Vì x, y là hai số nguyên dương nên x + y > 1 . Do đó, ta suy ra : ( 5x + y )3 =


xy 3
Do đó, ta suy ra x cũng là lập phương của một số nguyên dương.Đặt x = z 3 , ta có:
(5z + y) = ( zy ) ⇔ 5 z 3 + y = zy ⇔ y ( z − 1) = 5 z 3
3 3 3

Nếu z = 1 (ktm). Xét z ≠ 1 . Khi đó, ta có 5 z 3  ( z − 1) . Vì 5 z 3 ≡ 5 ( mod z − 1) ⇒ 5 z − 1


Từ đây ta tìm được z ∈ {2;6} . Suy ra :
( z; y ) ∈ {( 2; 40 ) ; ( 6; 216 )} ⇒ ( x; y ) ∈ {(8, 40 ) ; ( 216; 216 )}
2) Với a, b, c là những số thực dương thỏa mãn các điều kiện sau
c ≤ b < a ≤ 3; b 2 + 2a ≤ 10; b 2 + 2a + 2c ≤ 14
 2
( a + 1)( b + 1) + 4ab ≤ 2a + 2b + 2a + 2b
2 3 3

Tìm giá trị lớn nhất của biểu thức: P = 4a 4 + b 4 + 2b 2 + 4c 2


Ta có :
(a 2
+ 1)( b 2 + 1) + 4ab − 2a ( a 2 + 1) − 2b ( b 2 + 1) ≤ 0
⇔ ( a 2 + 1 − 2b )( b 2 + 1 − 2a ) ≤ 0 ⇔ b 2 + 1 ≤ 2a

Ta có : P= ( 2a ) + ( b 2 + 1) + ( 2c ) − 1 . Do đó :
2 2 2

P − 76 = ( 2a − 6 )( 2a + 6 ) + ( b 2 − 4 )( b 2 + 6 ) + ( 2c − 4 )( 2c + 4 )
= ( 2a − b 2 ) ( 2a − 6 ) + ( b 2 − 2c + 2 )( 2a + b 2 − 10 ) + ( 2c + 4 ) ( 2a + 2c + b 2 − 14 ) ≤ 0
Do đó P ≤ 76. Vậy Max P = 76 ⇔ ( a, b, c ) = ( 3, 2, 2 )
Câu III. (3 điểm)
Cho tam giác ABC nhọn, không cân, nội tiếp đường tròn (O). Điểm P nằm
trong tam giác ABC . Gọi E , F lần lượt là hình chiếu vuông góc của P trên các
cạnh CA, CB . Giả sử tứ giác BCEF nội tiếp trong đường tròn ( K )

Tài liệu sưu tầm và tổng hợp bản word đầy đủ liên hệ 0393732038 TÀI LIỆU TOÁN HỌC
6
Website:tailieumontoan.com

J
E
F O
Q S
T P R
B C
K

X
1) Chứng minh rằng AP vuông góc với BC
Do tứ giác BFEC nội tiếp dẫn đến ∠AEF = ∠ABC
Để ý rằng ∠OAC= 90° − ∠B dẫn đến OA ⊥ EF
Ta có ∠OAC = 90° − ∠AEF = ∠PAF (do AP là đường kính của ( AEF ))
Do đó dẫn đến ∠PAB + ∠B= 90° ⇒ AP ⊥ BC
2) Chứng minh rằng AP = 2OK
Gọi ( AEF ) cắt (O) tại điểm thứ hai là J. Gọi S , T lần lượt là trung điểm của EC và
FB, ta có ∠JFB = 180° − ∠JFA = 180° − ∠JEA = ∠JEC
Đồng thời, ∠JBF = ∠JCE , do đó : ∆JFB ∽ ∆JEC
⇒ ∆JTF ∽ ∆JES ( c.g .c ) ⇒ ∠JTA = ∠JSA
Do đó J , T , S , A, K cùng thuộc một đường tròn
Dẫn đến J , P, K thẳng hàng và đường thẳng này đi qua X là đối xứng của A qua O
Do đó chú ý rằng PECX là hình thang vuông mà SK là đường trung bình nên dẫn đến
K là trung điểm PX hay AP = 2OK
3) Đường thẳng qua P vuông góc với AP cắt đường tròn tại hai điểm Q, R .
Chứng minh rằng đường tròn tâm A bán kính AP tiếp xúc với đường tròn
ngoại tiếp ∆KQR
Gọi G đối xứng với P qua J
Ta có ∠AJP =90° dẫn đến AG = AP hay G thuộc ( A; AP )
Tài liệu sưu tầm và tổng hợp bản word đầy đủ liên hệ 0393732038 TÀI LIỆU TOÁN HỌC
7
Website:tailieumontoan.com
Ta có K là trung điểm của PX do đó
=
PX =
.PJ 2 PK =
.PJ PK =
.2 PJ PK .PG PQ.PR (do tứ giác JRXQ nội tiếp (O))
Suy ra G ∈ ( KQR )
Câu IV. (1 điểm)
Cho các điểm A1 , A2 ,....., A30 theo thứ tự nằm trên một đường thẳng sao cho
độ dài các đoạn Ak Ak +1 bằng k (đơn vị dài), với k = 1, 2,...., 29 . Ta tô màu mỗi đoạn
thẳng A1 A2 ,....., A29 A30 bởi 1 trong 3 màu (mỗi đoạn được tô bởi đúng 1 màu).
Chứng minh rằng với mọi cách tô màu,, ta luôn chọn được hai số nguyên dương
1 ≤ j ≤ i ≤ 29 sao cho hai đoạn Ai Ai +1 và Aj Aj +1 được tô cùng màu và i − j là bình
phương của số nguyên dương.
Gọi di là màu = Ai Ai +1 ; i 1..., 29, di ∈ {1; 2;3}

Phản chứng di ≠ d j ∀ i − j là số chính phương

di ≠ di +9 , di ≠ di +16 ≠ di + 25 ; di +9 ≠ di + 25 ; di +16 ≠ di + 25

Mà trong di ; di +9 , di +16 ; di + 25 có hai số bằng nhau, nên d=


i +9 di +16 , ∀i ≥ 1

Không mất tính tổng quát, giả sử=


d1 1,=
d 2 2; Có d10 ≠ d1 =
1.

• Nếu d10 =3 =di +9 ⇒ d17 =di +16 =d1+9 =d10 =3


d 26 ≠ d17= 3; d 26 ≠ d1 ⇒ d 26= 2
d11 ≠ d10 =3; d11 ≠ d 2 =2 ⇒ d11 =1
d 27 ≠ d 26 =2, d 27 ≠ d11 =⇒
1 d 27 =⇒
3 d 20 =3
d 26 =2 ⇒ d19 =2, d 20 =3 ⇒ d13 =3
Suy ra d3 = d17 (mâu thuẫn)
• Nếu d10 =2 ⇒ d 24 =d17 =d10 =2
d 26 ≠ d17 =2, d 26 ≠ d1 =1 ⇒ d 26 =3 =d19 =d12
Suy ra d11 = 1 do d11 ≠ d12 =3; d11 ≠ d10 =2 ⇒ d 25 =d18 =d11 =1
d3 ≠ d12 , d3 ≠ d 2 ⇒ d3 =
1 ⇒ d 28 ≠ d3 =
1, d 28 ≠ d 29 =⇒
2 d 28 =3
⇒ d 28 = d 29 = d17 = d10 = 3 nhưng do d10 = 2 , ta có điều mâu thuẫn
Vậy tồn tại i, j sao cho i − j là số chính phương và di = d j
Đề số 3

Bài 1. Giải phương trình 13 5 − x + 18 x + 8 = 61 + x + 3 ( 5 − x )( x + 8) .


Lời giải
Cách 1.
Điều kiện xác định: −8 ≤ x ≤ 5 .

Tài liệu sưu tầm và tổng hợp bản word đầy đủ liên hệ 0393732038 TÀI LIỆU TOÁN HỌC
8
Website:tailieumontoan.com
a = 5 − x ( a ≥ 0 ) a 2 + b 2 =13
Đặt:  . Khi đó, ta có:  .
b = x + 8 ( b ≥ 0 ) 61 + x = a + 2b + 40
2 2

Phương trình đã cho có thể viết lại thành:


13a + 18b = a 2 + 2b 2 + 40 + 3ab ⇔ ( a + b )( a + 2b ) + 40 = 8 ( a + b ) + 5 ( a + 2b ) .
a + b = 5
⇔ ( a + b − 5 )( a + 2b − 8 ) = 0 ⇔  .
 a + 2b = 8
Trường hợp 1: a + b = 5 . Thay a= 5 − b vào đẳng thức a 2 + b 2 = 13 , ta được:
b = 2  x = −4
( 5 − b ) + b2 =13 ⇔ 2 ( b − 2 )( b − 3) =0 ⇔  ⇔
2
(thỏa mãn).
b = 3 x = 1
Trường hợp 2: a + 2b = 8 . Thay a= 8 − 2b vào đẳng thức a 2 + b 2 =13 , ta được:
= b 3= x 1
(8 − 2b ) + b =13 ⇔ ( b − 3)( 5b − 17 ) =0 ⇔  17 ⇔  89 (thỏa mãn).

2 2

= b = x
 5  25
89
Vậy phương trình đã cho có ba nghiệm là x = 1 , x = −4 và x = .
25
5 − x ≥ 0
Cách 2. Điều kiện xác định :  ⇔ −8 ≤ x ≤ 5 .
x + 8 ≥ 0
Đặt a = 5 − x , a ≥ 0 .
Phương trình trở thành: 13a + 18 13 − a 2 = 61 + 5 − a 2 + 3a 13 − a 2
⇔ 3 13 − a 2 ( 6 − a ) =−a 2 − 13a + 66 ⇒ 9 (13 − a 2 ) ( 6 − a ) = ( a 2 + 13a − 66 )
2 2

⇔ 9 (13 − a 2 )( a 2 − 12a + 36 ) =a 4 + 26a 3 + 37 a 2 − 1716a + 4356


0 ⇔ ( a − 2 )( a − 3) (10a 2 − 32a + 24 ) =
⇔ 10a 4 − 82a 3 + 244a 2 − 312a + 144 = 0
 
=  a 2= x 1
 
⇔  a =⇔ 3 x = −4 (thỏa mãn).
 6  89
= a = x
 5  25
 89 
Vậy phương trình có tập nghiệm = S 1; −4;  .
 25 
 x 4 + y 4 + 6 x 2 y 2 =1 (1)
Bài 2. Giải hệ phương trình:  .
 x ( x + y ) =− x y ( 2)
4

Lời giải
Cách 1. Ta có:
( x + y ) =+
4
x 4 4 x3 y + 6 x 2 y 2 + 4 xy 3 + y 4 =+ ( )
x 4 y 4 + 6 x 2 y 2 + 4 xy x 2 + y 2 = (
1 + 4 xy x 2 + y 2 )
Thay vào phương trình ( 2 ) , ta được:
x 1 + 4 xy ( x 2 + y 2 )  = x − y ⇔ x + 4 x 2 y ( x 2 + y 2 ) = x − y
y = 0
⇔ 4x2 y ( x2 + y 2 ) + y =0 ⇔ y  4 x 2 ( x 2 + y 2 ) + 1 =0⇔ 2 2
 4 x ( x + y ) + 1 =0
2
.

Tài liệu sưu tầm và tổng hợp bản word đầy đủ liên hệ 0393732038 TÀI LIỆU TOÁN HỌC
9
Website:tailieumontoan.com

( )
Vì 4 x 2 x 2 + y 2 + 1 > 0 với mọi x, y nên phương trình 4 x 2 x 2 + y 2 + 1 = ( )
0 vô nghiệm.
Thay y = 0 vào hệ phương trình đầu, ta được:
= x 4 1= x 1
 5 ⇔ x4 =1⇔  .
 x = x  x = −1
Vậy tập nghiệm của hệ phương trình
= là S {(1;0 ) , ( −1;0 )} .
( )
2
Cách 2. Ta có: (1) ⇔ x 2 + y 2 + 4 x2 y 2 =
1
 y4 = 1
Xét x = 0 , thay vào hệ phương trình ta được:  (vô lý).
y=0
Xét x ≠ 0 , chia cả hai vế của ( 2 ) cho x ta được:
y y
( x + y) =1 −⇔ [( x + y ) 2 ]2 =1 −
4

x x
y y
⇔ ( x 2 + 2 xy + y 2 ) 2 = 1 − ⇔ ( x 2 + y 2 ) 2 + 4 x 2 y 2 + 2.2 xy.( x 2 + y 2 ) =
1−
x x
y
⇔ 1 + 4 xy.( x 2 + y 2 ) = 1 − ⇔ 4 x 2 y.( x 2 + y 2 ) + y = 0 , ( do x ≠ 0 )
x
⇔ y  4 x 2 ( x 2 + y 2 ) + 1 = 0 ⇔ y = 0 ( vì 4 x 2 ( x 2 + y 2 ) + 1 > 0. )
Thay y = 0 vào hệ phương trình ban đầu ta được:
=
 x 1=
4
x 1
 5 ⇔ x4 =1⇔  (tm).

 x = x  x = −1
Vậy tập nghiệm của hệ phương trình
= là S {(1;0 ) ; ( −1;0 )} .
Cách 3. Từ hệ phương trình, ta có:
(
x ( x + y ) = ( x − y ) x4 + y 4 + 6x2 y 2
4
)
⇔ x5 + 4 x 4 y + 6 x3 y 2 + 4 x 2 y 3 + xy 4 =+
x 5 xy 4 + 6 x 3 y 2 − yx 4 − y 5 − 6 x 2 y 3
⇔ 5 x 4 y + 10 x 2 y 3 + y 5 =
0
⇔ y ( 5 x 4 + 10 x 2 y 2 + y 4 ) =
0 (*).
Nếu y ≠ 0 thì 5 x 4 + 10 x 2 y 2 + y 4 > 0 , ∀x ∈  nên (*) vô nghiệm.
Nếu y = 0 thì ta thay vào hệ phương trình ban đầu được:
= x 4 1=
 x 1
 5 ⇔ x4 =1⇔  .
x = x
  x = −1
Vậy tập nghiệm của hệ phương trình
= là S {(1;0 ) , ( −1;0 )} .
Bài 3. Tìm số nguyên dương n nhỏ nhất, biết rằng khi chia n cho 7, 9, 11, 13 được các số dư
tương ứng là 3, 4, 5, 6 .
Lời giải
Vì n chia 7 dư 3 nên 2n chia 7 dư 6 .
Vì n chia 9 dư 4 nên 2n chia 9 dư 8 .
Vì n chia 11 dư 5 nên 2n chia 11 dư 10 .
Vì n chia 13 dư 6 nên 2n chia 13 dư 12 .

Tài liệu sưu tầm và tổng hợp bản word đầy đủ liên hệ 0393732038 TÀI LIỆU TOÁN HỌC
10
Website:tailieumontoan.com
Suy ra ( 2n + 1) chia hết cho 7, 9, 11, 13 .
Mà n là số nguyên dương nhỏ nhất nên
2n + 1 BCNN ( 7, 9, 11,
= = 13) 7.9.11.13= ⇒ n 4504 .
Bài 4. Cho tam giác nhọn ABC có điểm P nằm trong tam giác ( P không nằm trên các cạnh).
Gọi J , K , L lần lượt là tâm đường tròn nội tiếp các tam giác PBC , PCA , PAB .
1. Chứng minh rằng BJC  + CKA+ ALB =450° .
2. Giả sử PB = PC và PC < PA . Gọi X , Y , Z lần lượt là hình chiếu vuông góc của
J , K , L trên các cạnh BC , CA, AB . Dựng hình bình hành XYWZ . Chứng minh W nằm
trên phân giác BAC.
Lời giải
A

K
Z
L

P C

X
B

 + CKA
1. Chứng minh rằng BJC + ALB =450° .
 1
 BJC= 90° + 2 BPC


Ta có: CKA

1
= 90° + CPA
2
 + CKA
; do đó BJC + = 270° + BPC
ALB
1   
2
(
+ CPA + APB= 450° . )
 1
 ALB= 90° + 2 APB

2. Giả sử PB = PC và PC < PA . Gọi X , Y , Z lần lượt là hình chiếu vuông góc của J , K , L trên
các cạnh BC , CA, AB . Dựng hình bình hành XYWZ . Chứng minh W nằm trên phân giác BAC .
Cách 1.

Tài liệu sưu tầm và tổng hợp bản word đầy đủ liên hệ 0393732038 TÀI LIỆU TOÁN HỌC
11
Website:tailieumontoan.com

P I C

Tính chất: Cho tam giác ABC có K là tâm đường tròn nội tiếp, các điểm tiếp xúc là I , J , Y như
AC + AP − PC
hình vẽ. Khi đó AY = .
2
Chứng minh:
Ta có: AY = AJ , PI = PJ , CY = CI .
AC + AP − PC AY + CY + AJ + PJ − ( PI + IC )
= Ta có: = AY .
2 2
Trở lại bài toán:
A

S
R
W

M
Y
E

Z L

P C

X
B

Gọi R, S lần lượt là hình chiếu vuông góc của W lên AB, AC . Ta sẽ chứng minh
WR = WS .
Thật vậy, gọi M là trung điểm của AC , E là hình chiếu vuông góc của Y lên MX .
Do tam giác BPC cân tại P nên X là trung điểm của BC , suy ra XM // AB .
Lại có: YE // WR, YX // WZ và YX = WZ nên ∆YEX = ∆WRZ (cạnh huyền – góc nhọn)
Từ đó ta có: WR= YE= MY sin YME 
= MY sin BAC =
Tài liệu sưu tầm và tổng hợp bản word đầy đủ liên hệ 0393732038 TÀI LIỆU TOÁN HỌC
12
Website:tailieumontoan.com

=  AC + AP − PC AC    AP − PC  sin BAC
 . (1)
=( AY − AM ) sin BAC  −  sin BAC =  
 2 2   2 
 AP − BP  
Chứng minh tương tự ta có: WS =   sin BAC . (2)
 2 
.
Do BP = CP nên từ (1) và (2) ta có: WR = WS . Vậy W nằm trên phân giác BAC
Cách 2.
A

E
F W

Z L

P C

X
B

Lấy E ∈ AC ; F ∈ AB sao cho BE // XY , CF // XZ .


Tam giác PBC cân tại P nên X là trung điểm của BC suy ra Y , Z là trung điểm của
CE , BF .
 XZ //CF
Ta có:  ⇒ YW //CF , suy ra W là trung điểm của EF .
 XZ //YW
 AE = AC − CE = AC − 2CY = AC − ( AC + CP − AP ) = AP − CP
Từ đây ta có:  .
 AF = AB − BF = AB − 2 BZ = AB − ( AB + BP − AP ) = AP − BP
Mà PB = PC suy ra AE = AF nên tam giác AEF cân tại A .
Mặt khác W là trung điểm EF suy ra W nằm trên đường phân giác của góc BAC .
Bài 5. Cho tập A = {1; 2; 3;...; 2021} . Tìm số nguyên dương k lớn nhất ( k > 2 ) sao cho ta có
thể chọn được k số phân biệt từ tập A mà tổng của hai số phân biệt bất kỳ trong k số
được chọn không chia hết cho hiệu của chúng.
Lời giải
Cách 1:
Gọi k số các số nguyên dương lớn nhất có thể chọn được là a1 ; a2 ;...; ak không mất tính
tổng quát ta giả sử a1 < a2 < ... < ak .
Dễ thấy ai +1 − ai > 1 ∀i ∈ {1; 2;...; k − 1}
Mặt khác nếu tồn tại i ∈ {1; 2;...; k − 1} sao cho ai +1 − ai =
2 khi đó ai và ai +1 cùng tính
chẵn lẻ nên ta có ai +1 + ai  ai +1 − ai (trái với giả thiết) từ đó suy ra ai +1 − ai ≥ 3
2023
⇒ ak ≥ ( k − 1) .3 + a1 ≥ 3k − 2 ta lại có ak ≤ 2021 ⇒ 3k − 2 ≤ 2021 ⇒ k ≤ ⇒ k ≤ 674
3
.

Tài liệu sưu tầm và tổng hợp bản word đầy đủ liên hệ 0393732038 TÀI LIỆU TOÁN HỌC
13
Website:tailieumontoan.com
Nhận xét nếu có hai số cùng chia cho 3 dư 2 thì tổng của chúng chia cho 3 dư 1, còn
hiệu của chúng chia hết 3. Nên tổng hai số này không chia hết cho hiệu của chúng. Các
số thuộc tập A có tất cả 674 số chia cho 3 dư 2.
Vậy giá trị lớn nhất của k là 674.
Cách 2:
Gọi B là tập con của tập A thỏa mãn hai phần tử bất kỳ của B có tổng không chia hết
cho hiệu.
Dễ thấy trong 3 số tự nhiên liên tiếp ta chỉ có thể chọn 1 phần tử vào B . Thật vậy với 3
số
x, x + 1, x + 2 nếu có 2 phần tử trong B thì
x + ( x + 2 ) = 2 x + 2 chia hết cho ( x + 2 ) − x =2
x + ( x + 1) = 2 x + 1 chia hết cho ( x + 1) − x =
1
( x + 1) + ( x + 2 ) = 2 x + 3 chia hết cho ( x + 2 ) − ( x + 1) =
1
Vậy với cách xây dựng tập B như vậy thì số phần tử của B không thể lớn hơn
 2021 
 3 + 1 = 674

Tập B = {1, 4, 7,..., 2020} có 674 phần tử thỏa mãn yêu cầu bài toán.
Vậy giá trị lớn nhất của k là 674.
Đề số 4

Câu 1.
1) Với a, b, c là các số thực thỏa mãn a + b + c ≠ 0 và (a + b)(b + c)(c + a ) =
1.
a b 1 + abc + ab(a + b + c)
Chứng minh rằng: + 2 =
a (a + b + c) + 1 + abc b (a + b + c) + 1 + abc
2
( a + b + c) 2
Lời giải
Ta có:
= a ( a 2 + ab + ac ) + abc + ( a + b )( b + c )( c + a )
a 2 ( a + b + c ) + 1 + abc
= a ( a 2 + ab + bc + ca ) + ( a + b )( b + c )( c + a ) = a ( a + b )( a + c ) + ( a + b )( b + c )( c + a )
= ( a + b + c )( a + b )( a + c )
Từ đó ta có
a a (b + c ) a (b + c )
= =
a ( a + b + c ) + 1 + abc
2
( a + b + c )( a + b )( a + b )( c + a )( b + c ) a + b + c
Biến đổi tương tự ta có
b b( a + c )
=
b (a + b + c) + 1 + abc a + b + c
2

Suy ra
a b a (b + c) + b(a + c)
+ 2 = (1)
a (a + b + c) + 1 + abc b (a + b + c) + 1 + abc
2
a+b+c
Mặt khác từ đẳng thức
(a + b)(b + c)(c + a ) + abc= ( ab + bc + ca )( a + b + c )
Tài liệu sưu tầm và tổng hợp bản word đầy đủ liên hệ 0393732038 TÀI LIỆU TOÁN HỌC
14
Website:tailieumontoan.com
Nên ta lại có:
1 + abc + ab(a + b + c) = (a + b)(b + c)(c + a ) + abc + ab(a + b + c)
= ( ab + bc + ca )( a + b + c ) + ab ( a + b + c ) = ( a + b + c )( 2ab + bc + ca )
= ( a + b + c )  a ( b + c ) + b ( a + c ) 
Từ đó ta có
1 + abc + ab(a + b + c) b(a + c) + a (b + c)
= (2)
(a + b + c) a+b+c
2

Từ (1) và (2) ta có
a b 1 + abc + ab(a + b + c)
+ 2 = .
a (a + b + c) + 1 + abc b (a + b + c) + 1 + abc
2
( a + b + c) 2
Suy ra điều phải chứng minh.

 x + 4 y + 4 xy + 2 x y =
2 2 2 2
11
Câu 2. Giải hệ phương trình: 
3 xy ( x + 2 y ) + 31 = 9 x + 18 y + 13 xy

Lời giải.
Cách 1:
( x + 2 y )2 + 2 x 2 y 2 =11
Hệ đã cho ⇔ 
3 xy ( x + 2 y ) + 31 = 9 ( x + 2 y ) + 13 xy

a 2 + 2b 2 =
11 
a + 2b =
2 2
11 (1)
Đặt a = xy ta được hệ mới: 
x + 2 y, b = ⇔
3ab + 31 = 9a + 13b 3a ( b − 3) = 13b − 31 (2)

Dễ thấy b = 3 không thỏa mãn phương trình (2).
Xét b ≠ 3 :
13b − 31
Từ phương trình (2) ta có a = , thay vào phương trình (1) ta được:
3 ( b − 3)
2
 13b − 31 
 + 2b =11 ⇔ (13b − 31) + 18b . ( b − 3) =99 ( b − 3)
2 2 2

2 2

 3 ( b − 3) 
 
b 1
=  a 3;=
= b 1
 
5 7 5
0 ⇔ b = ⇒  a = ; b = .
⇔ 18b 4 − 108b3 + 232b 2 − 212b + 70 =
 3  3 3
 7  1 7
=b =a = ;b
 3  3 3
 x = 1

3  x= 3 − 2 y
x + 2 y =  y = 1
Trường hợp 1:= b 1⇒
a 3;= ⇔ ⇔
1   x = 2
.
 xy = 1 ( 3 − 2 y ) y =
 y = 1
  2

Tài liệu sưu tầm và tổng hợp bản word đầy đủ liên hệ 0393732038 TÀI LIỆU TOÁN HỌC
15
Website:tailieumontoan.com

 7  7
 x + 2 y =  x= − 2y
7 5  3  3
Trường hợp 2:=a = ;b ⇒ ⇔ , hệ vô nghiệm.
3 3  xy = 5  7 − 2 y  y =
5
 3  3  3

 1  1
 x + 2 y =  x= − 2y
1 7  3  3
Trường hợp 3:=
a = ;b ⇒ ⇔ , hệ vô nghiệm.
3 3  xy = 7  1 − 2 y  y =
7
 3  3  3
 1
Vậy hệ có hai cặp nghiệm ( x; y ) = (1;1) , ( x; y ) =  2;  .
 2

 x + 4 y + 4 xy + 2 x y =
2 2 2 2
11 (*)
Cách 2: 
3 xy ( x + 2 y ) + 31 = 9 x + 18 y + 13 xy (**)

Xét phương trình (*) ta có: ( x + 2 y ) =−


2
11 2 x 2 y 2 .
Xét phương trình (**) ta có:
3 ( x + 2 y )( xy − 3)= 13 xy − 31 ⇒ 9 ( x + 2 y ) ( xy − 3) = (13xy − 31)
2 2 2
.

⇔ −2 ( 3 xy − 5 )( 3 xy − 7 )( xy − 1) =0 .
2

Ta có các trường hợp:


• xy = 9 ( x + 2 y ) + 13 ⇒ x =
1 ⇒ 3 ( x + 2 y ) + 31 = 3 − 2 y . Khi đó
y =1
( 3 − 2 y ) y =
1 ⇔ 
y = 1
 2
 x = 1

 y = 1
⇔   x = 2 . Thử lại ta thấy hai cặp nghiệm này thỏa mãn hệ phương trình.

 y = 1
  2
 7  7
 x + 2 y =  x= − 2y
5 7  3  3
• xy = ⇒ x + 2 y = ⇒ ⇔ , hệ vô nghiệm.
3 3  xy = 5  7 − 2 y  y =
5
 3  3  3
 1  1
x + 2 y =  x= − 2y
7 1  3  3
• xy = ⇒ x + 2 y = ⇒  ⇔ , hệ vô nghiệm.
3 3  xy = 7  1  7
 − 2 y  y =
 3  3  3
 1
Vậy hệ có hai cặp nghiệm ( x; y ) = (1;1) , ( x; y ) =  2;  .
 2
Câu 2. 1) Tìm x, y nguyên dương thỏa mãn 3x + 29 =
2y

Tài liệu sưu tầm và tổng hợp bản word đầy đủ liên hệ 0393732038 TÀI LIỆU TOÁN HỌC
16
Website:tailieumontoan.com
Lời giải.
Xét x = 1 thay vào phương trình ta được 31 + 29 = 2 y ⇔ y = 5
Xét x > 1 ta có 3x chia hết cho 9
⇒ 2 y ≡ 2 (mod 9)
Để ý:
• y = 6k thì ta được 2 y ≡ 1 (mod 9)
• y 6k + 1 thì ta được 2 y ≡ 2 (mod 9)
=
• y 6k + 2 thì ta được 2 y ≡ 4 (mod 9)
=
• y 6k + 3 thì ta được 2 y ≡ 8 (mod 9)
=
• y 6k + 4 thì ta được 2 y ≡ 7 (mod 9)
=
• y 6k + 5 thì ta được 2 y ≡ 5 (mod 9)
=
Từ đó ta thấy được y ≡ 1 (mod 6)
Với y ≡ 1 (mod 6) thì 2 y ≡ 2 (mod 7) ⇒ 3x ≡ 1 (mod 7)
Để ý:
• x = 6k thì ta được 3x ≡ 1 (mod 7)
• =x 6k + 1 thì ta được 3x ≡ 3 (mod 7)
• =x 6k + 2 thì ta được 3x ≡ 2 (mod 7)
• =x 6k + 3 thì ta được 3x ≡ 6 (mod 7)
• =x 6k + 4 thì ta được 3x ≡ 4 (mod 7)
• =x 6k + 5 thì ta được 3x ≡ 5 (mod 7)
Từ đó ta thấy được x ≡ 0 (mod 6) ⇒ 3x ≡ 1 (mod 4) ⇒ 2 y ≡ 2 (mod 4) ⇒ y =
1 (vô lý)
Vậy có duy nhất cặp ( x; y ) = (1;5 ) thỏa mãn đề bài.

2) Với a, b, c là các số thực dương thỏa mãn điều kiện 2 ( a + b + c ) + ab + bc + ca =


9.
a +1 b +1 c +1
Tìm giá trị lớn nhất của biểu thức : M = + 2 + 2 .
a + 10a + 21 b + 10b + 21 c + 10c + 21
2

Lời giải.
Ta có 2 ( a + b + c ) + ab + bc + ca =
9 ⇒ ( ab + a + b + 1) + ( bc + b + c + 1) + ( ca + c + a + 1) =
12

⇒ ( a + 1)( b + 1) + ( b + 1)( c + 1) + ( c + 1)( a + 1) =


12

Và a 2 + 10a + 21 = ( a + 1) + 8 ( a + 1) + 12 . Đặt a + 1 =x ; b + 1 =y ; c + 1 =z .
2

Khi đó: xy + yz + zx =
12 . Suy ra
x y z
M= + 2 + 2
x + 8 x + 12 y + 8 y + 12 z + 8 z + 12
2

x y z
= + +
( x + y )( x + z ) + 8 x ( y + x )( y + z ) + 8 y ( z + x )( z + y ) + 8 z
12 ⇒ ( x + y + z ) ≥ 3 ( xy + yz + zx ) = 36 ⇒ x + y + z ≥ 6 .
Với xy + yz + zx =
2

Tài liệu sưu tầm và tổng hợp bản word đầy đủ liên hệ 0393732038 TÀI LIỆU TOÁN HỌC
17
Website:tailieumontoan.com
1 1 4
Áp dụng BĐT : + ≥ ta có :
x y x+ y
1 x 1 y 1 z 1
M≤  + + + + + 
4  ( x + y )( x + z ) 8 ( y + x )( y + z ) 8 ( z + x )( z + y ) 8 
1  2 xy + 2 yz + 2 zx 3 1  24 3
≤ .  = +  .  + 
4  ( x + y )( y + z )( z + x ) 8  4  ( x + y )( y + z )( z + x ) 8 

Ta cũng có : 12 = xy + yz + zx ≥ 3 3 x 2 y 2 z 2 ⇒ 3 x 2 y 2 z 2 ≤ 4 ⇒ x 2 y 2 z 2 ≤ 64 ⇒ xyz ≤ 8
Suy ra: ( x + y )( y + z )( z + x ) = ( x + y + z )( xy + yz + zx ) − xyz ≥ 6.12 − 8 = 64

1  24 3  3
⇒ M ≤ .  +  =. Dấu đẳng thức xảy ra tại a= b= c= 1 .
4  64 8  16
3
Vậy M max = khi và chỉ khi a= b= c= 1 .
16
 nhọn có đường tròn nội tiếp ( O ) . Các điểm M , N lần lượt
Câu 3. Cho hình thoi ABCD có BAD
thuộc các cạnh CB, CD sao cho MN tiếp xúc (O) tại P và tam giác CMN không cân. MN
lần lượt cắt AB, AD tại E , F . Gọi K , L lần lượt là trực tâm các ∆BME , ∆DNF .
1. Chứng minh OP đi qua trung điểm I của KL .
OI EF 1
2. Gọi H là trực tâm tam giác CMN . Chứng minh − =
− .
CH 2 MN 2
3. Gọi EK , FL lần lượt cắt BD tại S , T . NS cắt MT tại Q . Đường tròn nội tiếp tam giác
CMN tiếp xúc MN với tại G . Chứng minh PQ song song với GH .
Lời giải.
1) Chứng minh OP đi qua trung điểm I của KL :
Ta có K , L lần lượt là trực tâm các ∆BME , ∆DNF và MN tiếp xúc (O) tại P
⇒ BK / / DL / / OP (cùng vuông góc với FE ).
Trong hình thang BDLK có đáy BD có O là trung điểm BD và OP / / BK ⇒ OP đi qua
trung điểm I của đoạn thẳng KL.

Tài liệu sưu tầm và tổng hợp bản word đầy đủ liên hệ 0393732038 TÀI LIỆU TOÁN HỌC
18
Website:tailieumontoan.com

B
E K

I
M
P D
H
N L
C

F
OI EF 1
2) Gọi H là trực tâm tam giác CMN . Chứng minh − =
− :
CH 2 MN 2
Ta có:
1
+) OI là đường trung bình hình thang BDLK ⇒ OI= ( BK + DL ) .
2
+) EF = EM + MN + NF
OI EF BK + DL ME + MN + NF 1  BK DL ME NF 
Khi đó − = − =  + − −1−  . ( ∗)
CH 2 MN 2CH 2 MN 2  CH CH MN MN 
Mặt khác:
BK MK MK ME BK ME
+) ∆MBK  ∆MCH ⇒ = ; ∆MKE  ∆MHN ⇒ = . Do đó = .
CH MH MH MN CH MN
DL NL NL NF DL NF
+) ∆NDL  ∆NCH ⇒ = ; ∆NLF  ∆NHM ⇒ = . Do đó = .
CH NH NH NM CH MN
OI EF 1 1 OI EF 1
Từ ( ∗ ): − = . ( −1) =− . Vậy − =
− .
CH 2 MN 2 2 CH 2 MN 2
3)

Tài liệu sưu tầm và tổng hợp bản word đầy đủ liên hệ 0393732038 TÀI LIỆU TOÁN HỌC
19
Website:tailieumontoan.com
A

O Y1
S
T

Q D
K
I

L
G
E M P N F X

Gọi Y1 là giao điểm của TF và QP, Y2 là giao điểm của SE và QP.


Áp dụng định lý Menelaus cho ∆QMP có T ∈ QM , Y1 ∈ QP , F ∈ MP thẳng hàng:
TQ FM Y1 P Y1 P TM FP
⋅ ⋅ 1⇒ =
= ⋅ (1)
TM FP Y1Q Y1Q TQ FM
Tương tự cho ∆PQN ( S ∈ QN , Y2 ∈ QP , E ∈ PN thẳng hàng):
SQ EN Y2 P Y P SN EP
⋅ ⋅ 1 ⇒ 2=
= ⋅ (2)
SN EP Y2Q Y2Q SQ EN
Y1 P Y2 P TM FP SN EP
Để chứng minh Y1 ≡ Y2 ta cần = hay ⋅ = ⋅ ⇔
Y1Q Y2Q TQ FM SQ EN
TM SQ EP FM
⋅ = ⋅ (**)
TQ SN EN FP
Áp dụng định lý Menelaus cho ∆QMN có T ∈ QM , S ∈ QN , X ∈ MN (=
X BD ∩ MN ) và
cho ∆CMN có B ∈ CM , D ∈ CN , X ∈ MN :
TM SQ XN TM SQ XM
⋅ ⋅ 1⇒
= ⋅ = ;
TQ SN XM TQ SN XN
BM DC XN BM DC XM
⋅ ⋅ 1⇒
= ⋅ =
BC DN XM BC DN XN
TM SQ BM DC BM
Suy ra: ⋅ = ⋅ = (vì DC = BC). (***)
TQ SN BC DN DN

Tài liệu sưu tầm và tổng hợp bản word đầy đủ liên hệ 0393732038 TÀI LIỆU TOÁN HỌC
20
Website:tailieumontoan.com
EP EP.PN
Ta có =
EN EN .PN

Do EO là tia phân giác của  =1


AEN ⇒ OEN AEN
2
 ⇒ ONE
NO là tia phân giác của DNE  = 1 DNE

2

⇒ OEN = 1 
 + ONE
2
(
 = 900 (do AE // DN)
AEN + DNE )
EP OP 2
⇒ OE ⊥ ON ⇒ EP.PN = OP 2 và EN .PN = ON 2 ⇒ = .
EN ON 2
FM OM 2 EP FM OM 2
Chứng minh tương tự, ∆OFM vuông tại O: = . Suy ra ⋅ = (3)
FP OP 2 EN FP ON 2

= 90° − BCD= MBO
Xét ∆CNM có O là tâm đường tròn bàng tiếp ∆CNM ⇒ MON 
2
 nên ∆BMO  ∆OMN (g.g) ( MON
Mà MO là phân giác của BMN  , NMO
 = MBO  = BMO
)
Tương tự ∆DON  ∆OMN (g.g).
BM BO OM BM BO OM 2 BM OM 2
⇒ ∆BMO  ∆DON ⇒ = = ⇒ ⋅ = ⇒ = (do DO =
DO DN ON DO DN ON 2 DN ON 2
BO) (4)
BM EP FM
Từ (3) và (4) ⇒ = ⋅ (****).
DN EN FP
Từ (***) và (****) ⇒ (**) ⇒ Y1 ≡ Y2 ⇒ PQ, TF , SE đồng quy.
Ta có TF ⊥ AE (do AE // ND) và SE ⊥ AF (do AF / / BM ) ⇒ Y1 là trực tâm của ∆AEF .
Mà ∆AEF và ∆CNM có các cạnh tương ứng song song
⇒ HG // PY1 vì vai trò của HG và PY1 là như nhau ⇒ HG // PQ (đpcm).
a1 a a
Câu 4. Giả sử a1 , a2 ,....., a2021 là những số thực thỏa mãn + 2 2 + ...... + 2021 =
0.
1 + a1 1 + a2
2
1 + a2021
2

Chứng minh rằng tồn tại số nguyên k (1 ≤ k ≤ 2021) sao cho:

a1 2a2 kak 2k + 1
+ + ...... + ≤
1 + a1 1 + a2
2 2
1 + ak
2
8
Lời giải
ai
Đặt xi = ; i = 1,..., 2021 .
1 + ai2

1 (1 + ai )
2
1 ai 1
Xét xi += += ≥ 0 ⇒ xi ≥ − .
2 1 + ai 2 2 (1 + ai )
2 2
2

Tài liệu sưu tầm và tổng hợp bản word đầy đủ liên hệ 0393732038 TÀI LIỆU TOÁN HỌC
21
Website:tailieumontoan.com

1 − (1 − ai )
2
1 ai 1 1 1
Xét xi −= −= ≤ 0 ⇒ xi ≤ . Suy ra − ≤ xi ≤ ; i = 1,..., 2021 .
2 1 + ai 2 2 (1 + ai )
2 2
2 2 2

a1 a a
Vì + 2 2 + ...... + 2021 =0 ⇒ x1 + x2 + .... + x2021 =0
1 + a1 1 + a2
2
1 + a2021
2

Giả sử không tồn tại số nguyên k ;1 ≤ k ≤ 2021 sao cho


a1 2a2 kak 2k + 1
+ + ...... + ≤
1 + a1 1 + a2
2 2
1 + ak
2
8

a1 2a2 kak 2k + 1
Nghĩa là với mọi k ;1 ≤ k ≤ 2021 ta luôn có + + ...... + > .
1 + a1 1 + a2
2 2
1 + ak
2
8

Đặt gi = ∑ i.xi ; i = 1,..., 2021

Nhận thấy g1 ; g 2 ;.....; g 2021 luôn đồng dấu.


Thật vậy, giả sử nếu có 2 số gi −1 , gi (1 < i ≤ 2021) trái dấu.

2i + 1 2 ( i − 1) + 1 i 1
Do đó i.xi = gi − gi −1 = gi + gi −1 > + = ⇒ xi > (mâu thuẫn)
8 8 2 2
Nghĩa là g1 ; g 2 ;.....; g 2021 luôn đồng dấu
2021
gi
Lại có 0 = x1 + x2 + .... + x2021 = ∑ ≠ 0 (mâu thuẫn)
i =1 i
Vậy ta có điều phải chứng minh.

Đề số 7

Bài 1.

5
a, Điều kiện x ≥ − đặt a = 26 x + 5 và b = x 2 + 30 (a ≥ 0, b > 0)
26

Phương trình trở thành

a2
+ 2a = 3b ⇔ ( a − b )( a + 3b ) = 0 ⇔ a = b ⇔ 26 x + 5 = x 2 + 30
b

Vậy x = 1 hoặc 25 là nghiệm của phương trình.


Tài liệu sưu tầm và tổng hợp bản word đầy đủ liên hệ 0393732038 TÀI LIỆU TOÁN HỌC
22
Website:tailieumontoan.com
b, thay 2 = x2 + y2 vào phương trình thứ 2 ta được

(x + 2y)(x2 + y2 + 3y2 + 4xy) = 27

⇔ ( x + 2 y ) = 27
3

⇔ x = 3 - 2y thay vào PT thứ nhất ta được (3 - 2y)2 + y2 = 0

7
y = 1 hoặc y =
5

1
x = 1 hoặc
5

Bài 2.

a, từ biểu thức ( x 2 − x + 1)( y 2 + xy ) = 3 x − 1 ta nhận thấy 3x - 1 phải chia hết cho (x2 – x + 1)

ta có (3x - 1)(3x - 2) = 9x2 - 9x + 2 = 9(x2 – x + 1) - 7 cũng phải chia hết cho (x2 – x + 1)

suy ra 7 chia hết cho (x2 – x + 1)

(x2 – x + 1) = 1 hoặc 7

y = 0, 1, 3 và -2 lần lượt thay vào ta có y => (x,y) = (1, 1),(1, -2) và (-2, 1)

b, Từ giả thiết xy + 2 ≥ 2y => 4xy + 8 ≥ 8y

Mà ta lại có 4x2 + y2 ≥ 4xy

⇒ 4x2 + y2 + 8 ≥ 4xy + 8 ≥ 8y

⇒ 4(x2 + 4) ≥ 8y + 8 - y2

⇒ 4(x2 + 4) ≥ 4(y2 + 1) + (5y + 2)(2 - y) ≥ 4(y2 + 1)

x2 + 4
⇒M = ≥1
y2 + 1

Dấu “=” xẩy ra khi x = 1 và y = 2, Mmin = 1.

Bài 3.

a, Do đường tron (O) nội tiếp hình vuông ABCD nên E


và F là trung điểm các cạnh AB và AD => ∆ ABF và ∆
BCE bằng nhau => góc EBG bằng góc BCG => góc BGC
vông => AEGF cùng nằm trên một đường tròn, mà

Tài liệu sưu tầm và tổng hợp bản word đầy đủ liên hệ 0393732038 TÀI LIỆU TOÁN HỌC
23
Website:tailieumontoan.com
AEOF cũng nằm trên một đường tròn => AEGOF cùng nằm trên một đường tròn.

b, Ta có AB là tiếp tuyến của đường tròn (O) nên

góc BEM = góc EFM,

lại có góc EAG và EFG cùng chắn cung EG nên góc EAG = EFG

suy ra EM//AG trong khi E là trung điểm của AB => M cũng là trung điểm của BG
Bài 4.

3
1 1 1 2 x y z 
+ + ≥  + + 
1+ x 1+ y 1+ z
2 2 2
3  1 + x2 1 + y2 1 + z2 
 

Ta có:

1 +x 2 = xy + yz + xz + x2 = (x + y)(x + z)

1 + y2 = xy + yz + xz + y2 = (x + y)(y + z)

1 + z2 = xy + yz + xz + z2 = (z + y)(x + z)

2( x + y + z )
VT=
( x + y )( y + z )( z + x)

Ta có:

2
 x y z   x y z 
 + +  ≤ ( x + y + z)  + + 2 
 1 + x2 1 + y2 1 + z2  1+ x 1+ y 1+ z 
2 2
 

 x y z 
= ( x + y + z)  + + 
 ( x + y )( x + z ) ( x + y )( y + z ) ( z + y )( x + z ) 

2( x + y + z )
=
( x + y )( y + z )( z + x)

Do đó

4( x + y + z )  x y z 
VP=  + + 
3( x + y )( y + z )( z + x)  1 + x 2 1 + y2 1 + z2 
 

Bất đẳng thức trở thành

x y z 3
+ + ≤
1+ x 2
1+ y 2
1+ z 2 2

Tài liệu sưu tầm và tổng hợp bản word đầy đủ liên hệ 0393732038 TÀI LIỆU TOÁN HỌC
24
Website:tailieumontoan.com
Ta có:

x x 1 x x 
= ≤  + 
1 + x2 ( x + y )( x + z ) 2 x+ y x+ z 

y y 1 y y 
= ≤  + 
1 + y2 ( x + y )( y + z ) 2 x+ y y + z 

z z 1 z z 
= ≤  + 
1 + z2 ( x + z )( y + z ) 2 x+ z y + z 

x y z 3
=> + + ≤
1+ x 2
1+ y 2
1+ z 2 2

1
Dấu “=” xảy ra khi x = y = z =
3

Đề số 2

Câu 1.

a. Ta có
 3 x 2 + y 2 + 4 xy = 8  ( x + y )( 3 x + y ) = 8
 ⇔  .
( x + y ) ( x + xy + 2 ) =
2
8 ( x + y ) ( x 2 + xy + 2 ) =8

Do phương trình thứ nhất nên x + y ≠ 0 do đó ta kết hợp hai phương trình
lại ta có
 x =1
x 2 + xy + 2 = 3 x + y ⇔ ( x − 1)( x + y − 2 ) = 0 ⇔  .
 x= 2 − y
 y =1
TH1: x =1 ⇒ 3 + y 2 + 4 y = 8 ⇔  .
 y = −5
TH2: x= 2 − y thay vào phương trình thứ nhất ta có −4 ( y − 1) = 0 ⇔ y =1 .
Vậy hệ đã cho có hai cặp nghiệm ( x; y ) là (1;1) ; (1; −5 ) .
5
b. ĐK: x ≤ ; x 2 + x ≤ 5 .
2
Đặt a = 5 − x 2 + x và b =( )
5 − 2 x  ( a, b ≥ 0 ) . Ta có

32 − a 2 32 − b 2
= . (1)
2+a 2+b

Tài liệu sưu tầm và tổng hợp bản word đầy đủ liên hệ 0393732038 TÀI LIỆU TOÁN HỌC
25
Website:tailieumontoan.com
1 1
Ta thấy, nếu a > b ≥ 0 thì <
32 − a 2 < 32 − b 2 và
tức là VT<VP,
a+2 2+b
mâu thuẫn. Tương tự với a < b cũng mẫu thuẫn. Do đó a = b , tức là phương
trình ban đầu tương đương với
( )
 x = 1 TM
5 − ( x 2 + x ) =5 − 2 x ⇔  .
( )
 x = 0 TM
Vậy phương trình có hai nghiệm = =
x 1, 0
x .
Câu 2.

a. Trước hết ta chứng minh rằng x 7 ≡ x  ( mod 42 ) , ∀x ∈  .


(1)
Thật vậy, ta có x 7 − x= x ( x − 1)( x + 1) ( x 4 + x 2 + 1) .
Dễ thầy x ( x − 1)( x + 1) là tích 3 số nguyên liên tiếp nên nó chia hết cho 6.
Theo định lí Ơle thì x 7 − x ≡ 0 ( mod 7 ) , ∀x ∈  , tức là x 7 − x chia hết cho 7 .
Vậy x 7 − x chia hết cho BCNN ( 6;7 ) = 42 . Khẳng định (1) được chứng minh.
Từ đó
7 7 7
( 27 n + 5 )7 + 10  + (10n + 27 )7 + 5 + ( 5n + 10 )7 + 27 
     
≡ ( 27 n + 5 ) + 10 + (10n + 27 ) + 5 + ( 5n + 10 ) + 27 ( mod 42 )
7 7 7

≡ 27 n + 5 + 10 + 10n + 27 + 5 + 5n + 10 + 27 ( mod 42 )
≡ 42 ( n + 1)( mod 42 )
≡ 0 ( mod 42 ) .
Từ đó ta có khẳng định của bài toán.
( x + y)
2
a2
b. Đặt a= x + y . Sử dụng bất đẳng thức AM-GM, ta có xy ≤ =
4 4
2
5 
Hay  a + 1 ≥ 2 .
2 

Từ đó, ta có a ≥
2
5
(
2 − 1 . Suy ra )
9
( )
2
P=
17 x 2 + 17 y 2 + 16 xy =
17 a 2 − 18 xy ≥ 17 a 2 − a 2 ≥ 2 2 −1 =
6−4 2 .
2
2 −1
Dấu “=” xảy ra khi và chỉ khi x= y= .
5
Câu 3.

a. Sử dụng định lí Talet trong tam giác LKE với JH  EK , ta có


LH LJ
= .
HK JE
Tài liệu sưu tầm và tổng hợp bản word đầy đủ liên hệ 0393732038 TÀI LIỆU TOÁN HỌC
26
Website:tailieumontoan.com
Sử dụng định lí Talet trong tam giác JHE với FL  EK , ta cùng có
FL LJ
= .
EK JE
Do đó
FL LH
= .
EK HK
FL LH
Hai tam giác FLH và EKH có ∠FLH = 900 và
∠EKH = = nên
EK HK
∆FLH ∽ ∆EKH ⇒ ∠LFH = ∠KEH . Mặt khác, ta lại có ∠LFH =∠FHJ (so le
trong) và ∠KEH =
∠EHJ (so le trong). Do đó HJ là phân giác của góc EHF .
b. Go HJ  FL nên S FJL = S FLH . Suy ra S BFJL = S BFL + S FLH = S BFH . (1)
Chứng minh tương tự, ta cùng có SCEJK = SCEH .
(2)
Theo chứng minh câu a, hai tam giác FTL và EKH đồng dạng nên
∠FHB =
∠FHL = ∠EHC .
∠EHK =
Hai tam giác FHB và EHC có ∠FBH =
∠ECH và ∠FHB =
∠ECH nên đông
S FBH BF 2
dạng với nhau. Suy ra = .
S ECH CE 2
Ta kết hợp (1) và (2), ta thu được
S1 S FBH BF 2
= = .
S 2 S ECH CE 2
Điều phải chứng minh.

c. Không mất tính tổng quát, ta có thể giả sử P nằm cùng phíc với B so với
AD như hình vẽ ở trên. Gọi M là giao điểm của PJ và EK . Áp dụng định lí
Menelaus cho tam giác KFE với cát tuyến MJP , ta có

Tài liệu sưu tầm và tổng hợp bản word đầy đủ liên hệ 0393732038 TÀI LIỆU TOÁN HỌC
27
Website:tailieumontoan.com
MK PE JF
. . = 1.
ME PF JK
Mà hai tam giác  BFH và CEH đồng dạng với nhau có FL và EK là hai
JF FL BF
đường cao tương ứng nên = = .
JK EK CE
MK PE BF
Suy ra . . =1.
ME PF CE
(3)
J D thẳng hàng, ta chỉ cần chứng minh M , , 
Để chứng minh ba điểm P, ,  DJ
thẳng hàng. Theo định lí Menelaus đảo áp dụng cho tam giác LKE , điều này
tương đương với ta phải chứng minh
MK JE DL
. . = 1.
ME JL DK
JE EK CE DL DL DC DB AF AC AF
Lại có = = = và  .= . = . .1 .
JL FL BF DK DB DK DC AB AE AE
MK CE AF
Do đó, chỉ cần chứng minh . . =1
ME BF AE
(4)
CE AF PE BF
Kết hợp (3) và (4) , ta đưa bài toán về chứng minh . = . .
BF AE PF CE
AF PF BF 2
Hay . = .
AE PE CE 2
(5)
Gọi T, N lần lượt là tiếp điểm của đường tròn (I) với AB, AC . Đặt
= = AC=
a AB ,  = CD=
x BD , , = TF=
y PF = EN . Ta sẽ chứng minh
z PE
x ( x + y )( x + z )
a= (6)
x 2 − yz
Thật vậy, sử dụng định lí cosin trong các tam giác ABC và AEF , ta có
AE 2 + AF 2 − EF 2 AB 2 + AC 2 − BC 2
=2 cos A = .
AE. AF AB. AC
Suy ra
(a − x − y) + (a − x − z) − ( y + z)
2 2 2
2a 2 − 4 x 2
= ,
( a − x − y )( a − x − z ) a2
Hay
(a − x − y) + (a − x − z) − ( y + z)
2 2 2
2a 2 − 4 x 2
2− =
2− .
( a − x − y )( a − x − z ) a2
Từ đây, ta có
yz x2
= ,
a 2 − ( 2 x + y + z ) a + ( x + y )( x + z ) a 2
Tài liệu sưu tầm và tổng hợp bản word đầy đủ liên hệ 0393732038 TÀI LIỆU TOÁN HỌC
28
Website:tailieumontoan.com
Hay
(x 2
− yz ) a 2 − x 2 ( 2 x + y + z ) a + x 2 ( x + y )( x + z ) =0.

Như thế, ta có ( a − x ) ( x 2 − yz ) a − x ( x + y )( x + z )  =


0.
Do a > x nên (6) được chứng minh, Sử dụng (6) vừa chứng minh ta có
x ( x + y )( x + z )
−x− y
(=x + y)
2
AF PF a − x − y y x 2 − yz y BF 2
= . = . = . .
AE PE a − x − z z x ( x + y )( x − z ) z ( x + z)
2
CE 2
−x−z
x − yz
2

Đẳng thức (5) được chứng minh. Ta có điều phải chứng minh.
Bài 4. Đặt M n = { x|x ∈ ,x ≤ 2n − 1} . Ta chứng minh mệnh đề tổng quát: Trong 2n + 1 số
phân biệt từ tập hợp M n , luôn tồn tại ba số phân biệt có tổng bẳng 0 . Ta chứng minh bằng
phương pháp phản chứng. Giả sử tồn tại số nguyên dương n sao cho thể chọn ra 2n + 1 số
phân biệt từ tập hợp M n mà trong đó không có ba số phân biệt nào có tổng bằng 0 . Gọi n
là số nhỏ nhất có tính chất như vây. Khi đó n > 1 ( vì với n = 1 thì mệnh đề đúng). Vì n là
số nhỏ nhất làm cho mệnh đề không đúng nên mệnh đề đúng với n − 1 . Nếu trong các số
được chọn có ít nhất 2n − 1 số thuộc M n −1 thì do mệnh đề đúng với n − 1 , sẽ tồn tại ba số
phân biệt trong các số được chọn có tổng bằng 0 . Mẫu thuẫn. Vậy có tối đa 2n − 2 số được
chọn thuộc M n −1 . Suy ra trong bốn số −2n + 2, −2n + 1, 2n − 2, 2n − 1, có ít nhất ba số được
chọn. Suy ra 0 không được chọn.

• Nếu cả hai số của cặp ( −2n + 1, 2n − 1) được chọn. Chia tập


M n \ {−2n + 1, 2n − 1, 0} thành 2n − 2 cặp
(1; 2n − 2 ) , 2;
( 2n − 3) ,…, ( −1; −2n + 2 ) ,…, ( −n + 1, −n ) ta thấy từ mỗi cặp ta chỉ
chọn được tối đa một số. Suy ra chỉ lấy được tối đa 2 + 2n − 2 = 2n số. Mẫu
thuẫn.
• Nếu chỉ có một số của cặp ( −2n + 1, 2n − 1) được chọn thì theo lí luận ở trên,
cặp ( −2n + 2, 2n − 2 ) được chọn. Không mất tính tổng quát ta giả sử 2n − 1
được chọn còn 1 − 2n không được chọn. Lúc này chia các phần tử còn lại
thành 2n − 5 cặp
(1; 2n − 3) , 2;
( 2n − 4 ) ,…, ( n − 2; n ) , (−2; −2n + 3,…, ( −n + 3; −n − 1) , một bộ ba số
( −n + 2, −n + 1, −n ) và một phần tử lẻ cặp là n − 1 . Từ mỗi cặp ta lấy được tối
đa một số, từ bộ ba số ta cũng lấy được tối đa một số. Từ đó ta lấy được
tối đa 3 + 2n − 5 + 1 + 1 =2n số. Mẫu thuẫn.
Vậy trong mọi trường hợp đều dẫn đến mẫu thuẩn, tức điều giả sử sai. Mệnh đề
được chứng minh. Áp dụng mệnh đề cho n = 1010 ta có điều phải chứng minh.

Tài liệu sưu tầm và tổng hợp bản word đầy đủ liên hệ 0393732038 TÀI LIỆU TOÁN HỌC
29
Website:tailieumontoan.com

Đề số 3

Bài 1.

a) Giải phương trình x 2 − x + 2 x 3 + 1= 2 x + 1 .

Điều kiện xác định của phương trình là x ≥ −1 . Để ý rằng x 2 − x + 1 > 0 .

Đặt a = x + 1; b = x 2 − x + 1 ( a ≥ 0; b > 0 ) . Khi đó phương trình đã cho được viết lại thành

b 2 − 1 + 2ab= 2a ⇔ ( b − 1)( b + 1 + 2a )= 0

Do a ≥ 0; b > 0 nên ta có b + 1 + 2a > 0 . Khi đó từ phương trình trên ta được b = 1 .

Do đó ta có phương trình x 2 − x + 1 = 1 ⇔ x 2 − x = 0 ⇔ x ∈ {0;1} .

Kết hợp với điều kiện xác định ta được tập nghiệm của phương trình là S = {0;1} .

 2
xy + y =1 + y
b) Giải hệ phương trình  2 2
.

 x + 2y + 2xy 4
=+ x
2xy + 2y 2 =+
 2 2y
Hệ phương trình đã cho được viết lại thành  2 2
.

 x + 2y + 2xy 4
=+ x

Cộng theo vế hai phương trình của hệ phương trình trên ra thu được

x 2 + 4y 2 + 4xy =6 + x + 2y ⇔ ( x + 2y ) =x + 2y + 6 ⇔ ( x + 2y ) − ( x + 2y ) − 6 =0
2 2

 x + 2y − 3 = 0  x = 3 − 2y
⇔ ( x + 2y − 3 )( x + 2y + 2 ) =0 ⇔  ⇔
 x + 2y + 2 =0  x =−2y − 2

+ Thế x= 3 − 2y vào phương trình thứ nhất của hệ đã cho ta được

y ( 3 − 2y ) + y 2 = y + 1 ⇔ y 2 − 2y + 1 = 0 ⇔ ( y − 1) = 0 ⇔ y = 1
2

Từ đó tương ứng ta được x = 1 .

+ Thế x =
−2y − 2 vào phương trình thứ nhất của hệ đã cho ta được

 −3 − 5 −3 + 5 
y ( −2y − 2 ) + y 2 = y + 1 ⇔ y 2 + 3y + 1 = 0 ⇔ y ∈  ; 
 2 2 

−3 − 5 −3 + 5
Từ đó với y = ta được x= 1 + 5 và với y = ta được x= 1 − 5
2 2

   
(1;1) ,  1 + 5; −3 −2 5  ,  1 − 5; −3 +2 5  .
Vậy hệ phương trình có các nghiệm ( x; y ) =
   

Bài 2.
Tài liệu sưu tầm và tổng hợp bản word đầy đủ liên hệ 0393732038 TÀI LIỆU TOÁN HỌC
30
Website:tailieumontoan.com

a) Tìm tất cả các cặp số nguyên ( x; y ) thỏa mãn ( x + y )( 3x + 2y ) = 2x + y − 1 .


2

Để ý rằng 2x + y= ( 3x + 2y ) − ( x + y ) nên phương trình đã cho được viết lại thành


( x + y )( 3x + 2y ) = ( 3x + 2y ) − ( x + y ) − 1
2

Đặt a =+
x y; b =
3x + 2y . Khi đó ta có ab 2 = b − a − 1 hay a b 2 + 1 =b − 1 . ( )
Từ đó suy ra b − 1 chia hết cho b 2 + 1 . Do đó ta được b 2 + 1 − ( b − 1)( b + 1) chia hết cho

b 2 + 1 hay 2 chia hết cho b 2 + 1 . Suy ra b 2 + 1 ∈ {1; 2} nên b ∈ {−1; 0;1} .

+ Với b = −1 ta được a = −1 , khi đó ta được ( x; y=


) (1; −2 ) .
+ Với b = 0 ta được a = −1 , khi đó ta được ( x; y=
) ( 2; −3 ) .
+ Với b = 1 ta được a = 0 , khi đó ta được ( x; y=
) (1; −1) .
(1; −2 ) , (1; −2 ) , ( 2; −3 ) thỏa mãn yêu cầu bài toán.
Vậy các cặp số nguyên ( x; y ) =

b
b) Với a, b là các số thực dương thay đổi thỏa mãn điều kiện a + 2b =2 + . Tìm giá trị
3
a b
nhỏ nhất của biểu thức
= M . +
a + 2b b + 2a
+ Lời giải 1. Ta sẽ chứng minh M ≥ 2 với dấu đẳng thức xẩy ra chẳng hạn khi a= b= 3 .

Thật vậy, áp dụng bất đẳng thức AM – GM ta có

b b 3b 2b 3b b 3b
= ≥ =
b + 2a 3b ( b + 2a ) 3b + b + 2a a + 2b

a b 3b
Như vậy ta cần chỉ ra được + ≥ 2.
a + 2b a + 2b

b
Đặt x = a + 2b; y =
3
( x ≥ 0; y ≥ 0 ) . Khi đó giả thiết được viết lại thành x − y =2.

b 3y 2 ; =
Cũng từ trên ta có = a x 2 − 6y . Bất đẳng thức cần chứng minh trên được viết lại

x 2 − 6y 2 9y 3 x 2 − 6y 2 9y 3
thành + 2 ≥2⇔ + 2 ≥ x−y.
x x x x
Biến đổi tương đương bất đẳng thức trên ta được

( )
x x 2 − 6y 2 + 9y 3 ≥ x 2 ( x − y ) ⇔ x 3 − 6xy 2 + 9y 3 ≥ x 3 − x 2 y

( )
⇔ 9y 3 − 6xy 2 + x 2 y ≥ 0 ⇔ y 9y 2 − 6xy + x 2 ≥ 0 ⇔ y ( 3y − x ) ≥ 0
2

Tài liệu sưu tầm và tổng hợp bản word đầy đủ liên hệ 0393732038 TÀI LIỆU TOÁN HỌC
31
Website:tailieumontoan.com
Bất đẳng thức cuối cùng trên luôn đúng. Vậy bài toán được giải quyết hoàn toàn.

b a b b
+ Lời giải 2. Xét biểu thức M + = + + .
3 a + 2b b + 2a 3

Áp dụng bất đẳng thức Cauchy – Schwarz dạng phân thức ta có

(a + b + b)
2
b a2 b2 b2
M += + + ≥
3 a a + 2b b b + 2a b 3b a a + 2b + b b + 2a + b 3b

Áp dụng tiếp bất đẳng thức Cauchy – Schwarz ta lại có

b b + 2a + b 3b= b ( b + 2a + 3b ≤ b ) (1 + 1)( b + 2a + 3b=) 2b a + 2b

(a + b + b) ( a + 2b )
2 2
a + 2b
Từ đó ≥ = a + 2b .
=
a a + 2b + b b + 2a + b 3b a a + 2b + 2b a + 2b a + 2b

b b
Suy ra M + ≥ a + 2b nên M ≥ a + 2b − 2 . Vậy bài toán được giải quyết hoàn
=
3 3

toàn.

Bài 3.

a) Chứng minh hai tam giác BKM và DEF đồng dạng với nhau.

Đường tròn ( I ) nội tiếp tam giác ABC nên A

ta có BD và BF là các tiếp tuyến. Do đó BI là

đường trung trực của đoạn thẳng DF nên BI


E
vuông góc với DF tại M. Từ đó BMDK nội F
I

tiếp đường tròn, do đó BMK 
= BDK .
= CDE M
N

X
Cũng do CE là tiếp tuyến với đường tròn
C
 . Từ đó suy ra
 = DFE
( I ) tại E nên ta có CDE
B D

K
 = DFE
BMK 
 . Mặt khác BKM 
= BDM 
= DEF J
L
nên hai tam giác BKM và DEF đồng dạng.

b) Chứng minh hai đường thẳng MK và NL song song với nhau.


 = DBK
Ta có các tứ giác BKMD và CLDN nội tiếp đường tròn nên suy ra DMK  và

 = DLN
DCN  . Mặt khác do BK song song với CN nên ta có DBK
 = DCN
 . Từ đó suy ra

 = DLN
DMK  nên MK song song với LN.

Tài liệu sưu tầm và tổng hợp bản word đầy đủ liên hệ 0393732038 TÀI LIỆU TOÁN HỌC
32
Website:tailieumontoan.com
c) Chứng minh đường thẳng JX vuông góc với đường thẳng EF.
 =DCN
Ta có DMK  =90 0 − CDN
 =90 0 − DFE
 =90 0 − DMN
 , do đó KMN
 = 90 0 . Do vậy tứ

giác KMNL là hình thang vuông. Ta có J là trung điểm của KL nên J nằm trên đường trung

1
trực của đoạn thẳng MN hay JM = JN . Mặt khác XM
= XN
= ID nên suy ra X nằm trên
2
đường trung trực của MN. Do đó XJ vuông góc với MN. Trong tam giác DEF thì MN là

đường trung bình nên ta có MN song song với EF. Do đó suy ra JX vuông góc với EF

Bài 4.

Gọi m, n theo thứ tự là số đường thẳng đi qua P và Q. Gọi S số miền được tạo thành. Do

mỗi đường thẳng chỉ đi qua điểm P hặc điểm Q nên ta có m + n =


10 . Ta xét các trường
hợp sau.

+ Trường hợp 1. Nếu m = 0 hoặc n = 0 , chẳng hạn m = 0 thì tất cả 10 đường thẳng đã cho

cùng đồng quy tại P. Khi đó dễ thấy số miền được tạo ra trên mặt phẳng là 20. Do đó ta có

S = 20 .
+ Trường hợp 2. Nếu m > 0 và n > 0 , khi đó m ≥ 1 và n ≥ 1 . Từ mặt phẳng đã cho với hai

điểm P và Q ta vẽ thêm m đường thẳng đi qua điểm P, số miền được tạo thành là 2m.

Lần lượt vẽ thêm các đường thẳng đi qua điểm Q. Khi vẽ đường thẳng đầu tiên thì

đường thẳng này cắt m đường thẳng đi qua P tại m điểm phân biệt, m điểm phân biệt này

chia đường thẳng vừa vẽ thành m + 1 phần. Nói cách khác thì đường thẳng vừa vẽ đi qua

(vì thế chia đôi) đúng m + 1 miền trong 2m miền được tạo ra. Do đó lúc này số miền được

tạo ra là 2m + ( m + 1) .

Kể từ đường thẳng thứ hai đến đường thẳng thứ n đi qua điểm Q thì mỗi đường sẽ

cắt m đường thẳng phân biệt đi qua điểm P tại m điểm phân biệt khác Q. Các điểm phân

biệt đó cùng với điểm Q chia đường thẳng vừa vẽ thành m + 2 phần. Do đó mối lần vẽ

đường thẳng thì số miền tăng thêm m + 2 . Do đó số miền được tao ra từ các đường còn lại

đi qua Q là ( n − 1)( m + 2 ) . Như vậy ta có

S= 2m + ( m + 1) + ( n − 1)( m + 2 )= mn + 2m + 2n − 1= mn + 2 ( m + n ) − 1= mn + 19

1 1
( m + n)=
2
Áp dụng bất đẳng thức AM – GM ta có mn ≤ .100= 25 .
4 4

Tài liệu sưu tầm và tổng hợp bản word đầy đủ liên hệ 0393732038 TÀI LIỆU TOÁN HỌC
33
Website:tailieumontoan.com
Từ đó ta được S ≤ 25 + 19 =44 . Dấu bằng xẩy ra khi và chỉ khi m= n= 5 .
Vậy số miền được tạo ra tối đa là 44 khi số đường thẳng đi qua P là 5 và số đường thẳng đi
qua Q là 5.

Đề số 4

Câu 1.
 xy ( x + y ) =2
a) Giải hệ phương trình :  3
 x + y + x y + 7 ( x + 1)( y + 1) =
3 3 3
31
Ta có hệ phương trình:
 xy ( x + y ) = 2
⇔
( x + y )( x − xy + y ) + ( xy ) + 7( x + y + xy + 1) =
2 2 3
31
 xy ( x + y ) =2
⇔
( x + y ) ( x + y ) − 3 xy  + ( xy ) + 7 ( x + y ) + xy + 1 =31
 2
 3

ab = 2
Đặt a =+
x y; b =xy thì hệ trên trở thành: 
a ( a − 3b ) + b + 7 ( a + b + 1) =
2 3
31
ab = 2
⇔ 3
a − 3ab + b + 7 ( a + b + 1) =
3
31

⇒ ( a + b ) ( a + b ) − 3ab  − 3ab + 7 ( a + b + 1) =
2
31
 
⇔ ( a + b ) − 3ab(a + b) − 3ab + 7(a + b) − 24 =
3
0
⇒ ( a + b ) − 6(a + b) − 3.2 + 7 ( a + b ) − 24 =
3
0
⇔ ( a + b ) + ( a + b ) − 30 =
3
0
⇔ ( a + b ) − 27 + (a + b) =
3
3
⇔ (a + b − 3) ( a + b ) + 3(a + b) + 10  =
2
0
 
=
⇒ a + b 3 do ( (a + b)
2
+ 3(a + b) + 10 > 0 )
 a=
+b 3 =
a 2
(do a =( x + y ) ≥ 4 xy =4b)
2
⇒ ⇒ 2

= ab 2= b 1
x + y = 2
⇒ ⇒ x = y =1
 xy = 1
Vậy hệ có nghiệm duy nhất ( x; y ) = (1;1)

b) Giải phương trình: 9 + 3 x ( 3 − 2 x ) = 7 x + 5 3 − 2 x


3
Điều kiện xác định: 0 ≤ x ≤
2
Tài liệu sưu tầm và tổng hợp bản word đầy đủ liên hệ 0393732038 TÀI LIỆU TOÁN HỌC
34
Website:tailieumontoan.com

Đặt a =x,b =3 − 2 x ( a, b ≥ 0 ) . Khi đó phương trình tương đương với:


9 + 3ab =7 a + 5b
 2 ⇒ 2a 2 + b 2 + 6 + 3ab = 7 a + 5b
 2a + b =
2
3
⇔ 2a 2 + 2ab − 4a + ab + b 2 − 2b − 3a − 3b + 6 = 0
⇔ 2a (a + b − 2) + b ( a + b − 2 ) − 3 ( a + b − 2 ) =0
⇔ ( a + b − 2 )( 2a + b − 3) =0
 a + b = 2 ⇒ b = 2 − a ⇒ 9 + 3a(2 − a) = 2a + 10
⇔
 2a + b = 3 ⇒ b = 3 − 2a ⇒ 9 + 3a ( 3 − 2a ) = 7 a + 5 ( 3 − 2a )
( 3a − 1)( a − 1) =  1 1
0
 a= ⇒x= (tm)
⇔ ⇔ 3 9
( a − 1) =
2
0 
 a =1 ⇒ x =1 (tm)
1 
Vậy phương trình trên có tập nghiệm S =  ;1
9 
Câu 2.
a) Cho x,y…..
(x 2
− 2 xy − y ) + ( xy − 2 y 2 − x ) =x 2 − xy − 2 y 2 − x
Ta có:
= x 2 + xy − ( 2 xy + y 2 ) − ( x + y ) = ( x + y )( x − 2 y − 1)
Lại có: x 2 − 2 xy − y, xy − 2 y 2 − x chia hết cho 5
⇒ ( x + y )( x − 2 y − 1) chia hết cho 5
TH1: Nếu x + y chia hết cho5 thì y ≡ − x ( mod 5 )
⇒ 0 ≡ x 2 − 2 xy − y ≡ x 2 + 2 x 2 + x= x ( 3 x + 1) (mod 5) , do vậy x chia hết cho 5 hoặc chia 5
dư 3.
+)Nếu x chia hết cho 5 thì y cũng vậy, bài toán được chứng minh
+)Nếu x chia cho 5 dư 3 thì y chia 5 dư 2, thì
2 x 2 + y 2 + 2 x + y ≡ 2.9 + 4 + 2.3 = 30 ≡ 0(mod 5)
Ta cũng có điều phải chứng minh.
TH2) Nếu x − 2 y − 1 chia hết cho 5 thì x ≡ 2 y + 1( mod 5 )

⇒ 0 ≡ x 2 − 2 xy − y ≡ ( 2 y + 1) − 2 y ( y + 1) − y = y + 1( mod 5 )
2

Do đó y chia 5 dư 4 và x cũng chia 5 dư 4 nên:


2 x 2 + y 2 + 2 x + y = 2.16 + 16 + 2.4 + 4 = 60 ≡ 0 ( mod 5 )
Vậy ta có điều phải chứng minh.
b) Cho………
Nếu tồn tại n :1 ≤ n ≤ 50 : a1 + a2 + ...... + an =
50 thì kết luận bài toán hiểu nhiên

Tài liệu sưu tầm và tổng hợp bản word đầy đủ liên hệ 0393732038 TÀI LIỆU TOÁN HỌC
35
Website:tailieumontoan.com

a1 + a2 + ...... + an ≤ 49
Xét: 1 ≤ n ≤ 49 :  ⇒ an +1 ≥ 2
a +
 1 2 a + ....... + an +1 ≥ 51
TH 1: an+1 =2 ⇒ a1 + a2 + ..... + an =49
an+ 2 + an+3 + ...... + a50 =
49
Nên nếu n ≤ 24 ⇒ a1 ≤ an + 2 ; a2 ≤ an +3 ;.....; an ≤ a2 n +1
⇒ 49 =a1 + a2 + ..... + an ≤ an + 2 + an +3 + ..... + a2 n +1 < an + 2 + ...... + a49 + a50
Điều này vô lý nên:
n ≥ 25 ⇒ 49 = a1 + a2 + .... + an ≥ na1 ≥ 25a1 ⇒ a < 2 ⇒ a = 1
⇒ a2 + .....
= an 48; a2 + ...... + =
an+1 50
TH2: an +1 ≥ 3

an + 2 + an +3 + ..... + a50 = 100 − ( a1 + a2 + ..... + an +1 ) ≤ 49


⇒ 49 ≥ ( 49 − n ) an + 2 ≥ ( 49 − n ) .3 ⇒ n ≥ 33
⇒ 49 ≥ ( a1 + a2 + .... + a16 ) + ( a17 + ..... + an ) ≥ 16 + ( n − 16 ) a17 ≥ 16 + 17 a17
⇒ a17 < 2 ⇒ a17 =1 ⇒ a1 = a2 ..... = a17 =1
Nếu an +1 < 18 đặt a1 + a2 + ..... + an +1 = 50 + k ( k ≥ 1)
⇒ 18 ≥ an +1 ≥ ( 50 + k ) − 49 =k + 1
⇒ k ≤ 17 ⇒ ak +1 + ..... + an +1 =
50
Nếu an +1 ≥ 19
⇒ 49 ≥ ( 49 − n ) an + 2 ≥ ( 49 − n )19 → n ≥ 47
⇒ a1 = a2 = ..... = a45 = 1
Vì nếu
a45 ≥ 2 ⇒ ( a1 + a2 + ....... + a44 ) + ( a45 + ....... + an ) ≥ 44 + ( n − 44 ) a45 ≥ 44 + ( 47 − 44 ) .2 > 49
Đặt an +1 = 50 − k ( 0 ≤ k ≤ 31) ⇒ a1 + ..... + ak + an +1 = 50 ( do a1 = ...... = ak = 1)
Vậy ta có điều phải chứng minh.

Câu 3:

Tài liệu sưu tầm và tổng hợp bản word đầy đủ liên hệ 0393732038 TÀI LIỆU TOÁN HỌC
36
Website:tailieumontoan.com

a) Chứng minh 3 điểm K, M, Q thẳng hàng


 
= CBQ
Do các tứ giác BCKQ và BCDA nội tiếp nên: CKQ  ⇒ KQ / / AD. Mặt khác
= CAD
MK / / AD nên K, M, Q thẳng hàng
b) Đường tròn ngoại tiếp tam giác………….
Chứng minh tương tự ta có: R, M , L thẳng hàng
 
= RLD
MQ / / AD nên RMQ  ⇒ tứ giác RTMQ nội tiếp
= ETD
Chứng minh tương tự RMSQ nội tiếp do đó: M , S , Q, R, T cùng thuộc một đường tròn
c) Chứng minh đường tròn ngoại tiếp..
Bổ đề: cho tam giác ABC, M nằm trên d / / BC lấy E khác M trên d, AM cắt BC
tại I. Đường qua M / / AB cắt BE tại J , khi đó IJ / / AE
MA AG
Chứng minh MJ cắt AE, AC tại S và T, ME cắt AC tại G. Ta có MG//BC suy ra = ,
MI GC
MS AP AG MA
ME cắt AB tại P ta có: = = = ⇒ AE / / IJ
MJ PB GC MI
Quay trở lại bài toán:
AM cắt BC, (O) tại I và J khác A. Áp dụng bổ đề ta có: IR / / AE , IQ / / AB . Do đó

= 
IRE = 
AEC AJC ⇒ nên RIJC là tứ giác nội tiếp . Chứng minh tương tự ta có DQIJ là
tứ giác nội tiếp
 + IJQ
Do đó: RJI  + RPD
= 2 PCD
 + CPD
= 1800 nên RPQJ nội tiếp. Kẻ tiếp tuyến Jx của
(O).
Ta có:
 = xJA
xJR  − RJA
 =  =
ADJ − PDC 
ADP + MAC
= =
ADP + PAD APB
 = MAC
⇒ PEJ  = PED 
Suy ra : Jx tiếp xúc với ( PQR ) hay ta thu được: ( PQR ) tiếp xúc với ( O )
Tài liệu sưu tầm và tổng hợp bản word đầy đủ liên hệ 0393732038 TÀI LIỆU TOÁN HỌC
37
Website:tailieumontoan.com
Vậy ta có điều phải chứng minh
Câu 4: Áp dụng BĐT Cauchy-Schwarz ta có:

 ab bc   1 1   ab bc   1 1 
 +  +  ≤ 2.  +  . 2  + 
 a+b b + c  a + b b+c  a+b b+c a+b b+c
 a c  b b   a c   b b 
= 2  +  +  ≤ + + + 
 a + b b + c  a + b b + c   a + b b + c   a + b b + c 
 a b   c b 
= + + + = 2
a+b a+b b+c b+c
Vậy ta có điều phải chứng minh
Dấu " = " xảy ra khi và chỉ khi a= b= c

Đề số 5

Câu 1 (3.5 điểm).


x 2  y 2  xy  1

a) Giải hệ phương trình 
x  x 2y  2y 3


Phương trình thứ nhất của hệ được viết lại thành xy  x 2  y 2  1 . Thế vào phương trình

thứ hai của hệ ta được

 
x  x x 2  y 2  1  2y 3  x  x 3  xy 2  x  2y 3  x 3  xy 2  2y 3  0
3 3 2 3
 
 x  y  xy  y  0  x  y  x 2  xy  y 2  y 2 x  y   0
x  y  0 x  y
 

 x  y  x  xy  2y  0   2
2 2
 
x  y  0
2
x  xy  2y  0

Dễ thấy x  y  0 không thỏa mãn phương trình thứ nhất của hệ.

Do đó từ x  y thay vào phương trình thứ nhất của hệ ta được

x 2  1  x  1  x  y  1 .

Vậy hệ phương trình đã cho có các nghiệm là x ; y   1;1, 1; 1 .

b) Giải phương trình 2 x  1 x  1   


x  1  1 x 2  1 x2 
Tài liệu sưu tầm và tổng hợp bản word đầy đủ liên hệ 0393732038 TÀI LIỆU TOÁN HỌC
38
Website:tailieumontoan.com
Điều kiện xác định của phương trình là 1  x  1 . Biến đổi tương đương phương trình

ta được

2 x  1 x  1   
x  1  1 x 2  1 x2 
 2 x  1 x  1  2 x  1  2 1  x  x  1 1  x   1  x  1  x 
2 2

 2 x  1 x  1  2 x  1  2 1  x  x  1 x  1  1  x  x  1
 x  1  x 1  1x  2 1x 
Đặt a  x  1;b  1  x a  0;b  0 , khi đó ta có a 2  b 2  2 .

Phương trình trên được viết lại thành a 2 a  b   2b . Từ đó ta có hẹ phương trình

a 2  b 2  2

 2 .
a a  b   2b

+ Xét trường hợp b  0 , hệ phương trình trên vô nghiệm.

+ Xét trường hợp b  0 , khi đó phương trình thứ nhất của hệ tương đương với

 
b a 2  b 2  2b .


Khi đó ta có hệ hương trình  2

b a 2  b 2  2b
 

 3

a 2b  b 3  2b
.

a a  b   2b 
a  a 2b  2b

 

Từ đó ta được a 2b  b 3  a 3  a 2b  2b  a 3  b 3  a  b .

Thay vào phương trình thứ nhất của hệ và chú ý đến điều kiện ta được

a2  1  a  1  a  b  1 .

Từ đó ta được x  1  1  x  1  x  0 , thỏa mãn điều kiện xác định.

Vậy phương trình có nghiệm duy nhất là x  0 .

Câu 2(2.5 điểm).

a) Chứng minh rằng không tồn tại các số nguyên x, y thỏa mãn đẳng thức

12x 2  26xy  15y 2  4617

Trước hết ta chứng minh bổ đề: Với mọi số nguyên tố có dạng p  4k  3 thì ta luôn có

a  p
a 2  b 2  p   a,b  Z 
b  p

Thật vậy, ta xét hai trường hợp sau

Tài liệu sưu tầm và tổng hợp bản word đầy đủ liên hệ 0393732038 TÀI LIỆU TOÁN HỌC
39
Website:tailieumontoan.com
+ Trường hợp 1. Nếu một trong hai số a và b chia hết cho p thì ta suy ra điều cần chứng

minh.

+ Trường hợp 2. Nếu cả hai số a và b cùng khoog chia hết cho p. Khi đó ta có

a; p   b; p   1 .
a p1  1 mod p  a 4k 2  1 mod p 
Theo định lí Fecmat ta có  p1   4k 2  a 4k 2  b 4k 2  2 mod p 
b  1 mod p  b  1 mod p 
 

Mặt khác ta có a 4k 2  b 4k 2  a 2   
k 1 2k 1
 b2 chia hết cho a 2  b 2 nên chia hết cho p.

Từ đó suy ra 2 chia hết cho p, mà p là số nguyên tố nên ta được p  2 . Điều này mâu

thuẫn vì p là số nguyên tố lẻ.

Như vậy trường hợp 2 không xẩy ra hay bổ đề được chứng minh.

Trở lại bài toán. Do 4617 chia hết cho 19 nên 12x 2  26xy  15y 2 19 hay ta được

12x 2  12xy  15y 2  38xy 19  12x 2  12xy  15y 2 19


 
 3 4x 2  4xy  5y 2 19  4x 2  4xy  5y 2 19

   4y 19  2x  y   2y  19


2 2
2 2 2
 4x  4xy  y

Do 19 là số nguyên tố có dạng 4k  3 nên áp dụng bổ đề trên ta suy ra được


2x  y 19 3x  2y 19 3x 19 x 19
      
   
2y 19 2y 19 2y 19 y 19
   
Từ đó ta được 4x 2  4xy  5y 2 192 . Điều này dẫn đến mâu thuẫn vì 4617 không chia hết

cho 192 .

Vậy không tồn tại cặp số nguyên x ; y  thỏa mãn yêu cầu bài toán.

b) Với a, b là các số thực dương. Tìm giá trị lớn nhát của biểu thức
 1 1  1
M  a  b  3  3  
a  b b  a  ab

Áp dụng bất đẳng thức Bunhiacopxki ta có


1   
a  
 b   b   a  b  ; b 3  a b1  a   a  b 
2 2
3

a 

1 1
b a
a b a  b
Từ đó ta được 3 a ; 3 b . Do đó suy ra
a b a b b a a b
Tài liệu sưu tầm và tổng hợp bản word đầy đủ liên hệ 0393732038 TÀI LIỆU TOÁN HỌC
40
Website:tailieumontoan.com
1 1 1 1
 1  b a a b  
1 a b a b
a  b a 3  b  a  b 3   a 3  b  b 3  a  aa  b  ba  b  a ab b
 

1 1
a b  1  ab a  b   a  b  a  b   ab a  b   1
a b  
Suy ra M 
a b ab a  b ab ab a  b 

Vậy giá trị lớn nhất của M là 1, đạt được tại a  b  1 .

Câu 3(3.0 điểm).



Cho hình thoi ABCD có BAD  900 . Đường tròn tâm I nội tiếp tam giác ABD tiếp

xúc với BD và BA lần lượt tại J và L. Trên đường thẳng IJ lấy điểm K sao cho BK song song

ID.
  ABI
a) Chứng minh rằng CBK .
B

Ta có L

  CBD
CBK   KBD   ABD
; ABI   IBD A J
C
I
  CBD
. Lại có ABD  . Mặt khác do ID
K
song song với BK nên ta có D
  IDB
IBD   DBK
 . Từ đó suy ra

  ABI
CBK 

b) Chứng minh rằng KC  KB .

Dễ thấy KJC  . Lại có IJL


  LJI   IBJ
  LBI   ABI
 . Kết hợp với CBK  ta được

  CJK
CBK  nên tứ giác BCKJ nội tiếp đường tròn. Do đó BJC
  BKC
  900 hay

KC  KB .

c) Chứng minh rằng bốn điểm C, K, I ,L cùng nằm trên một đường tròn.
  IJ
Tam giác IJL cân tại I nên ta có ILJ  L  IBJ   JBK
 . Mà ta có IBJ   JBK
 nên ILJ 

  JCK
Mặt khác do tứ giác BCKJ nội tiếp đường tròn nên JBK .

  JBK
Từ đó ta được ILJ   JCK
 nên suy ra tứ giác CKIL nội tiếp đường tròn.

Câu 4. (1.0 điểm)

Trước hết ta chứng minh bổ đề: Với hợp số n  4 ta luôn có n  1 ! n .

Tài liệu sưu tầm và tổng hợp bản word đầy đủ liên hệ 0393732038 TÀI LIỆU TOÁN HỌC
41
Website:tailieumontoan.com
Thật vậy, do n là hợp số và n  4 nên ta viết được n  a.b trong đó a, b  N ;1  a, b  n .

Khi đó ta suy ra được 2  a, b  n  1 . Từ đó dễ thấy ta luôn có n  1 ! n .

Trở lại bài toán. Ta thấy an  n vì nếu an  n thì ai  n i  1; n  1 .


   
a .a ...a  n
Khi đó ta có  1 2 i , điều này mâu thuẫn với giả thiết của bài toán.
a1.a2 ...an  n

Do vậy an  n . Giả sử n là hợp số và n  4 , khi đó theo bổ đề ta có

a1.a2 ...an 1  n  1 ! n .

Mặt khác theo bài ra ta lại có a1.a2 ...an  n . Như vậy a1.a2 ...an 1 và a1.a2 ...an  n có cùng số dư

khi chia cho n. Điều này mâu thuẫn với giả thiết của bài toán.

Từ đó suy ra n  4 . Mà do n là hợp số nên ta được n  4 .

Ta thấy với n  4 thì bộ số 1; 3;2; 4 viết được dãy số 1;1.3;1.3.2;1.3.2.4 chia cho 4 có số dư

lần lượt là 1; 3;2; 0 .

Đề số 6

Câu 1.

x  y   x  3y
2

1) Hệ phương trình tương đương với 


 x  y 2  3  xy
 
x  3
Do đó ta có phương trình x  3y  3  xy  x  3y  1  0  
y  1

x  3
 x  3

+ Với 
 2  
 2 , hệ phương trình vô nghiệm.

x  y 2
 xy  3 
y  3y  6  0

 

y  1 y  1 x  1; y  1

+ Với  2   2 
x  y  xy  3
2
x  x  2  0
 x  2; y  1 .
  
Vậy hệ phương trình đã cho có nghiệm là x ; y   1;1 .

2. Với a, b là các số thực dương thỏa mãn ab  a  b  1 . Chứng minh rằng:

a b 1  ab
 
  
2
1a 1  b2 2 1  a 2 1  b2

Cách 1. Do ab  a  b  1 nên ta được


Tài liệu sưu tầm và tổng hợp bản word đầy đủ liên hệ 0393732038 TÀI LIỆU TOÁN HỌC
42
Website:tailieumontoan.com
a 2  1  a 2  ab  a  b  a  b a  1;b 2  1  b 2  ab  a  b  a  b b  1

Đẳng thức cần chứng minh tương đương với

a b 1  ab
 
a  b a  1 a  b b  1 2 a  b  a  1b  1 2

a b  1  b a  1 1  ab
 
a  b a  1b  1 2 a  b  a  1b  1 2

 a  1b  1  2 a  1b  1  a  1b  1  2  ab  a  b  1

Do đẳng thức cuối cùng luôn đúng nên đẳng thức cần chứng minh đúng.

Cách 2. Đẳng thức cần chứng minh tương đương với

 
a b2  1  b a 2  1   1  ab
.
a 2

1 b 1 2
  
2 a 2  1 b2  1 
   
Mà ta có a b 2  1  b a 2  1  a  b ab  1 nên đẳng thức trên tương đương với

a b 1
  
 2 a  b   a 2  1 b 2  1  a 2  b 2  4ab  a 2b 2  1
2

a 2

 1 b2  1  2

 a  b   ab  1  a  b  ab  1  ab  a  b  1
2 2

Do đẳng thức cuối cùng luôn đúng nên đẳng thức cần chứng minh đúng.

Câu 2.

1. Câu 2 (2.5 điểm).

1. Giả sử p và q là các số nguyên tố thỏa mãn đẳng thức p p  1  q q 2  1 .  


a) Chứng minh rằng tồn tại số nguyên dương k sao cho p  1  kq, q 2  1  kp .

p  q  0
Nếu p  q thì ta có p  1  q 2  1   , điều này vô lí vì p, q là các số nguyên tố.
 p  q  1

Do vậy p  q , khi đó do p và q là các số nguyên tố nên p  1q và q 2  1 p .

Như vậy tồn tại các số nguyên dương m, n thỏa mãn p  1  mq; q 2  1  np , thay vào


p  1  kq
đẳng thức đã cho ta được m  n . Do vậy tồn tại số nguyên dương k sao cho 
 2 .

q  1  kp

b) Tìm tất cả các số nguyên tố p, q thỏa mãn đẳng thức p p  1  q q 2  1 .  


Tài liệu sưu tầm và tổng hợp bản word đầy đủ liên hệ 0393732038 TÀI LIỆU TOÁN HỌC
43
Website:tailieumontoan.com
Thế p  kq  1 vào hệ thức q 2  1  kp ta được q 2  1  k kq  1  q 2  k 2q  k  1  0 .

Xem phương trình là phương trình bậc hai ẩn q, khi đó để phương trình có nghiệm

nguyên dương thì

  k 4  4 k  1  k 4  4k  4 phải là số chính phương.

Ta có k 4  k 4  4k  4  k 2  2 nên ta được   k 2  1 .
2 2

Từ đó ta được k 4  4k  4  k 2  1  k  k 2  k  1 .
2

Thay vào hệ thức đã cho ta được q 2  q  2  0  q  2  p  3 .

Vậy các số p  3; q  2 là các số nguyên tố cần tìm.

2. Với a, b, c là các số thực dương thỏa mãn ab  bc  ca  abc  2 . Tìm giá trị lớn nhất

của biểu thức

a 1 b 1 c 1
M  2
 2  2
a  2a  2 b  2b  2 c  2c  2

Biến đổi giả thiết ab  bc  ca  abc  2 ta được

1  a 1  b 1  c   1  a   1  b   1  c 
1 1 1
   1
1  a 1  b  1  a 1  c  1  b 1  c 
1 1 1
Đặt x  ;y  ;z  , khi đó ta thu được xy  yz  zx  1 .
1a 1 b 1c

Biểu thức M được viết lại thành

1 1 1
a 1 b 1 c 1 x y z
M      
a  1  1 b  1  1 c  1  1
2 2 2
1 1 1
1 1 1
x2 y2 z2
x y z
 2
 2  2
x 1 y 1 z 1

Để ý ta có x 2  1  x 2  xy  yz  zx  x  y x  z  . Áp dụng tương tự ta được

x y z
M   
x  y x  z  y  z x  y  x  z y  z 
x y  z   y z  x   z x  y  2 xy  yz  zx  2
  
x  y y  z z  x  x  y y  z z  x  x  y y  z z  x 

Tài liệu sưu tầm và tổng hợp bản word đầy đủ liên hệ 0393732038 TÀI LIỆU TOÁN HỌC
44
Website:tailieumontoan.com
Ta chứng minh được 9 x  y y  z z  x   8 x  y  z xy  yz  zx  .

Vì x  y  z   3 xy  yz  zx   3 nên x  y  z  3 . Nên ta được


2

2 9 9 3 3
M   
8 4 x  y  z  4 3 4
9
x  y  z xy  yz  zx 
3 3 1
Vậy giá trị lớn nhất của M là , đạ được tại x  y  z   a  b  c  3 1.
4 3

Câu 3 (3.0 điểm).

Cho tam giác ABC nhọn với AB  AC . Gọi E, F lần lượt là trung điểm của CA, AB.
 và nằm ngoài tam
Đường trung trực của EF cắt BC tại D. Giả sử P nằm trong góc EAF
 D
giác AEF sao cho PEC    DFE
EF và PEB  . Đường thẳng PA cắt đường tròn ngoại

tiếp tam giác PEF tại Q khác P.

A
I E
F
R
M X

P Y
F E
B C
D Z

N K
P

Hình 1 B
D
C

Hình 2
  BAC
a) Chứng minh rằng EQF   EDF
.

Vì tứ giác PEQF nội tiếp đường tròn nên ta có


  1800  EPF
EQF  P   DEC
EF  PFE   DFB

 EAD 
  EDA
  FAD
  FDA
 
  EDF
  BAC 

Tài liệu sưu tầm và tổng hợp bản word đầy đủ liên hệ 0393732038 TÀI LIỆU TOÁN HỌC
45
Website:tailieumontoan.com
b) Tiếp tuyến tại P của đường tròn ngoại tiếp tam giác PEF cắt CA, AB lần lượt tại M, N.

Chứng minh rằng bốn điểm C, M, B, N cùng nằ trên một đường tròn. Gọi đường tròn này

là đường tròn K  .

Không mất tính tổng quát ta giả sử M nằm giữa A, C và N nằm trên tia đối của tia BA (các

trường hợp còn lại chứng minh tương tự). Khi đó ta có


  1800  NPF
MNB   PFN
  1800  P  
EF  DFE
 D
 1800  DEC  
EF  1800  C   ACB
EF  AEF 

Do đó tứ giác NCMB nội tiếp đường tròn K  .

c) Chứng minh rằng đường tròn K  tiếp xúc với đường tròn ngại tiếp tam giác AEF.

  DAC
Ta có nhận xét: PAB  . Thật vậy, gọi X, Y, Z lần lượt là điểm đối xứng với P qua

 D
EF, AE, AF thì từ giả thiết PEC   D
EF suy ra DEY 
EX , mà ta có EX  EY nên

DX  DY . Tương tự thì ta có DX  DZ nên DX  DY  XZ , lạ có AY  AP  AZ ta


  DAY
được DAZ   DAC
 . Kết hợp tính đối xứng ta được PAB .

Đường tròn ngoại tiếp tam giác PEM cắt đường tròn ngoại tiếp tam giác AEF tại R khác E.
  
Ta thấy RPN  R EM  RFA nên tứ giác PRFN nội tiếp đường tròn. Lại do tứ giác
  1800  NBC 0 
  180 .
BNCM nội tiếp đường tròn nên BAC  CMN  AMN
    
Từ đây ta thu được A R E  AFE  ABC  AMN  1800  PRE nên ba điểm A, R, P

thẳng hàng.

Gọi giao điểm của EF và AD là I, theo tính chất đường trung bình thì I là trung điểm của

AD.
  AR
Ta lại có AEI  F , kết hợp với nhận xét ta được AEI ∽ ARF .

Từ đó ra suy ra được AED ∽ ARB . Ta thu được


  ADE
ABR   DEC
  DAE
 P  P
EF  PAB   P
EF  FER  
ER  RMP
Từ đó tứ giác NMRB nội tiếp đường tròn hay năm điểm M, N, R, B, C cùng nằm trên

đường tròn K  .


Lại có ER   EF
M  EPM  
P  EF   RAE
R  RFP   RNM
.

Vậy đường tròn ngoại tiếp tam giác REF và đường tròn K  tiếp xúc nhau tại R.
Tài liệu sưu tầm và tổng hợp bản word đầy đủ liên hệ 0393732038 TÀI LIỆU TOÁN HỌC
46
Website:tailieumontoan.com
Câu 4 (1.0 điểm).

Cho n là số nguyên dương với n  5 . Xét đa giác lồi n cạnh. Người ta muốn kẻ một

số đường chéo của đa giác mà các đường chéo này chia đa giác thành đúng k miền, mõi

miền là mọt ngũ giác lồi (hai miền bất kì không có điểm chung trong).

a) Chứng minh rằng ta có thể thực hiện được với n  2018, k  672 .

Kí hiệu đa giác 2018 cạnh là A1A2A3 ...A2018 , kẻ các đường chéo A1A5 ; A1A8 ; A1A11;...; A1A2015 khi

đó đa giác A1A2A3 ...A2018 được chia thành 672 ngũ giác lồi gồm:

A1A2A3A4A5 ; A1A5A6A7A8 ;...; A1A2012A2013A2014A2015 ; A1A2015A2016A2017A2018

b) Với n  2017, k  672 ta có thể thực hiện được không? Hãy giải thích.

Giả sử ta có thể chia đa giác lồi 2017 cạnh thành 672 ngũ giác lồi bằng các đường chéo của

nó. Gọi p là số giao điểm của các đường chéo nằm trong đa giác. Do mỗi đỉnh của ngũ giác

lồi là là đỉnh của đa giác đã cho hoặc là một trong p giao điểm của các đường chéo nên

tổng số góc các ngũ giác này là

p.3600  2017  21800  2p  20151800

Mặt khác số ngũ giác lồi là 672, mỗi ngũ giác lồi có tổng số góc ở đỉnh là 3.1800 nên tổng

số góc của các ngũ giác lồi là 672.3.1800 .

1
Từ đó ta được 2p  20151800  672.3.1800  p  , vô lý.
2
Vậy ta không thể thực hiện được với n  2017, k  672 .

Đề số 5

Câu I.

 x 2 + y 2 + xy = 7 (1)
1) Giải hệ phương trình:  3
9 x =xy + 70 ( x − y )
2
(2)
 7
3 x 2 = 7
 x = ±
Nếu x = y, hệ phương trình trở thành  ⇔ 3 (Vô nghiệm), do đó x ≠ y

8 x 3
= 0 x = 0

Nhân cả hai vế của phương trình (1) với x − y ≠ 0 ta có:

(1) ⇔ ( x − y ) ( x 2 + y 2 + xy ) = 7 ( x − y ) ⇔ x3 − y 3 = 7 ( x − y ) ⇔ 10 ( x3 − y 3 ) = 70 ( x − y )

Tài liệu sưu tầm và tổng hợp bản word đầy đủ liên hệ 0393732038 TÀI LIỆU TOÁN HỌC
47
Website:tailieumontoan.com
Thế vào phương trình ( 2 ) ta có:

( 2 ) ⇔ 9 x3 = xy 2 + 10 ( x3 − y 3 ) ⇔ x3 + xy 2 − 10 y 3 = 0
x − 2y = 0 ( 3)
⇔ ( x − 2 y ) ( x 2 + 2 xy + 5 y 2 ) =0 ⇔  2
 x + 2 xy + 5 y =
2
0 ( 4)
Ta có: ( 3) ⇔ x = 2y
 y =1 ⇒ x =2
Thế vào phương trình (1) ta có: 4 y + y + 2 y =7 ⇔ 7 y =7 ⇔ 
2 2 2 2

 y =−1 ⇒ x =−2
0 ( x + 2y)
( 4 ) ⇔ x 2 + 2 xy + y 2 + 4 y 2 =⇔ + 4 y2 =
2
0
x + 2 y =
0
⇔ ( x + 2 y) + (2 y) = 0 ⇔  ⇔ x = y = 0(ktm)
2 2

 y = 0
Vậy nghiệm của hệ phương trình là ( x; y ) ∈ {( 2;1) ; ( −2; −1)}
2) Giải phương trình: 11 5 − x + 8 2 x − 1 = 24 + 3 ( 5 − x )( 2 x − 1)
11 5 − x + 8 2 x − 1 = 24 + 3 ( 5 − x )( 2 x − 1) (*)
5 − x ≥ 0 1
ĐKXĐ:  ⇔ ≤ x≤5
2 x − 1 ≥ 0 2
 5 − x= a ( a ≥ 0 ) a 2= 5 − x
Đặt :  ⇒ 2
 2 x − =
1 b ( b ≥ 0 ) b= 2 x − 1
⇒ 2a 2 + b 2= 2 ( 5 − x ) + 2 x − 1= 9
11a + 8b = 24 + 3ab (1)
Khi đó ta có: 
 2a + b =
2 2
9 (2)
Giải phương trình (1) ta có: (1) ⇔ 11a − 3ab = 24 − 8b ⇔ a (11 − 3b ) = 24 − 8b (*)
11 16 11
Với 11 − 3b =0 ⇔ b = ⇒ (*) ⇔ 0a =− (vô lý) ⇒ b = không là nghiệm của phương
3 3 3
trình (*)
24 − 8b 8b − 24 8b − 24

= a = , Thay a = vào ( 2 ) ta được:
11 − 3b 3b − 11 3b − 11

Tài liệu sưu tầm và tổng hợp bản word đầy đủ liên hệ 0393732038 TÀI LIỆU TOÁN HỌC
48
Website:tailieumontoan.com

8b − 24 
2

( 2 ) ⇔ 2   +b =
2
9
 3b − 11 
⇔ 2 ( 64b 2 − 384b + 576 ) + b 2 ( 9b 2 − 66b + 121)= 9 ( 9b 2 − 66b + 121)
⇔ 128b 2 − 768b + 1152 + 9b 4 − 66b3 + 121b 2 − 81b 2 + 594b − 1089 =
0
⇔ 9b 4 − 66b3 + 168b 2 − 174b + 63 =0 ⇔ 3b 4 − 22b3 + 56b 2 − 58b + 21 =0
⇔ ( b − 1) ( 3b3 − 19b 2 + 37b − 21) =0 ⇔ ( b − 1)( b − 1)( b − 3)( 3b − 7 ) =0
   
=b − 1 0 = b 1  2 x − 1 =
1 =2x − 1 1 = x 1(tm)
   
⇔ b − 3 = 0 ⇔ b = 3 ⇔  2 x − 1 = 3 ⇔ 2 x − 1 = 9 ⇔ x = 5(tm)

3b − 7 =0  7  7  49  29
= b  =2x − 1 =2x − 1 =x (tm)
 3  3  9  9
 29 
Vậy phương trình có tập nghiệm S = 1; ;5
 9 

Câu II.

1) Tìm x, y nguyên dương thỏa mãn: x 2 y 2 − 16 xy + 99 = 9 x 2 + 36 y 2 + 13 x + 26 y


x 2 y 2 − 16 xy + 99 = 9 x 2 + 36 y 2 + 13 x + 26 y
⇔ x 2 y 2 + 20 xy + 99 = 9 x 2 + 36 xy + 36 y 2 + 13 x + 26 y
⇔ ( x 2 y 2 + 20 xy + 100 ) − 1= ( 3x + 2 y ) 13 ( x + 2 y )(*)
2

 x + 2 y = a ( a > 0 )
Đặt 
 xy + 10 = b ( b > 10 )
⇒ (*) ⇔ b 2 − 1= 9a 2 + 13a
13 169 169
⇔ 9a 2 + 2.3a. + − =b 2 − 1
6 36 36
2
 13  133
⇔  3a +  − b 2 = ⇔ (18a + 13) − 362 = 133
2

 6 36
⇔ (18a − 6b + 13)(18a + 6b + 13) =
133 (1)
Ta lại có : a, b > 0 ⇒ 18a + 6b + 13 > 18a − 6b + 13 > 0
Lại có= = 19.7
133 133.1

Tài liệu sưu tầm và tổng hợp bản word đầy đủ liên hệ 0393732038 TÀI LIỆU TOÁN HỌC
49
Website:tailieumontoan.com

 b = 11
 18a + 6b + 13 = 0  18a + 6b = 120  (tm)
 a = 3
 
18a − 6b + 13 = 1 18 a − 6b =−12 
⇒ (1) ⇔  ⇔ ⇔  a = 19
 18a +=6b + 13 19  18
= a + 6b 32 
    6
(ktm)
 18a − 6b + 13 = 7  18a − 6b = −6  25
 b = − 18
 
 x + 2 y =3  x =3 − 2 y  x= 3 − 2 y  x= 3 − 2 y
⇒ ⇔ ⇔ ⇔ 2
 xy=+ 10 11 = xy 1  y (3 − 2 y ) =
1 2 y − 3 y + 1 =0
 x= 3 − 2 y
 x= 3 − 2 y   x = 1(tm)

⇔   y = (ktm) ⇔ 
1
⇔
( 2 y − 1)( y − 1) =
0 

2  y = 1(tm)
  y =1
Vậy phương trình có nghiệm ( x; y ) = (1;1)
2) Với a, b là những số thực dương thỏa mãn 2 ≤ 2a + 3b ≤ 5 (1) ;

8a + 12b ≤ 2a 2 + 3b 2 + 5ab + 10 . Chứng minh rằng 3a 2 + 8b 2 + 10ab ≤ 21( 2 )


Giải

( 2 ) ⇔ 8a + 12b ≤ ( 2a + 3b )( a + b ) + 10 ≤ 5 ( a + b ) + 10
⇔ 3a + 7b ≤ 10. Mặt khác 2a + 3b ≤ 5
Dự đoán dấu " = " xảy ra ⇔ a = b = 1

Ta có: 3a + 8

2
ab = ( 3a + 4b ) .( a + 2b )
b + 10 2

(I )

( A + B)
2

Áp dụng bất đẳng thức AB ≤ , ta có:


4

( 9a + 12b + 7a + 14b )
2

21.( I ) = 3 ( 3a + 4b )  . 7 ( a + 2b ) 


≤
4

(16a + 26b )
2

⇒ 21.( I ) ≤ (8a + 13b )


=
2

4
Ta biểu diễn 8a + 13b theo 3a + 7b và 2a + 3b bằng cách đồng nhất hệ số

Xét 8a + 13b = x ( 3a + 7b ) + y ( 2a + 3b )

Tài liệu sưu tầm và tổng hợp bản word đầy đủ liên hệ 0393732038 TÀI LIỆU TOÁN HỌC
50
Website:tailieumontoan.com

⇔ 8a + 13b = ( 3 x + 2 y ) .a + ( 7 x + 3 y ) .b
 2
 x =
3 x + 2 y =
8 5
⇔ ⇔
7 x + 3 y =
13 
y=
17
 5
2 2
2 17  2 17 
⇒ 21.( I ) ≤ ( 8a + 13b )=  .( 3a + 7b ) + .( 2a + 3b )  ≤  .10 + .5 = 212
2

5 5  5 5 
⇒ ( I ) ≤ 21.

Dấu " = " xảy ra ⇔ a = b = 1

Tài liệu sưu tầm và tổng hợp bản word đầy đủ liên hệ 0393732038 TÀI LIỆU TOÁN HỌC
51
Website:tailieumontoan.com
Câu III.

A P
M
Q K
E
F
N

O
C

D
B

1) Chứng minh rằng AM = AN


 = DAB
Ta có: NBA  (so le trong do BN / / AD)
 = DAC
DAB =
 ( gt ) ; DAC ACM (so le trong do CM / / AD)
 = MCA
⇒ NBA  ⇒ sd  AN = sd 
AM (trong một đường tròn, hai góc nội tiếp bằng nhau thì chắn
hai cung bằng nhau).
Vậy AM = AN (trong một đường tròn, hai dây bằng nhau căng hai cung bằng nhau)
2) Chứng minh rằng 4 điểm B, C , E , F cùng thuộc một đường tròn.

=
Ta có:
1

AEF
2
sd  (
AN + sdCM )
 (góc có đỉnh ở bên trong đường tròn)

=
1
2
sd  (
AM + sdCM
=  1  
2
= )
sd AC ABC (góc nội tiếp bằng nửa số đo cung bị chắn)
Vậy tứ giác BCEF là tứ giác nội tiếp (tứ giác có góc ngoài và góc trong tại đỉnh đối diện bằng
nhau) hay B, C , E ,F cùng thuộc một đường tròn.
Tài liệu sưu tầm và tổng hợp bản word đầy đủ liên hệ 0393732038 TÀI LIỆU TOÁN HỌC
52
Website:tailieumontoan.com
3) Chứng minh các đường thẳng EQ, FP, AD đồng quy
Áp dụng định lý Mê-lê-na-uýt trong tam giác AHN , cát tuyến EKQ , ta có:
EN KH QA EN KH
. . = 1⇒ . = 1(do Q là trung điểm của AN ( gt ) nên QA = QN )
EH KA QN EH KA
EN KA
⇒ =( I )
EH KH
Gọi AD ∩ PE = {K '}. Ta đi chứng minh K ' ≡ K
Áp dụng định lý Mê-lê-na-uýt trong tam giác AHM , cát tuyến PKF ta có:
FM K ' H PA FM K ' H
. . = 1⇒ . = 1 (Do P là trung điểm của AM ( gt ) nên PA = PM )
FH K ' A PM FH K ' A
FM K ' A
⇒ = ( II )
FH K ' H
EN FM FM FH FM − FH HM
Ta sẽ chứng minh = ⇔ = = = (*) (tính chất dãy tỉ số
EH FH EN EH EN − EH HN
bằng nhau)
HM DC
=
Vì BN / / AD / / CM nên áp dụng định lý Ta – let ta có:
HN DB
DC AC HM AC
Lại có : = (định lý đường phân giác), do đó: = (1)
DB AB HN AB
Xét ∆AEF và ∆ABC có: AEF =   chung
ABC (cmt ), BAC
AC AF
⇒ ∆AEF  ∆ABC ( g .g ) ⇒ = ( 2)
AB AE
HM AF
Từ (1) và (2) ⇒ = ( 3)
HN AE
AF HF
Tiếp tục áp dụng định lý đường phân giác trong tam giác AEF ta có: = ( 4)
AE HE
HM HF EN FM
Từ (3) và (4) ta suy ra = , do đó (*) được chứng minh, tức là = ( III )
HN HE EH FH
KA K ' A
Từ ( I ) , ( II ) , ( III ) suy ra = , do đó K ≡ K '
KH K ' H
Vậy EQ, FP, AD đồng quy tại K
Câu IV.
Với a, b, c > 0, a + b + c =3 ta có:

Tài liệu sưu tầm và tổng hợp bản word đầy đủ liên hệ 0393732038 TÀI LIỆU TOÁN HỌC
53
Website:tailieumontoan.com

a ( a + bc ) b ( b + ca ) c ( c + ab ) a 2 ( a + bc ) b 2 ( b + ca ) c 2 ( c + ab )
2 2 2 2 2 2

P= + + = + +
b ( ab + 2c 2 ) c ( bc + 2a 2 ) a ( ca + 2b 2 ) ab ( ab + 2c 2 ) bc ( bc + 2a 2 ) ca ( ca + 2b 2 )
a 2 b2 c2 ( a + b + c )
2

Áp dụng BĐT + + ≥ ta có:


x y z x+ y+z

(a + b 2 + c 2 + 3abc ) (a + b 2 + c 2 + 3abc )
2 2 2 2

P≥ ⇒P≥
a 2b 2 + b 2c 2 + c 2 a 2 + 2abc ( a + b + c ) ( ab + bc + ca )
2

a + b + c = p

q , áp dụng BĐT Schur ta có: 9r ≥ p ( 4q − p 2 )
Đặt  ab + bc + ca =
abc = r

⇒ 9abc ≥ 3  4 ( ab + bc + ca ) − 9  ⇔ 3abc ≥ 4 ( ab + bc + ca ) − 9
Khi đó ta có:

 a 2 + b 2 + c 2 + 4 ( ab + bc + ca ) − 9 
2

P≥
( ab + bc + ca )
2

2
( a + b + c )2 + 2 ( ab + bc + ca ) − 9 
P≥  
( ab + bc + ca )
2

32 + 2 ( ab + bc + ca ) − 9 
2
4 ( ab + bc + ca )
2

P≥ ⇒P≥ =
4
( + + ) ( + + )
2 2
ab bc ca ab bc ca

Dấu " = " xảy ra ⇔ a = b = c =1

Vậy P ≥ 4( dfcm)

Đề số 6

Câu I. (4 điểm)
( x + y )( x + 1) =
4
1) Giải hệ phương trình:  2
(
 y + xy + x + y + 5 x + y + 12 y + 13 =
3 3
243)( )
2) Giải phương trình: ( x − 12)7 + (2 x − 12)7 + (24 − 3 x)7 =
0
Lời giải
4 vào phương trình (2) , ta có:
1) Thay ( x + y )( x + 1) =

y 2 + xy + x + y + 5 = ( x + y )( y + 1) + ( x + y )( x + 1) + 1 = ( x + y )( x + y + 2) + 1 = ( x + y + 1) 2

Tài liệu sưu tầm và tổng hợp bản word đầy đủ liên hệ 0393732038 TÀI LIỆU TOÁN HỌC
54
Website:tailieumontoan.com
và x3 + y 3 + 12 y + 13 = x 3 + y 3 + 3( x + y )( x + 1)( y + 1) + 1 = ( x + y + 1)3 ⋅ ⇒ ( x + y + 1)5 = 243
⇒ x + y +1 =3.
⇒ x+ y =2
Thay vào phương trình (1) ta có: x= y= 1

2) Đặt x − 12 = b ta có: a 7 + b7 − (a + b)7 =


a và 2 x − 12 = 0

( )
⇒ (a + b) a 6 − a 5 b + a 4 b 2 − a 3 b3 + b 4 a 2 − ab5 + b6 − (a + b)(a + b)6 =
0.

( )
⇒ (a + b) a 6 − a 5 b + a 4 b 2 − a 3 b3 + b 4 a 2 − ab5 + b6 − a 6 − 6a 5 b − 15a 4 b 2 − 20a 3 b3 − 15a 2 b 4 − 6b5 a − b6 =
0
⇒ (a + b) ( −7 a b − 14a b − 21a b − 14a b − 7 ab ) =
5 4 2
0.3 3 2 4 5

⇒ (a + b)7 ab ( a + 2a b + 3a b + 2b a + b ) =
4 3 2 2
0 3 4

⇒ 7 ab(a + b) ( a + ab ) + a b + ( b + ab )  =
2 2
2 2 2 2
0
 
Từ đây ta suy ra a = 0 hoặc b = 0 hoặc a + b =0.
⇒ x= 12; x= 6 hoặc x = 8
Câu II. (2 điểm)
1) Tìm tất cả các số nguyên dương a, b, c sao cho cả ba số 4a 2 + 5b; 4b 2 + 5c; 4c 2 + 5a đều là bình
phương của số nguyên dương.
2) Từ một bộ bốn số thực (a, b, c, d ) ta xây dựng bộ số mới (a + b, b + c, c + d , d + a ) và liên tiếp
xây dựng các bộ số mới theo quy tắc trên. Chứng minh rằng nếu ở hai thời điểm khác nhau, ta thu
được cùng một bộ số (có thể khác thứ tự) thì bộ số ban đầu phải có dạng (a, −a, a, −a )
Lời giải
1) Không mất tổng quát, giả sử a = max{a, b, c} . Ta có: (2a ) 2 < 4a 2 + 5b < (2a + 2) 2 ⇒
16a + 9 25c − 9
4a 2 + 5b = (2a + 1) 2 . ⇒ 5b = 4a + 1 . Tương tự ta có: 5c ≥ 4b + 1 = hay a ≤ < 2c .
5 16
{ }
Từ đó (2c) 2 < 4c 2 + 5a < (2c + 3) 2 nên ta phải có 4c 2 + 5a ∈ (2c + 1) 2 , (2c + 2) 2 . Từ đây, xét hai
trường hợp
a 4c + 1 và do a ≥ c nên trong trường hợp này thì a= b= c= 1
- Nếu 4c 2 + 5a = (2c + 1) 2 thì 5=

2) Giả sử ở thời điểm thứ n ta thu được bộ số ( an , bn , cn , d n ) và nếu đặt S n = an + bn + cn + d n thì


S n = 2 S n −1 nên suy ra S n = 2n S0 với S0 = a + b + c + d . Vì tồn tại hai thời điểm ta thu được cùng
một bộ số nên S0 = 0 , kéo S n = 0 với mọi n ∈  . Nếu đặt Pn = an2 + bn2 + cn2 + d n2 thì

Pn +1= 2 Pn + 2 ( an + cn )( bn + d n =
) 2 Pn + 2Sn2−1= 2 Pn ∀n ≥ 1.
Suy ra Pn = 2n −1 P1 với mọi n ≥ 2 . Cũng vì tồn tại hai thời điểm thu được hai bộ số giông nhtau nên
P1 = 0 , suy ra a=
1 b=
1 c=
1 d=1 0 . Điều đó có nghĩa là bộ số ban đầu phải là (a, −a, a, −a ) .

Tài liệu sưu tầm và tổng hợp bản word đầy đủ liên hệ 0393732038 TÀI LIỆU TOÁN HỌC
55
Website:tailieumontoan.com
Câu III. (3 điểm) Cho tam giác ABC cân tại A với ∠BAC < 90o . Gọi P là giao điểm của BE với
trung trực của BC. Gọi K là hình chiếu vuông góc của P lên AB. Gọi Q là hình chiếu vuông góc
của E lên AP. Gọi giao điểm của EQ và PK là F .
1) Chứng minh rằng bốn điểm A, E, P, F cùng thuộc một đường tròn.
2) Gọi giao điểm của KQ và PE là L. Chứng minh LA vuông góc LE.
3) Gọi giao điểm của FL và AB là S . Gọi giao điểm của KE và AL là T . Lấy R là điểm đối xứng
với A qua L . Chứng minh rằng đường tròn ngoại tiếp tam giác AST và đường tròn ngoại tiếp tam
giác BPR tiếp xúc với nhau.
Lời giải
1) Ta có ∠PAE =
∠PAK =
∠EFK nên tứ giác AEPF nội tiếp.
2) Điểm A nằm trên đường tròn ngoại tiếp tam giác PEF nên theo định lý về đường thẳng Simson,
hình chiếu vuông góc của A trên ba cạnh của tam giác PEF thẳng hàng. Do K, Q lần lượt là hình
chiếu vuông góc của A trên PF và PE, KQ cắt PE tại L nên AL ⊥ PE .
3) Ta phát biểu một bổ đề như sau.
Bổ đề. Cho tam giác ABC. Đường cao AD. P là điểm bất kì trên AD. BP, CP cắt AC, AB lần lượt tại
E, F. Khi đó DA là phân giác ∠EDF .
Chúng minh. Qua A kẻ đường song song với BC, cắt DE, DF tại X, Y.
AX AE AY AF AE CD BF
= =
Ta có , , mà ⋅ ⋅ 1 (định lý Céva) nên AX = AY . Lại có
=
DC EC DB FB EC DB FA
AD ⊥ XY nên $D A$ là phân giác ∠EDF .
Trở lại bài toán:
Ta có ∠KLE =
∠QLE =
∠QAE = ∠EFK nên tứ giác ELFK nội tiếp. Gọi T ′ là giao của
∠QAK =
đường thẳng qua S song song với BC với AL. Ta có ∠AT ′ S =∠APL = ∠AKL nên tứ giác SLT ′ K
nội tiếp. Suy ra ∠LKT ′ =
∠LST ′ = ∠LFE = ∠LKE , suy ra K , E , T ′ thẳng hàng. Từ đó T ′ ≡ T .

Gọi Y là hình chiếu vuông góc của P trên AC. BY cắt ( AKP) tại X , cắt AL tại T ′′ . Ta có
PY = PK nên LP là phân giác ∠KLY . Mà BL ⊥ AT ′′ nên theo bổ đề trên, K , E , T ′′ thẳng hàng.
Suy ra T ′′ ≡ T .
Ta có ∠AXT =
∠APY =
∠APK = ∠AST nên X ∈ ( AST ) .
∠ABC =

∠PXB =
∠PAY =
∠PAB =
∠PRB nên X ∈ ( BPR) .

Do ∠AXP = 90° = ∠ATX + ∠PBX nên kẻ tiếp tuyến $X t$ của ( AST ) thì Xt cũng là tiếp tuyến của
( BPR) . Vậy ( AST ) tiếp xúc với ( BPR) tại X .
Câu IV. (1 điểm) Với a, b, c là những số thực dương thỏa mãn a + b + c =3 . Chứng minh rằng
2
1 1 1  4  a b c 
3  + + − 1 + 1 ≥ + 3 + + 
a b c  abc  bc ca ab 
Lời giải:

Tài liệu sưu tầm và tổng hợp bản word đầy đủ liên hệ 0393732038 TÀI LIỆU TOÁN HỌC
56
Website:tailieumontoan.com
Bài toán cần chứng minh tương đương với:

1 1 1
2

3 + +  + 4 ≥
4
+
3 a 2 + b2 + c2( 1 1 1
+ 6 + + 
)
a b c abc abc a b c

3(a + b + c) 2
2
1 1 1 4
⇔ 3 + +  + 4 ≥ +
a b c abc abc
2
1 1 1 31
⇔ 3 + +  + 4 ≥
a b c abc
Quy đồng và rút gọn ta đưa về chứng minh:

( )
3 a 2 b 2 + b 2 c 2 + c 2 a 2 + 4a 2 b 2 c 2 ≥ 13abc

( )
⇔ 3 a 2 b 2 + b 2 c 2 + c 2 a 2 − abc(a + b + c) ≥ 4abc(1 − abc)

( ) (
⇔ 81 a 2 b 2 + b 2 c 2 + c 2 a 2 − abc(a + b + c) ≥ 4abc (a + b + c)3 − 27 abc )
Không mất tính tổng quát, giả sử a ≤ b ≤ c , ta có các phân tích sau:

a 2 b 2 + b 2 c 2 + c 2 a 2 − abc(a + b + c=
) c 2 (a − b) 2 + ab(a − c)(b − c)

(a + b + c)3 − 27 abc = (a + b + 7c)(a − b) 2 + (4a + 4b + c)(a − c)(b − c) .

Khi đó, bài toán cần chứng minh tương đương với:
81c 2 (a − b) 2 + 81ab(a − c)(b − c) ≥ 4abc(a + b + 7c)(a − b) 2 + 4abc(4a + 4b + c)(a − c)(b − c) .

Với a ≤ b ≤ c , kết hợp a + b + c =3 ta dễ có 2 đánh giá sau:

81c 2 ≥ 4abc(a + b + 7c)

và 81ab ≥ 4abc(4a + 4b + c)

Kết hợp (a − c)(b − c) ≥ 0 ta có đpcm.

Đề số 7

Câu 1(3.5 điểm).

x 3  y 3  xy x  y   4

a) Giải hệ phương trình 

 xy  1x 2  y 2   4

Tài liệu sưu tầm và tổng hợp bản word đầy đủ liên hệ 0393732038 TÀI LIỆU TOÁN HỌC
57
Website:tailieumontoan.com
 Phân tích và lời giải. Hệ phương trình đã cho là hệ phương trình đối xứng dạng 1, do

đó ta có thể sử dụng phép đặt ẩn phụ S  x  y; P  xy . Tuy nhiên để đơn giản hóa ta cần

biến đổi hệ phương trình trước.Biến đổi tương đương hệ phương trình ta được


x  y   3xy x  y   xy x  y   4
3
x 3  y 3  xy x  y   4

  

xy  1 x 2  y 2  4 xy  1 x  y 2  2xy   4

 
  

 
x  y   2xy x  y   4
3

x  y   2xy x  y   4
3

 
   
xy  1 x  y   2xy   4
2
 
    2xy xy  1  4
2

 
 xy  1 x  y

  

Đến đây ta có thể sử dụng phép đặt ẩn phụ, tuy nhiên để ý đến vế phải hai phương trình

ta lấy hiệu theo vế của hai phương trình của hệ thì được

x  y   2xy x  y   xy  1x  y   2xy xy  1


3 2

 x  y  x  y  xy  1  2xy x  y  xy  1  0
2

 
 x  y  xy  1 x  y   2xy   0
2

 
x  1
x  y  xy  1  0 x  1y  1  0  x  1; y  1
  y  1 
  x 2  y 2  0    x  y  0
x  y   2xy  0
2

   
x  y  0

Thay vào hệ phương trình đã cho ta được x ; y   1;1 là nghiệm của hệ.

8x  3
b) Giải phương trình 7x  2  5  x  .
5

 Phân tích. Để ý rằng 7x  2  5  x   8x  3 , khi đó ta đặt 7x  2  a ; 5  x  b

a 2  b2  a  b 
thì phương trình đã cho được viết lại thành a  b   a  b 1   0.
5  5 

2
 Lời giải. Điều kiện xác định của phương trình là  x  5.
7

Đặt 7x  2  a; 5  x  b a  0;b  0 . Khi đó ta có a 2  b 2  8x  3 .

Như vậy phương trình đã cho được viết lại thành

a 2  b2  a  b  a  b
a b   a  b 1  0  
5  5  a  b  5


3
+ Với a  b ta có phương trình 7x  2  5  x  x 
8
Tài liệu sưu tầm và tổng hợp bản word đầy đủ liên hệ 0393732038 TÀI LIỆU TOÁN HỌC
58
Website:tailieumontoan.com

+ Với a  b  5 ta có phương trình 7x  2  5  x  5 hay ta được

7x  2  5  x  2 7x  2 5  x  25  33x  7x 2  10  9  3x
  x  3
9  3x  0 
 2   x 1
33x  7x 2  10  9  3x  16x 2  87x  71  0
 

 3 
Kết hợp với điều kiện xác định ta được tập nghiệm của phương trình là S   ;1
 8 

Câu 2(2.5 điểm)

a) Tìm tất cả các giá trị của m sao cho tồn tại cặp số nguyên x ; y  thỏa mãn hệ phương

trình
2  mxy 2  3m


 
2  m x 2  y 2  6m



Lấy hiệu theo vế hai phương trình của hệ ta được m xy 2  x 2  y 2  3m . 
+ Nếu m  0 , ta thấy hệ phương trình vô nghiệm.

+ Nếu m  0 , khi đó từ phương trình ta được


xy 2  x 2  y 2  3  y 2 x  1  x 2  1  2 
  
 y  x  1 x  1  2  y  x  1 1  x   2
2 2

Do x ; y  nhận xác giá trị nguyên nên ta xét các trường hợp sau

1  x  1 x  0
+ Trường hợp 1. Với  2   2 , hệ phương trình không có nghiệm
y  x  1  2 y  3
 
nguyên.
1  x  1 x  2 x  2

+ Trường hợp . Với  2  2   
y  x  1  2 y  1 y  1
  
1  x  2 x  1 x  1
+ Trường hợp 1. Với  2   2  
y  x  1  1 y  1 y  1
  
1  x  2 x  3
+ Trường hợp 1. Với  2    2 , hệ phương trình không có nghiệm
y  x  1  1 y  3
 
nguyên.
Vậy phương trình trên có các nghiệm nguyên là x ; y   1; 1, 1;1, 2; 1, 2;1 .

1
Thay các nghiệm trên vào hệ phương trình đã cho ta tìm được m  ; m  2 thỏa mãn.
2

Tài liệu sưu tầm và tổng hợp bản word đầy đủ liên hệ 0393732038 TÀI LIỆU TOÁN HỌC
59
Website:tailieumontoan.com

 1 
Vậy m   ;2 thỏa mãn yêu cầu bài toán.
 2 

b) Với x, y là những số thực thỏa mãn các điều kiện 0  x  y  2;2x  y  2xy . Tìm giá trị

 
lớn nhất của biểu thức P  x 2 x 2  1  y2 y2  1 .  
1 2 1
  2 . Áp dụng bất đẳng thức a 2  b 2  a  b 
2
Từ giả thiết 2x  y  2xy ta được
x y 2
ta có
2
1 4 1  1 2  1 4 1 4
      2  2  2  2  2  2
x 2
y 2
2  x y  x y x y
2
1 16 1  1 4 1 16
Hoàn toàn tương tự ta được 4  4   2  2   2  4  2  4 .
x y 2  x y  x y
 x2  4x 2
Do 0  x  y  2 nên ta có 1  2  y 2  4  0  y 2  4  x 2  2 .
 y  y
1 4
Từ đó kết hợp với 2
 2  2 ta được
x y
4x 2 4x 2 
2 4  1
y2  4  x 2   x 2
 y 2
 4  2 x 2
  4  x 2  2
  4  x 2. 2  5
y 2
y 2 
 y  x
 x4  16x 4
Hoàn toàn tương tự ta cũng có 1  4  y 4  16  0  y 4  16  x 4  4 .
 y  y

1 16
Từ đó kết hợp với 4
 2  4 ta được
x y
4x 4 16x 4 4
 16  4 1
y 4  16  x 4   x 4
 y 4
 16  2x 4
  16  x 2  
  16  x . 4  17
y4 y4  y 4  x
   
Do vậy P  x 2 x 2  1  y2 y2  1  x 4  y 4  x 2  y 2  17  5  22 . Dấu bằng xẩy ra khi

và chỉ khi x  1; y  2 .

Vậy giá trị lớn nhất của P là 22, đạt được tại x  1; y  2

Câu 3(3.0 điểm). Cho tam giác ABC nhọn nội tiếp đường tròn O  với AB  AC . Phân

 cắt BC tại D và cắt đường tròn


giác của góc BAC O  tại E khác A. Gọi M là trung điểm
của đoạn thẳng AD. Đường thẳng BM cắt đường tròn O  tại P khác B. Giả sử các đường

thẳng EP và AC cắt nhau tại N.

a) Chứng minh rằng tứ giác APNM nội tiếp và N là trung điểm của đoạn thẳng AC.

Tài liệu sưu tầm và tổng hợp bản word đầy đủ liên hệ 0393732038 TÀI LIỆU TOÁN HỌC
60
Website:tailieumontoan.com
 nên ta
Do AE là phân giác của góc BAC
A P

có E là điểm chính giữa cung BC.


  ANP
Từ đó ta được AMP  nên tứ giác
M N
AMNP nội tiếp đường tròn.
O Q
  ANM
Do đó APM  . Lại có R

  ACB
APM  nên suy ra ANM
  ACB
. B D C

Từ đó dẫn đến MN dong song với BC,

mà M là trung điểm AD nên suy ra N là E

trung điểm AC

b) Giả sử đường tròn K  ngoại tiếp

tam giác EMN cắt đường thẳng AC tại

Q khác N. Chứng minh B và Q đối xứng

nhau qua AE.

Không mất tính tổng quát ta giả sử Q nằm giữa N và C(các trương hợp còn lại chứng

minh tương tự).


  MNA
Do tứ giác EMNQ nội tiếp nên MEQ  . Mà ta có MNA
  ACB
 và ACB
  AEB

  AEB
nên ta suy ra được AEQ  . Lại có BAE  và AE chung nên suy ra
  CAE

ABE  ACE .

Do đó AB  AG và EB  EQ nên AE là đường trung trực của BQ, suy ra Q và B đối

xứng nhau qua đường thẳng AE.

c) Giả sử đường tròn K  cắt đường thẳng BM tại M. Chứng minh rằng RA vuông góc RC

  EMR
Tứ giác ERMN nội tiếp đường tròn nên ta có ENR   AMP
.

  ANP
Lại có ENC   AMP
 nên ta được ERN
  ENC
.

  PMN
Ta có REN   PAN
  PEC   CEN
 và REN 

Kết hợp với cạnh NE chung ta suy ra được REN  CEN

1
Suy ra RN  NC  NA nên RN  AC , điều này dẫn đến tam giác RAC vuông tại R
2
hay ta được RA vuông góc với RC.
Tài liệu sưu tầm và tổng hợp bản word đầy đủ liên hệ 0393732038 TÀI LIỆU TOÁN HỌC
61
Website:tailieumontoan.com
Câu 4(1.0 điểm). Số nguyên a được gọi là số “đẹp” nếu với mọi cách sắp xếp theo thứ tự

tùy ý của 100 số 1, 2, 3,…, 100 luôn tồn tại 10 số hạng liên tiếp có tổng lớn hơn hoặc bằng a.

Tìm số “đẹp” lớn nhất

1  100100
 Lời giải. Tổng của 100 số của dãy số là  5050 . Chia 100 số thành 10 bộ số
2
gồm 10 số liên tiếp thì trung bình tổng của 10 bộ số này là 505. Khi đó tồn tại ít nhất một

bộ số mà tổng 10 số đó lớn hơn hoặc bằng 505. Ta sẽ chứng minh a lớn nhất chỉa có thể

bằng 505 bằng cách cách chọn ra ví dụ mà tổng 10 số liên tiếp bất kỳ nhỏ hơn hoặc bằng

505, khi đó mọi số a lớn hơn 505 đều không thỏa mãn.

Thật vậy, xét cách sắp xếp sau 100,1, 99,2, 98, 3, , 51, 50 (chia thành các cặp có tổng

bằng 101, viết số lớn đứng trước rồi xếp các cặp cạnh nhau theo thứ tự giảm dần của số

lớn hơn). Nếu 10 số liên tiếp gồm 5 cặp số như vậy thì tổng 10 số này là 505. Nếu không

10 số gồm số đầu nhỏ hơn trong một cặp và kết thúc là số lớn hơn trong một cặp khác. Các

số này thuộc sáu cặp khác nhau là x,101  x, x  1,102  x, , x  4,105 – x và 10 số được

chọn là các số được chọn là các số 101  x đến x  5 (trong dãy trên ). Dễ thấy tổng 10 số

liên tiếp bất kỳ đều không vượt quá 505. Vậy a  505

Đề số 8

Câu 1 (3.5 điểm).

x 2  4y 2  5

a) Giải hệ phương trình  2
4x  8xy 2  5x  10y  1


Biến đổi tương đương hệ phương trình ta có

x 2  4y 2  5  
x  2y   4xy  5 x  2y   4xy  5
2 2

 2  
4xy x  2y   5 x  2y   1

4x  8xy 2  5x  10y  1  4xy  5x  2y   1
 
Đặt a  x  2y;b  4xy , khi đó hệ phương trình trên trở thành

a 2  b  5 b  a 2  5 b  a 2  5 b  a 2  5 b  4


    
   2   3   3  
ab  5a  1
   
a a  5  5a  1 a  5a  5a  1

a  1

a  1


 x  1; y  1
x  2y  1 
Từ đó ta có hệ phương trình    .

xy   1 x  2; y   1
  2
Tài liệu sưu tầm và tổng hợp bản word đầy đủ liên hệ 0393732038 TÀI LIỆU TOÁN HỌC
62
Website:tailieumontoan.com
 1
Vậy hệ phương trình đã cho có các nghiệm là x ; y   1;12;   .
 2 

64x 3  4x
b) giải phương trình 5x 2  6x  5 
5x 2  6x  6

Dễ thấy 5x 2  6x  5  5x 2  6x  5  2x 2  2  3 x  1  0 với mọi x. Do đó điều kiện


2

xác định của phương trình là x  R . Phương trình đã cho được viết lại thành

 
5x 2  6x  5 5x 2  6x  5  1  4x   4x
3

Đặt a  5x 2  6x  5;b  4x a  0 . Khi đó phương trình trên được viết lại thành

   
a a 2  1  b 3  b  a  b  a 2  ab  b 2  1  0  a  b

Từ đó ta được 5x 2  6x  5  4x , phương trình tương đương với

x  0 x  0
   x 1
5x 2  6x  5  16x 2 11x 2  6x  5  0
 
Vậy phương trình đã cho có nghiệm duy nhất là x  1 .

Câu 2(2.5 điểm).

x 2  1 y2  1
a) Với x, y là những số nguyên thỏa mãn đẳng thức  . Chứng minh
2 3

x 2  y 2  40 .

x 2  1 y2  1
Biến đổi giả thiết
2

3
   
 3 x 2  1  2 y 2  1  3x 2  2y 2  1

Vì số chính phương chia 5 dư 0 hoặc 1 hoặc 4, mà 3x 2  2y2  1 nên x 2 và y 2 chia cho 5 có

cùng số dư 1, từ đó ta được x 2  y 2  5

Vì số chính phương chia 8 dư 0 hoặc 1 hoặc 4, mà 3x 2  2y2  1 nên x 2 và y 2 chia cho 8

có cùng số dư 1, từ đó ta được x 2  y 2  8

Do 5 và 8 nguyên tố cùng nhau nên ta được x 2  y 2  40

b) Tìm tất cả các cặp số nguyên x ; y  thỏa mãn đẳng thức sau x 4  2x 2  y 3 .

Ta có x 4  2x 2  y 3  x 4  2x 2  1  y 3  1  x 2  1  y  1y2 – y  1 .
2

Tài liệu sưu tầm và tổng hợp bản word đầy đủ liên hệ 0393732038 TÀI LIỆU TOÁN HỌC
63
Website:tailieumontoan.com

 
Gọi d  y  1; y2  y  1 . Khi đó ta có y  1 d và y 2  y  1d nên ta được
2

y  1  y 
2
2
 y  1 d  3y d

Do d là nguyên tố nên ta có hai trường hợp

+ Khi 3 d ta được x 2  1  y  1y2 – y  1 9 nên x 2  1  9  x 2  1 3 . Điều này


2 2

vô lý vì số chính phương chia cho 3 không thể có số dư là 2.

+ Khi 3 d ta được y d , kết hợp với y  1d ta suy ra được d  1 .


Do đó y  1; y2 – y  1  1 . 
 
Khi đó do y  1 y2 – y  1 là số chính phương nên ta đặt y  1  a 2 ; y2 – y  1  b2

trong đó a, b là các số nguyên dương và a;b   1 . Tứ đó ta được

      
2
b2  a 2  1 – a 2  1  4b2  4a 4  12a 2  12  2b – 2a 2  3 2b  2a 2  3  3

Vì 2b  2a 2  3  2b  2a 2  3 nên ta xét các trường hợp sau


2 2

2b  2a 2  3  1 b  1
+ Trường hợp 1. Với 
  
 , hệ không có nghiệm nguyên.
2b  2a 2  3  3 a 2  2
 
2b  2a 2  3  3 b  1 a  1 y  1  1
+ Trường hợp 2. Với        2  x  y  0.
  2 
2b  2a 2  3  1 a  1 b  1 y – y  1  1
   
Thử lại vào phương trình ban đầu ta thấy thỏa mãn. Vậy phương trình có nghiệm duy

nhất là 0; 0 .

Câu 3 (3.0 điểm). Cho hình vuông ABCD nội tiếp đường tròn O  . P là điểm thuộc cung

nhỏ AD của đường tròn O  và P khác A, D. Các đường thẳng PB, PC lần lượt cắt AD tại

AD tại M, N. Đường trung trực của AM cắt đường thẳng AC, PB lần lượt tại E, K. Đường

trung trực DN cắt các đường thẳng BD, PC lần lượt tại F, L.

Lời giải.

Tài liệu sưu tầm và tổng hợp bản word đầy đủ liên hệ 0393732038 TÀI LIỆU TOÁN HỌC
64
Website:tailieumontoan.com

a) Chứng minh rằng ba điểm K, O, L thẳng hàng.


  KMA
Ta có KA  KM suy ra tam giác AKM cân. Do đó ta được KAM ,

  KBA
Mà ta lại có KMA   900 và KAB
  KAM
  900 nên suy ra   hay tam
KAB  KBA

giác AKB cân ta K. Do đó ta được KA  KB  KM .

Lại có OB  OD nên OK là đường trung bình của tam giác DKM, suy ra OK // MD.

Chứng minh tương tự ta có OL là đường trung bình của tam giác NCA, suy ra OL // AD

Theo tiên đề Ơclit thì ba điểm K, O, L thẳng hàng.

b) Chứng minh đường thẳng PO đi qua trung điểm của EF


  450 nên tam giác EAM vuông cân.
Ta có E thuộc đường trung trực AM và EAM

Do đó suy ra ME vuông góc với AC. Hoàn toàn tương tự ta cũng có NF vuông góc với BD.

PB PC
Ta có MN song song với BC nên theo định lí Talet ta có  .
MB NC
Hạ PX vuông góc với AC và PY vuông góc với BD, khi đó ta có PX, EM, BO cùng song

song với nhau.

XO PB
Do đó ta được  .
EO PM

Tài liệu sưu tầm và tổng hợp bản word đầy đủ liên hệ 0393732038 TÀI LIỆU TOÁN HỌC
65
Website:tailieumontoan.com
YO PC
Lại có PY, FN, CO cùng song song với nhau nên ta cũng có  .
FO NC

XO YO
Từ đó dẫn đến  nên suy ra XY và EF song song với nhau.
EO FO
  PYO
Ta có PXO   XOY
  900 nên tứ giác PXOY là hình chữ nhật, do đó PO đi qua

trung điểm của XY. Do XY song song với EF nên PO đi qua trung điểm của EF.

c) Giả sử đường thảng EK cắt đường thẳng FL và AC cắt nhau tại T. Đường thẳng ST cắt

các đường thẳng PB, PC lần lượt tại U và V. Chứng minh rằng bốn điểm K, L, V, U cùng

thuộc một đương tròn.


  OAD
Ta có LK song song với AD nên LK vuông góc với ES. Do đó KOA   450 nên

  450 .
KEO
  900 nên OK là phân giác của góc EOS
Mà ta có EOS .

Suy ra tam giác EOS cân nên ta có KS  KE , suy ra KL là đường trung trực của ES hay E

và S đối xứng với nhau qua KL. Hoàn toàn tương tự ta có F và T đối xứng qua KL.
  STO
Từ đó ta được EOF  SOT nên EFO .

Gọi giao điểm của OP và EF là I, ta có I là trung điểm của EF.

Do tam giác OEF cân nên ta có IO  IE  IE .


  IF
Suy ra tam giác IOF cân nên IOF  .
O  OTS
  IOF
Mà IOE   EOF
  900 nên IOE
  OTS
  900 .

  IOE
Gọi giao điểm của OP và ST là H nên ta cóTOH   HTO
 , suy ra TOH   900 .

  900 hay PO vuông góc với ST.


Từ đó suy ra THO
  PCD
Ta có PLF   PBD
 và PCD   BPO
 nên PLF
  PBO
  PVH
.

  HVP
Lại có PH vuông góc với UV nên VPH   900 .

  PLK
Mà ta lại có PLF   PLK
  900 nên   .
PLK  HVP
  UVP
Từ đó suy ra PLK  hay tứ giác KLUV nội tiếp đường tròn.

Tài liệu sưu tầm và tổng hợp bản word đầy đủ liên hệ 0393732038 TÀI LIỆU TOÁN HỌC
66
Website:tailieumontoan.com
Câu 4(1.0 điểm). Chứng minh rằng với mọi số tự nhiên n  3 luôn tồn tại cách xếp bộ n số

xi  xk
1, 2, 3,...,n thành bộ số x 1, x 2 , x 3 ,..., x n sao cho x j  với mọi bộ chỉ số i; j ; k  mà
2
1i  j k n .

Lời giải.

a j  ak
Dãy a1 , a2 , a 3 ,..., as chiều dài s  3 tùy ý được gọi là dãy “tốt” nếu a j  với mọi
2
chỉ số (i, j, k) thỏa mãn ( 1  i  j  k  s ).

Nếu dãy a1 , a2 , a 3 ,...., as là dãy tốt thì dãy 2a1 , 2a2 , 2a 3 ,..., 2as và dãy

2a1  1, 2a2  1, 2a 3  1,..., 2as  1 cũng là dãy tốt.

Từ nhận xét trên ta suy ra nếu dãy x 1 , x 2 , x 3 ,..., x s là dãy tốt của các số 1, 2, 3, ... , s ( s  3 )

thì dãy 2x 1 , 2x 2 , 2x 3 ,..., 2x s , 2x 1  1, 2x 2  1,..., 2x s  1 là dãy tốt của các số 1, 2, 3, ..., 2s (chú ý

2x k  2x m  1
rằng không là số nguyên).
2
+) (1, 3, 2) là dãy tốt của các số 1, 2, 3.

+) Với n  3 luôn tồn tại k để 3.2k 1  n  3.2k . . Theo nhận xét trên, ta xây dựng được dãy

tốt từ các dãy tốt từ các số 1, 2, 3,..., 3.2k sau đó ta bỏ đi các số n  1, n  2, n  3,...., 3.2k

chúng ta nhận được dãy tốt từ các số 1, 2, 3, ..., n (trên dãy tốt ta bỏ đi các số hạng bất kì

thì dãy còn lại vẫn là dãy tốt).

Cách khác:

Với n  3 ta có cách sắp xếp 1, 3, 2.

Ta chứng minh rằng nếu bài toán đúng với n sẽ đúng với 2n.

Thật vậy, giả sử ta có cách xắp xếp đúng với n thì cách sắp xếp đó có dạng

xi  xk
x 1, x 2 , x 3 , x 4 ,..., x n thỏa mãn với mọi 1  i  j  k  n ta có x j  . Ta chứng minh
2
tồn tại dãy 2n thỏa mãn đề bài .

Xét dãy sau 2x1,2x 2 ,2x 3 ,...,2x n , 2x1  1,2x 2  1,2x 3  1,...,2x n  1 , dãy số gồm tất cả các số

từ 1 đến 2n. Xét a  b bất kỳ cùng thuộc dãy

a b
+ Nếu a, b khác tính chẵn lẻ thì không thuộc dãy (thỏa mãn)
2
Tài liệu sưu tầm và tổng hợp bản word đầy đủ liên hệ 0393732038 TÀI LIỆU TOÁN HỌC
67
Website:tailieumontoan.com
a b
+ Nếu a, b cùng tính chẵn lẻ thì thuộc dãy
2
Nếu a, b cùng chẵn (trường hợp a, b cùng lẻ chứng minh tương tự )

a b a b
Trường hợp 1. Khi lẻ thì không thể nằm giữa a, b do cách xây dựng dãy (thỏa
2 2
mãn)

a b
Trường hợp 2. Khi chẵn.
2

a b a b
Giả sử rằng nằm giữa a, b trong dãy khi đó a  2x i , b  2x k ,  2x j với
2 2

i  j  k suy ra dãy ban đầu x1,x 2 ,x 3 ,..., x n là cách sắp xếp không thỏa mãn đề bài (mâu

thuẫn )

a b
Vậy điều giả sử là sai nên không nằm giữa a và b trong dãy.
2
Vậy với mọi trường hợp trung bình cộng của a, b không thể nằm giữa a, b suy ra đã xây

dựng được cách xếp thỏa mãn cho trường hợp 2n. Như vậy đã chứng minh được rằng nếu

bài toán đúng với n thì đúng với 2n.

Mặt khác bài toán đúng với n thì đúng với n  1 . Nên theo nguyên lí quy nạp ta có điều

phải chứng minh.

Đề số 9

Câu 1.
1). Giả sử a; b là hai số thực phân biệt thỏa mãn
 2
 a  3b  2
a).  2  a 2  b2  3a  b  0 .
b  3a  2

 a  ba  b  3a  b  0  a  ba  b  3  0
 a  b  0 ( l)

 a  b  3
.

3
b). Với a  b  3  a  b  27
 a 3  b3  3ab a  b  27  a 3  b3  9 ab  27
2
 a 2  3a  b2  3b  4  a  b  2 ab  3a  b  4  ab  2 .
Vậy a 3  b3  45 .

Tài liệu sưu tầm và tổng hợp bản word đầy đủ liên hệ 0393732038 TÀI LIỆU TOÁN HỌC
68
Website:tailieumontoan.com
Nhận xét: Đây là bài toán hết sức cơ bản, đề yêu cầu gì ta làm đó, với một vài phép biến
đổi tương đương cùng kết hợp hằng đẳng thức quen thuộc ta sẽ suy ra được điều phải
chứng minh.
Nhắc lại kiến thức và phương pháp:
2
• Hằng đẳng thức a  b  a2  2ab  b2
3
và a  b  a3  3a2 b  3ab2  b3 .
ma 2  nb  p (1)
• Hệ phương trình đối xứng dạng  2 .
mb  na  p (2)

Lấy (1) + (2), ta có: ma2  b2   na  b  2 p .
a  b

Lấy (1) – (2), ta có: ma  b   na  b  
2 2
.
a  b  n
 m
Từ đó suy ra:
   n2  n
2 p  m a  b  2 ab  na  b  m 2  2 ab  n.  ab  ...
2

   m  m
Bài tập kết thúc.
Bài tập tương tự:
1. Cho a; b thỏa mãn a2  4b  3; b2  4a  3 . Tính a5  b5 .
2. Cho a; b thỏa mãn a2  2b2  4b  7; b2  2a2  4a  7 . Tính giá trị của biểu thức
a3  b3  ab .

2). Ta thấy x  y  0 là nghiệm của phương trình.


Nếu y  0 , nhân hai vế của phương trình với y , ta được
 
2 xy  3 y  5 xy  2 x  3 y  5 xy
2 2

 2 
  2 2 2
4 x  y  5 xy
2 2
4 x  y  5 xy

2 x  3 y  5 xy 2 x  3 y  5 xy

  2  
 2

 2 x  xy  y 2
 0  2
4 x  y  5 xy
2
 
2 x  3 y  5 xy 2 x  3 y  5 xy
   
 x  y2 x  y  0  x  y2 x  y  0
 
2 x  3 y  5 xy
  x y1
 x  y  0
 
 .
2 x  3 y  5 xy 2  4
 
2 x  y  0  x  5 , y  5

Nhận xét: Bài toán sử dụng phương pháp thế tích từ phương trình một xuống phương
trình hai của hệ, và đưa về phương trình đẳng cấp bậc hai để tìm mối liên hệ giữa hai biến
rồi thế ngược lại phương trình một, tìm nghiệm của hệ phương trình.
Nhắc lại kiến thức và phương pháp:
• Phương trình đẳng cấp bậc hai ẩn x , y là ax 2  bxy  cy 2  0 (*).
Làm nháp: Chia phương trình (*) cho y 2 ta có:

Tài liệu sưu tầm và tổng hợp bản word đầy đủ liên hệ 0393732038 TÀI LIỆU TOÁN HỌC
69
Website:tailieumontoan.com
2
 x  x x x
a    b    c  0 . Coi đây là một phương trình bậc hai ẩn , giả sử tìm được  m
 y   y  y y
x
hoặc  n , do đó ta phân tích được
y
 x  my
(*)  ax 2  bxy  cy 2  0   x  my x  ny  0   .

 x  ny
Ý tưởng: Dạng hệ sử dụng phương pháp thế khá là đặc trưng nhưng cái khó là tìm các đại
lượng biểu diễn nhân tử chung giữa hai biến x; y . Nhìn vào vế trái của từng phương
trình, với phương trình một là bậc nhất, còn phương trình hai là bậc hai, tương tự ở vế
phải, bậc của phương trình hai cũng lớn hơn một bậc so với phương trình một. Điều nó sẽ
làm ta nghĩ đến phép nhân để đồng bậc hóa chúng, nhân ở đây sẽ là nhân chéo từng
phương trình. Tuy nhiên nếu quan sát kỹ hơn một chút, cả hai phương trình đều xuất hiện
5xy nên ta sẽ thế 5 xy  2 x  3 y xuống phương trình hai (thực chất đây cũng là việc nhân
chéo nhưng ta đã giảm biến y , khi đó hệ phương trình đã cho trở thành:
2 x  3 y  5 xy
2 x  3 y  5 xy 2 x  3 y  5 xy  
    x  y .
4 x 2  y 2  2 x  3 y y 2 x 2  xy  y 2  0 
  
 2 x  y  0
Việc còn lại chỉ là tìm nghiệm của hệ phương trình đã cho.
Bài toán kết thúc.
Bài tập tương tự:

x  3 y  5 xy
1. Giải hệ phương trình  2 .

2 2
x  y  5 xy

2 1  1
Đáp số:  x; y  0; 0 ,  ;   , 1;  .
5 5  2

3 x  3 y  6 xy
2. Giải hệ phương trình  2 .
 2 2
9 x  y  6 xy

1   5 5
Đáp số:  x; y  0; 0 ,  ;  1 ,  ;  .
3  6 4
Câu 2.
1). Ta có xy  1   x  1 y  1 , suy ra xy  1  xy  1  x  y .
Mà xy  1  x  y  xy  1  x  y   x  1   y  1   x  1 y  1 , suy ra x  1  y  1 và y  1  x  1 ,
nên x  y
2
x 2  1   x  1 ta có x  1  x  1 , suy ra 2  x  1 , nên x  2 hoặc x  3 .
Nhận xét: Bài toán tìm số nguyên thỏa mãn một điều kiện, sử dụng tính chất chia hết của
một tổng, hiệu, tích,…
Nhắc lại kiến thức và phương pháp:
• Biến đổi đại số.
Ta có ( x − 1)( y − 1) = xy − x − y + 1 nên:
( xy − 1)  ( x − 1)( y − 1) ⇔ ( xy − 1)  ( xy − x − y + 1) .
• Một số luôn chia hết cho chính nó.
Tài liệu sưu tầm và tổng hợp bản word đầy đủ liên hệ 0393732038 TÀI LIỆU TOÁN HỌC
70
Website:tailieumontoan.com

Ta có ( xy − x − y + 1)  ( xy − x − y + 1) .
• Hai số chia hết cho một số thì hiệu của hai số đó.
( xy − 1)  ( xy − x − y + 1)
 ⇒ ( x + y − 2 )  ( xy − x − y + 1)
( xy − x − y + 1)  ( xy − x − y + 1)
⇔ ( x − 1) + ( y − 1)  ( xy − x − y + 1) ⇔ ( x − 1) + ( y − 1)  ( x − 1)( y − 1) .
• Tính chất chia hết a  b thì ka  b với a; b; k là các số nguyên.
( x − 1) + ( y − 1)  ( x − 1)( y − 1) ⇒ ( x − 1) + ( x − 1)( y − 1)  ( x − 1)( y − 1) .
2

• Một tổng hai số hạng chia hết cho một số trong đó có một số hạng chia hết cho số đó
thì số hạng còn lại cũng chia hết.
( x − 1)2 + ( x − 1)( y − 1)  ( x − 1)( y − 1) 2
   x  1   x  1 y  1   x  1   y  1 ;
( x − 1)( y − 1)  ( x − 1)( y − 1)
hoàn toàn tương tự ta có ( y − 1)  ( x − 1) .
• Có hai số thỏa mãn: Số thứ nhất chia số thứ hai và số thứ hai chia hết cho số thứ nhất
thì hai số bằng nhau.

 x  1   y  1
Ta có   x 1  y 1  x  y .



 y  1   x  1

Thay lại vào đề bài ta có ( x 2 − 1)  ( x − 1) ⇔ ( x − 1)( x + 1)  ( x − 1)


2 2

⇔ ( x + 1)  ( x − 1) ⇔ ( x − 1) + 2   ( x − 1) ⇒ 2  ( x − 1) .
• Số thứ nhất chia hết cho số thứ hai thì số thứ hai là ước của số thứ nhất.
2  ( x − 1) ⇒ ( x − 1) ∈ U( 2 ) ={±1; ± 2} .

vì x ≥ 2 ⇔ x − 1 ≥ 1 suy ra ( x − 1) ∈ {1; 2}
x − 1 = 1 x = 2 ⇒ y = 2
⇔ ⇔ .
x − 1 = 2 x = 3 ⇒ y = 3
Vậy x= y= 2 hoặc x= y= 3 .
x 2 y 2  1
2). Ta có x 2 y 2  2 y  1  0  2 y  x 2 y 2  1  y 
2
xy xy
P 
3x y  1  2 3 x 2 y 2  1
2 2

 3 Px 2 y 2  2 xy  P  0 .
Ta có  4  12P 2 .
Phương trình có nghiệm khi  0
1 1 1
 4  12 P 2  0  1  3 P  1  P P .
3 3 3
3 12 3 7
Vậy MaxP = ⇔ x =− và y = − .
3 21 6
Nhận xét: Bài toán tìm giá trị của một biểu thức sau khi biến đổi biểu thức trở thành tham
số của một phương trình bậc hai theo một ẩn
Nhắc lại kiến thức và phương pháp:

Tài liệu sưu tầm và tổng hợp bản word đầy đủ liên hệ 0393732038 TÀI LIỆU TOÁN HỌC
71
Website:tailieumontoan.com
• Biến đổi giả thiết.
x 2 y 2  1
Ta có x 2 y 2  2 y  1  0  2 y  x 2 y 2  1  y  .
2
• Đưa biểu thức đã cho về một ẩn hoặc một biểu thức cố định.
−x2 y 2 − 1 xy
Thay y = vào biểu thức P ta có P = 2 2
2 −x y − 1
3 +1
2
2 xy 2 xy
= = .
( 2 2
3 − x y − 1 + 2 −3 x y − 1
2 2
)
2a
Đặt xy = a ta được P = .
−3a 2 − 1
• Tìm giá trị lớn nhất, nhỏ nhất của một biểu thức bằng cách đưa biểu thức đó trở thành
tham số của một phương trình bậc hai
2a
=P 2
+ P 0 Khi đó ta có ∆′ = 12 − 3P 2 .
⇒ 3 Pa 2 − 2 a=
−3a − 1
• Phương trình bậc hai có nghiệm khi và chỉ khi ∆ hoặc ∆ ' không âm.
Ta có Vì phương trình có nghiệm nên ∆′ ≥ 0 ⇔ 1 − 3P 2 ≥ 0
( )(
⇔ 1 − 3P 1 + 3P ≥ 0 ⇔ ) ( 3P − 1 )( )
3P + 1 ≥ 0

−1 1 1
⇔ −1 ≤ 3 P ≤ 1 ⇔ ≤P≤ ⇒P≤ .
3 3 3
• Khi xét đến giá trị nhỏ nhất, giá trị lớn nhất của biểu thức cần phải xét xem “dấu bằng
xảy ra khi nào?”.
1
Ta có dấu bằng xảy ra khi vào chỉ khi P = ⇔ ∆ ' = 0 ⇒ Phương trình bậc hai có
3
2
nghiệm kép a = , thay vào biểu thức ban đầu ta có:
3
2
2
 2  7 7 xy 3 12
 
 3   2 y  1  0  2 y   3  y   6  x  y  7   7 3 .

6
3 12 3 7
Vậy MaxP = ⇔ x =− và y = − .
3 21 6
Câu 3.

Tài liệu sưu tầm và tổng hợp bản word đầy đủ liên hệ 0393732038 TÀI LIỆU TOÁN HỌC
72
Website:tailieumontoan.com

BD AB
1). Ta có AD là phân giác   mà BED; CDF là tam giác cân,
DC AC
BE AB
   BC  FE .
CF AC
Nhận xét. Bài toán chứng minh hai đường thẳng song song sử dụng định lý Ta-lét đảo
Nhắc lại kiến thức và phương pháp.
• Tâm đường tròn nội tiếp tam giác là giao điểm của ba đường phân giác.
Trong ∆ABC có I là tâm đường tròn nội tiếp nên suy ra AI hay AD là phân giác của
.
BAC
• Tính chất đường phân giác trong tam giác.
DB AB
Trong ∆ABC có AD là phân giác nên = .
DC AC
• Hai điểm đối xứng nhau qua một đường thẳng thì đường thẳng đó là trung trực của
đoạn thẳng nối hai điểm.
+ E là điểm đối xứng của D qua BI nên BI là trung trực của DE ;
+ F là điểm đối xứng của D qua CI nên CI là trung trực của DF .
• Một điểm thuộc trung trực của một đoạn thẳng thì điểm đó cách đều hai đầu mút của
đoạn thẳng đó.
+ B ∈ BI mà BI là trung trực của DE nên BD = BE ;
+ C ∈ CI mà CI là trung trực của DF nên CD = CF ;
BD BE BE AB BE CF
suy ra = do đó = ⇔ =
CD CF CF AC BA CA
• Định lý Ta-lét đảo: Hai điểm thuộc hai đoạn thẳng, chia hai đoạn thành các đoạn
thẳng tương ứng tỷ lệ thì hai đường thẳng chứa hai đoạn thẳng đó song song.
BE CF
Theo định lý Ta-lét đảo ta có
= thì EF  BC .
BA CA
2). Ta có   EDB
BC  EF  EFD   BED
 .
  180 0  AEM
Mà APM   APM
  BED   DEF
.
  APN
Tương tự: DFE   APN
  APM
  DFE
  FED
  MPN
.
  MDN
Mà MJN   EDF
  MJN
  MPN
  180 0  MPNJ nội tiếp.

Tài liệu sưu tầm và tổng hợp bản word đầy đủ liên hệ 0393732038 TÀI LIỆU TOÁN HỌC
73
Website:tailieumontoan.com
Nhận xét: Bài toán chứng minh tứ giác nội tiếp bằng cách chứng minh tổng hai góc trong
bằng 180° .
Nhắc lại kiến thức và phương pháp:
• Hai đường thẳng song song tạo ra các góc ở vị trí so le trong bằng nhau.
 = EDB
Ta có EF  BC , suy ra FED .
• Tam giác có hai cạnh bằng nhau là tam giác cân, tam giác cân có hai góc kề cạnh đáy
bằng nhau.
 = BED
Tam giác ∆BDE có BD = BE nên ∆BDE cân tại B suy ra BDE  , suy ra

FED 
= EDB .
= BED
• Tứ giác nội tiếp có tổng hai góc đối diện bằng 180° .
Tứ giác APME là tứ giác nội tiếp nên  + AEM
APM = 180 0
 =180 0 − AEM
⇔ APM  =BED
 ⇒ APM .
 =DEF

Hoàn toàn tương tự có


  APN
DFE   APN
  APM
  DFE
  FED
  MPN
.
• Trung điểm; Đường trung bình của tam giác.
 MJ  DF

+ MJ là đường trung bình của ∆DFE nên  1 ;
 MJ = DF
 2
DF
+ N là trung điểm của DF nên DN = ;
2
 MJ  DN ( DF )
suy ra  .
=MJ DN =( DF )
• Tứ giác có một cặp cạnh song song và bằng nhau là hình bình hành.
Tứ giác MJND có MJ  DN và MJ = DN nên MJND là hình bình hành.
• Hình bình hành có hai góc đối diện bằng nhau.
  MDN
+ MJND là hình bình hành nên MJN ;
  EDF
+ MDN  (hai góc so le trong của MN  FE suy ra tính chất đường trung bình);
  MPN
suy ra MJN   180 0 .

• Tứ giác có tổng hai góc trong đối diện bằng 180° là tứ giác nội tiếp.
  MPN
Tứ giác MPNJ có MJN   180 0 nên tứ giác MPNJ là tứ giác nội tiếp.

  DEF
3). Ta có APM   JNM
 và JPM   JEM
  JPM
  APM
 , suy ra 3 điểm A; P ; J thẳng

hàng.
Nhận xét: Bài toán chứng minh ba điểm thẳng hàng bằng cách chứng minh ba điểm cùng
thuộc một đường thẳng đặc biệt
Nhắc lại kiến thức và phương pháp:
• Tứ giác nội tiếp có góc ngoài và góc trong không kề với nó bằng nhau.
 = BED
Tứ giác MPAE là tứ giác nội tiếp nên MPA  = DEF
 mà BED  ( = BDE
 ), suy ra
 = DEF
MPA .
• Trung điểm, Đường trung bình, Hình bình hành,…(đã nhắc lại phần trên).

Tài liệu sưu tầm và tổng hợp bản word đầy đủ liên hệ 0393732038 TÀI LIỆU TOÁN HỌC
74
Website:tailieumontoan.com
 MN  FE

+ MN là đường trung bình của ∆DEF nên  1 ;
 MN = FE
 2
1
+ J là trung điểm FE nên EJ = EF ; suy ra MN  JE và JE = MN nên MNJE là hình
2
    (hai góc nội tiếp cùng chắn cung MJ
bình hành nên MEJ = MNI mà MEJ = MNP  của
 , suy ra 
  DEF
đường tròn ngoại tiếp tứ giác MNPJ ) suy ra JPM  nên
JPM = APM
PJ ≡ PA hay ba điểm A; P ; J thẳng hàng.
Câu 4.
1). Theo đề bài, các số nguyên dương được sắp xếp theo từng hàng chéo của bảng: Hàng
chéo thứ nhất có 1 số, hàng chéo thứ hai có 2 số,……
Giả sử số x nằm ở hàng chéo thứ k thì ta có:
k  k  1 k  k  1 1  1  8 x 1  1  8x
x  k
2 2 2 2
1  1  8 x
k .
2

1  1  8.2015
Áp dụng x  2015 ta có k   63 .
2
k  k  1
Số đầu tiên ở hàng chéo thứ k  63 là  1  1954 .
2
Như vậy số 2015 nằm ở vị trí thứ 2015  1954  1  62 của hàng chéo thứ 63 (vị trí áp chót)
Tọa độ của nó là 2,62 .
2). Theo Côsi cho 4 số ta có
4  abc  ab  bc  ac  4 4 a 3 b3 c 3  1  abc  a  b  c  3 3 abc  3 3 a 2 b2 c 2 .
BĐT tương đương a 2  b2  c 2  3 3 a 2 b2 c 2  2 ab  bc  ac (1).
Đặt 3
a 2  x; 3 b2  y ; 3 c 2  z ( x; y ; z  0 ).
 x 3  y 3  z 3  3 xyz  2 x 3 y 3  2 z 3 x 3  2 z 3 y 3 .
Áp dụng bất đẳng thức Schur bậc 3:
x 3  y 3  z 3  3 xyz  xy  x  y  yz  y  z  xz  x  z
 x  x  y x  z  y  y  x y  z  z  z  x z  y  0
với mọi số thực không âm x; y; z .
Chứng minh bất đẳng thức
Do vai trò x; y; z như nhau, giả sử x  y  z  z  z  x z  y  0 .
Ta xét x  x  z  y  y  z  x 2  xz  yz  y 2   x  y x  y  z  0
 x  x  z x  y  y  y  z x  y  0  x  x  z x  y  y  y  z y  x  0
 x  x  y x  z  y  y  x y  z  z  z  x z  y  0 .
(điều phải chứng minh).
Ta có x  y  z  3xyz  xy  x  y  yz  y  z  xz  x  z
3 3 3

Tài liệu sưu tầm và tổng hợp bản word đầy đủ liên hệ 0393732038 TÀI LIỆU TOÁN HỌC
75
Website:tailieumontoan.com

 2 x3 y 3  2 z3 x3  2 z3 y 3 .
xyz 
Dấu “=” xảy ra khi   a b c 1.
 x  y; z  0
Nhận xét: Bài toán sử dụng bất đẳng thức Cosi, phép đặt ẩn phụ đồng thời chứng minh
qua bất đẳng thức trung gian hay là bất đẳng thức bổ đề (bất đẳng thức Schur bậc ba) để
suy ra điều phải chứng minh.
Nhắc lại kiến thức và phương pháp:
• Bất đẳng thức Cosi cho ba số thực dương a  b  c  3 3 abc .
• Bất đẳng thức Schur bậc ba cho các số thực dương
x 3  y 3  z 3  3 xyz  xy  x  y  yz  y  z  xz  x  z .
Bài toán kết thúc.
Bài tập tương tự:
1. Chứng minh rằng với mọi a; b; c  0 , ta luôn có:
2 a 2  b2  c 2   abc  8  5a  b  c .
2. Cho a; b; c  0 là các số thực dương thỏa mãn a  b  c  3 . Chứng minh rằng
 1 1 1 12
5     3.
 a b c  abc

ĐỀ SỐ 10

Câu 1.
3a  b  c  x

1). Đặt 3b  c  a  y .

3c  a  b  z
3 3 3 3
Ta có 3a  3b  3c  24  3a  b  c  3b  c  a  3c  a  b
3
  x  y  z  24  x 3  y 3  z 3
3 3
  x  y  z  24   x  y  z  3 x  y y  z x  z
 24  3 x  y y  z x  z  0  24  32 a  4b2b  ac2c  4 a  0
 24  24 a  2bb  2cc  2 a  0  a  2bb  2cc  2 a  1 .
Nhận xét: Bài toán sử dụng phương pháp đặt ẩn phụ, khai thác biến đổi từ giả thiết, điểm
nhấn ở đây chính là hằng đẳng thức bậc ba để suy ra điều phải chứng minh.
Nhắc lại kiến thức và phương pháp:
3
• Hằng đẳng thức  x  y  x 3  3x 2 y  3xy 2  y 3
• Hằng đẳng thức
3 3 2
 x  y  z   x  y  3 x  y z  3 x  y z 2  z 3
2
 x 3  3 x 2 y  3 xy 2  y 3  3 x  y z  3 x  y z  z 3
2
 x 3  y 3  z 3  3 xy  x  y  3 x  y z  3 x  y z
 x 3  y 3  z 3  3 x  y xy  xz  yz  z 2 

Tài liệu sưu tầm và tổng hợp bản word đầy đủ liên hệ 0393732038 TÀI LIỆU TOÁN HỌC
76
Website:tailieumontoan.com
 x 3  y 3  z 3  3 x  y y  z z  x .
Ý tưởng: Bài toán cho giả thiết khá là cồng kềnh, phức tạp thậm chí bậc to, nhưng quan sát
các đại lượng bên vế phải, ta thấy các biểu thức 3a  b  c ,
3b  c  a và 3c  a  b độc lập so với nhau mặt khác:
3a  b  c  3b  c  a  3c  a  b  3a  3b  3c
Vì thế nếu đặt x  3a  b  c; y  3b  c  a; z  3c  a  b thì giả thiết bài cho tương đương
3
với  x  y  z  24  x 3  y 3  z 3 . Khoan nói đến chuyện biến đổi nói, bây giờ ta sẽ quan
sát đến biểu thức cần chứng minh, với phép ẩn phụ trên ta có thể hoàn toàn rút a; b; c theo
x; y ; z . Xét biểu thức a  2b biểu thức này không chứa c vậy nên ta cần tìm mối liên hệ
giữa x; y; z sao cho c triệt tiêu, dễ thấy ở x chứa c còn y chứa c nên suy ra
x  y  2 a  4b  2 a  2b , tương tự y  z  2 b  2c ; z  x  2 c  2 a . Do đó ta cần chứng
minh  x  y y  z z  x  8 . Bây giờ ta quay ngược lại với giả thiết, xuất hiện hằng đẳng
3
thức bậc ba, nên ta sẽ khai triển nó, ta được:  x  y  z  x 3  y 3  z 3  3 x  y y  z z  x
nên suy ra:
x 3  y 3  z 3  3 x  y y  z z  x  24  x 3  y 3  z 3
  x  y y  z z  x  8  điều phải chứng minh.
Bài toán kết thúc.
Bài tập tương tự:
1. Cho a; b; c là ba số thực dương thỏa mãn điều kiện
3 3 3 3
 a  b  c  12  a  b  c  b  c  a  a  c  b . Chứng minh rằng 2 abc  1 .


2 x  2 y  xy  5
2). Ta có 
27  x  y  y  7  26 x 3  27 x 2  9 x
3


 x  2 y  2  9


 


 27 x  y  y 3  7  26 x 3  27 x 2  9 x
 y 3  x 3  7  3 x  y x  2 y  2  27 x 3  27 x 2  9 x
2 3
 y 3  x 3  8  3 xy  x  y  12  x  y  6  x  y  3 x  1
3 3
  x  y  2  3 x  1  x  y  2  3 x  1
x  1  y  1

 y  1  2 x   x  22 x  1  9   .
 x   7  y  8
 2
 7 
Vậy  x; y  1; 1 ,  ;  8 .
 2 
Nhận xét: Bài hệ phương trình rất hay ở chỗ kết hợp cả hai phương trình, sau đó nhóm lại
hằng đẳng thức đưa được về phương trình biểu diễn mối liên hệ giữa x; y và thế ngược
lại tìm nghiệm của hệ phương trình.
Nhắc lại kiến thức và phương pháp:
3
• Hằng đẳng thức  x  y  x 3  3x 2 y  3xy 2  y 3 .

Tài liệu sưu tầm và tổng hợp bản word đầy đủ liên hệ 0393732038 TÀI LIỆU TOÁN HỌC
77
Website:tailieumontoan.com
3 3 2
• Hằng đẳng thức  x  y  z   x  y  3 x  y z  3 x  y z 2  z 3
2
 x 3  3 x 2 y  3 xy 2  y 3  3 x  y z  3 x  y z  z 3
2
 x 3  y 3  z 3  3 xy  x  y  3 x  y z  3 x  y z
 x 3  y 3  z 3  3 x  y xy  xz  yz  z 2 
 x 3  y 3  z 3  3 x  y y  z z  x
3 3
• Giải phương trình tổng quát  f  x   g  x
  f  x  g  x  f 2  x  f  x g  x  g 2  x  0  f  x  g  x .
  
Ý tưởng: Phương trình thứ hai trong hệ khá phức tạp, ta sẽ đi khai thác nó trước, để ý bên
vế phải của phương trình hai, xuất hiện các tổng 27 x 2  9 x mà ta thấy
2 3
27 x 2  9 x  3.3 x  3.3 x có bóng dáng của hằng đẳng thức nếu thêm đại lượng 3 x  13
3
khi đó : 26 x 3  27 x 2  9 x  3x  1  x 3  1
Chuyển đại lượng x 3  1 sang vế trái của phương trình hai, ta được:
3
x 3  y 3  8  27  x  y  3 x  1 ( i ).
Điều còn lại là ta chưa khai thác tới phương trình một, nhưng quan sát ( i ) một chút, ta
thấy vế phải là một lũy thừa bậc ba, ta sẽ biến đổi vế trái ( i ) cũng về dạng đó. Bởi lẽ
3
x 3  y 3  8  x 3  y 3  2 3 ta sẽ nghĩ đến chuyện đưa vế trái ( i ) về dạng  x  y  2 . Hay nói
cách khác, kết hợp phương trình một của hệ, ta cần chứng minh:
3
x 3  y 3  8  27  x  y   x  y  2 .
Sẽ có hai cách để chứng minh, một là đi xuôi, hai là chứng minh ngược:
• Từ phương trình một của hệ, có  x  2 y  2  9 thế vào vế trái của ( i ), ta được:
3
x 3  y 3  2 3  3 x  y x  2 y  2   x  y  2 .
3
• Ta có  x  y  2  x 3  y 3  2 3  3 x  y x  2 y  2
 3 x  y x  2 y  2  27  x  y  2 x  2 y  xy  5
3 3
Khi đó ta có  x  y  2  3x  1  x  y  2  3x  1  y  2 x  1 , thế ngược lại phương
trình một của hệ ta sẽ tìm được nghiệm của hệ phương trình ban đầu.
Bài toán kết thúc.
Bài tập tương tự:

2 x  2 y  xy  5
1. Giải hệ phương trình  .
27  x  y  y  7  7 x 3  12 x 2  6 x
3

Câu 2.

n  5  x 2

1). Đặt  ( x , y  ?; x , y  0 ).

n  30  y
2

 y 2  x 2  25   y  x y  x  1.25 vì ( x , y  ?; x , y  0 ).
y  x  1
  y  13

Lại có y  x  y  x nên  
 .

 y  x  25
 
x  12

Tài liệu sưu tầm và tổng hợp bản word đầy đủ liên hệ 0393732038 TÀI LIỆU TOÁN HỌC
78
Website:tailieumontoan.com
Thay vào ta tính được n  139 thảo mãn.
Nhận xét: Bài toán số học sử dụng tính chất số chính phương, phương trình ước số
Nhắc lại kiến thức và phương pháp:
• Số chính phương được viết dưới dạng a 2 với a là số nguyên.
+ n + 5 là số chính phương nên có dạng x 2 với x là số nguyên.
+ n + 30 là số chính phương nên có dạng y 2 với y là số nguyên
• Hằng đẳng thức A 2 − B2 = ( A + B )( A − B ) .

n  5  x 2
Trừ vế theo vế của  cho nhau ta được
 2
n  30  y

y 2 − x 2 = ( n + 30 ) − ( n + 5 ) ⇔ ( y − x )( y + x ) = 25 .
• Phương trình ước số.
( 1) . ( −25 ) =5.5 =−
( y − x )( y + x ) =25 =1.25 =− ( 5 ) . ( −5 ) ;
= y − x 1 =  2 y 26 =   y 13  =n + 30 169
   
  y + x = 25 x = y − 1 =   x 12  n + 5 144
=
=   y − x 25 =  2 y 26 =   y 13  =n + 30 169
   
y + x = 1   x = y − 25   x =−12  n + 5 =144
   
= y − x 5 =  2 y 10 = y 5  =n + 30 25
y + x = 5 x = y − 5 = x 0  
   = n + 5 0
suy ra  ⇔ ⇔ ⇔
  y − x =−5  2 y =−10 y = −5  n + 30 =25
   
  y + x =−5   x =y + 5 = x 0  n + 5 0
=
  y − x =−1  2 y =−26 y = −13  n + 30 =169
   
  y + x =−25   x =y + 1   x =−12  n + 5 =144
   
  y − x =−25  2 y =−26 y = −13  n + 30 =169
  y + x =−1   x =y + 25=   x 12  
    n + 5 144
=
 =n + 30 169 =  n 139
 
= n + 5 144= n 139 n = 139
⇔ ⇔  ⇒ 139 (do n là số tự nhiên).
⇒n=
n + 30 = 25 n =−5  n = 139
 
 n + 5 =0  n =−5

2). Ta thấy 1  x  y  3  x  y và x , y  ?  x , y là số chính phương.


 x  y  3; x ; y  ?
Đặt x  a; y  b; x  y  3  c a , b , c  ?

 a b  c 1

 
a  b  c  1
 x  y  a 2  b 2  
  2

  2 2
c  a  b  3
 x  y  3  c 2 


2
 a  b  1  a 2  b2  3  2 a  2b 2 ab  3
 
a  2  x  4
b  3  y  9
 a  1b  1  2   
 
a  3 x  9
 
 
b  2  y  4
Tài liệu sưu tầm và tổng hợp bản word đầy đủ liên hệ 0393732038 TÀI LIỆU TOÁN HỌC
79
Website:tailieumontoan.com

Nhận xét: Bài toán số học sử dụng tính chất số chính phương, phương trình ước số.
Nhắc lại kiến thức và phương pháp:
• Khi a là số nguyên ta có được a là số chính phương.
Ta có x; y; x + y + 3 là các số nguyên nên ta có thể biển diễn như sau
 x = a ⇒ x = a2

 y =b ⇒ y =b
2
với a , b , c là các số nguyên dương.
 2
 x + y + 3 = c ⇒ x + y + 3 = c
Từ đây ta có

 a b  c 1

 
a  b  c  1

x  y  a  b  
2 2
 2
2
 a  b  1  a 2  b2  3 .

  2 2
c  a  b  3
2 
x  y  3  c


• Hằng đẳng thức ( x + y − z ) = x 2 + y 2 + z 2 + 2 xy − 2 xz − 2 yz
2

( a + b − 1)
2
− a 2 − b2 =3 ⇔ a 2 + b2 + 1 + 2 ab − 2 a − 2b − a 2 − b2 =3
⇔ 2 ab − 2 a − 2b =2 ⇔ ab − a − b =1 ⇔ a ( b − 1) − ( b − 1) =2
⇔ ( a − 1)( b − 1) =
2.
• Phương trình ước số ( a − 1)( b − 1) =2 =2.1 =1.2 =( −2 ) . ( −1) =( −1) . ( −2 ) .
Vì a; b ≥ 0 nên ( a − 1) ; ( b − 1) ≥ −1 .
=a − 1 2 =  a 3 = x 9
  
= b −1 1 = b 2 = y 4
Ta có   ⇔  ⇒  .
a − 1
= 1 = a 2 = x 4
  
=b − 1 2 = b 3 =  y 9
Vậy ( x; y ) ∈ ( 9; 4 ) , ( 4; 9 ) .
x y z
3). Ta có P   
y  z4 z x4 x y4
4x 4y 4z
P  
4 y  z4 4 z x4 4 x y4
4x 4y 4z
  
y  z44 x z44 x y44
 x y z 
 4     6 .
 y  z x  z x  y 
x  4
Dấu = xảy ra khi x  y  z  4   .
 y  9
Nhận xét: Bài toán sử dụng kỹ thuật chọn điểm rơi, kết hợp với bất đẳng thức Cosi đưa về
được một bất đẳng thức quen thuộc gọi là bất đẳng thức Nesbitt, bất đẳng thức có tới 45
cách chứng minh từ đó tìm ra giá trị nhỏ nhất của biểu thức đã cho.
Nhắc lại kiến thức và phương pháp:
• Bất đẳng thức Cosi cho hai số thực dương a  b  2 ab .

Tài liệu sưu tầm và tổng hợp bản word đầy đủ liên hệ 0393732038 TÀI LIỆU TOÁN HỌC
80
Website:tailieumontoan.com

• Bất đẳng thức Cosi cho ba số thực dương a  b  c  3 3 abc .


• Bất đẳng thức Nesbitt cho ba số thực dương a; b; c là
a b c 3
   .
bc ca ab 2
Chứng minh: Bất đẳng thức đã cho tương đương với:
     
 a  1   b  1   c  1  9
 b  c   c  a   a  b  2
abc bca cab 9
   
bc ca ab 2
 1 1 1  9
 a  b  c   
 a  b b  c c  a  2
 1 1 1 
 a  b  b  c  c  a     9 .
  a  b b  c c  a 
Áp dụng bất đẳng thức Cosi cho ba số thực dương, ta có:
a  b  b  c  c  a  3 3 a  bb  cc  a
1 1 1 3
  
ab bc ca 3
a  bb  cc  a
Nhân hai bất đẳng thức trên với nhau suy ra điều phải chứng minh.
Ý tưởng: Bài toán là một bất đẳng thức đối xứng, vai trò các biến x; y; z là như nhau,
không khó để thấy nếu P  m , dấu đẳng thức xảy ra khi và chỉ khi x  y  z  k . Với giá trị
x  y  z  k thay ngược lại P , ta có:
3k
P  m  9 k 2  2 m2 k  4 m2  0 (*).
2k  4
Để bất phương trình (*) có nghiệm với mọi m , k thì (*)  0 khi đó ta được
m4  36 m2  0  m2 m2  36  0  m2  36  m  6 . Do đó ta tìm được P  6 và dấu đẳng
thức xảy ra khi k  4  x  y  z  4 . Bây giờ, quan sát biểu thức P , chứa ba phân thức
đồng thời căn thức xuất hiện ở mẫu số mỗi phân thức, bằng cách nào đó ta sẽ đánh giá
khử căn bậc hai. Với điểm rơi đã tìm được là x  y  z  4 , ta thấy y  z  4  4 , vì vậy đã
khử căn bậc hai ta áp dụng bất đẳng thức Cosi như sau:
yz
 y  z  4  4  2 4. y  z  4  4 y  z  4  y  z  4  .
4
Tương tự cho các biểu thức còn lại, ta suy ra:
 x y z  x y z 3
P  4      6     (*).
 y  z z  x x  y  yz zx xy 2
Và nếu chứng minh được bất đẳng thức (*) bài toán sẽ được hoàn thành. Nhận thấy nếu
lấy tử cộng mẫu trong từng phân thức của (*) ta sẽ được đại lượng x  y  z chung, chính
vì thế, mỗi phân thức ta sẽ cộng thêm 1 khi đó, ta có (*)
xyz xyz xyz 9
   
yz zx xy 2
 1 1 1  9
  x  y  z   
 x  y y  z z  x  2
Tài liệu sưu tầm và tổng hợp bản word đầy đủ liên hệ 0393732038 TÀI LIỆU TOÁN HỌC
81
Website:tailieumontoan.com
 1 1 1 
  x  y   y  z   z  x     9 .
   x  y y  z z  x 
Đặt a  x  y; b  y  z; c  z  x ta có:
1 1 1 1
a  b  c    3 3 abc .3 3  9  (*) luôn đúng (điều phải chứng minh).
a b c abc
Bài toán kết thúc.
Bài tập tương tự:
1. Cho x; y; z là các số thực dương lớn hơn 1 . Tìm giá trị nhỏ nhất của biểu thức
x y z
P   .
yz2 z x2 x y2
2. Cho x; y; z là các số thực dương lớn hơn 3 . Tìm giá trị nhỏ nhất của biểu thức
2x y z
P   .
y  z6 z  2x  6 2x  y  6

Câu 3.

1). Gọi P là điểm đối xứng của A qua M


 HP  HM  MB  2 HM  AH  HN
 H là trung điểm của NP .
Mà BH  NP , suy ra tam giác PNB cân tại B  BN  BP .
Mặc khác lại có M là trung điểm của BC ; AP . Do đó tứ giác ACPB là hình bình hành, suy
ra AC  BP  AC  BN .
Nhận xét: Chứng minh hai đoạn thẳng bằng nhau ta chứng minh cho chúng cùng bằng
một đoạn thẳng thứ ba
Nhắc lại kiến thức và phương pháp:
• Hai điểm đối xứng nhau qua một điểm thì điểm đó là trung điểm của đoạn thẳng nối
hai điểm đã cho.
Tài liệu sưu tầm và tổng hợp bản word đầy đủ liên hệ 0393732038 TÀI LIỆU TOÁN HỌC
82
Website:tailieumontoan.com
P là điểm đối xứng với A qua M nên M là trung điểm của AP do đó ta có
PM = AM ⇔ PM + HM = AM + HM
⇔ PH = AH + HM + HM = AH + 2 HM = AH + AN = HN
⇒ H là trung điểm của PN .
• Tam giác có đường trung tuyến cũng là đường cao thì tam giác đó là tam giác cân.
Tam giác ∆PNB có BH vừa là đường trung tuyến H là trung điểm của PN ) vừa là
đường cao ( BH ⊥ NP ) nên ∆PNB cân tại B
• Tam giác cân có hai cạnh bên bằng nhau.
Tam giác ∆PNB cân tại B nên BP = BN .
• Tứ giác có hai đường chéo cắt nhau tại trung điểm mỗi đường là hình bình hành.
Tứ giác ACPB có M vừa là trung điểm của BC vừa là trung điểm của AP nên ACPB
là hình bình hành.
• Hình bình hành có các cạnh đối diện bằng nhau.
 AC = PB
ACPB là hình bình hành nên AC = PB , suy ra  BN (điều phải chứng
⇒ AC =
 BP = BN
minh).
  APB
2). Do tứ giác ACPB là hình bình hành, suy ra PAC .
  ANB
Mà tam giác PBN cân tại B  APB   ANB
  PAC

  BNQ
 CAN 

Ta có AC  NB; NQ  AN
  NCD
BNQ CAN  NBD   N ; B; C ; D cùng thuộc một đường tròn C ; G là giao điểm
  BQG


DQG  với 
DBC CAG , suy ra .
 GBQ ∽ GCA  GA  GQ  GA  GQ
  GCA
Mà GBQ .
AC QB NB NC
  BDC
Mà BNC   AGQ
 NBC ∽ GAQ
  NCB
 GQA   NCB
  GDC
  GC  NB  NG  BC .
Nhận xét: Chứng minh hai đoạn thẳng bằng nhau ta chứng minh cho chúng cùng bằng
một đoạn thẳng thứ ba.
Nhắc lại kiến thức và phương pháp:
• Hình bình hành có hai cạnh đối diện song song.
  APB
ACPB là hình bình hành nên AC  PB suy ra PAC  (hai góc so le trong).
• Tam giác cân có hai góc ở đáy bằng nhau.
 = ANB
Tam giác PBN cân tại B nên APB  , suy ra PAC = ANB
 hay CAN = BNQ
.

Có AC  NB (chứng minh phần 1).


NQ  AN ( Q đối xứng với A qua N ).
• Hai tam giác có một góc bằng nhau và hai cạnh kề góc tương tứng bằng nhau thì bằng
nhau theo trường hợp “cạnh - góc - cạnh” (c – g – c).
Xét ∆CAN và ∆BNQ có:
+ CA = NB ;
 = BNQ
+ CAN ;

Tài liệu sưu tầm và tổng hợp bản word đầy đủ liên hệ 0393732038 TÀI LIỆU TOÁN HỌC
83
Website:tailieumontoan.com
  BQG
+ AN = NQ ; CAG 

Suy ra ∆CAN =    = NBD


∆BNQ (c – g – c), suy ra ACN = NBQ hay DCN .

• Tứ giác có hai đỉnh liên tiếp cùng nhìn một cạnh dưới hai góc bằng nhau là tứ giác nội
tiếp.
 = NBD
Tứ giác NBCD có hai đỉnh B; C liên tiếp cùng nhìn cạnh DN với hai góc DCN 

nên NBCD là tứ giác nội tiếp hay N ; B; C ; D cùng thuộc một đường tròn.
  BQG
3). Ta có CAG .

 GBQ ∽ GCA  GA  GQ  GA  GQ ;
  GCA
Mà GBQ
AC QB NB NC
  BDC
và BNC   AGQ
 NBC ∽ GAQ ;
  NCB
 GQA   NCB
  GDC
  GC  NB  NG  BC .
Nhận xét: Chứng minh hai đoạn thẳng bằng nhau ta chứng minh cho chúng cùng bằng
một đoạn thẳng thứ ba
Nhắc lại kiến thức và phương pháp:
• Tứ giác nội tiếp có góc trong bằng góc ngoài tại đỉnh đối diện.
  BQG
DNGB là tứ giác nội tiếp nên CAG .

• Hai góc nội tiếp cùng chắn một cung thì bằng nhau.
  GCA
GBQ  (hai góc nội tiếp cùng chắn cung DG
 của đường tròn ngoại tiếp tứ giác

BCGD ).
• Hai tam giác có hai cặp góc bằng nhau thì đồng dạng theo trường hợp “góc - góc” (g –
g); Tam giác đồng dạng có các cặp cặp tương ứng tỷ lệ và các góc tương ứng bằng
nhau.
Xét GBQ và GCA có:
  BQG
+ CAG ;
  GCA
+ GBQ ;

GA GQ GA GQ
Suy ra GBQ ∽ GCA (g – g), suy ra    .
AC QB NB NC
• Hai tam giác có hai góc bằng nhau và hai cặp cạnh kề góc tương ứng tỷ lệ thì đồng
dạng theo trường hợp “cạnh - góc - cạnh” (c – g – c); Tam giác đồng dạng có các cặp
cặp tương ứng tỷ lệ và các góc tương ứng bằng nhau.
Xét NBC và GAQ có:
GA GQ
+  ;
NB NC
  AGQ
+ BNC   BDC

;  
  NCB
Suy ra NBC ∽ GAQ (c – g – c), suy ra GQA ,
  GDC
 NCB   GC  NB  NG  BC .

Câu 4. Giả sử trên mặt phẳng có n điểm thẳng hàng thì tồn tại một đường thẳng.Theo bài
ra các điểm đã cho không cùng nằm trên một đường thẳng nên tồn tại ít nhất một điểm
không cùng nằm trên đường thẳng đó nối điểm đó với n  1 điểm đã cho ta được n  1

Tài liệu sưu tầm và tổng hợp bản word đầy đủ liên hệ 0393732038 TÀI LIỆU TOÁN HỌC
84
Website:tailieumontoan.com
đường thẳng với đường thẳng đi qua n  1 điểm ta được n đường thẳng, thay n  2015 thì
tồn tại ít nhất 2015 đường thẳng.
Nhận xét: Bài toàn tư duy từ việc lập đường thẳng đi qua các điểm kết hợp với tính toán
số đường thẳng được tạo thành.
Nhắc lại kiến thức và phương pháp:
• Với n điểm thẳng hàng luôn tồn tại một đường thẳng duy nhất đi qua n điểm đó.
• Với n 1 điểm thẳng hàng và 1 điểm nằm ngoài đường thẳng đi qua n 1 điểm đó.
Khi đó từ 1 điểm đó ta kẻ được n 1 đường thẳng tới n 1 điểm. Trường hợp này ta
có n đường thẳng được tạo thành. Đây là trường hợp có ít đường thẳng được tạo ra
nhất.
Thay n  2015 ta có 2015 đường thẳng phân biệt được tạo thành.

Đề số 11

Câu 1. 1) Điều kiện: −1 ≤ x ≤ 1.

( )
2
Ta có: 1+ x + 1− x = 2 + 2 1 − x2 .

Do đó phương trình đã cho tương đương với

( )
3
1+ x + 1− x = 8 ⇔ 1+ x + 1− x = 2

⇔ 2 + 2 1 − x 2 =4 ⇔ 1 − x 2 =1 ⇔ x 2 =0
⇔ x = 0 ∈ [ −1,1] .

Vậy phương trình đã cho có nghiệm duy nhất x = 0.

2) Từ HPT ra ta có:
4 ( x 2 − xy + y 2 ) =x 2 + xy + 2 y 2 ⇔ 3 x 2 − 5 xy + 3 y 2 =0 ⇔ ( x − y )( 3 x − 2 y ) =0.

Với x – y = 0 hay x = y, thay vào PT thứ nhất của HPT ta có x 2 − x 2 + x 2 =⇔


1 x=±1.

3
Với 3x – 2y = 0 hay y =
x , thay vào PT thứ nhất của HPT ta có:
2
3 9 2 3
x 2 − x 2 + x 2 =⇔
1 x= ± ⇒y= ± .
2 4 7 7

 2 3   2 3 
Vậy HPT có bốn nghiệm (x; y) là: (1;1) , ( −1; −1) ,  ; , − ;− .
 7 7  7 7

1 1 1
Câu 2. 1) Đặt=a = ,b = ,c .
x y z

Từ giả thiết ta có ab + bc + ca = 1. Do đó, đẳng thức cần chứng minh tương đương với:

Tài liệu sưu tầm và tổng hợp bản word đầy đủ liên hệ 0393732038 TÀI LIỆU TOÁN HỌC
85
Website:tailieumontoan.com
a 2b 3c 5bc + 4ca + 3ab
+ + =
1+ a 1+ b 1+ c
2 2 2
( a + b )( b + c )( c + a )
a 2b 3c 5bc + 4ca + 3ab
⇔ + + =
( a + b )( a + c ) ( b + c )( b + a ) ( c + a )( c + b ) ( a + b )( b + c )( c + a )

⇔ a ( b + c ) + 2b ( c + a ) + 3c ( a + b ) 5bc + 4ca + 3ab (luôn đúng)


Vậy đẳng thức được chứng minh.
2) Đặt u = x + y, v = xy phương trình đã cho trở thành:
v+3
v2u + u = 3 + v ⇔ u = 2 .
v +1
Do x; y ∈ Z + nên u; v ∈ Z+ suy ra v 2 + 1 là ước của v + 3 ⇒ v 2 + 1 là ước của v 2 − 9 ⇒ v 2 + 1 là
ước của v 2 + 1 − 10 ⇒ v 2 + 1 là ước của 10.

Lần lượt xét các ước nguyên của 10 để xác định v, u rồi tìm nghiệm nguyên dương x, y
tương ứng, thử lại với PT ban đầu.

Phương trình có ba nghiệm nguyên dương (x; y) là: (0; 3), (3; 0), (1; 1).

Câu 3.

1) Ta có ∆ABF và ∆ACE đồng dạng do chúng lần lượt cân tại F, E và


∠FBA = ∠BAD = ∠DAC = ∠ECA.

GF BF AB DB
2) Gọi G là giao điểm của BE và CF. Ta có: = = = ⇒ DG / / BF . Mặt khác,
GC CE AC DC
DA//BF suy ra A, D, G thẳng hàng, suy ra đpcm.

3) Ta có ∠BQG = ∠QGA = ∠GAE = ∠GAC + ∠CAE = ∠GAB + ∠BAF = ∠GAF suy ra AGQF
là tứ giác nội tiếp. Mặt khác, ∠QPG = ∠GFQ nên QGPF là tứ giác nội tiếp.
∠GCE =

Tài liệu sưu tầm và tổng hợp bản word đầy đủ liên hệ 0393732038 TÀI LIỆU TOÁN HỌC
86
Website:tailieumontoan.com
Suy ra đpcm

Câu 4. Áp dụng BĐT Cauchy ta có:

1 1
a 4 b 2 + abc 2 + ca ≥ a 2 bc;
3 9
1 1
b 4 c 2 + bca 2 + ab ≥ b 2 ca;
3 9
1 1
c 4 a 2 + cab 2 + bc ≥ c 2 ab.
3 9

Cộng theo từng vế của ba BĐT trên thay ab + bc + ca = 1 vào và rút gọn ta được:

2 1
abc ( a + b + c ) ≤ a 4 b 2 + b 4 c 2 + c 4 a 2 + . (1)
3 9
Ta có
1 1 4 4
abc ( a + b + c ) = ab.ca + bc.ab + ca.bc ≤ ( ab + bc + ca ) = ⇒ abc ( a + b + c ) ≤ . ( 2)
2

3 3 3 9

Cộng theo từng vế (1) và (2) ta có đpcm.

3
Đẳng thức xảy ra ⇔ a = b = c = .
3

Đề số 12

Câu 1.
1). Dễ thấy đẳng thức sau đúng với a  b .
b b 2 b 2 b b 2b 2
  , suy ra   .
a  b a  b a2  b2 a  b a  b a2  b2
Do đó đẳng thức đã cho tương đương với
y 2y2  y2 2 y 4   y4 2 y 8  8 y8
 
  2     4     4
x  y x2  y 2  x 2  y 2 x 4  y 4   x 4  y 4 x8  y 8  x8  y 8
y
  4  y  4 x  4 y  5 y  4 x , điều phải chứng minh.
xy
Nhận xét: Bài toán sử dụng đẳng thức (bổ đề luôn đúng) để ghép vào biểu thức đã cho để
chứng minh.
Ý tưởng: Trước hết, xét ở đẳng thức cần chứng minh ta có 5 y  4 x vì thế ta sẽ tách thành
y
y  4x  4 y   4 . Việc tách này là có cơ sở vì số 4 xuất hiện ở vế phải của giả thiết,
xy
y
cũng như các mối liên hệ giữa mẫu số các phân thức ở vế trái. Với 4  thế ngược lại
xy
y 2y2 4y4 8 y8 y
giả thiết của bài toán, tức là ta sẽ cần chứng minh  2   
xy x y 2 4
x y 4 8
x y 8
xy
(*).

Tài liệu sưu tầm và tổng hợp bản word đầy đủ liên hệ 0393732038 TÀI LIỆU TOÁN HỌC
87
Website:tailieumontoan.com
Đẳng thức (*) hoàn toàn có thể chứng minh bằng phương pháp biến đổi tương đương đó
y y 2y2 4y4 8 y8
là:   2   0
x  y x  y x  y 2 x4  y 4 x8  y 8
2y2 2y2 4y4 8 y8
    0
x2  y 2 x2  y 2 x4  y 4 x8  y 8
4y4 4y4 8 y8 8 y8 8 y8
    0    0 .
x4  y 4 x4  y 4 x8  y 8 x8  y 8 x8  y 8
Với cách biến đổi trên, để làm xuôi ngược lại thì ta đã sử dụng một bổ đề đẳng thức rất
b b 2 b 2
đẹp đó là   2 .
a  b a  b a  b2
Bài toán kết thúc.


 x  y2 x  3 y  12
2). Hệ đã cho tương đương với  ,
 


6 x  y  xy  x  y  12
x  y  0
suy ra  x  y2 x  3 y   x  y6  xy  
 2 x  3 y  6  xy. (loại)
x  3
Ta có 2 x  3 y  6  xy   x  3 y  2  0   .
 y  2
+ Với x  3 , thay vào phương trình đầu của hệ ta có
 y  1
18  3 y 2  3 y  12   .
y  2

+ Với y  2 , thay vào phương trình đầu của hệ ta có
x  3
2 x 2  2 x  12  12   .
 x  4

Vậy hệ có nghiệm  x; y  3;  1 , 3; 2 , 4; 2 .
Nhận xét: Bài toán sử dụng phương pháp thế hằng số ở cả hai phương trình, sau đó
phương trình thu được phân tích thành nhân tử và thế ngược lại một trong hai phương
trình của hệ tìm nghiệm của hệ phương trình.
Ý tưởng: Thoạt nhìn, ta sẽ nghĩ đến hướng xét delta ẩn x hoặc y ở phương trình thứ hai
của hệ và mong muốn đenta chính phương. Nhưng hướng đi này sẽ thất bại, vì dễ thấy
cũng từ phương trình hai, ta tách được rằng 6  x  y  xy  x  y  12   x  y xy  6  12 .
Mặt khác, xét vế trái của phương trình một nếu coi đây là một phương trình đẳng cấp bậc
hai, ta sẽ có được 2 x 2  3 y 2  xy   x  y2 x  3 y . Khi đó hệ phương trình đã cho tương

 x  y2 x  3 y  12
đương với  .



 x  y xy  6  12
Đây là một hệ rất đẹp vì nhân tử x  y; 12 đều xuất hiện ở cả hai phương trình, chính vì
thế suy ra:
x  y
 x  y2 x  3 y   x  y xy  6  
 2 x  3 y  xy  6 (*)

Tài liệu sưu tầm và tổng hợp bản word đầy đủ liên hệ 0393732038 TÀI LIỆU TOÁN HỌC
88
Website:tailieumontoan.com
x  3
Với (*), dễ thấy nhân tử như sau: (*)   x  3 y  2  0   .
y  2
Việc còn lại là thế ngược lại tìm nghiệm của hệ phương trình.
Bài toán kết thúc.

Câu 2.
1). Do x; y là các số nguyên lớn hơn 1 nên x; y  2
 4 xy  1  7 x  7 y  4 xy  1
 4 x 2 y 2  4 xy  1  4 x 2 y 2  7 x  7 y  4 x 2 y 2  4 xy  1
2 2
 2 xy  1  4 x 2 y 2  7 x  7 y  2 xy  1 .

Mà 4 x 2 y 2  7 x  7 y là số chính phương và 1  2 xy  1  2 xy  1 ;
2
nên ta có 4 x 2 y 2  7 x  7 y  2 xy  x  y , điều phải chứng minh.
Nhận xét. Bài toán chứng minh đẳng thức từ những điều kiện đã cho.
Nhắc lại kiến thức và phương pháp.
• Xét x  y
x  y  0 7 x  7 y  0
+ Với x  y ta có    7 x  7 y  4 xy  1 .
xy  0 4 xy  1  0
+ Với x; y  2 ta có 4 xy  1  8 x  1  7 x  x  1  7 x  y  1  7 x  7 y
Suy ra 4 xy  1  7 x  7 y  4 xy  1
 4 x 2 y 2  4 xy  1  4 x 2 y 2  7 x  7 y  4 x 2 y 2  4 xy  1
2 2
 2 xy  1  4 x 2 y 2  7 x  7 y  2 xy  1
+ Bình phương của hai số nguyên có giá trị tuyệt đối là hai số tự nhiên liên tiếp được
2 2
gọi là hai số chính phương liên tiếp. Giữa n 1 và n  1 có duy nhất số chính
phương n2 với n là số nguyên.
2 2 2
+ Ta có giữa 2 xy  1 và 2 xy  1 có duy nhất số chính phương 2xy mà
2 2
2 xy  1  4 x 2 y 2  7 x  7 y  2 xy  1 suy ra
2
4 x 2 y 2  7 x  7 y  2 xy  4 x 2 y 2  7 x  7 y  4 x 2 y 2
 7 x  7 y  0  x  y (trái với điều kiện x  y )
• Xét y  x
Chứng minh hoàn toàn tương tự như trên, ta được x  y (trái với điều kiện y  x ).
Mối quan hệ giữa các số nguyên a  b hoặc a  b hoặc a  b .
Ta có x  y và y  x đều không tồn tại nên chỉ có x  y (điều phải chứng minh).

2). Ta có x 3  y 3  x 2  y 2  xy
  
  x  y x 2  y 2  xy  x 2  y 2  xy   x  y  1 x 2  y 2  xy  0 

Tài liệu sưu tầm và tổng hợp bản word đầy đủ liên hệ 0393732038 TÀI LIỆU TOÁN HỌC
89
Website:tailieumontoan.com
 x 2  y 2  xy  0  x  y  0
  
x  y  1 .
 x  y  1  0 
5
+ Với x  y  0  P  .
2
+ Với x  y  1  0  x; y  1 ,
1 1 2 1
suy ra P    4 , Dấu “=” xảy ra khi và chỉ khi x  1; y  0 .
2 0 1 0
1 0 2 0 4
P   , Dấu “=” xảy ra khi và chỉ khi x  0; y  1 .
2  1 1 1 3
4
Vậy Pmax  4  x  1; y  0 và Pm in   x  0; y  1 .
3
Nhận xét: Khai thác giả thiết cũng như biểu thức bài cho, tìm điều kiện chặn của biến để
tìm giá trị nhỏ nhất, giá trị lớn nhất của biểu thức.
Nhắc lại kiến thức và phương pháp:
• Hằng đẳng thức a3  b3  a  ba2  ab  b2 
m  p f  x
• Xét biểu thức P  , với 0  a  f  x  b , ta có:
n  q f  x
mp a mp a mp b mp b
P  Pmin  ; P  Pmax  .
nq b nq b nq a nq a
Ý tưởng: Đi từ giả thiết của bài toán, sự xuất hiện của x 3  y 3 làm ta nghĩ đến hằng đẳng
thức x 3  y 3   x  y x 2  xy  y 2  , khi đó giả thiết trở thành:
x 3  y 3  x 2  xy  y 2   x  y x 2  xy  y 2   x 2  xy  y 2
x  y  0
  x  y  1 x 2  xy  y 2   0  
x  y  1

Ta chỉ cần xét với x  y  1 , mặt khác kết hợp với điều kiện ở biểu thức ta có được điều

0  x  1
kiện chặn của x , y là 0  x , y  1   .

0  y  1


Suy ra
1 
 x 1 
 x 2
 2  1 x 2 x 4
•  ; P   .
 y 3  y 2 2 y 1 y 3
2 

 1 


1 
 2 
x 2  x 3
  1 x 2 x
•  ; P  4 .
 y 2  y 1 2 y 1 y
2 

 1 


4
Do đó Pmax  4  x  1; y  0 và Pm in   x  0; y  1 .
3
Bài toán kết thúc.

Câu 3.

Tài liệu sưu tầm và tổng hợp bản word đầy đủ liên hệ 0393732038 TÀI LIỆU TOÁN HỌC
90
Website:tailieumontoan.com

F O
E
P

B D C

K
  AEB
1). Ta có AFC   ADC
  ADB
  180 suy ra tứ giác AEPF nội tiếp.
Nhận xét. Để chứng minh bốn điểm cùng thuộc một đường tròn ta đưa về chứng minh tứ
giác AEPF bằng cách chỉ ra tứ giác này có tổng hai góc đối diện bằng 180 .
Nhắc lại kiến thức và phương pháp.
• Các góc nội tiếp cùng chắn một cung của một đường tròn thì bằng nhau.
  ADC
+ AFC  (hai góc nội tiếp cùng chắn cung AC của đường tròn ngoại tiếp tam
giác ACD ).
  ADB
+ AEB  (hai góc nội tiếp cùng chắn cung AB của đường tròn ngoại tiếp tam giác
ABD ).
• Tứ giác có tổng hai góc đối diện bằng 180 là tứ giác nội tiếp.
  ADB
ADC   180 (hai góc kề bù) nên AFC   AEB
  180 , suy ra tứ giác AEPF là tứ
giác nội tiếp hay bốn điểm A ; E ; P ; F cùng thuộc một đường tròn.

 = LFC
2). Từ tứ giác AEPF nội tiếp, suy ra AEB  (1).
 = FCB
Ta lại có FCL   BCL   BAQ
 = PBC  = DAE
  BAQ = BAE
 (2).

Từ (1) và (2), suy ra FCL ∽ EAB .


Nhận xét. Nhớ lại các trường hợp đồng dạng của hai tam giác kết hợp với dữ kiện đề bài
cùng các ý đã chứng minh được từ ý trên để tìm ra hướng chứng minh của bài toán. Đối
với bài toán này, ta chứng minh hai tam giác đồng dạng theo trường hợp “góc - góc”
Nhắc lại kiến thức và phương pháp.
• Tứ giác nội tiếp có góc trong bằng góc ngoài tại đỉnh đối diện.
  LFC
Tứ giác AEPF là tứ giác nội tiếp (chứng minh trên) nên AEB 
• Các góc nội tiếp cùng chắn một cung của một đường tròn thì bằng nhau.

Tài liệu sưu tầm và tổng hợp bản word đầy đủ liên hệ 0393732038 TÀI LIỆU TOÁN HỌC
91
Website:tailieumontoan.com
  BAQ
+ BCL  (hai góc nội tiếp cùng chắn cung BQ của đường tròn (O) ).
  EBC
+ EAD  (hai góc nội tiếp cùng chắn cung DE của đường tròn ngoại tiếp tam giác
ABD ).
• Tam giác cân có hai góc kề đáy bằng nhau.
  PCB
Tam giác PBC có PB  PC nên tam giác PBC cân tại P suy ra PBC .
  FCB
Kết hợp lại, ta được BCL   ABQ
  QAE
  FCL  BAE
.

• Hai tam giác có hai góc tương ứng bằng nhau thì đồng dạng.
Xét FCL và EAB có: AEB   LFC (chứng minh trên) và FCL
  BAE
 (chứng minh
trên), suy ra FCL ∽ EAB (g – g).
FL FC
3). Từ FCL ∽ EAB , suy ra = hay FL.EA = FC.EB (3).
BE AE
Chứng minh tương tự EK.FA = FC.EB (4).
FL EK
Từ (3) và (4), suy ra FL.EA = EK.FA hay = , suy ra EF  KL .
FA EA
Ta lại có  = ALK
QLK   ALQ
  AFE  ABE
  APE
  ABE
  PAB  .
  PAC
Tương tự ta có QKL .

Suy ra QKL   QLK


  PAB   PAC
.

Nhận xét. Với các bài toán chứng minh hai tổng bằng nhau, ta dựa vào mối quan hệ giữa
các góc để đưa các vế cùng bằng một lượng nào đó.
Nhắc lại kiến thức và phương pháp.
• Tính chất hai tam giác đồng dạng .

 FL = FC  FL. AE  FC.EB
FCL ∽ EAB   BE AE .
  
FLC  ABE
Chứng minh hoàn toàn tương tự ta có EK.FA = FC.EB
• Định lý Ta-lét đảo.
FL EK
Từ chứng minh trên, ta được FL.AE  EK.FA  = , suy ra EF  KL .
FA EA
• Đường thẳng cắt hai đường thẳng song song tạo ra các góc đồng vị bằng nhau.
  ALK
EF  KL (chứng minh trên), suy ra AFE .

• Các góc nội tiếp cùng chắn một cung của một đường tròn thì bằng nhau.
  APE
AFE  (hai góc nội tiếp cùng chắn cung AE của đường tròn ngoại tiếp tứ giác
AFPE ).
Kết hợp các ý chứng minh trên ta có được điều đã trình bày trong bài giải trên
 = ALK
QLK   ALQ
  AFE
  ABE
  APE  ABE  PAB
 . Hoàn toàn chứng minh tương tự ta
  PAC
có được QKL  . Từ đó ta có điều cần chứng minh QKL
  PAB
  QLK
  PAC
.

Câu 4.
Từ giả thiết dễ thấy m tập con thuộc dãy là phân biệt. Vì A có 31 phần tử nên số tập con
31  30
có đúng 2 phần tử của A là  Ký hiệu ak 2  k  31 là số các tập có đúng k phần
2

Tài liệu sưu tầm và tổng hợp bản word đầy đủ liên hệ 0393732038 TÀI LIỆU TOÁN HỌC
92
Website:tailieumontoan.com
tử, nằm trong dãy đã cho, suy ra m  a2  a3    a31 . Xét một tập hợp có k phần tử suy
k  k  1 k  k  1
ra số các tập con có 2 phần tử của tập đó là  ak tập này sẽ có ak tập con 2
2 2
phần tử. Mà theo giả thiết với 2 phần tử bất kỳ của A thì chúng không thể đồng thời
thuộc 2 tập có k phần tử của dãy  các tập con 2 phần tử nói trên là phân biệt.
k  k  1 31  30 1
Suy ra ak   ak  31  30 
2 2 k  k  1
 1 1 1 
 a2  a3    a31  31.30    
 1.2 2.3 30.31
 1 1 1 1 1
 m  31.30 1        .
 2 2 3 30 31
Vậy m  900, điều phải chứng minh.
Nhận xét. Bài toán về phần nguyên
Nhắc lại kiến thức và phương pháp.
• Bài toán cách chọn số từ một tập hợp số: “Trong n số bất kỳ ta chọn n cách chọn số thứ
n n 1
nhất, n 1 cách chọn số thứ hai, khi đó có cách chọn hai số khác nhau từ n số
2
đã cho”.
Từ giả thiết dễ thấy m tập con thuộc dãy là phân biệt. Vì A có 31 phần tử nên số tập
31  30
con có đúng 2 phần tử của A là  Ký hiệu ak 2  k  31 là số các tập có đúng k
2
phần tử, nằm trong dãy đã cho, suy ra m  a2  a3    a31 . Xét một tập hợp có k phần
k  k  1 k  k  1
tử suy ra số các tập con có 2 phần tử của tập đó là  ak tập này sẽ có ak
2 2
tập con 2 phần tử. Mà theo giả thiết với 2 phần tử bất kỳ của A thì chúng không thể
đồng thời thuộc 2 tập có k phần tử của dãy  các tập con 2 phần tử nói trên là phân
biệt.
• Phân số có mẫu là tích hai số tự nhiên liên tiếp: “Với x là số tự nhiên thỏa mãn điều
1 1 1
kiện xác định ta có   ”
x  x  1 x x  1
k  k  1 31  30 1
ak   ak  31  30 
2 2 k  k  1
 1 1 1 
 a2  a3    a31  31.30    
 1.2 2.3 30.31
 1 1 1 1 1  1
 m  31.30 1          31.30.1    900 .
 2 2 3 30 31  31

Tài liệu sưu tầm và tổng hợp bản word đầy đủ liên hệ 0393732038 TÀI LIỆU TOÁN HỌC
93
Website:tailieumontoan.com

Đề số 13

Câu 1.
1
1). Điều kiện:   x  2 .
3
Phương trình đã cho tương đương với 2 x  3  2 3x  12  x  9

3  x  0
 3 x 2  5 x  2  3  x  

 2 2
3 x  5 x  2  x  6 x  9

x  1

2
 4 x  11x  7  0   .
x  7
 4
7
Đối chiếu với điều kiện ta được được nghiệm: x  1; x  .
4
Nhận xét: Bài toán sử dụng phương pháp nâng lũy thừa (bình phương) hai vế tìm nghiệm
của phương trình.
Nhắc lại kiến thức và phương pháp:

 
2
• Phương trình dạng f  x  g  x  m  0  f  x  g  x  m 2

 f  x   g  x  2 f  x g  x  m 2  2 f  x g  x  m 2  f  x  g  x

 m 2  f  x  g  x  x  x1


   .
  
4 f  x g  x   m  f  x  g  x
2
2 x  x


    2

• Phương trình trên có cách giải khác như sau:


f  x  g  x  m  f  x  m  g  x
m  f  x m  f  x
   
 f x  m2  2 m g x  g  x 2 m g  x  g  x  m2  f  x

    

m  f  x ; g  x  m 2  f  x

  x  x1
   .
 
 x  x2
2
4 m2 g  x   g  x  m2  f  x


Ý tưởng: Đây là một bài phương trình cơ bản, dạng toán một vế chứa hai căn thức vế còn
lại là một hằng số thì phương pháp nâng lũy thừa hai vế là một phương pháp tối ưu nhất.
Bài toán kết thúc.
Bài tập tương tự:
1. Giải phương trình 3x  1  x  1  8 .
Đáp số: x  8 .
2. Giải phương trình 7 x  4  x  1  3 .
Đáp số: x  3 .

Tài liệu sưu tầm và tổng hợp bản word đầy đủ liên hệ 0393732038 TÀI LIỆU TOÁN HỌC
94
Website:tailieumontoan.com
 1   1  9

 x  y    y  x   2
 
2). Hệ phương trình tương đương với  .
 1 3  1   1  1 
   x     x   y    2
 4 2  y   y  x 

 1

u x
 y
Đặt  .

 1

 vy

 x

 9 
 9
u  v  
 v  u

  2
Hệ phương trình trở thành  2 
 1 3

 9 3u 9  .
  u  uv  2 
  
 u  u
  2

4 2
 
4
 2 
9 3u 9u 9
Suy ra    u2  u2  3u   0
4 2 2 4

 1 3
2  3 
 x 
 3 u   y 2
 u    0   2 

 
2  
 1
v  3 
 y 3

 x

 3y  1

 xy  1  3y y 2 y  1  x 
2.
2
 2   3x  x   y   3  y  3 y  2  0  

 2 2 y 
xy  1  3 x
  y  2  x  1
1 
Hệ phương trình có nghiệm  x; y   ; 1 , 1; 2 .
2 
Nhận xét: Bài toán sử dụng phương pháp đặt hai ẩn phụ, đưa về hệ phương trình bậc hai
cơ bản giải bằng phương pháp thế. Sau đó từ nghiệm ẩn phụ suy ngược lại nghiệm của hệ
phương trình.
Ý tưởng: Hình thức bài toán khá phực tạp vì sự xuất hiện của phân thức, quan sát ta thấy
1
ở cả hai phương trình của hệ đều xuất hiện biểu thức x  . Ta sẽ nghĩ đến chuyện thế
y
1 9  1 1
x    y   xuống phương trình hai nhưng còn đại lượng xy  chưa biết xử lý
y 2  x xy
như thế nào. Có lẽ tác giá đã gợi mở theo con đường đặt ẩn phụ, nếu đặt
1 1 1
u  x  ; v  y  thì bây giờ ta chỉ cần biểu diễn xy  qua u; v thì hệ phương trình đã
y x xy

 1

u  x   uy  xy  1
 y
cho sẽ được giải quyết. Ta có 

v  y  1  vx  xy  1



 x
2 1
 uvxy   xy  1  uv  xy  2.
xy

Tài liệu sưu tầm và tổng hợp bản word đầy đủ liên hệ 0393732038 TÀI LIỆU TOÁN HỌC
95
Website:tailieumontoan.com

 9
uv 
 2
Khi đó, hệ phương trình đã cho tương đương với  .

 1 3
  u  uv  2

4 2

Hệ phương trình trên là hệ phương trình cơ bản, hoàn toàn giải quyết được bằng phương
pháp thế.
Bài toán kết thúc.
Bài tập tương tự:
x 2 1  y 2   2

1. Giải hệ phương trình  .
 2 2 2
1  xy  x y  3 x


 7 5   7 5 

Đáp số:  x; y  1; 1 , 1;  1 ,  ;  ,  ; .
 4 7   4 7 

 2 2

 x  1  y  1  27 xy
2. Giải hệ phương trình  2 .

 x  1 y 2  1  10 xy


1   1 
2 
 
Đáp số:  x; y   ; 2  3  , 2; 2  3 , 2  3;

.
2 
Câu 2.
1). Cách 1: Đẳng thức cần chứng minh tương đương với
a  b  b  c  c  a  3
1  
 1  
 1  
a  b b  c b  c  c  a c  a  a  b  4
ac ba cb 3
   
a  bb  c b  cc  a c  aa  b 4
3
 ac a  c  ba b  a  cb c  b  a  bb  cc  a
4
 ac a  c  ba b  a  cb c  b  6 abc
 ac a  c  b2 a  c  ba 2  abc  c 2 b  abc  8 abc

 a  cac  b2  ab  bc  8 abc

 a  cc  bb  a  8 abc , điều phải chứng minh.


 
x  a 1  x  b
 a  b  ba
 
b c
Cách 2: Đặt  y   1  y  .
 b  c  bc
 
z  c 1  z  a
 c  a  ca

1
Từ điều kiện suy ra xyz  , ta được
8
xyz  1  x1  y1  z  1   x  y  z   xy  yz  zx  xy
 2 xyz  1   x  y  z   xy  yz  zx
3
 xyz  xy  yz  zx .
4

Tài liệu sưu tầm và tổng hợp bản word đầy đủ liên hệ 0393732038 TÀI LIỆU TOÁN HỌC
96
Website:tailieumontoan.com
Nhận xét: bài toán sử dụng phép biến đổi tương đương hoặc ẩn phụ để chứng minh đẳng
thức đã cho.
Ý tưởng: Nhìn đẳng thức cần chứng minh ( gọi là (*)) khá là cồng kềnh, tuy nhiên nếu tinh
ý một chút, ta thấy rằng bên vế trái (*) có tổng của ba thừa số, đồng thời vế phải (*) xuất
hiện tổng hoán vị của tích hai thừa số . Vì thế nếu chuyển vế ta sẽ nhóm được nhân tử
chung là:
a  b  b  c  c  a  3
1 


 1 


 1 


a  b b  c b  c  c  a c  a  a  b  4
ac ba cb 3
    ( i ).
a  bb  c b  cc  a c  aa  b 4
Với biểu thức ( i ), hướng tối ưu nhất có lẽ là quy đồng mẫu số và biến đổi tương đương,
kết hợp với giải thiết a  bb  cc  a  8 abc thì ta có:
3
( i )  ac a  c  ba b  a  cb c  b  a  bb  cc  a
4
 ac a  c  ba b  a  cb c  b  6 abc
 ac a  c  b2 a  c  ba 2  abc  c 2 b  abc  8 abc

 a  cac  b2  ab  bc  8 abc .


Hoặc, ta có thể đi với hướng tư duy ẩn phụ hóa để đơn giản bài toán hơn một chút. Vẫn là
a b c
hướng phát hiện như bên trên, ta sẽ đặt ẩn phụ các thừa số là x  ; y ; z .
ab bc ca
1 3
Khi đó giả thiết  xyz  và ta cần chứng minh x  y  z   xy  yz  zx (**). Nếu chỉ
8 4
dựa vào giả thiết để chứng minh (**) là chưa đủ, ta cần phải khéo léo kết hợp với giải thiết
b c a
bài toán như sau 1  x  ; 1 y  ; 1 z  . Và từ đó suy ra được đẳng thức
ba bc ca
xyz  1  x1  y1  z . Khai triển tích số, ta sẽ được điểu phải chứng minh.
Bài toán kết thúc.

2). Ta có abcde  abc00  de  abc  100  de


 abc 101  1  de  abc  101  abc  de
Suy ra abcde chia hết cho 101  abc  de  abc  10d  e chia hết cho 101 .
99999 9
Ta có 101  m  99999  m   990 
101 101
Vậy số có 5 chữ số lớn nhất chia hết cho 101 là 990  101
999
Ta có 101  n  9999  n   99
101
Vậy số có 5 chữ số nhỏ nhất chia hết cho 101 là 100  101
Số các số có 5 chữ số thỏa mãn yêu cầu của bài toán là: 990  100  1  891.
Đáp số: 891 số.
Nhận xét. Bài toán chứng minh đẳng thức từ những điều kiện đã cho.
Nhắc lại kiến thức và phương pháp.

Tài liệu sưu tầm và tổng hợp bản word đầy đủ liên hệ 0393732038 TÀI LIỆU TOÁN HỌC
97
Website:tailieumontoan.com
• Cấu tạo số
abcde  abc 00  de  abc  100  de  abc 101  1  de  abc  101  abc  de
• Tính chất chia hết của một tích: Trong một tích có một thừa số chia hết cho một số thì
tích chia hết cho số đó.
Ta có 101  101  abc  101  101
• Tính chất chia hết của một tổng: Tổng của hai số hạng, trong đó có một số hạng chia
hết cho một số thì số hạng còn lại chia hết cho số đó.
 
 abc  101  101


Ta có  , suy ra de  abc   101
  
abcde   abc.101  de  abc
   101


 
 abc  10d  e  101 .
• Số lớn nhất có năm chữ số chia hết cho 101.
99999 9
Ta có 101  m  99999  m   990  suy ra số có năm chữ số lớn nhất chia hết
101 101
cho 101 là 990  101 .
• Số bé nhất có năm chữ số chia hết cho 101 .
999
Ta có 101  n  9999  n   99 suy ra số có năm chữ số nhỏ nhất chia hết cho 101 là
101
100  101
cd
• Số các số của dãy số viết theo quy luật được tính theo công thức s   1 trong đó
h
c là số cuối, d là số đầu, h khoảng cách giữa 2 số liên tiếp của dãy.
Số các số có 5 chữ số thỏa mãn yêu cầu của bài toán là:
990.101  100.101
 1  990  101  1  891 .
101
Câu 3.
E
A
F

M O

N
C
B

1). Ta có góc nội tiếp bằng nhau   BCF


BDM  (1) và   BFA
BMA  suy ra
  180 0  BFA
180 0  BMA  hay BMD
  BFC
 (2).

Tài liệu sưu tầm và tổng hợp bản word đầy đủ liên hệ 0393732038 TÀI LIỆU TOÁN HỌC
98
Website:tailieumontoan.com
Từ (1) và (2) , suy ra BDM và BCF đồng dạng (g - g).
Nhận xét. Đây là bài toán tương đối cơ bản và thường gặp trong các bài toán chứng minh
tam giác đồng dạng ứng dụng của góc nội tiếp.
Nhắc lại kiến thức và phương pháp.
• Các góc nội tiếp cùng chắn một cung của một đường tròn thì bằng nhau.
  BCF
+ BDM  (hai góc nội tiếp cùng chắn cung AB của đường tròn (O) ).
  BFA
+ BMA  (hai góc nội tiếp cùng chắn cung AB của đường tròn ngoại tiếp tam giác
ABM )
  180 0  BFA
 180 0  BMA   BMD
  BFC
.
• Hai tam giác có hai cặp góc tương ứng bằng nhau thì đồng dạng.
  BCF
+ BDM và BCF có BDM  và BMD
  BFC
 , suy ra BDM ∽ BCF (g – g).

 suy ra DB  DC vậy DE vuông góc với BC tại trung điểm N


2). Từ AD là phân giác BAC
của BC .
DM BD
Từ 1). BDM ∽ BCF , ta có  .
CF BC
DA 2 DM 2 BD CD DE
Vậy ta có biến đổi sau     (3).
CF CF BC CN CE
  FCE
Ta lại có góc nội tiếp ADE  (4).

Từ (3) và (4) , suy ra EAD ∽ EFC suy ra EFC  EAD


  90 .

Vậy EF  AC .
Nhận xét. Với bài toán này ta đưa về chứng minh EF tạo với AC một góc vuông. Dựa vào
các góc đã biết và kết nối bởi tam giác đồng dạng.
Nhắc lại kiến thức và phương pháp.
• Hai góc nội tiếp bằng nhau chắn hai cung thì hai cung đó bằng nhau và hai dây cung
của cung đó bằng nhau.
Từ AD là phân giác BAC suy ra BAD
  DAC suy ra DB  DC kết hợp với OB  OC (
 R ) suy ra DO hay DE là trung trực của BC hay DE vuông góc với BC tại trung
điểm N của BC .
• Các góc nội tiếp cùng chắn một cung của đường tròn thì bằng nhau.
  FCE
ADE  (hai góc nội tiếp cùng chắn cung AE của đường tròn (O) )

• Các dữ liệu được suy ra, biến đổi từ những điều đã chứng minh.
DM BD 2 DM 2 BD DA CD
BDM ∽ BCF      
CF BC CF 2CN CF CN
• Hệ thức lượng trong tam giác vuông “Cạnh huyền  Đường cao = Tích hai cạnh góc
vuông”.
Áp dụng hệ thức lượng vào tam giác CDE vuông tại C , đường cao CN ta có
CD DE DA DE
CN .DE  CE.CD   kết hợp với chứng minh trên ta được  kết hợp
CN CE CF CE
  EFC
với trên, ta suy ra EAD ∽ EFC (g – c – g)  EAD .
• Góc nội tiếp chắn nửa đường tròn là góc vuông.

Tài liệu sưu tầm và tổng hợp bản word đầy đủ liên hệ 0393732038 TÀI LIỆU TOÁN HỌC
99
Website:tailieumontoan.com
 chắn nửa đường tròn (O) được chia bởi đường kính ED nên EAD
Góc EAD   90 suy
  90 hay EF vuông góc với AC.
ra EFC

Câu 4.
Với  là số thực dương ta có
d3 a3 b3 dab d 3 b3 c3 dbc d 3 c3 a3 dca a 3  b3  c 3 abc
 3 3 2 ;  3 3 2 ;  3 3 2 ;  2 .
3 3 3  3 3 3  3 3 3  3 2 
Cộng bốn đẳng thức trên ta thu được
 2 1  1
d 3   3  2 a 3  b3  c 3   2 dab  dbc  dca  abc .
 3 3  
2 1 4 4
Ta tìm   0 sao cho  2   2     3  4 3  3  6
3 3
3 9 3
3
1 1 1 1 3 1
Chọn    x   , ta được  x     x    6
2 x 2  x  2  x
1 1  3 1 3  1
  x 3  3    x     x    6  x 6  12 x 3  1  0 .
2  x  2  x  2  x
Ta có nghiệm dương là
1 
x  3 6  35 ; x  3 6  35     3 6  35  3 6  35 
2 
Với  xác định như trên ta thu được
4 3 1
d3   a  b3  c 3   2
9 
9 36
 9d 3  4  a 3  b 3  c 3   2  2
.
 3 3 
 6  35  6  35 
 
 1
Đẳng thức xảy ra khi a  b  c  3 ; d= 3 3
3   3 2
36
Vậy giá trị nhỏ nhất của P là 2 .
3 
 6  35  3 6  35 
 

Đề số 14

Câu 1.
1). Cộng hai phương trình của hệ ta thu được
3
x 3  y 3  6 xy  8  x 3  y 3  2  3 xy 2  0

 
  x  y  2 x 2  y 2  4  xy  2 y  2 x  0
2
Ta luôn có x 2  y 2  2  xy  2 y  2 x đẳng thức xảy ra khi và chỉ khi x  y  2 .
Vậy nếu x 2  y 2  4  xy  2 y  2 x  0 ta suy ra x  y  2 (loại) vì không thỏa mãn phương
trình 7 xy  y  x  7 .

Tài liệu sưu tầm và tổng hợp bản word đầy đủ liên hệ 0393732038 TÀI LIỆU TOÁN HỌC
100
Website:tailieumontoan.com

x  y  2  y  2  x
Vậy thu được hệ  .

7 xy  y  x  7

x  1  y  1
2 
Suy ra 7 x 2  x  2  2 x  7  7 x  12 x  5  0   .
x  5  y  9
 7 7
Nhận xét: Bài toán sử dụng phương pháp thế (hay cộng vế) để ra được phương trình có
mối liên hệ giữa các biến. Sau đó thế ngược lại tìm nghiệm của hệ phương trình.
Nhắc lại kiến thức và phương pháp:
• Tổng các đại lượng không âm:
2 2 2
a  b  b  c  a  c  0  a2  b2  c 2  ab  bc  ca
• Đẳng thức:

a3  b3  c 3  3abc  a  b  c a2  b2  c 2 ab  bc  ca 
Ý tưởng: Cả hai phương trình của hệ, đều xuất hiện nhân tử x  y vì thế ta sẽ nghĩ đến
chuyện thế x  y từ phương trình một vào phương trình hai (hoặc ngược lại), do đó ta có
được x 3  y 3  6 xy  8  0 ( i ). Đến đây ta mong muốn sẽ biểu diễn mối quan hệ giữa x , y ,
3
quan sát phương trình ( i ), ta thấy rằng 2  8 và 6 xy  3xy 2 do đó nếu đặt z  2
thì ( i )  x 3  y 3  z 3  3xyz  0 . Một biểu thức đối xứng rất đẹp, bằng cách nhóm nhân tử,
ta có:
3
x 3  y 3  z 3  3 xyz  0   x  y  z 3  3 xy  x  y  z  0

 
  x  y  z x 2  2 xy  y 2  xz  yz  z 2  3 xy  x  y  z  0

 
  x  y  z x 2  y 2  z 2  xy  yz  xz  0 (*).

1 2 2 2
Dễ thấy x 2  y 2  z 2  xy  yz  xz   x  y   y  z   z  x  do đó, phương trình
2  
x  y  z  0 x  y  2
   . Công việc còn lại là thay z  2 suy ra  . Nhưng x  y  2
x  y  z  x  y  2
loại vì không thỏa mãn phương trình hai trong hệ. Với x  y  2 thay xuống phương trình
5 9 
hai, ta tìm được nghiệm của hệ phương trình là  x; y  1; 1 ,  ; .
 7 7 
Bài toán kết thúc.
Bài tập tương tự:
x 3  y 3  1  3 xy
1. Giải hệ phương trình  2 .
x  y 2  x  2 y  3  0

 1  33 5  33   1  33 5  33 
  , 
Đáp số:  x; y  1; 1 ,  ;
  4
;  .

 4 4   4 

x 3  y 3  x  2 xy  3
2. Giải hệ phương trình  .

x  xy  4

Đáp số: vô nghiệm

Tài liệu sưu tầm và tổng hợp bản word đầy đủ liên hệ 0393732038 TÀI LIỆU TOÁN HỌC
101
Website:tailieumontoan.com
2). Điều kiện 1  x  1 .
Phương trình tương đương với
2
2  x 1   1  x2  x  1  1  x  2 x  1

  
x  1  1 x  2 x  1  2  x  1  x  1  1 x    x  1  1 x  2  
x  1 1  0 .

+ Giải x  1  1  x  2  2  2 1  x2  4  1  x2  1  x  0 .
+ Giải x 1 1 x  0 .
Đáp số x  0 .
Nhận xét: bài toán sử dụng phương pháp nhóm nhân tử chung, sau đó nâng lũy thừa bậc
hai để tìm nghiệm của phương trình.
Nhắc lại kiến thức và phương pháp:
 f  x  0
• Giải phương trình f  x.g  x  0   .
 g  x  0

• Giải phương trình a  f  x  a  f  x  b



a  f  x  a

a  f  x  a
    2 2 .
2 a  2 a  f  x  b 4  a  f  x  b  2 a
2 2 2 2 2

Ý tưởng: Bài toán xuất hiện ba căn thức, nhưng có điều đặc biệt ở đây là căn thức còn lại là
tích của hai căn thức kia. Mặt khác 1  x 2 , 1  x có sự đồng nhất hệ số, do đó ta sẽ nhóm

hai căn này lại nên ta được nhân tử chung như sau: 1  x2  1  x  1  x  
x  1  1 . Và

ta mong muốn biểu thức x  3  3 x  1 sẽ phân tích được biểu thức có chứa x  1  1.
Thật vậy, nếu coi h  x  x  3  3 x  1 là một phương trình bậc hai ẩn x  1 ta sẽ thấy:

h  x  x  1  3 x  1  2   x 1 1  
x  1  2 . Chính vì thế bài toán của ta được giải

quyết như sau:


x  3  1  x2  3 x  1  1  x
  x 1 1  
x  1  2  1 x  
x 1 1  0
 x11
  x 1 1  x  1  1  x  2  0   
 x  1  1  x  2
.

Phần còn lại chỉ là việc bình phương các phương trình và tìm nghiệm như ở trên đã nêu.
Ta được nghiệm của phương trình là x  0.
Bài toán kết thúc.
Bài tập tương tự:
1. Giải phương trình x  4  4  x 2  3 x  2  2  x .
Đáp số: x  1 .
2. Giải phương trình 2 x  3  1  4 x 2  3 2 x  1  1  2 x .

Tài liệu sưu tầm và tổng hợp bản word đầy đủ liên hệ 0393732038 TÀI LIỆU TOÁN HỌC
102
Website:tailieumontoan.com
Đáp số: x  0 .
Câu 2.
1). Nhận xét: a; b là các số nguyên thỏa mãn a2  b2  3 thì a; b 3 thật vậy, vì
a 2  0,1mod 3 ; b2  0,1mod 3 .
 2
a  0 mod 3

suy ra a2  b2  0 mod 3    a , b 3 .

b 2
 0  mod 3

  
Phương trình tương đương với 6 x 2  9 y 2  x 2  y 2  28  9 3 . 

 x 2  0 mod 3
2 2 

suy ra x  y  0 mod 3    x  3 x1 ; y  3 y1 ( x1 ; y1   ).

 y 2
 0  mod 3


Thay vào phương trình ta thu được 5  9 x12  8  9  y12  28  9 3
 5  x12  8  y12  28  9 2 .
Lập luận tương tự ta thu được x1  3x2 ; y1  3 y2 ( x2 ; y2   ).
Và nhận được phương trình 5  9 x22  8  9  y22  28  9 2
 5  x22  8  y22  28  9 .
Tương tự ta có x2  3x3 ; y2  3 y3 ( x3 ; y3   ) và thu được 5  x32  8  y32  28 .
28
Từ phương trình suy ra y32   22 .
8
 2 28
 y3  0  x32  2 2 2
Suy ra 
 5  x3  2 ; y 3  1 .
 y 2  1  x2  22
 3 3

 x22  9  2 2 ; y22  9 .
 x12  9 2  2 2 ; y12  9 2  x 2  9 3  2 2 ; y 2  9 3 .
Đáp số: x  2  33 ; y  33 , x  2  33 ; y  33 , x  2  33 ; y  33 , x  2  33 ; y  33 .
Nhận xét. Bài toán nghiệm nguyên giải bằng phương pháp xét số dư hay đồng dư thức
Nhắc lại kiến thức và phương pháp.
• Một số chính phương chia hết cho 3 chỉ tồn tại dư 0 hoặc 1. Do đó tổng của hai số
chính phương chia cho 3 dư 0 hoặc dư 2. Nên a; b là các số nguyên thỏa mãn a2  b2  3
thì a; b  3 ,
 2
2 2 a  0 mod 3


suy ra a  b  0 mod 3    a , b 3 .

b 2
 0  mod 3


• Một số chia hết cho 3 có dạng x = 3 k .
  
Phương trình tương đương với 6 x 2  9 y 2  x 2  y 2  28  9 3 . 
Suy ra

 x 2  0 mod 3
2 2 

x  y  0 mod 3    x  3 x1 ; y  3 y1 ( x1 ; y1   ).

 y 2
 0  mod 3


Tài liệu sưu tầm và tổng hợp bản word đầy đủ liên hệ 0393732038 TÀI LIỆU TOÁN HỌC
103
Website:tailieumontoan.com

Thay vào phương trình ta thu được 5  9 x12  8  9  y12  28  9 3


 5  x12  8  y12  28  9 2 .
Lập luận tương tự ta thu được
x1  3 x2 ; y1  3 y2 ( x2 ; y2   ).
Và nhận được phương trình 5  9 x22  8  9  y22  28  9 2
 5  x22  8  y22  28  9 .
Tương tự ta có x2  3x3 ; y2  3 y3 ( x3 ; y3   ) và thu được 5  x32  8  y32  28 .
• Hai số hạng không âm luôn nhỏ hơn hoặc bằng tổng.
28
Từ phương trình suy ra y32   22 ,
8
 2 28
 y3  0  x32  2 2 2 2 2 2
suy ra  5  x3  2 , y 3  1  x2  9  2 ; y 2  9
 y 2  1  x2  22
 3 3

 x12  9 2  2 2 ; y12  9 2  x 2  9 3  2 2 ; y 2  9 3 .
Đáp số: x  2  33 ; y  33 , x  2  33 ; y  33 , x  2  33 ; y  33 , x  2  33 ; y  33 .
2 1
2). Ta có P  1  x 2 y 2  2 xy  .
xy xy

 x  y 2 1
Đặt t  xy     .
 2  4
P 1 1 15 17
Ta thu được  t   16t   15t  2 16    P  17 .
2 t t 4 2
Dấu “=” xảy ra khi
1
xy  Pmin  17 .
2
Nhận xét: Bài toán sử dụng bất đẳng thức Cosi kết hợp với giả thiết tìm giá trị nhỏ nhất
của thức bài cho.
Nhắc lại kiến thức và phương pháp:
• Bất đẳng thức Cosi cho hai số thực dương a  b  2 ab .
2
1 1 2  a  b 
• Các hệ quả từ bất đẳng thức Cosi   ; ab    .
a b ab  2 
Ý tưởng: Bài toán có sự đối xứng giữa hai biến x; y ( vai trò của chúng như nhau ) vì thế
1
điểm rơi ta khẳng định là x  y , kết hợp với giả thiết suy ra được tại x  y  thì Pmin . Mặt
2
khác, xét với biểu thức P , có xuất hiện đại lượng xy trong căn, vì vậy ra nghĩ đến bất
1 1
đẳng thức Cosi để đánh giá đại lượng  về f  xy .
x y
1 1 2 2 1  x2 y 2 1
Ta có   P 2  xy
x y xy xy xy

Tài liệu sưu tầm và tổng hợp bản word đầy đủ liên hệ 0393732038 TÀI LIỆU TOÁN HỌC
104
Website:tailieumontoan.com
Và khai thác giả thiết, ta cũng sẽ đánh giá về xy , với điểm rơi x  y thì ta có đánh giá
2
2  x  y  1
 x  y  0  xy     . Và nếu đặt t  xy thì ta cần tìm giá trị nhỏ nhất của biểu
 2  4
1 1
thức P  2  t với t  . Bằng sự khéo léo trong chọn điểm rơi, ta đánh giá như sau:
t 4
1 1 15
2 16t   15t  2 2 16t.  15t  2 2 16   17
t t 4
 Pmin  17 .
1
Dấu “=” xảy ra khi x  y  .
2
Bài toán kết thúc.
Bài tập tương tự:
1. Cho x; y là các số thực dương thỏa mãn x  y  2 . Tìm giá trị nhỏ nhất của biểu thức
1 1
P     1  x 2 y 2 .
 x y 
2. Cho x; y là các số thực dương thỏa mãn x  y  2 . Tìm giá trị nhỏ nhất của biểu
4x 2 y 2 x2 y2
thức P  2
  2 .
x 2
 y2  y2 x

Câu 3.

M
Q
Q M
N N E
E F O
F
O H
H P
P
B C
B C K

1). Ta có   BHC
BPC   180  BAC
 , suy ra tứ giác AEPF nội tiếp, nên BFC
  BEC
  1800 .

Mặt khác từ các tứ giác AQFN ; AQEM nội tiếp ta có


  MQA
MQN   NAQ
  MEA
  NFA
  1800 .

Vậy M ; N ; Q thẳng hàng.


Nhận xét. Từ ba điểm bất kỳ tạo thành hai tia có chung gốc. Khi hai tia đó tạo với nhau
một góc tù thì ba điểm đã cho thẳng hàng.
Nhắc lại kiến thức và phương pháp.

Tài liệu sưu tầm và tổng hợp bản word đầy đủ liên hệ 0393732038 TÀI LIỆU TOÁN HỌC
105
Website:tailieumontoan.com
• Hai góc nội tiếp cùng chắn một cung của một đường tròn thì bằng nhau.
  BHC
+ BPC  của đường tròn ngoại tiếp tam giác
 (hai góc nội tiếp cùng chắn cung BC

HBC ).
  AFN
+ AQN  (hai góc nội tiếp cùng chắn cung AN
 của đường tròn ngoại tiếp tam

giác FAN ).
  AEM
  của đường tròn ngoại tiếp tam
+ AQM (hai góc nội tiếp cùng chắn cung AM
giác AME ).
• Tứ giác có tổng hai góc đối diện bằng 180 là tứ giác nội tiếp. Tứ giác nội tiếp có tổng
hai góc đối diện bằng 180 .
Gọi chân đường cao từ đỉnh B và C của tam giác ABC là E và F  .
 
Tứ giác AE HF  có AE H  AF  H  90  90  180 , suy ra AE HF  là tứ giác nội tiếp, khi
   
 mà F  HE  BHC
 (hai góc đối đỉnh) nên
đó F  AE  F  HE  180  F  HE  180  BAC
  180 BAC
BHC  kết hợp với trên ta được BPC
  180 BAC
  BPC
  BAC
  180 suy

ra tứ giác AEPF là tứ giác nội tiếp.


  AEP
Suy ra AFP   180  180 AFP

  180 AEP
  180
  
  BEC
 BFC   180  NFA
  AEM
  180 kết hợp với trên, ta có
  AQM
NAQ   180  NQM
  180 hay ba điểm N ; Q; M thẳng hàng.

  ANQ
2). Ta có các góc nội tiếp bằng nhau AFQ   ANM
  ABM
 suy ra FQ  BE . Tương tự

EQ  CF .
  QFP
Từ đó tứ giác EQFP là hình bình hành, suy ra QAN   QEP
  QAM
 hay AQ là phân

.
giác MAN
 thì A , P , Q thẳng hàng.
Nếu AP là phân giác MAN
  QAC
Từ đó nếu PQ giao BC tại K thì KAC   QME
  NMB 
  PCK

Vậy AKC ∽ CKP , suy ra KC 2  KP.KA .


Tương tự KB2  KP.KA .
Từ đó KB  KC hay K là trung điểm.
Nhận xét. Chứng minh một đường thẳng đi qua trung điểm của một đoạn thẳng khác ta
chứng minh giao điểm của đường thẳng với đoạn thẳng là trung điểm của đoạn thẳng.
Nhắc lại kiến thức và phương pháp.
• Một đường thẳng cắt hai đường thẳng khác tạo ra cặp góc đồng vị bằng nhau thì hai
đường thẳng song song.
  ANQ
Từ kiến thức ở trên, ta có AFQ   ANM   ABM  và ABM
 mà AFQ  ở vị trí đồng

vị của hai đường thẳng FQ và BE nên suy ra FQ  BE . Hoàn toàn tương tự ta có


EQ  CF .
• Tứ giác có hai cặp cạnh song song là hình bình hành.

Tài liệu sưu tầm và tổng hợp bản word đầy đủ liên hệ 0393732038 TÀI LIỆU TOÁN HỌC
106
Website:tailieumontoan.com
Tứ giác EQFP có FQ  BE và EQ  CF suy ra EQFP là hình bình hành khi đó
  QFP
QAN   QEP
  QAM
 suy ra AQ là phân giác MAN
 . Do đó A , Q , P thẳng hàng.
  QME
KAC  (hai góc nội tiếp cùng chắn cung QE
 của đường ngoại tiếp tam giác

AME ).
  PCK
  của đường tròn O ),
QME (hai góc nội tiếp cùng chắn cung NB
  PCK
 KAC  chung của hai tam giác AKC và CKP nên suy ra
 mà lại có góc PKC
AKC ∽ CKP (g – g)
AK KC 2
   KC 2  AK.KP hoàn toàn tương tự ta có KB  KP.KA . Từ đó suy ra
CK KP
KC 2  KB2  KC  KB hay K là trung điểm của BC .

Câu 4.
Giả sử k là chỉ số mà x1  x2    xk  0  xk1    x192 .
Ký hiệu S  x1  x2    xk ; S  xk1  xk2    x192
2013
 S  S  0  S  S  2013  S  S  .
2
Do x1  x2    x192 suy ra S  kx1 ; S  192  k x192
S S S S
 x1    x1  ; x192 
k k k 192  k
S S 2013 2013 2013  192
 x192  x1      .
192  k k 2 192  k  2k 2 k 192  k 

 192  k   k  2 192 2
Ta có 2 k 192  k  2   

 2  2
2013  192 2013
 x192  x1   .
192 2 96
2
Dấu “=” xảy ra khi
2013 2013
x1  x2    x96   ; x97  x98    x192 
192 192

Đề số 15

Câu 1.
1). Điều kiện x  6.
Phương trình đã cho tương đương với  x  9  2012  
x  6 1  0 .
2
+ Giải x  9  2012  0  x  2012  9  4048135 .

+ Giải x  6  1  0  x  5 .
Vậy phương trình có hai nghiệm: x  4048135; x  5 .

Tài liệu sưu tầm và tổng hợp bản word đầy đủ liên hệ 0393732038 TÀI LIỆU TOÁN HỌC
107
Website:tailieumontoan.com
Nhận xét: Bài toán sử dụng phương pháp nhóm nhân tử chung và nâng lũy thừa tìm
nghiệm của phương trình.
Nhắc lại kiến thức và phương pháp:
• Cách giải phương trình dạng
 f x m 
   f  x  m 2
 
f  x  m . 
g  x  n  0  
 g  x  n
 
 g  x  n
2
.

Ý tưởng: Bài toán cho hết sức đơn giản, với sự xuất hiện của hai căn thức
x  9; x  6; x  9x  6 nên không khó để nhóm được nhân tử chung như sau:
x  9  2012 x  6  2012  x  9x  6
 x9  x  9x  6  2012 x  6  2012  0
 
 x  9 1  x  6  2012 1  x  6  0  

 1 x  6  
x  9  2012  0 .
Bài toán kết thúc.
Bài tập tương tự:
1. Giải phương trình 2 x  6  x  7  2  x 2  13x  42 .
Đáp số: x  3; x  5 .
2. Giải phương trình x  4  2 x  3  2  x 2  7 x  12 .
Đáp số: x  0; x  2 .
 2 2
x   y  1  5
2). Cách 1: Hệ đã cho tương đương với  .
x  y  1  x   y  1  5


u  x   y  1
Đặt 
2 2
 x 2   y  1   x  y  1  2 x  y  1  u2  2 v .


 v  x  y  1


u2  2 v  5
Thu được 

u  v  5

u  3  v  2
 u2  2 5  u  5  u2  2u  15  0   .
u  5  v  10


x   y  1  3  x  y  1
+ Giải  

.
 


 x y  1  2  x  2, y  0

x   y  1  5
+ Giải  (vô nghiệm).
 


 x y  1  10

Vậy hệ phương trình có hai nghiệm 1; 1 và 2; 0 .



x 2  y 2  2 y  4
Cách 2: Hệ tương đương với  .

4 x  2 y  2 xy  8

Cộng vế với vế hai phương trình ta thu được

Tài liệu sưu tầm và tổng hợp bản word đầy đủ liên hệ 0393732038 TÀI LIỆU TOÁN HỌC
108
Website:tailieumontoan.com
2 x  y  2
 x  y  4  x  y  12  0  
 x  y  6
.

x  y  2 x  y  2 x  1  y  1
+ Giải     .
2 x  y  xy  4 x  x 2  x  2  x  2  y  0

x  y  6 x  y  6
+ Giải    (vô nghiệm).
2 x  y  xy  4 x  x 6  x  10

Vậy hệ phương trình có nghiệm  x; y  1; 1 , 2; 0 .

Nhận xét: Bài toán sử dụng phương pháp ẩn phụ sau đó từ ẩn phụ tìm ngược lại nghiệm
của hệ phương trình.
Ý tưởng: Sự xuất hiện của y 2  2 y ở phương trình một của hệ, làm ta nghĩ đến hằng đẳng
2 2
thức y 2  2 y  1   y  1 hay nói cách khác, từ phương trình một ta có: x 2   y  1  5 .

Đây là phương trình có dạng là tổng các bình phương, dễ làm ta suy đoán đến hệ phương
trình đối xứng loại I, tức là đặt ẩn phụ theo định lý Vi-et ( đặt tổng và tích ) như sau:
u  x  y  1 và v  x  y  1 .

Nhưng đây cũng chỉ là suy đoán ban đầu, bây giờ ta sẽ đi xét phương trình hai để xuất
hiện u, v .
Thật vậy, ta có phương trình hai trong hệ tương đương với:
2 x  y  xy  4  x  y  1  x  y  1  5 .

u2  2 v  5 u  3  v  2

Do đó hệ phương trình đã cho trở thành  
 .
u  v  5
 u  5  v  10

Thế ngược lại tìm hệ của phương trình ban đầu.
Hoặc, ta có thể suy luận như sau: ta đi kết hợp cả hai phương trình trong hệ, vẫn với sự
xuất hiện x 2  y 2 ở phương trình một, đồng thởi có tích ở phương trình xy ta sẽ liên tưởng
2
đến hằng đẳng thức  x  y . Vì thế lấy phương trình hai nhân 2 rồi cộng với phương trình

một ta được:
2 x  y  2
 x  y  4  x  y  12  0  

.
 x  y  6
Thế ngược lại một trong hai phương trình trong hệ ban đầu để tìm nghiệm của hệ ban
đầu.
Bài toán kết thúc.
Bài tập tương tự:

Tài liệu sưu tầm và tổng hợp bản word đầy đủ liên hệ 0393732038 TÀI LIỆU TOÁN HỌC
109
Website:tailieumontoan.com

x 2  y 2  4 y  1
1. Giải hệ phương trình  .
3 x  xy  y  3

Đáp số:  x; y  1; 0 , 2;  1 .



x 2  2 x  y 2  4 y  0
2. Giải hệ phương trình  .

3 x  xy  2 y  0

Đáp số:  x; y  0; 0 , 1;  1 .

Câu 2.
1). Phương trình tương đương với
x  y  1xy  x  y  2 x  y  1  3
  x  y  1 xy  x  y  2  3 .

 x  y  1 là ước của 3.

x  y  1  1 x  y  0
+ Giải    (vô nghiệm).
xy  x  y  2  3 xy  5

x  y  1  1 x  y  2 x  1


+ Giải      .
xy  x  y  2  3 xy  1  y  1

x  y  1  3 x  y  2 x  1


+ Giải      .
xy  x  y  2  1 xy  1  y  1

x  y  1  3 x  y  4
+ Giải    (vô nghiệm).
xy  x  y  2  1 xy  5

Vậy  x; y  1;  1 , 1; 1 .

Nhận xét. Bài toán nghiệm nguyên giải bằng phương pháp đưa về phương trình ước số
Nhắc lại kiến thức và phương pháp.
• Phân tích đưa về phương trình ước số
x  y  1xy  x  y  5  2 x  y
  x  y  1 xy  x  y  3  2  2  x  y

  x  y  1 xy  x  y  3  2  x  y  1

  x  y  1 xy  x  y  2  x  y  1  3

  x  y  1 xy  x  y  2  3

• Phân tích số thành tích của 2 ước số


3  1.3  3.1 .
• Cho mỗi thừa số chứa biến ở vế này đồng nhất với thừa số ở vế kia

Tài liệu sưu tầm và tổng hợp bản word đầy đủ liên hệ 0393732038 TÀI LIỆU TOÁN HỌC
110
Website:tailieumontoan.com

2). Ta có  x 1  
y  1  4  xy  x  y  3.

Theo bất đẳng thức Cô si


x  y x 1 y 1
3  xy  x  y     xy2.
2 2 2
x2 

 y  2 x 2
 P  x  y  x y 2
2
y
Theo bất đẳng thức Cô si 2  .
y 
 y x
 x  2 y
x 

Vậy giá trị nhỏ nhất của P là 2 khi x  y  1 .
Nhận xét: bài toán sử dụng bất đẳng thức Cosi dựa trên điểm rơi đã được suy đoán cũng
như kết hợp với điều kiện bài toán để tìm giá trị nhỏ nhất.
Nhắc lại kiến thức và phương pháp:
• Bất đẳng thức Cosi cho hai số thực dương x  y  2 xy .
2
a 2 b2 a  b
• Mở rộng ra một đánh giá   , a ; b; x ; y  0 .
x y xy
Ý tưởng: Đây là một bài toán có sự đối xứng rõ ràng giữa x , y nên ta mạnh dạn dự đoán
2
điểm rơi tại x  y  k . Thay ngược lại giả thiết bài toán, ta có  
k  1  4  k  1 . Với điểm
rơi x  y  1 , ta sẽ dễ dàng đánh giá hơn khi vận dụng bất đẳng thức Cosi, chính vì vậy,
khi đi khai khác giả thiết, ta suy ra:  x 1  
y  1  4  xy  x  y  1  4
x  y x 1 y 1
 xy  x  y  3  3     xy2.
2 2 2
x2 y 2
Vậy nên, bây giờ ta sẽ đánh giá biểu thức P theo   f  x  y  2 . Hiển nhiên có con
y x
số 2 vì điểm rơi x  y  1 . Vậy nên ta cần tìm f  x  y thỏa mãn f  x  y  2 . Mà như bên
x2 y 2
trên, ta đã tìm được x  y  2 do đó, ta cần chứng minh rằng   x  y . Đánh giá này
y x
ta có các cách như sau:
• Biến đổi tương đương, ta có:
x2 y2 x  x  y y  x  y 2
x y0    0   x  y  0 .
y x y x
• Sử dụng bất đẳng thức Cosi, ta có:
 x 2 x2

 y  y  2 .y  2 x
 y x2 y 2
  xy.
 2 y x
y y2
  x  2 .x  2 y
 x x
• Sử dụng đánh giá mở rộng như đã nêu, ta có:
2
x 2 y 2  x  y
  xy.
y x xy
Tài liệu sưu tầm và tổng hợp bản word đầy đủ liên hệ 0393732038 TÀI LIỆU TOÁN HỌC
111
Website:tailieumontoan.com
Bài toán kết thúc.
Bài tập tương tự:
1. Cho a; b là hai số thực dương thỏa mãn ab  1 . Tìm giá trị nhỏ nhất của biểu thức
a2 b2
P   2 a 2  b2  .
b a
2. Cho a; b; c là các số thực dương thỏa mãn a  b  c  3 . Tìm giá trị nhỏ nhất của biểu
a b c
thức P  2
 2
 .
1 b 1 c 1  a2
Câu 3.

1). Vì MP là đường kính suy ra PN  MN (1).


Vì MD là đường kính suy ra DN  MN (2).
Từ (1) và (2), suy ra N ; P; D thẳng hàng.
Nhận xét. Chứng minh ba điểm thẳng hàng ta quy về chứng minh chúng cùng thuộc một
đường thẳng.
Nhắc lại kiến thức và phương pháp.
• Góc nội tiếp chắn nửa đường tròn là góc vuông.
 là góc nội tiếp chắn nửa đường tròn được chia đôi bởi đường kính PM
+ Góc PNM
  90 hay PN  NM .
của đường tròn đường kính PM nên PNM
 là góc nội tiếp chắn nửa đường tròn được chia đôi bởi đường kính DM
+ Góc DNM
  90 hay DN  NM
của đường tròn (O) nên DNM
• Từ một điểm nằm ngoài một đường thẳng đã kẻ được một và chỉ có một đường thẳng
vuông góc với đường thẳng đó.
Ta có từ N ta kẻ được PN và DN vuông góc với MN , suy ra PN  DN hay ba điểm
P ; N ; D thẳng hàng.

Tài liệu sưu tầm và tổng hợp bản word đầy đủ liên hệ 0393732038 TÀI LIỆU TOÁN HỌC
112
Website:tailieumontoan.com
  MAD
2). Tứ giác APQD nội tiếp ( PQD   90 0 ),

  PDQ
suy ra PAQ   NDM
 (3).
  NAM
Xét (O) , ta có NDM  (4).
  NAP
Từ (3) và (4) PAQ  (*).
 , suy ra AP là phân giác của góc NAQ

  AMD
Xét (O) , ta có AND .

Xét đường tròn đường kính MP có QMP   ANP


  QNP   QNP
 , nên NP là phân giác của

 (**).
góc ANQ
Từ (*) và (**), suy ra P là tâm đường tròn nội tiếp tam giác ANQ.
Nhận xét. Chứng minh một điểm là tâm đường tròn nội tiếp một tam giác ta chứng minh
điểm đó là giao điểm của hai đường phân giác trong của tam giác đó.
Nhắc lại kiến thức và phương pháp.
• Tứ giác có tổng hai góc đối diện bằng 180 là tứ giác nội tiếp.
  90
  90 (góc nội tiếp chắn nửa đường tròn (O) ) và DQP
Tứ giác APQD có DAP
(góc ngoài tại đỉnh đối diện bằng góc trong không kề với nó của tứ giác nội tiếp) suy
  DQP
ra DAP   90  90  180 do đó tứ giác APQD là tứ giác nội tiếp.

• Hai góc nội tiếp cùng chắn một cung của một đường tròn thì bằng nhau.
  QAP
Tứ giác APQD là tứ giác nội tiếp nên QDP  (hai góc nội tiếp cùng chắn cung QP

  NDM
của đường tròn (O) ) hay PAQ  .

  NAM
+ NDM  của đường tròn (O) ).
 (hai góc nội tiếp cùng chắn cung MN

  AMD
+ AND  (hai góc nội tiếp cùng chắn cung AD
 của đường tròn (O) ).

  QNP
+ QMP  (hai góc nội tiếp cùng chắn cung QP
 của đường tròn đường kính PM ) ,

suy ra PAQ  hay AP là phân giác của NAP


  NAP   QNP
 và AND  hay ND là phân

.
giác của QNA
• Giao điểm của hai đường phân giác của hai góc trong một tam giác là tâm đường tròn
nội tiếp tam giác đó.
 , ta có
 và ND là phân giác của QNA
Tam giác ANQ có AP là phân giác của NAP
AP và ND cắt nhau tại P nên suy ra P là tâm đường tròn nội tiếp của ANQ .

Câu 4. Ta có

Tài liệu sưu tầm và tổng hợp bản word đầy đủ liên hệ 0393732038 TÀI LIỆU TOÁN HỌC
113
Website:tailieumontoan.com

abc  ab  ac  a  abc  bc  ba  b abc  ca  cb  c


Q
a  1b  1c  1
a b  1c  1  bc  1a  1  c a  1b  1
Q
a  1b  1c  1
a b c
Q   .
a 1 b 1 c 1
Ta chứng minh rằng
a b c 1 2 3 5
      .
1  a 1  b 1  c 1  1 1  2 1  3 12
Bất đẳng thức trên tương đương với
 3 c   b 2   a 1 
       0
 1  3 1  c   1  b 1  2   1  a 1  1

3c b2 a 1
   0
4 1  c 3b  1 2 1  a

   
1 1   1 1 
 3  c     
 3  c  
 b  2  
   
 4 c  1 3b  1   3b  1 2 1  a
   
1
 3  c  b  2  a  1 0
  2 1  a

3b  4c  1 2a  3b  1 1
 3  c   b  1  c   a  b  c  0.
12 b  1c  1 6 b  1a  1 2 a  1

3b  4c  1
Vì c  3; 0  b  c  3  c  0 (1).
12 b  1c  1

2a  3b  1
Vì b  1  c; 0  a  b  b  1  c  0 (2).
6 b  1a  1

1
Vì a  b  c; 0  a  a  b  c   0 (3).
2 a  1

Từ (1), (2) và (3), suy ra điều phải chứng minh.


5
Vậy giá trị nhỏ nhất của Q là khi a  1; b  2; c  3 .
12
Đề số 16

Câu 1.
1). Hệ phương trình đã cho tương đương với

xy  x  y  2

 3
x  y 3  6  27 x 3  y 3  9 xy 3 x  y



Tài liệu sưu tầm và tổng hợp bản word đầy đủ liên hệ 0393732038 TÀI LIỆU TOÁN HỌC
114
Website:tailieumontoan.com
xy  x  y  2 xy  x  y  2
 
 3
 
x  y  3 xy  x  y  3 x  y
3 3  x  y3  3 x  y3
 
xy  x  y  2 x  y
     x y1.
x  y  3 x  y xy  x  y  2
 
Vậy nghiệm của hệ là x  y  1 .
Nhận xét: Bài toán sử dụng phép thế hằng số từ một phương trình vào phương trình còn
lại sau đó sử dụng hằng đẳng thức tìm nhân tử.
Nhắc lại kiến thức và phương pháp:
3
• Hằng đẳng thức bậc ba ax  by  a 3 x 3  3a 2 bx 2 y  3ab2 xy 2  b3 y 3 .
• Phương trình dạng f 3  x; y  g 3  x; y  f  x; y  g  x; y .
Ý tưởng: Ở cả hai phương trình của hệ, các biến x; y đều nằm trong các biểu thức bậc 3.
Và đặc biệt là cả hai phương trình cũng đều chứa hằng số. Vì vậy nếu thế hằng số này vào
hằng số của phương trình kia thì rõ ràng ta sẽ thu được một phương trình bậc ba đẳng cấp
của hai biến x; y .
Cụ thể như sau: 9 xy 3x  y  3xy  x  y  26 x 3  2 y 3 (*)
 x 3  3 x 2 y  3 xy 2  y 3  27 x 3  27 x 2 y  9 xy 2  y 3
3 3
  x  y   3 x  y  x  y  3 x  y  x  y .
Tuy nhiên, phương trình (*) là một phương trình đẳng cấp bậc ba nếu ta chia phương
trình cho y 3 , sẽ thu được một phương trình bậc ba. Ta có thể sử dụng máy tính cầm tay để
giải quyết phương trình đó. Với x  y , thế ngược lại phương trình một ta sẽ tìm được
nghiệm của hệ là x  y  1 .
Bài toán kết thúc.
Bài tập tương tự:

 y  2 x  y  3
1. Giải hệ phương trình:  2 .
x  4 xy  3  4 x 2  y 2

 2 15 15   2 15 15 
 
Đáp số:  x; y  0;  3 ,  
 5
;  , 
5   5
; .
5 
 2 2
x  y  2
2. Giải hệ phương trình:  .
 x  y1  xy4  32

Đáp số: x  y  1 .

2). Điều kiện 4  x  4.


x
Phương trình đã cho tương đương với
x4 2
 
4  x  2  2x .

+Với x  0 là nghiệm.
+ Giải 4x 2  2  x4 2 
Đặt u  x  4; v  4  x ta thu được

Tài liệu sưu tầm và tổng hợp bản word đầy đủ liên hệ 0393732038 TÀI LIỆU TOÁN HỌC
115
Website:tailieumontoan.com

 v  2u  2

 2
2
 u2  2u  2  8  5u2  8u  4  0
 2
u  v  8

 2 14
u  ; v  2 96
 5 5  x4   x  (thỏa mãn).
 5 25
 u   2 ( l )
96
Vậy phương trình có hai nghiệm x  0; x   .
25
Nhận xét: Sử dụng phương pháp nhân liên hợp, sau đó đặt ẩn phụ tìm nghiệm của bài
toán.
Nhắc lại kiến thức và phương pháp:
ab
• Hằng đẳng thức a  b   a b  
a b  a b
a b
.

• Giải phương trình tổng quát dạng f  x  g  x  m


 f  x ; g  x  0  f  x  g  x  m2
    .
 f  x  g  x  2 m g  x  m2  f  x  g  x  m2  2  4 mg  x
  
Ý tưởng: Không khó để nhận thấy, phương trình có một nghiệm là x  0 . Đồng thời vế trái
của phương trình có xuất hiện biểu thức x  4  2 , dễ thấy rằng
2
x  
x  4  22   x4 2  
x  4  2 . Vì thế, phương trình đã cho tương đương với:
 x4 2  0  x  0
 x4 2  4x 2  2   x4 2  
x  4  2  
 4  x  2 x  4  2
Phương trình còn

lại có thể giải bằng cách tổng quát nêu ở trên, hoặc có thể giải quyết bằng cách đặt ẩn phự

v  4  x
như sau:  ( u; v  0 ). Ta có hệ phương trình

u  x  4



 v 2  u2  8 2 14 2 96

 u ; v  x4   x  .
 5 5 5 25
 v  2u  2

Bài toán kết thúc.
Bài tập tương tự:
1. Giải phương trình  x  1 1  
1  x  1  2x .
24
Đáp số: x  0; x  
25
2. Giải phương trình  x  9 1  
9  x  1  3x .
Đáp số: x  0 .
Câu 2.
2
1). +) 412  40  1  40 2  80  1  81 (mod 100).
414  812 (mod 100)  80 2  160  1 (mod 100)  61 (mod 100).
 415  61.41 (mod 100)  60.40  100  1 (mod 100)  1 (mod 100).
21
 415   41105  1 (mod 100)  41106  41 (mod 100).

Tài liệu sưu tầm và tổng hợp bản word đầy đủ liên hệ 0393732038 TÀI LIỆU TOÁN HỌC
116
Website:tailieumontoan.com
503
+) 57 4  1 (mod 100)  57 2012  57 4   1 (mod 100).
Suy ra A  41106  57 2012  41  1 (mod 100).
Vậy 2 chữ số cuối cùng của A là 42.
Nhận xét. Muốn tìm hai chữ số tận cùng của một số (thường gặp là một lũy thừa) ta xét số
dư của số đó khi chia cho 100
Nhắc lại kiến thức và phương pháp.
2
• Hằng đẳng thức bình phương của một tổng a  b  a 2  2 ab  b2 .
2
Ta có 412  40  1  40 2  80  1  4.100  81  0  81  81 (mod 100).
• Tính chất của đồng dư thức: a  b (mod n )  ac  bc (mod n ) với a ; b ; c và n đều là
các số nguyên dương.
Ta có 412  81mod100  414  812 mod100
2
812  80  1  8.100  160  1  0  60  1  61mod100
• Tính chất của đồng dư thức: a  b mod n  ac  bc mod n với a , b , c và n đều là các
số nguyên dương.
Ta có 414  61 mod100  415  61.41  2501  1 mod100
21
 415   121  1 mod100  41105  1 mod100  41106  41 mod100 Hoàn toàn áp dụng
503
các kiến thức như trên ta có 57 4  1(mod100)  57 2012  57 4   1(mod100) .

• Tính chất đồng dư thức: a  b mod n và c  d mod n khi đó a  c  b  d mod n .


Ta có 41106  41mod100 và 57 2012  1 mod100 .
Khi đó 41106  57 2012  41  1  42 mod100 .
Vậy hai chữ số cuối cùng của số A  41106  57 2012 là 42 .

1 5
2). Tập xác định x .
2 2
x 2  5  4x 2 5  3x 2
Ta có x 5  4x 2
 .
2 2
2x  1  1 3( x 2  1)
3 2x  1  3( )  3x  .
2 2
Cộng hai bất đẳng thức trên ta thu được
y  3 2x  1  x 5  4x 2  4.
Vậy ymax  4 khi x  1.
Nhận xét: bài toán sử dụng việc kết hợp đánh giá điểm rơi cũng như bất đẳng thức Cosi
để tìm giá trị lớn nhất của biểu thức ban đầu.
Nhắc lại kiến thức và phương pháp:
• Bất đẳng thức Cosi cho hai số thực dương a  b  2 ab .
• Giả sử m là điểm rơi của một bài toán, ta sử vận dụng các đánh giá quen thuộc xung
2
quanh điểm rơi là x 2  m2  2 mx   x  m  0 .

Tài liệu sưu tầm và tổng hợp bản word đầy đủ liên hệ 0393732038 TÀI LIỆU TOÁN HỌC
117
Website:tailieumontoan.com
Ý tưởng: Biểu thức bài cho chứa hai căn thức bận hai, đồng thời yêu cầu là tìm giá trị nhỏ
nhất vậy nên ta sẽ đánh giá qua hai bước, đó là: sử dụng đánh giá cosi để khử căn thức,
khéo léo biến đổi theo điểm rơi để khử dần biến số. Vậy nên việc quan trọng nhất đó là dự
đoán điểm rơi của bài toán, tuy nhiên như đã nói ta cần sử dụng Cosi cho từng căn thức
một, nên ta sẽ đánh giá cho mỗi căn như sau:
32 x  1  
2 x  1    2  2 x  1  3 2 x  1 
2 

2
m x  n 5  4 x
2 2 2 2
 2
m 2
 4n2  x 2  5n2
mx.n 5  4 x   x 5  4x 
2 2 mn
32 x  1   m 2
 4 n  x  5n
2 2 2

Do đó, suy ra P   . Bây giờ, ta sẽ có hướng tư duy là sử


2  2 mn
dụng đánh giá nào đó để khử hết biến. Biểu thức cuối có sự xuất hiện của x; x 2 do đó ta sẽ
nghĩ là nên đánh giá x về x 2 hay ngược lại. Và ta sẽ chọn giải pháp đầu tiên
x2  k 2 3 x 2  k 2  k  k m2  4n2  x 2  5n2
2 2
x  k  2 kx  2 x  . Khi đó, ta có: P   . Và nếu
2k 2k  2 mn
muốn khử được hết biến thì đầu tiên ta quan sát được là hai mẫu số phải bằng nhau và
tổng hệ số của x 2 bằng 0. Chính vì thế, ta được:
k   mn; 4n2  m2  3 m  n  k    1
 
 
2 x  1   ; m2 x 2  n2 5  4 x 2  x  1

Và từ đó, ta có đánh giá sau:
2x  1  1 x2  5  4x2
2
P  3 2 x  1  x 5  4 x  3. 
2 2
2 2 2
5  3x 3x  3 5  3x
 3x     4  Pmax  4 .
2 2 2
Bài toán kết thúc.
Bài tập tương tự:
1. Cho x; y; z là các số thực dương thỏa mãn xy  yz  zx  5 . Chứng minh rằng:
3 x 2  3 y 2  z 2  10 .
2. Chứng minh rằng với mọi 0  x  1 ta có bất đẳng thức sau:
9 x 1  x 2  13 x 1  x 2  16 .

Câu 3.

Tài liệu sưu tầm và tổng hợp bản word đầy đủ liên hệ 0393732038 TÀI LIỆU TOÁN HỌC
118
Website:tailieumontoan.com

A
R

Q S
P

B C

N M

  TQA
1). Do TPA   90 0 , nên tứ giác TAPQ nội tiếp.
  QTP
Do đó MTC   QAP
 (do tứ giác TAPQ nội tiếp)  BAN
  MAC  (do MN  BC ), suy ra

tứ giác MTAC nội tiếp, suy ra T  (O) .


Nhận xét. Có nhiều cách để chứng minh một điểm nằm trên một đường tròn, trong đó có
cách áp dụng tính chất “Có một và chỉ một đường tròn đi qua ba điểm không thẳng
hàng”.
Nhắc lại kiến thức và phương pháp.
• Tứ giác có hai đỉnh liên tiếp cùng nhìn cạnh đối dưới hai góc bằng nhau là tứ giác nội
tiếp.
Tứ giác TAPQ có đỉnh P và Q cùng nhìn cạnh AT dưới hai góc TPA   TQA
  90 0 suy

ra TAPQ là tứ giác nội tiếp.


• Hai góc nội tiếp cùng chắn một cung của một đường tròn thì bằng nhau.
  QAP
QTP  hay MTC
  BAN
 (hai góc nội tiếp cùng chắn cung QP
 của đường tròn

ngoại tiếp tứ giác TAPQ ).


• Hai dây cung song song của một đường tròn tạo ra hai dây bị chắn hai bên bằng nhau
(Chứng minh dựa vào tính chất hình thang cân).
Trong đường tròn (O) có hai dây cung MN và BC song song nên ta có BN   MC.

• Hai góc nội tiếp chắn hai cung bằng nhau của một đường tròn thì bằng nhau.
Trong đường tròn (O) có góc nội tiếp BAN  và góc nội tiếp MAC
 chắn cung BN  chắn
 . Kết hợp với trên ta suy ra BAN
cung MC   MAC
.
Áp dụng kiến thức trên, tứ giác MTAC có hai đỉnh liên tiếp T và A cùng nhìn cạnh
  MAC
MC dưới hai góc bằng nhau MTC  (  BAN
 ), suy ra MTAC là tứ giác nội tiếp.
• Có một và chỉ một đường tròn đi qua ba điểm không thẳng hàng.
Tài liệu sưu tầm và tổng hợp bản word đầy đủ liên hệ 0393732038 TÀI LIỆU TOÁN HỌC
119
Website:tailieumontoan.com
Theo chứng minh trên ta có bốn điểm M ; T ; A ; C cùng thuộc đường tròn mà ba
điểm M ; A ; C cùng nằm trên đường tròn (O) , nên suy ra T cũng là điểm nằm trên
đường tròn (O) .

  PTA
2). Từ tứ giác TAPQ nội tiếp ta có PQA   CTA
  ABC
  PQ  BC  MN .

Từ đó Q  (1).
SA  NMA
  AMN
Mà tứ giác AMNR nội tiếp, suy ra ARN   180 0 (2).
 Q
Từ (1) và (2), suy ra QRA SA  180 0 , suy ra tứ giác ARQS nội tiếp, ta có điều phải chứng
minh.
Nhận xét. Có nhiều cách để chứng minh bốn cùng điểm nằm trên một đường tròn, trong
đó đưa về chứng minh bốn điểm tạo thành một tứ giác nội tiếp là thông dụng nhất.
Nhắc lại kiến thức và phương pháp.
Kiến thức nhắc lại trên
  PTA
PQA  (hai góc nội tiếp cùng chắn cung PA
 của đường tròn ngoại tiếp tứ giác TAPQ

).
  ABC
CTA  của đường tròn (O) )
 (hai góc nội tiếp cùng chắn cung AC
  PTA
Suy ra PQA   CTA  ABC.

• Một đường thẳng cắt hai đường thẳng đã cho tạo ra hai góc ở vị trí đồng vị bằng nhau
thì hai đường thẳng đã cho song song.
  ABC
Theo chứng minh trên ta có PQA  mà hai góc này ở vị trí đồng vị của hai đường

thẳng QS và BC suy ra QS  BC kết hợp với giải thiết MN  BC ta có QS  BC  MN .


• Một đường thẳng cắt hai đường thẳng song song tạo ra cặp góc đồng vị bằng nhau.
  AMN
Theo chứng minh trên ta có QS  MN nên hai góc ở vị trí đồng vị ASQ .

• Tứ giác nội tiếp có tổng hai góc đối diện bằng 180 .
  AMN
Tứ giác AMNR nội tiếp đường tròn (O) nên ta có ARN   180 0 . Kết hợp với
  ASQ
trên, ta có ARN   180 0 .

• Tứ giác có tổng hai góc đối diện bằng 180 là tứ giác nội tiếp.
Tứ giác ARQS có ARN  ASQ
  180 0 suy ra ARQS là tứ giác nội tiếp hay bốn điểm A ;

R ; Q ; S cùng thuộc một đường tròn.

Câu 4. Giả sử các số của tập hợp X được sắp theo thứ tự (đánh số lại)
x1  x2    xn .
Ta có x1  x2  x1  x3    x1  xn  x2  xn  x3  xn    xn1  xn , suy ra đối với một tập n

số thực phân biệt bất kỳ ta luôn có ít nhất n  1  n  2  2n  3 giá trị phân biệt của các

tổng xi  x j .

Vậy C X   2n  3 .

Xét tập X1  1; 2; ; n , khi đó với mọi 1  i  j  n


Tài liệu sưu tầm và tổng hợp bản word đầy đủ liên hệ 0393732038 TÀI LIỆU TOÁN HỌC
120
Website:tailieumontoan.com
xi  x j  i  j  3; 4; ; 2n  1  C X1   2n  3

Vậy C X  min  2n  3 .

nn  1 nn  1
Số các tổng xi  x j ( 1  i  j  n ) bằng , suy ra C X   .
2 2

Xét tập X2  2; 2 2 ;; 2n  , thì với mọi 1  i  j  n

xi  x j  2 i  2 j .

Giả sử tồn tại 1  r  s  n : xr  xs  xi  x j  2r  2 s  2i  2 j

2 r |2 i nn  1
 2 r 1  2 sr   2 i 1  2 ji    i r  r  i  s  j  C X2   .
2 |2 2

nn  1
Vậy C X   .
max 2

Đề số 17

Câu 1.
1). Hệ phương trình tương đương với

 x  1 y   x  1  2  y
 2 


 x  1 y 2  1  2  y (1)
  .



 y  2  x 2
  y  2 )  x  1 



 y  2  x 2
 1  x  1 (2)

+) Nếu x  1 , suy ra  x  1 y 2  1  0 nên từ (1)  2  y  0


 y  2   y  2 x 2  1  0 ;
Do đó từ (2)  x  1  0  x  1 (mâu thuẫn).
+) Nếu x  1 , tuơng tự suy ra x  1 (mâu thuẫn).
+) Nếu x  1  y  2 (thỏa mãn).
Đáp số: x  1; y  2
Nhận xét: Bài toán sử dụng kỹ thuật đánh giá theo miền nghiệm khi đoán trước được
nghiệm của hệ phương trình.
Nhắc lại kiến thức và phương pháp:
• Cho hai biểu thức f  x; y và g  x; y , trong đó g  x; y  0 .
Xét biểu thức: P  f  x; y.g  x; y .
Có hai trường hợp sau xảy ra đó là P  0  f  x; y  0 và P  0  f  x; y  0 .
• Kỹ thuật nhẩm nghiệm.
Ý tưởng: Bài toán này không phải là một hệ phương trình đồng bậc, nếu là đồng bậc hai
thì ta có thể giải quyết bằng cách đưa về hệ số bất định. Nhưng một điều đáng lưu ý ở bài
toán này đó chính là các biểu thức x  1; y  2 được gắn với hai đại lượng không âm. Nên

Tài liệu sưu tầm và tổng hợp bản word đầy đủ liên hệ 0393732038 TÀI LIỆU TOÁN HỌC
121
Website:tailieumontoan.com

nhiều khả năng sẽ xảy ra  x  1 y 2   y  2 x 2  0 . Xét các trường hợp thì thấy  x; y  1; 2
là nghiệm duy nhất của hệ phương trình. Hoặc ta có thể sử dụng kỹ thuật nhẩm nghiệm
như sau, đó là giả sử x  k , bây giờ ta sẽ thay thử các giá trị của k , tất nhiên sẽ lấy các giá
trị k nguyên và đẹp. Và cũng cho ta được nghiệm  x; y  1; 2 . Với cặp nghiệm này, thực
x  y  3 x  1  y  2  0
chất bài toán quy về giải hệ phương trình    .
 y  x  1  y  2  x  1
Vì thế ta tách hệ phương trình ban đầu, và nhóm nhân tử như sau:
• Hệ phương trình đã cho
 x  1 y 2  x  1  y  2  0  x  1 y 2  1  2  y 1

  
 y  2 x 2  y  2  x  1  y  2 x 2  1  x  1
   2
• Đến đây, ta sẽ đánh giá miền nghiệm:
1   x  1 y 2  1  0  2  y  0  y  2

TH1. Nếu x  1   .
2   y  2 x  1  0  y  2
2

Hệ bất phương trình này vô nghiệm.


1   x  1 y 2  1  0  2  y  0  y  2

TH2. Nếu x  1   .
2   y  2 x  1  0  y  2
2

Hệ bất phương trình vô nghiệm


• Vậy x  1; y  2 là nghiệm duy nhất của hệ phương trình.
Bài toán kết thúc.
Bài tập tương tự:

2  x  1 y  x  y  4
2

1. Giải hệ phương trình  .
 y  3 x  y  x  2
 2


Đáp số:  x; y  1; 3 .

2). Điều kiện x  0 .


3
Phương trình tương đương 2  x  1 x   x2  7 .
x
Chia hai vế cho x  0 , ta được
 1 3 7  3  1 3 4  3  3 2 
2 1   x   x    x    2 1   x    0   x   2 x     0 .
 x x x  x  x x x  x  x x 

3 3 x  1
+ Giải x  2  x   4  x2  4x  3  0   .
x x x  3

3 2 3 4
+ Giải x   x   2  x3  3x  4  0
x x x x
  x  1 x 2  x  4  0  x  1 .
Đáp số x  1; x  3 .
Nhận xét: Bài toán sử dụng phương pháp ẩn phụ không hoàn toàn, sau đó nâng lũy thừa
tìm nghiệm của phương trình ban đầu.
Nhắc lại kiến thức và phương pháp:
Tài liệu sưu tầm và tổng hợp bản word đầy đủ liên hệ 0393732038 TÀI LIỆU TOÁN HỌC
122
Website:tailieumontoan.com
• Phương pháp đặt ẩn phụ không hoàn toàn: xét một phương trình bậc hai có dạng
mf  x.t 2  ng  x.t  k  0 (*), trong đó t là ẩn phụ được biểu diễn dưới dạng t  h  x .
Khi đó, ta có
2
t  ng  x  4 kmf  x , với t bắt buộc là một số chính phương. Nên ta tìm được
 
nghiệm của (*) đó là
ng  x  t ng  x  t
t  h  x ; t   h  x .
mf  x mf  x

 f  x ; g  x  0

• Cách giải phương trình f  x  g  x   .




f  x   g 2
 x 
Ý tưởng: Trước hết, ta cần quy đồng mẫu số bài toán, như vậy ta sẽ được phương trình có
dạng f  x. g  x  h  x và nếu nâng lũy thừa hai vế, ta sẽ thu được một phương trình bậc
5. Và phương trình bậc 5 nếu không có nghiệm nguyên thì sẽ rất khó để giải quyết. Vậy
3
nên ta cần nghĩ đến hướng tư duy khác, đó là bài toán xuất hiện căn thức x nên ta
x
2
 3 
mong muốn sẽ tạo được lượng k  x   để có thể đưa về phương trình bậc hai, sau đó
 x 
3
đặt t  x  để sử dụng phương pháp ẩn phụ không hoàn toàn. Tức là sẽ chia cả hai vế
x
của phương trình, ta được:
3 x2  7 3
• Ta có x    2  x  1 x   x 2  7
x 2( x  1) x
 1 3 7 3  1 3 4
 2 1   x   x   x   2 1   x    0 (*).
 x x x x  x x x
3  1 4
• Đặt t  x   0 , khi đó ta có (*)  t 2  2 1  t   0 .
x  x x
2 2
 1 4  1
Có 't  1     1   nên suy ra được
 x  x  x 

 1
t  1   1 
1 t  2  x 3 2 i 
 x x  x
 2   .
 1 1  t
t  1   1    x 3  2
 x x
x  ii
 x x
3 x  1
• Giải ( i ), ta có ( i )  x   4  x 2  4 x  3  0   .
x x  3
3 4
• Giải ( ii ), ta có ( ii )  x   2  x 3  3x  4  0  x  1 .
x x
Bài toán kết thúc.
Bài tập tương tự:
1. Giải phương trình:  x  3 4  x12  x  28  x .
Đáp số: x  4  
2  1 ; x  31  3 .

Tài liệu sưu tầm và tổng hợp bản word đầy đủ liên hệ 0393732038 TÀI LIỆU TOÁN HỌC
123
Website:tailieumontoan.com

2. Giải phương trình: x3  x  2x2  x  2 .


1 5 1  65
Đáp số: x  ; x .
2 8
Câu 2.
1). Giả sử tồn tại các số nguyên x , y , z thỏa mãn
x 4  y 4  7 z 4  5  x 4  y 4  z 4  8 z 4  5 (1) .
Ta có a 4  0,1 (mod 8) với mọi số nguyên a
x 4  y 4  z 4  0,1,2,3 (mod 8)
  4 .
8 z  5  5(mod 8)

Mâu thuẫn với (1).
Vậy không tồn tại  x; y; z thỏa mãn đẳng thức.
Nhận xét. Để giải bài toán trên cần sử dụng phương pháp phản chứng: “Giả sử xảy rồi
biến đổi thấy điều mâu thuẫn với giả sử”.
Nhắc lại kiến thức và phương pháp.
• Thêm cùng một lượng vào hai vế của đẳng thức ta được một đẳng thức mới tương
đương với đẳng thức ban đầu
x4  y 4  7 z4  5  x4  y 4  z4  8z4  5 .
• Lũy thừa bậc bốn của một số nguyên khi chia cho 8 dư 0 hoặc 1 tức là a 4  0, 1 mod 8
với mọi số nguyên a
 4
x  0, 1 mod 8  4
x  y  z  0, 1, 2, 3 mod 8
4 4
 4
 y  0, 1 mod 8   4 .
 8 z  5  5 mod 8


z  0, 1 mod 8
4

• Hai vế của một đẳng thức có số dư khi chia cho cùng một số nhận được số dư khác
nhau thì đẳng thức này không thể tồn tại.
Ta thấy VT  x 4  y 4  z 4 chia 8 dư 0 hoặc 1 hoặc 2 hoặc 3 .
Mà VP  8 z 4  5 chia 8 dư 5 do đó không thể tồn tại đẳng thức x 4  y 4  z 4  8 z 4  5
hay x 4  y 4  z 4  8 z 4  5 .
Vậy không tồn tại các bộ ba số nguyên  x; y; z thỏa mãn đẳng thức
x4  y 4  7 z4  5 .

2). Phương trình tương đương với


 2  2
 x  1   x  1   x  1   x  1   y  2 x  24 x  y  8 x  8 x  y .
2 2 3 2 3 3 3

  
3 3 3
+) Nếu x  1  8 x 3  8 x 3  8 x  2 x  1  2 x  y 3  2 x  1 (mâu thuẫn vì y nguyên).
+) Nếu x  1 và  x; y là nghiệm, ta suy ra x;  y cũng là nghiệm, mà x  1  mâu
thuẫn.
+) Nếu x  0  y  0 (thỏa mãn).
Vậy x  y  0 là nghiệm duy nhất.
Nhận xét. Để giải bài toán trên cần sử dụng phương pháp biến đổi tương đương đưa về
xét khoảng giá trị của nghiệm.
Tài liệu sưu tầm và tổng hợp bản word đầy đủ liên hệ 0393732038 TÀI LIỆU TOÁN HỌC
124
Website:tailieumontoan.com
Nhắc lại kiến thức và phương pháp.
• Hằng đẳng thức A 2  B2   A  B A  B
4 4  2 2  2 2
VT   x  1   x  1   x  1   x  1   x  1   x  1 
  
2
• Hằng đẳng thức  A  B  A 2  2 AB  B2
VT   x 2  2 x  1  x 2  2 x  1 x 2  2 x  1  x 2  2 x  1

 2 x 2  24 x  8 x 3  8 x .

Khi đó phương trình đã cho tương đương với phương trình 8 x 3  8 x  y 3


Giải phương trình nghiệm nguyên bằng cách xét khoảng giá trị của nghiệm
• Không tồn tại lũy thừa bậc ba nào giữa hai lập phương (lũy thừa bậc ba) liên tiếp.
3 3 3
+) Nếu x  1  8 x 3  8 x 3  8 x  2 x  1  2 x  y 3  2 x  1
3 3
(mâu thuẫn vì y nguyên và 2x và 2 x  1 là hai lập phương liên tiếp).
3 3 3
+) Nếu x  1  8 x 3  8 x 3  8 x  2 x  1  2 x  y 3  2 x  1
3 3
(mâu thuẫn vì y nguyên và 2x và 2 x  1 là hai lập phương liên tiếp).
+) Nếu x  0  y  0 (thỏa mãn).
Vậy x  y  0 là nghiệm duy nhất.

Câu 3.

B C
E

I
O

A
D

 , suy ra OBD
1). Tứ giác OBCD nội tiếp và CO là phân giác góc BCD   OCD
  OCB
  ODB
,

nên tam giác OBD cân tại O , do đó OB  OD (1).


  OBE
Tứ giác OBCD nội tiếp ODC  (cùng bù với góc OBC
 ) (2).

Trong tam giác CEF có CO vừa là đường cao vừa là đường phân giác nên tam giác
CEF cân tại C .
  AFC
Do AB  CF  AEB   EAB
 , suy ra tam giác ABE cân tại B , nên BE  BA  CD (3).

Từ (1), (2) và (3), suy ra OBE  ODC (c – g – c).

Tài liệu sưu tầm và tổng hợp bản word đầy đủ liên hệ 0393732038 TÀI LIỆU TOÁN HỌC
125
Website:tailieumontoan.com
Nhận xét. Có ba trường hợp bằng nhau cơ bản của hai tam giác. Ở bài này, chúng ta sử
dụng trường hợp bằng nhau “cạnh-góc-cạnh” từ đó ta sẽ đi tìm ra các cạnh và góc bằng
nhau.
Nhắc lại kiến thức và phương pháp.
• Hai góc nội tiếp cùng chắn một cung của một đường tròn thì bằng nhau.
  ODB
+ BCO  của đường tròn ngoại tiếp tam giác
 (hai góc nội tiếp cùng chắn cung OB
BCD ).
  CBD
+ OCD  của đường tròn ngoại tiếp tam giác
 (hai góc nội tiếp cùng chắn cung DO
BCD ).
  OCD
Mà BCO  (vì CO là tia phân giác của BCD ), suy ra OBD
  ODB
.
• Tam giác có hai góc bằng nhau là tam giác cân.
Tam giác OBD có OBD   ODB
 (chứng minh trên) nên OBD cân tại O .
• Tam giác cân có hai cạnh bên bằng nhau.
Tam giác OBD cân tại O suy ra OB  OD .
• Tứ giác nội tiếp có góc ngoài tại một đỉnh bằng góc trong tại đỉnh không kề với nó.
 là góc ngoài tại đỉnh B
Tứ giác BCDO nội tiếp đường tròn ngoại tiếp BCD có EBD
 là góc trong tại đỉnh D không kề B suy ra EBD
và CDO   CDO
.
• Tam giác có đường cao cũng là đường phân giác thì tam giác đó cân.
Tam giác CEF có CO vừa là đường cao vừa là đường phân giác nên tam giác CEF
cân tại C .
• Tam giác cân có hai góc kề cạnh đáy bằng nhau.
  CFE
Tam giác CFE cân tại C , suy ra CEF  hay AEB
  AFC.
• Một đường thẳng cắt hai đường thẳng song song tạo ra hai góc ở vị trí đồng vị bằng
nhau.
  EAB
AFC
  EAB
 (hai góc ở vị trí đồng vị của AB  FC ), suy ra AEB  , nên EBA cân tại

B (tam giác có hai góc bằng nhau), do đó BE  BA mà ABCD là hình bình hành nên
AB  CD suy ra BE  CD .
Xét OBE và ODC có: OB  OD ; EBD   CDO
 ; BE  CD suy ra OBE  ODC (c – g
– c).

2). Từ OBE  ODC  OE  OC .


Mà CO là đường cao tam giác cân CEF , suy ra OE  OF .
Từ đó OE  OC  OF , vậy O là tâm đường tròn ngoại tiếp tam giác CEF .
Nhận xét. Đường tròn ngoại tiếp tam giác cách đều ba đỉnh của tam giác do đó ta chứng
minh điểm O cách đều các điểm E ; C ; F hay OE  OC  OF .
Nhắc lại kiến thức và phương pháp.
• Hai tam giác bằng nhau có các cặp cạnh và cặp góc tương ứng bằng nhau.
OBE  ODC  OE  OC .
• Tam giác cân có đường cao cũng là đường trung trực của cạnh tương ứng
CO là đường cao của tam giác cân CFE nên CO là đường trung trực của FE .
• Một điểm thuộc trung trực của một đoạn thẳng thì cách đều hai mút của đoạn thẳng.

Tài liệu sưu tầm và tổng hợp bản word đầy đủ liên hệ 0393732038 TÀI LIỆU TOÁN HỌC
126
Website:tailieumontoan.com
Điểm O thuộc đường trung trực CO của đoạn thẳng FE nên OE  OF , suy ra
OE  OC  OF .
• Điểm cách đều ba đỉnh của một tam giác là tâm đường tròn ngoại tiếp tam giác đó.
Ta có OE  OC  OF suy ra O là tâm đường tròn ngoại tiếp CEF .

3). Theo trên, ta có BE  CD mà CE  CF  BC  DF .


 , nên
Ta có CI là đường phân giác góc BCD
IB CB DF
   IB.BE  ID.DF .
ID CD BE
Mà CO là trung trực EF và I  CO , suy ra IE  IF .
Từ hai đẳng thức trên, suy ra IB.BE.EI  ID.DF.FI .
Nhận xét. Chứng minh một đẳng thức ta kết hợp các đẳng thức đã cho, đã chứng minh
được để ghép vào đẳng thức cần chứng minh.
Nhắc lại kiến thức và phương pháp.
Ta có BE  CD (chứng minh trên) và CE  CF ( ECF cân tại C ) suy ra
CB DF
CE  BE  CF  CD  BC  DF suy ra  .
CD BE
• Đường phân giác trong của một tam giác chia cạnh dối diện thành hai đoạn thẳng có
tỷ số bằng với tỷ số hai cạnh tương ứng của tam giác.
 của tam giác CBD nên IB  CB , suy ra ta
Ta có CI là phân giác của góc trong BCD
ID CD
IB DE
được   BE.BI  DE.DI .
ID BE
• Nhắc lại kiến thức.
Ta có I nằm trên trung trực CO của đoạn thẳng FE nên suy ra IE  IF nhân vế theo
vế với đẳng thức BE.BI  DE.DI ta được BE.BI .IE  DE.DI .IF

x3 x2
Câu 4. Ta chứng minh  (1) .
x3  8 y 3 x2  2 y 2

x3 x4 2
  x 2  2 y 2   x  x 3  8 y 3   4 x y  4 y  8 xy
2 2 4 3
 3 3
 2
x  8y x  2 y 
2 2

 x 2  y 2  2 xy (đúng).

y3 y2
Ta chứng minh 3
 (2).
y 3   x  y x2  2 y 2

y3 y4  3 2
  x 2  2 y 2   y  y 3   x  y    x 2  2 y 2   y 4  y  x  y
3
 3
 2
 2y2   
y 3   x  y x 2

  x 2  y 2  x 2  3 y 2   y  x  y
3

1 2
Ta có x 2  y 2   x  y
2

Tài liệu sưu tầm và tổng hợp bản word đầy đủ liên hệ 0393732038 TÀI LIỆU TOÁN HỌC
127
Website:tailieumontoan.com
x 2  3 y 2  x 2  y 2  2 y 2  2 xy  2 y 2  2 y  x  y
1
  x 2  y 2  x 2  3 y 2  
2 3
 x  y .2 y x  y  y  x  y  (2) (đúng).
2
Từ (1) và (2), suy ra P  1 .
Dấu “=” xảy ra khi và chỉ khi x  y .
Vậy Pmin  1 .
Nhận xét: Bài toán sử dụng phương pháp dự đoán điểm rơi, từ đó phát hiện tư duy bất
đẳng thức phụ cần thiết để tìm giá trị nhỏ nhất của bài toán.
Nhắc lại kiến thức và phương pháp:
• Bất đẳng thức Cosi cho hai số thực dương x  y  2 xy .
1 2
• Hệ quả của bất đẳng thức Cosi, đó là: x 2  y 2   x  y .
2
Ý tưởng: Đây là một bài toán chứa các biểu thức đồng bậc, nên điểm rơi của bài toán sẽ có
dạng x  ky . Từ đó thay ngược lại biểu thức P , ta có:
k3 4 k 4
P 3
 k  .
k 8 3
1   k  1
3
k 8 1   k  1
3

3
Các biểu thức k 3  8; 1   k  1 đều xuất hiện trong căn, nên ta mong muốn nó là một số
3 
m  3
phương trình và bằng nhau, do đó k 3  8  1   k  1 và k 3  8  m2 nên dễ thấy  .


 k  1
Tức là điểm rơi của bài toán tại x  y và giá trị nhỏ nhất của P là 1 . Việc dự đoán điểm
rơi này là cần thiết, nó giúp ta có nhiều sự lựa chọn hơn trong các việc đánh giá như
1 2
x  y  2 xy hay x 2  y 2   x  y . Với điểm rơi đó, hai mẫu số trong đã bằng nhau P
2
nên nếu có đánh giá P , ta cần đánh giá hai căn về hai biểu thức cùng mẫu, ví dụ như cùng
mẫu số x  2 y; 2 x  y; x 2  2 y 2 ; 2 x 2  y 2 . Bây giờ, quan sát từng căn thức một:
x3 x
• Với f  x; y  3 3
x 3 , trong căn thức mẫu số có bậc ba, trên tử là bậc
x  8y x  8y3
nhất, vậy để đồng hóa bậc thì ta cần đánh giá căn thức về một biểu thức dạng bậc
không trên bậc nhất . Hơn nữa lại xuất hiện 8y 3 nên ta sẽ chọn đánh giá x  2 y để tối
thiểu hóa ẩn y , tức là ta sẽ cần chứng minh:
x 1 2
  x  x  2 y  x 3  8 y 3  y  y  x x  2 y  0 .
3
x  8y 3
x  2y
Nhưng điều này chưa hoàn toàn đúng, vì cần phải có điều kiện y  x , vậy nên hướng
tư duy như trên chưa đúng. Tức là ta sẽ lựa chọn biểu thức x 2  2 y 2 thay vì x  2 y như
ta đã chọn, vì thế:
x x 2
3 3
 2 2
 xx2  2 y 2   x2 x3  8 y 3 
x  8y x  2y
2
 xy 2  x  y  0 .

Tài liệu sưu tầm và tổng hợp bản word đầy đủ liên hệ 0393732038 TÀI LIỆU TOÁN HỌC
128
Website:tailieumontoan.com

4y3 4y
• Với g  x; y  3
y 3
, với hướng tư duy tương tự, chúng ta sẽ
y   x  y
3
y   x  y
3

có:
4y3 2y2
  x 2  y 2  x 2  3 y 2   y  x  y .
3
3
 2 2
y 3   x  y x  2y

Điều này luôn đúng do theo bất đẳng thức Cosi, ta có:

 1 2
x  y   x  y
 2 2
  x 2  y 2  x 2  3 y 2   y  x  y .
3
 2

 2 2 2 2 2 2
x  3 y  x  y  2 y  2 xy  2 y


Bài toán kết thúc.
Bài tập tương tự:
1. Cho a; b là hai số thực dương thỏa mãn a  b  2 . Tìm giá trị nhỏ nhất của biểu thức
2 a 2  3b 2 2 b 2  3 a 2
P  .
2 a 3  3b3 2b3  3a 3
2. Cho a; b là hai số thực dương. Chứng minh rằng:
 2     
a  b  3 b2  a  3   2 a  1 2b  1  .
 4  4   
2  2 

Đề số 18

Câu 1.
1). Điều kiện 0  x  1 .
Phương trình tương đương với
3
x3  x
 
1 x  1  1  3  
1 x  1  x  x  3 .

 
Nếu 0  x  1  3 1  x  1  3 , đồng thời x  x  3  1  4  3,
suy ra VT  VP (loại).
Thử lại ta thấy x  1 là nghiệm.
Nhận xét: bài toán kết hợp giữa phương pháp nhân liên hợp và phương pháp đáng giá để
tìm nghiệm của phương trình.
Nhắc lại kiến thức và phương pháp:
• Biểu thức liên hợp x  m  x   xm x  xm  x 
m
 xm x  với x  0; x  m  0 .
xm  x

m  f  x  0
• Đánh giá: m  f  x  n  g  x  h  x với  .

n  2 m

Tài liệu sưu tầm và tổng hợp bản word đầy đủ liên hệ 0393732038 TÀI LIỆU TOÁN HỌC
129
Website:tailieumontoan.com
 m  f x  n  n
  
Ta có: 0  f  x  m   , suy ra phương trình vô nghiệm. Vậy x  m
 g x  h x  2 m
    
là nghiệm của phương trình đã cho.
Ý tưởng: Bài toán xuất hiện ba căn thức nằm trong một tích, sẽ rất khó để định hình ra
hướng giải, ẩn phụ sẽ rất phức tạp. Nhưng nếu xét hai căn thức đầu tiên ta thấy
2 2
 x3   x  3 . Vì thế ta sẽ nghĩ ngay đến chuyện dùng hằng đẳng thức dạng

a 2  b2  a  ba  b . Khi đó phương trình đã cho tương đương với:


3
x3  x
 
1  x  1  1  3 1  x  3  x  3  x ( i ).

Với phương trình ( i ), ta sẽ đi nhẩm một vài giá trị nghiệm đẹp thỏa mãn các yêu cầu là
1  x  0 và các biểu thức trong căn thức là số chính phương vì thế ta khẳng định nó có
nghiệm duy nhất x  1 , đồng thời 1 lại là miền chặn của biến do đó ta sẽ đi đánh giá
phương trình ( i ). Tức là với 0  x  1 ta sẽ đi chứng minh ( i ) vô nghiệm như sau:


3 1  x  3  3
0  x 1  VT  VP  ( i ) vô nghiệm.

 x  3  x  3


Vậy ta kết luận x  1 là nghiệm của phương trình đã cho.
Bài toán kết thúc.
Bài tập tương tự:
3. Giải phương trình  x  2  x 1  
2  x  1  1.
Đáp số: x  2 .
4. Giải phương trình  x 1 x2  
3 x 1  1.
Đáp số: x  3 .

2).
+ Xét x  y  0 là nghiệm.
+ Xét x  0; y  0 hệ phương trình tương đương với
1
 1 1
 1

  2 2 
  2 2 (1)

 x 2
y 
x
2
y

   .

  1 1    1  

 1 1 
   2 
1    4   2    8 (2)
 x  y 

 xy    xy 


  x y 



3
 1  1  2
1 1 x y
Thay (1) vào (2) ta thu được     8    x y 1.
 x y   1
 1
 xy

Nhận xét: bài toán sử dụng phép chia các biến, sau đó kết hợp hai phương trình tìm mối
liên hệ giữa hai biến để tìm nghiệm của hệ phương trình.
Ý tưởng: quan sát thấy hệ phương trình có dáng dấp của hệ phương trình đối xứng loại
hai, tức là sự xuất hiện của tổng x  y và tích xy sẽ làm ta nghĩ đến phép đặt Viet là
S  x  y ; P  xy để từ đó giải hệ hai ẩn S , P . Nhưng nếu làm như thế ta thu được hệ

Tài liệu sưu tầm và tổng hợp bản word đầy đủ liên hệ 0393732038 TÀI LIỆU TOÁN HỌC
130
Website:tailieumontoan.com
 2 2
S  2 P  2 P


S 1  P  4 P 2 , hệ này giải bằng phương pháp thế sẽ thu được phương trình lũy thừa bậc
 


4 phức tạp, vì thế ta sẽ nghĩ đến phương án khác đó chính phương pháp “ chia để trị “.
Trước hết là xét phương trình hai, vế trái của nó xuất hiện tích đồng thời vế phải cũng
xuất hiện tích số dạng xy.xy . Do đó ta sẽ nghĩ đến việc chia một biểu thức bên vế trái cho
 1 1  1 1 1
xy và ta được như sau:   1    4 . Phương trình này chủ đạo là hai biến ;

 x y  xy  x y vì
1 1
vậy ta cũng chia phương trình một để xuất hiện hai biến này, đó là x 2  y 2  2 . Và nếu đặt
ax  1  2 2
a 2  b2  2
 a  b  2  

by  1 hệ phương trình đã cho trở thành a  b1  ab  4 a  b2  2 ab  4
 
a  b  2
2 2
a  b  2
2 2

  
  a  b  a 2
 2 ab  b 2
  4  3

a  b  8

a 2  b 2  2 1 1
   ab1  1 x y 1
 x y
a  b  2

Bài toán kết thúc.
Bài tập tương tự:

 2 2

 x  1  y  1  27 xy
1. Giải hệ phương trình  2 .

 x  1 y 2  1  10 xy


1   1 
2 
 
Đáp số:  x; y   ; 2  3  , 2; 2  3 , 2  3;

.
2 
 
3
 3 1 
 x  y 1    16
3 

  xy 
2. Giải hệ phương trình   2 .
 2  1 
 x  y 1    8
2 


 xy 
Câu 2.
 1 1
1). Ký hiệu K   3 n    , do n  1  K  1 .
 27 3 
3 3
1 1  1 1  2
Ta có K  n    K  1  K    n   K  
3
27 3  3  27  3 
K 1 1 4 8
 K3  K2    n   K 3  2K 2  K 
3 27 27 3 27
K 4 1 3
 K 3   n  K 2  K 3  3K 2  K   K 3  n  K 2  K  1
3 3 3
2
 1 1
suy ra n  K 2  n   3 n    không biểu diễn được dưới dạng lập phương của một số
 27 3 
nguyên dương.
Nhận xét. Áp dụng kiến thức về phần nguyên, quan hệ giữa phần nguyên của một số với
số đó.

Tài liệu sưu tầm và tổng hợp bản word đầy đủ liên hệ 0393732038 TÀI LIỆU TOÁN HỌC
131
Website:tailieumontoan.com
Nhắc lại kiến thức và phương pháp.
• Áp dụng giả thiết của bài toán đề tìm ra điều kiện.
1 1
Ta có n là số nguyên dương nên n  1 khi đó 3 n  1.
27 3
• Giá trị phần nguyên của một số không vượt qua số đó.
1 1  1 1
3 n   1 nên  3 n     1 hay K  1 .
27 3  27 3 
• Một số thực bất kỳ có giá trị lớn hơn hoặc bằng giá trị của phần nguyên của nó và nhỏ
hơn số nguyên liền sau số nguyên biểu diễn phần nguyên của số thực đó.
Ta có
   
 3 n 1  1  3 n 1  1   3 n 1  1  1
 27 3  27 3  27 3 

1 1
 K  3 n   K  1.
27 3
3
Tiếp tục biến đổi như trên bài giải ta được K 3  n  K 2  K  1 .
• Không tồn tại một lập phương nào giữa hai lập phương liên tiếp.
3 3
Ta có K 3 và K  1 là hai lập phương liên tiếp. Mà K 3  n  K 2  K  1 nên n  K 2
không thể viết được dưới dạng lập phương của một số nguyên dương.
2
 1 1
Vậy với mọi số nguyên dương n , biểu thức n   3 n    không biểu diễn được
 27 3 
dưới dạng lập phương của một số nguyên dương.

2). Ta có 6  x 2  5  6  y 2  5  z 2  5

 6  x  y x  z  6  y  z y  x  z  xz  y


3  x  y   2  x  z  3  x  y   2  y  z   z  x   z  y 
  
2 2 2
9x  9 y  6z 3
   3 x  3 y  2 z ,
2 2
3x  3 y  2 z 2
suy ra P  
6  x 2  5  6  y 2  5  6  z 2  5 3

Đẳng thức xảy ra khi x  y  1; z  2


2
Vậy Pmin  .
3
Nhận xét: Bài toán sử dụng phép thế từ giả thiết và bất đẳng thức Cosi cho hai số thực
dương để tìm giá trị nhỏ nhất của biểu thức.
Ý tưởng: Quan sát thấy, bài toán có sự đối xứng giữa hai kiến x; y nên điểm rơi sẽ xảy ra
tại x  y  kz . Thế lại giả thiết ta sẽ tìm được x  y  1; z  2 . Giả thiết cho xy  yz  zx  5
đồng thời số 5 này cũng xuất hiện ở biểu thức P nên ta sẽ nghĩ đến chuyện thế giả thiết
vào P . Khi đó ta có:

Tài liệu sưu tầm và tổng hợp bản word đầy đủ liên hệ 0393732038 TÀI LIỆU TOÁN HỌC
132
Website:tailieumontoan.com
3x  3 y  2 z
P .
6  x  y x  z  6  y  z y  z  z  xz  y
Và điều ta cần là sử dụng đánh giá nào đó để triệt tiêu tử số và mẫu số, tức là tìm số m
thỏa mãn:
6  x  y x  z  6  y  z y  z  z  xz  y  m3x  3 y  2 z .
1
Thì lúc đó P  . Câu chuyện tiếp theo là tìm m , quan sát thấy các biểu thức trong căn
m
đều là tích của hai thừa số dương, ta sẽ sử dụng bất đẳng thức Cosi cho hai số thực dương
nhưng phải thỏa mãn điều kiện điểm rơi. Với căn thức cuối, với điểm rơi
x  y  z  x  z  y thì ta có ngay rằng
x  y  2z
 z  x z  y  .
2
Cũng với tư duy đó, ta sẽ thấy:
3  x  y   2  x  z
6  x  y x  z 
2
3  y  z   2  y  x
6  y  z y  x 
2
3
Nên 6  x  y x  z  6  y  z y  z   z  x z  y  3x  3 y  2 z .
2
2 2
Tức là P   Pmin  tại x  y  1; z  2 .
3 3
Bài toán kết thúc.
Bài tập tương tự:

1. Cho a; b; c  0 và thỏa mãn a  b  c  1 . Chứng minh rằng:


bc ca ab 1
P    .
a  bc b  ca c  ab 2
2. Cho a; b; c  0 và thỏa mãn a  b  c  1 . Chứng minh rằng:
bc ca ab 3
P    .
a  bc b  ca c  ab 2

Câu 3.

Tài liệu sưu tầm và tổng hợp bản word đầy đủ liên hệ 0393732038 TÀI LIỆU TOÁN HỌC
133
Website:tailieumontoan.com
B P C

M
I N

A D

Q
  BPM
1). Tứ giác BPIM nội tiếp và AD  BC , suy ra MAD   BIM
 , nên tứ giác AMID nội

tiếp.
Tương tự tứ giác DNIA nội tiếp.
Vậy các điểm A; M ; I ; N ; D thuộc một đường tròn ( K ) .
Nhận xét. Chứng minh năm điểm cùng thuộc một đường tròn ta chứng minh cho bốn
điểm trong đó tạo thành một tứ giác nội tiếp.
Nhắc lại kiến thức và phương pháp.
• Hai góc nội tiếp cùng chắn một cung của một đường tròn thì bằng nhau.
  BIM
BPM  của đường tròn ngoại tiếp tứ giác
 (hai góc nội tiếp cùng chắn cung BM
BPIM ).
• Một đường thẳng cắt hai đường thẳng song song tạo ra các cặp góc so le trong bằng
nhau.
  MAD
BPM  (hai góc so le trong của BP  AD ), suy ra BIM
  MAD
.

• Tứ giác có góc ngoài tại một đỉnh đối diện bằng góc trong không kề với nó thì tứ giác
đó là tứ giác nội tiếp.
 là góc ngoài tại đỉnh I và góc MAD
Tứ giác MIDA có góc BIM  là góc trong tại đỉnh

A không kề với I thỏa mãn BIM   MAD


 nên suy ra MIDA là tứ giác nội tiếp.
Hoàn toàn tương tự ta chứng minh được DNIA là tứ giác nội tiếp.
Do đó 5 điểm A; M ; I ; N ; D cùng thuộc một đường tròn.

2). Do các tứ giác BPIM và CPIN nội tiếp nên ta có QMI   BPI
  CNI  , suy ra tứ giác MINQ

nội tiếp.
Mà M ; I ; N  ( K ) , suy ra tứ giác MINQ nội tiếp đường tròn ( K ) .
Vậy Q thuộc đường tròn ( K ) .
Nhận xét. Chứng minh một điểm nằm trên một đường tròn ta chứng minh điểm đó cùng
ba điểm nằm trên đường tròn tạo thành một tứ giác nội tiếp.
Nhắc lại kiến thức và phương pháp.
• Tứ giác nội tiếp có góc ngoài tại một đỉnh bằng góc trong tại đỉnh không kề với nó.

Tài liệu sưu tầm và tổng hợp bản word đầy đủ liên hệ 0393732038 TÀI LIỆU TOÁN HỌC
134
Website:tailieumontoan.com
+ Tứ giác BPIM là tứ giác nội tiếp có góc ngoài tại đỉnh M và góc trong tại đỉnh I
  BPI
không kề với nó nên QMI .

+ Tứ giác CPIN là tứ giác nội tiếp có góc ngoài tại đỉnh P và góc trong tại đỉnh N
  CNI
không kề với nó nên BPI   CNI
 , suy ra QMI  .

• Áp dụng nhắc lại kiến thức.


 là góc trong tại đỉnh
 là góc ngoài tại đỉnh N và góc QMI
Tứ giác MINQ có góc INC
  QMI
M không kề với N thỏa mãn INC  nên suy ra MINQ là tứ giác nội tiếp.

• Qua ba điểm không thẳng hàng xác định được một và chỉ một đường tròn.
Bốn điểm M ; I ; N ; Q cùng thuộc một đường tròn mà ba điểm M ; I ; N cùng thuộc
đường tròn ( K ) nên suy ra Q cũng thuốc đường tròn ( K ) .

  PIC
3). Khi P; I ; Q thẳng hàng, kết hợp với Q thuộc đường tròn ( K ) ta có AIQ  (đối đỉnh);
  PNC
PIC  (do tứ giác NIPC nội tiếp).
  QND
PNC  (đối đỉnh); QND  (do tứ giác
  QID INDQ nội tiếp).
   .
 AIQ  QID , suy ra IQ là phân giác DIA nên IP là phân giác góc BIC
PB IB ID IB  ID BD PB BD
Do đó       .
PC IC IA IC  IA AC PC CA
Nhận xét. Chứng minh đẳng thức là tỷ số của các đoạn thẳng ta thường sử dụng mối liên
hệ giữa các đoạn thẳng thông qua định lý Ta-lét, tính chất đường phân giác trong/ngoài
tam giác, tính chất của tỷ lệ thức.
Nhắc lại kiến thức và phương pháp.
• Hai góc đối đỉnh thì bằng nhau.
 và góc PIC
+ Góc AIQ   PIC
 là hai góc đối đỉnh nên AIQ .
 và góc QND
+ Góc PNC  là hai góc đối đỉnh nên PNC
  QND
.
• Áp dụng nhắc lại kiến thức.
  PNC
+ PIC  của đường tròn ngoại tiếp tứ giác
 (hai góc nội tiếp cùng chắn cung PC
PINC ).
  QID
+ QND  (hai góc nội tiếp cùng chắn cung QD
 của đường tròn ngoại tiếp tứ giác
 
QNID ), suy ra AIQ  QID .
• Một tia nằm giữa hai tia và chia góc tạo bởi hai tia đó thành hai góc bằng nhau thì tia
này là tia phân giác.
  QID
Ta có IQ nằm giữa IA và ID mà AIQ .
 suy ra IQ là phân giác của góc AID
• Hai góc đối đỉnh có chung đường phân giác hay tia phân giác của góc này là tia đối
của tia phân giác góc kia.
+ IQ là phân giác của góc AID.
+ IQ là tia đối của tia IP .
 và góc BIC
Góc AID  là hai góc đối đỉnh, suy ra IP là phân giác của góc BIC

• Đường phân giác trong của một tam giác chia cạnh dối diện thành hai đoạn thẳng có
tỷ số bằng với tỷ số hai cạnh tương ứng của tam giác.
 của tam giác CBI nên PB  IB
Ta có IP là phân giác của góc trong BIC
PC IC

Tài liệu sưu tầm và tổng hợp bản word đầy đủ liên hệ 0393732038 TÀI LIỆU TOÁN HỌC
135
Website:tailieumontoan.com
• Định lý Ta-lét.
IB IC IB ID PB IB ID
Ta có BC  AD nên    , suy ra   .
ID IA IC IA PC IC IA
PB IB ID IB  ID BD PB BD
• Tỷ lệ thức       (điều phải chứng minh).
PC IC IA IC  IA AC PC AC

Câu 4. Giả sử A có n số, chúng ta xếp chúng theo thứ tự 1  x1  x2  x2    xn  100


(1).
Suy ra với mỗi k  1,2,3, , n  1 ta có xk1  xi  x j  xk  xk  2 xk (2), với 1  i , j  k.
Áp dụng kết quả 2 ta thu được x2  1  1  2 ; x3  2  2  4 ; x4  8 ; x5  16 ; x6  32 ;
x7  64 , suy ra tập A phải có ít nhất 8 phần tử.
+) Giả sứ n  8  x8  100 .
Vì x6  x7  32  64  96  x8  2 x7  x7  50.
Vì x5  x6  16  32  48  x7  2 x6  x6  25.
25
Vì x4  x5  8  16  24  25  x6  2 x5  x5  (mâu thuẫn).
2
+) n  9 ta có tập 1; 2; 3; 5; 10; 20; 25; 50; 100 thỏa mãn yêu cầu bài toán .
Đáp số: n  9 .
Nhận xét. Các bài toán dạng này chủ yếu đánh giá tư duy và không có cách giải tổng quát.
Nhắc lại kiến thức và phương pháp.
• Từ 1 đến 100 có 100 số tự nhiên. Tập hợp A là tập hợp con của tập  có phần tử nhỏ
nhất bằng 1 và lớn nhất bằng 100 nên tập hợp A không vượt quá 100 phần tử.
Tổng quát, tập hợp A có 2  n  100 phần tử, sắp xếp các phần tử này theo thứ tự
1  x1  x2  x2    xn  100
x  a
• Theo đề bài có x  a  b với x, a, b đều là thuộc tập hợp A nên ta có  do đó mỗi

x  b

k  1,2,3, , n  1 ta có xk1  xi  x j  xk  xk  2 xk với 1  i , j  k.
• Áp dụng kết quả xk1  xi  x j  xk  xk  2 xk ta được
x2  1  1  2 ; x3  2  2  4 ; x4  8 ; x5  16 ; x6  32 ; x7  64 ,
suy ra tập A phải có ít nhất 8 phần tử.
• Giả sử n  8 theo thứ tự giả sử ta suy ra được x8  100 .
• Áp dụng các kiến thức trên, cùng cách tính toán giải phương trình bậc nhất.
+ Vì x6  x7  32  64  96  x8  2 x7  x7  50.
+ Vì x5  x6  16  32  48  x7  2 x6  x6  25.
25
+ Vì x4  x5  8  16  24  25  x6  2 x5  x5  (mâu thuẫn).
2
• Vì A là tập hợp có ít nhất 8 phần tử mà xét trường hợp có 8 phần tử cho kết quả mâu
thuẫn nên tập hợp A có ít nhất 9 phần tử.
Với n  9 theo thứ tự giả sử ta suy ra được x9  100 từ đó ta tìm được một tập hợp
1,2,3,5,10,20,25,50,100 thỏa mãn yêu cầu bài toán.
Đáp số: n  9 .

Tài liệu sưu tầm và tổng hợp bản word đầy đủ liên hệ 0393732038 TÀI LIỆU TOÁN HỌC
136
Website:tailieumontoan.com

Đề số 19

Câu 1.

1) Cộng hai vế của hệ ta được (2x + 3y)2 = 25


Ta có hệ:

2 x + 3 y = 5 2 x + 3 y = −5
 2 và  2
x + y = 2 x + y = 2
2 2

−1
2) Điều kiện x ≥
2
Đặt 2 x + 1 = a (a ≥ 0); 4 x 2 − 2 x + 1 = b(b > 0)

Ta có (1- b)(a - 3) = 0

1
B = 1 thì x1 = 0; x 2 = ; a = 3 thì x3 = 4
2

Câu 2.

1) Phá ngoặc
(1 + x )(1 + y ) + 4 xy + 2 ( x + y )(1 + xy ) = 25. ⇔ ( xy + 1)
2 2 2
+ 2 ( x + y )(1 + xy ) + ( x + y ) 2 = 25
⇔ ( xy + 1 + x + y ) 2 = 25 ⇔ ( x + 1) 2 ( y + 1) 2 = 25
Vì x, y không âm nên (x + 1)(y + 1) = 5 ta có (x; y) = (0; 4) ; (4; 0)
k 2 + k +1 k2 k +1 k 1 1 1
2) Xét = + = + = 1− + (k ∈ N )
k (k + 1) k (k + 1) k (k + 1) (k + 1) k k +1 k
Thay k lần lượt từ 1 ta có
 3 7 n 2 + n + 1  1   n 
 + + ...  = n + 1 − = n + = n (đpcm)
1.2 2.3 n(n + 1)   n + 1  n + 1

Câu 3.

Tài liệu sưu tầm và tổng hợp bản word đầy đủ liên hệ 0393732038 TÀI LIỆU TOÁN HỌC
137
Website:tailieumontoan.com

1) Ta có
AC
cot ∠ACB = ⇔ AC = AB. cot 30o = 2 3R
AB
AB AB
sin ∠ACB = ⇒ BC = = 4R
BC sin 30o
1 1 1 1 1 1
= + = + = ⇒ AH = R 3
AH2 AB2 AC2 12R 2 4R 2 3R 2
2) Ta có ∠ACB = ∠HAB (cùng phụ với ∠CAH )
1
Mà ∠HAB = ∠HNB (cùng bằng số đo cung HB)
2
∠HBN = ∠ACB
Từ đó tứ giác CMNH nội tiếp. Tâm đường tròn nội tiếp CMNH thuộc trung trực của HC
cố định.
Câu 4.

Áp dụng bất đẳng thức Bu-nhi-a-cốp –xki ta có


a2 + 4 1
( a ;1 ) và (1; 4) ta có 17( a + 1) ≥ ( a + 4) ⇔ a + 1 ≥ (1); Dau :" =" ⇔ a =
2 4 2 2 4

17 2
b2 + 4 1
( b ;1 ) và (1; 4) ta có 17(b + 1) ≥ (b + 4) ⇔ b + 1 ≥ (1); Dau :" =" ⇔ b =
2 4 2 2 4

17 2
a2 + b2 + 8 5
Từ (1) và (2) ta có P ≥ (*) Mặt khác Từ GT ta có a + b + ab =
17 4
Lại áp dụng bất đẳng thức Cô – si ta có:

Tài liệu sưu tầm và tổng hợp bản word đầy đủ liên hệ 0393732038 TÀI LIỆU TOÁN HỌC
138
Website:tailieumontoan.com

 2 1
a + 4 ≥ a

 2 1 3 2 1 5 1 1
b + ≥ b ⇔ (a + b ) + ≥ (a + b + ab) = ⇔ a + b ≥ ; Dau :" =" ⇔ a = b =
2 2 2

 4 2 2 4 2 2
a + b
2 2

 ≥ ab
 2
1
+8
2 17
Thay Vào (*) ta có P ≥ =
17 2
17 1
Vậy Min( P) = ⇔a=b=
2 2
Đề số 20

Câu I.

1
1) Điều kiện: x ≥ −
3
Với x = 1 là nghiệm của phương trình
Với x > 1 vế trái lớn hơn 4. Phương trình vô nghiệm
Với x < 1, vế trái nhỏ hơn 4. Phương trình vô nghiệm
Vậy nghiệm của phương trình là x = 1.
2) Giải hệ phương trình
 5x2 + 2y2 + 2xy = 26  5x2 + 2y2 + 2xy = 26
 ⇔ 
 (
3x + 2x + )(
y x − y = )
11. 
3x + 2x2 − 2xy + xy − y2 =11

 5x2 + 2y2 + 2xy =26


 2 ( )
⇒ 5x2 + 2y2 + 2xy + 2 2x2 + 3x − y2 − xy = 26 + 2.11 = 48
2x + 3x − y − xy =
2
11

 x=2
⇔ 9x + 6x − 48 =0 ⇔ 
2
x = − 8
 3

Với x = 2 ta có:

 y =1
2.22 + 3.2 − y2 − 2y = 11 ⇔ y2 + 2y − 3 = 0 ⇔ 
 y = −3

8
Với x = − ta có
3
Tài liệu sưu tầm và tổng hợp bản word đầy đủ liên hệ 0393732038 TÀI LIỆU TOÁN HỌC
139
Website:tailieumontoan.com
2
 8  8 8 8 43
2  −  + 3  −  − y2 + y = 11 ⇔ y2 − y + = 0 Phương trình vô nghiệm
 3  3 3 3 9

Vậy nghiệm của hệ phương trình là (2; 1); (2; -3)

Câu II.

1) Giả sử n2 + 391 =
a 2 với a nguyên dương. Ta có
  n + a =1  n =−195
  (L)
 n − a =−391   a =196
( )(
n−a n+a = )
−391 ⇔ 
= n + a 391=
⇔
n 195
   TM ( )
  n − a =−1  a =196
   
Vậy số nguyên dương n thỏa mãn đề bài là 195.
2) Ta có

xy + z + 2x2 + 2y2 (
xy + z x + y + z + x + y)

1 + xy 1 + xy

( x + z )( y + z ) + x + y xy + z + x + y
≥ ≥ =
1
1 + xy 1 + xy
1
Dấu “=” xảy ra khi x= y= z=
3

Câu III.

1) Chứng minh rằng M là trực tâm của tam giác ABC.


Ta có các tứ giác BEPH và PHQM là tứ giác nội tiếp. Từ đó

Tài liệu sưu tầm và tổng hợp bản word đầy đủ liên hệ 0393732038 TÀI LIỆU TOÁN HỌC
140
Website:tailieumontoan.com

∠H1 =∠P1 =∠P2 =∠H2 mà ∠H2 =∠C1 (cùng phụ với ∠QHC )
∠H1 =∠C1 nên CM // EH ⇒ CM ⊥ AB tương tự BM ⊥ AC . Vậy M là trực tâm
của tam giác ABC.
2) Chứng minh rằng BEFC là tứ giác nội tiếp.
∠HPF (cùng bù với góc ∠HPE )
∠EBH =
∠HPF = ∠PFA ⇒ ∠EBH = ∠PFA
Vậy tứ giác BEFC nội tiếp.

Câu IV.

Số các số được đánh dấu ≥ 1

Nếu tất cả các số được đánh dấu là số dương ta có đpcm.

Nếu các số đánh dấu có số âm giả sử là a n thì số a n +1 là số dương cũng được đánh dấu và
a n + a n +1 > 0 , mọi số âm đều có số có tổng dương, các cặp số này không trùng nhau. Vậy
tổng các số được đánh dấu là dương.

Đề số 21

Câu 1.

1)ĐKXĐ: ∀x ∈ R
x 2 − x + 2 = 2 x 2 − x + 1 ⇔ x 2 − x + 1 − 2 x 2 − x + 1 + 1 = 0 ⇔ ( x 2 − x + 1 − 1) 2 = 0
⇔ x 2 − x + 1 = 1 ⇔ x( x − 1) = 0 ⇔ x = 0; hoac : x = 1
Phương trình có 2 nghiệm x1= 0;x2 = 1
2)
 x − y + xy = 1  y = 1 − x − xy  y 2 = 1 − x 2 − xy
2 2 2 2

 ⇔  2 ⇔ 
3 x + y = y 2 + 3  x + xy − 3 x − y + 2 = 0 ( x − 1)( x + y − 2) = 0
 y = 0
 x = 2 − y 
 y = 1 − x − xy
2 2
 2  x = 1
  y + 2 y − 3 = 0  x = 1
⇔  x = 1 ⇔ ⇔ 
 x = 2 − y 
x =1  y = 1
  y 2 − y = 0 
 x = 5
 y = −3
Vậy hệ có 3 nghiệm (x;y) = (1; 0); (1; 1); (5;-3)
Câu 2.

1) Ta có
13
( )
13 = 134 .13 ≡ 3(mod10);6 6 ≡ 6(mod10);2009 2009 = (2009 )
3 2008
.2009 ≡ 9(mod10)
1313 + 6 6 + 2009 2009 ≡ 3 + 6 + 9 ≡ 8(mod10)
Tài liệu sưu tầm và tổng hợp bản word đầy đủ liên hệ 0393732038 TÀI LIỆU TOÁN HỌC
141
Website:tailieumontoan.com
nên 1313 + 6 6 + 2009 2009 có tận cùng là 8
2)áp dụng BĐT Cô si cho 2 số dương
9a + 4a + 5b 13a + 5b
9a và 4a+5b ta có 9a (4a + 5b) ≤ =
2 2
9b + 4b + 5a 13b + 5a
9b và 4b+55 ta có 9b(4b + 5a ) ≤ =
2 2
3(a + b) 1 1
Nên P ≥ = nên Min(P) = khi a = b
18a + 18b 3 3
2
Cách khác áp dụng bất đẳng thức Bunhicôpsky 2 dãy
a; b và dãy 4a + 5b ; 4b + 5a ta có
( a (4a + 5b) + b(4b + 5a ) )
2
≤ (a + b)(9a + 9b) ⇔ a (4a + 5b) + b(4b + 5a ) ≤ 3(a + b)
a+b 1
⇔ ≥
a (4a + 5b) + b(4b + 5a ) 3
1
Min (P)=
3
4a + 5b 4b + 5a 4a + 5b 4b + 5a
khi = ⇔ = ⇔ 4ab + 5b 2 = 4ab + 5a 2 ⇔ a = b
a b a b
Câu 3.

1 O1
B H D

O2

a-Tam giác BKO1 dd tam giác O2KA nên


a O1 B O1 K O1 K
= = = = TgB1
b O2 A AK BK
AH a AH
Mà = tgB1 nên =
BH b BH
b
b-Theo a ta có HB = .HA
a
trong ∆ vuông BO2H ta có BH2+O2H2=O2B2
nên
b2 b2 b2
2
. AH 2
+ (b − AH ) 2
= b 2
⇔ 2
. AH 2
+ b 2
− 2bAH + AH 2
= b 2
⇔ 2
. AH 2 − 2bAH + AH 2 = 0
a a a
2 2
b 2a b 2ab 2
⇔ 2 . AH − 2b + AH = 0 ⇔ (a 2 + b 2 ) AH = 2a 2 b ⇔ AH = 2 suyra : BH =
a (a + b 2 ) (a 2 + b 2 )

Tài liệu sưu tầm và tổng hợp bản word đầy đủ liên hệ 0393732038 TÀI LIỆU TOÁN HỌC
142
Website:tailieumontoan.com

8a 3b 3
S ABCD = 2. AH .BH =
(a 2 + b 2 ) 2
Câu 4.

4a + 6b
Ta có 3a 2 + 8b 2 + 14ab = (3a + 2b)(a + 4b) ≤ = 2a + 3b
2
4b + 6c
3b 2 + 8c 2 + 14cb = (3b + 2c)(b + 4c) ≤ = 2b + 3c
2
4c + 6a
3c 2 + 8a 2 + 14ac = (3c + 2a )(c + 4a ) ≤ = 2c + 3a
2
Ta có
a2 b2 c2
P= + +
3a 2 + 8b 2 + 14ab 3b 2 + 8c 2 + 14bc 3c 2 + 8a 2 + 14ca
a2 b2 c2
P≥ + + =Q
2a + 3b 2b + 3c 2c + 3a

Áp dụng BĐT Bunhiacopsky cho 2 dãy


a b c
; ; và 2a + 3b ; 2b + 3c ; 2c + 3a
2a + 3b 2b + 3c 2c + 3a
1 1
Ta có Q(5a + 5b + 5c) ≥ (a + b + c) 2 ⇔ Q ≥ (a + b + c) vậy P ≥ Q ≥ (a + b + c)
5 5
Dấu “=” xảy ra khi khi a=b=c

1
Cách khác: Chứng minh Q ≥ (a + b + c) áp dụng BĐT Cô-Si
5
a 2
2a + 3b 2a a 2
2a 2a + 3b 8a − 3b
+ ≥ ⇔ ≥ − = tương tự
2a + 3b 25 5 2a + 3b 5 25 25
b2 8b − 3c c2 8c − 3a
≥ ; ≥
2b + 3c 25 2c + 3a 25
Vậy
8a − 3b 8b − 3c 8c − 3a a + b + c
Q≥ + + =
25 25 25 5
Đề số 22

Câu 1.
1)ĐKXĐ: x ≥ −1
Đặt x + 35 = a.; x + 1 = b; (a > 0; b ≥ 0)
14 x + 35 + 6 x + 1 = 84 + x 2 + 36 x + 35 ⇔ 14a + 6b − 84 − ab = 0
a = 6  x = 1; (T / m)
⇔ (a − 6)(14 − b) = 0 ⇔  ⇔
b = 14  x = 195; (T / m)
2).Dùng phương pháp quy nạp toán học
* Với n = 1 đúng giả sử đúng với n = k ta có

Tài liệu sưu tầm và tổng hợp bản word đầy đủ liên hệ 0393732038 TÀI LIỆU TOÁN HỌC
143
Website:tailieumontoan.com
k2 (k + 1) 2
Sk = ; ta phải chứng minh đúng với n=k+1 nghĩa là S k +1 =
4k 2 + 1 4(k + 1) 2 + 1
2(k + 1) − 1 (k + 1) 2 k2 2(k + 1) − 1
S k +1 = S K + ⇔ = +
4 + (2k + 1) 4
4(k + 1) + 1 4k + 1 4 + (2k + 1) 4
2 2

(k + 1) 2 k2 2(k + 1) − 1
⇔ − =
4(k + 1) + 1 4k + 1 4 + (2k + 1) 4
2 2

(k + 1) 2

k2
=
[
(k + 1) 2 (4k 2 + 1) − k 2 4(k + 1) 2 + 1) ]
Ta có :
4(k + 1) 2 + 1 4k 2 + 1 [ ]
4(k + 1) 2 + 1) (4k 2 + 1)
4k + 8k + 4k + k + 2k + 1 − 4k − 8k − 4k 2 − k 2
4 3 2 2 4 3
2k + 1
= = ; ( DPCM )
16k + 32k + 24k + 8k + 5
4 3 2
4 + (2k + 1) 4
Cách khác:
a 1 1 1 
đặt a = 2n - 1( n ∈ N * ) xét tổng quát =  2 − 2  thay n lần lượt từ
4+a 4
4  a − 2a + 2 a + 2a + 2 
1 ;2;3;4;…. Ta có a lần lượt 1;3;5;7;…..

1 1 1 1 1 1 4n 2
Ta có 4S = − + − + ............... − = 1 − =
1 5 5 13 (2n − 1 + 1) 2 + 1 (2n − 1 + 1) 2 + 1 4n 2 + 1
Câu 2.

1) Với n < 6 không có số nào thoả mãn


Với n = 6 thoả mãn Với n>6 n có dạng n = 7k;7k+1;7k+2;7k+3;7k+4;7k+5;7k+6 ( (k ∈ N * )
n = 7k thì n+7  7; n=7k+1 thì n+13  7; n=7k+2 thì n+5  7
n=7k+3 thì n+25  7; n=7k+4 thì n+17  7; n=7k+5 thì n+37  7; n=7k+6 thì n+1  7
Vậy chỉ có duy nhất n=6 thoả mãn đề bài
2) Ta thấy a-b  7;(a+c)-(b+d)=((a-b)+(c-d))  7 mà 2014-844=1170=167x7+1
Không chia hết cho 7 vậy sau một số hữu hạn lần thay thế ta không thể nhận được tập
hợp các cặp số M 1 = {(2018,702), (844,2104), (1056,2176), (2240,912)}
Câu 3.

P
A
C

K O O'

B
Q

1) ta có ∠ DCB= ∠ DAB (Chắn cungBD)


∠ DAB = ∠ QPB (chắn cungBQ)
Tài liệu sưu tầm và tổng hợp bản word đầy đủ liên hệ 0393732038 TÀI LIỆU TOÁN HỌC
144
Website:tailieumontoan.com
Nên ∠ DCB= ∠ QPB(1)
∠ DBC= ∠ PAQ( cùng bù ∠ DAC)
∠ PAQ = ∠ PBQ ( chăn cung PQ)
Nên ∠ DBC= ∠ PBQ(2)
Từ (1) ;(2) ta có
∆ BCD đ d với ∆ BPQ (g.g)
2)theo a) thì ∠ DCB= ∠ QPB nên tứ giác KPCB nội tiếp nên đường tròn ngoại tiếp tam giác
KCP luôn đi qua điểm B cố định khi M thay đổi
Câu 4.

Giả sử x,y,z là những số thực thoả mãn điều kiện


0 ≤ x, y, z ≤ 2 và x + y + z = 3
Tìm giá trị nhỏ nhất và lớn nhất của biểu thức :
M = x 4 + y 4 + z 4 + 12(1 − x )(1 − y )(1 − z )
Giải: cách 1 đặt 1-x = a;1 - y = b;1 – z = c thì a + b + c = 0 nên a3 + b3 + c3 = 3abc
và − 1 ≤ a; b; c ≤ 1
M=(1-a)4+(1-b)4+(1-c)4+12abc=a4+b4+c4-4(a3+b3+c3-3abc)+6(a2+b2+c2)-4(a+b+c)+3
M= a4+b4+c4+6(a2+b2+c2)+3
áp dụng BĐT Bunhiacopsky cho 2 dãy a;b;c và 1;1;1 ta có 3(a2+b2+c2) ≥ (a+b+c)2=0
Nên a2+b2+c2 ≥ 0 Dấu “=” xảy ra khi a=b=c=0
áp dụng BĐT Bunhiacopsky cho 2 dãy a2;b2;c2 và 1;1;1 ta có 3(a4+b4+c4) ≥ (a2+b2+c2)2=0 ⇔
a4+b4+c4 ≥ 0 Dấu “=” xảy ra khi a = b = c = 0
Vậy M ≥ 3 suy ra min (M)= 3 khi a=b=c=0 khi đó x=y=z=1
Mặt khác a2+b2+c2=(1-x)2+(1-y)2+(1-z)2=3-2(x+y+z)+x2+y2+z2= x2+y2+z2-3
Giả sử 0 ≤ x ≤ y ≤ z ≤ 2 ; ta có x2+y2+z2-3=(x+y)2-2xy+z2 -3 ≤ (3-z)2 +z2-3=2z2-6z+6
2
 3 3
x +y +z -3=2z -6z+6= 2 z −  + ≤ 2 vì 0<z ≤ 2 suy ra a2+b2+c2 ≤ 2
2 2 2 2

 2 2
ta có a2;b2;c2 ∈ [0;1] nên (a4+b4+c4) ≤ ( a2+b2+c2) suy ra M ≤ 7(a2+b2+c2)+3 ≤ 17
Max(M)=17 khi (a;b;c)=(-1;0;1) và các hoán vị hay (x;y;z) =(0;1;2)và các hoán vị

Cách 2: thay x4+y4+z4=(x2+y2+z2)2-2(x2y2+y2z2+z2x2)


Mà x2+y2+z2=(x+y+z)2-2(xy+yz+zx)=9-2a (đặt xy+yz+zx=a)
x y +y z +z x =(xy+yz+xz) -2xyz(x+y+z)=a -6xyz
2 2 2 2 2 2 2 2

12(1-x)(1-y)(1-z)=12(1-x-y-z+xy+yz+zx-xyz)=12a-12xyz-24
Nên M=(9-2a)2-2(a2-6xyz)+ 12a-12xyz-24=81-36a+4a2-2a2+12xyz+12a-12xyz-24
M=2a2-24a+57=2(a-6)2-15 (*)
Ta có x2+y2+z2 ≥ xy+yz+zx ⇔ (x+y+z)2 ≥ 3(xy+yz+zx) nên a ≤ 3
Ta cũng có (2-x)(2-y)(2-z) ≥ 0 ⇔ 8-4(x+y+z)+2(xy+yz+zx)-xyz ≥ 0
⇔ xy+yz+zx ≥ 2+xyz ≥ 2 hay a ≥ 2 vậy 2 ≤ a ≤ 3 thay suy ra -4 ≤ (a-6) ≤ -3
Hay 16 ≥ (a-6)2 ≥ 9 vào (*) 3 ≤ M ≤ 17
Vậy Min(M)= 3 khi a=3 khi đó x=y=z=1;
Max(M) =17 khi a=2 khi đó (x;y;z)=(0;1;2) và các hoán vị

Tài liệu sưu tầm và tổng hợp bản word đầy đủ liên hệ 0393732038 TÀI LIỆU TOÁN HỌC
Website:tailieumontoan.com

ĐẠI HỌC QUỐC GIA HÀ NỘI ĐỀ THI TUYỂN SINH LỚP 10


TRƯỜNG ĐẠI HỌC KHOA HỌC TỰ NHIÊN HỆ THPT CHUYÊN NĂM 2008

MÔN : TOÁN (Vòng 2)


Thời gian làm bài: 150 phút (không kể thời gian phát đề)

Câu I: ( 3 điểm )
−6 xy ( 2 x − y ) =
−6
1 ( 2 điểm ) Hệ đã cho tương đương với 
(2 x) − y =
3 3
7
Cộng hai phương trình của hệ ta thu được (2 x − y )3 =1 ⇒ 2 x − y =1
Từ phương trình thứ nhất của hệ ⇒ xy = 1
 1
2 x =
−y 1 = x 1 x = −
Từ  ⇒ và  2
=  xy 1= y 1  y = −2
2 ( 1 điểm )
Cách thứ nhất:
1 1 3
y = x + 2 (1 − x ) ≤ x + + (1 − x ) =
2 2 2
1
Dấu bằng đạt được khi x =
2
Cách thứ hai:
Đặt 1 − x = t ⇒ 0 ≤ t ≤ 1 và x = 1 − t 2
2
 1  3 3
⇒ y =−t + 2t + 1 =−  t −
2
 + ≤
 2 2 2
1 1
Dấu bằng đạt được khi t= ⇔ x=
2 2
3
⇒ ymax =
2
Câu II: ( 3 điểm )
1 ( 2 điểm ) Phương trình đã cho tương đương với ( 2 x + y + 1)( x + y + 1) =−1
2 x + =
y +1 1 = x 2
a)  ⇒
 x + y + 1 =−1  y =−4
2 x + y + 1 =−1  x =−2
b)  ⇒
 x=+ y +1 1 = y 2
2 ( 1 điểm ) : Ta phải có

Liên hệ tài liệu word môn toán: 039.373.2038 TÀI LIỆU TOÁN HỌC
Website:tailieumontoan.com

abc∣( ab − 1)( bc − 1)( ca − 1)


 ⇔ abc∣−1 + ab + bc + ca − abc ( a + b + c ) + (abc) 2
⇔ abc∣ab + bc + ca −
 1
Không mất tổng quát có thể giả sử 1 ≤ a ≤ b ≤ c
Ta có abc ≤ ab + bc + ca − 1 < ab + bc + ca < c ( 2a + b )
> ab < 2a + b ≤ 3b ⇒ a < 3 ⇒ a ∈ {1, 2}
=
Xét trường hợp a = 1 ta có:
bc bc + b + c − 1 ⇒ bc b + c − 1
⇒ bc ≤ b + c − 1 ⇒ ( b − 1)( c − 1) ≤ 0
⇒ ( b − 1)( c − 1) =
0
( ab − 1)( bc − 1)( ca − 1)
Với =
a 1,=
b 1 ta có : = 0  ∀c
abc
Vậy trong trường hợp này ta thu được nghiệm : = a 1,= b 1, c nguyên dương tùy ý.
Do vai trò của a, b, c như nhau trong ba số a, b, c có hai số bằng 1 và một số nguyên dương tùy
ý là đáp số của bài toán.
Xét trường hợp a = 2
2bc 2 ( b + c ) + bc − 1 ⇒ 2bc 4 ( b + c ) + 2bc − 2
⇒ 2bc 4 ( b + c ) − 2 ⇒ bc 2 ( b + c ) − 1
⇒ bc ≤ 2 ( b + c ) − 1 ⇒ ( b − 2 )( c − 2 ) ≤ 3
⇒ ( b − 2 )( c − 2 ) ∈ {0,1, 2,3}
a) Nếu ( b − 2 )( c − 2 ) =0 ⇒ nếu c = 2 → b = 2
3 ( ab − 1)( bc − 1)( ca − 1) 3.3.3 27
Lúc đó
= P = = ( loại)
abc 2.2.2 8
3(2c − 1) 2
Nếu b = 2 ⇒ P = không là số nguyên => loại
4c
b) Nếu ( b − 2 )( c − 2 ) =1 ⇒ b =3, c =3
25.8 100
=P = ( loại )
2.3.3 9
5.11.7
c) Nếu ( b − 2 )( c − 2 ) = 2 ⇒ b = 3, c = 4 Lúc đó P = ( loại )
2.3.4
5.14.9
d) Nếu ( b − 2 )( c − 2 ) = 3 ⇒ b = 3, c = 5 Lúc=
đó P = 21 ( thỏa mãn)
2.3.5
Vậy ta được đáp số= a 2,= b 3,=c 5
Do vai trò của a, b, c như nhau nên trong trường hợp này các nghiệm của bài toán là các hoán vị
của ( 2,3,5 ) .
Câu III: ( 3 điểm )

Liên hệ tài liệu word môn toán: 039.373.2038 TÀI LIỆU TOÁN HỌC
Website:tailieumontoan.com

1 (1,5 điểm )
Giả sử phân giác của góc KBQ cắt PQ tại I .
Nối IC ta chứng minh rằng CI là phân giác của góc KCQ
= IBQ + IQB
Ta có BIP
Nhưng IBQ = IBK ( phân giác )
QB = KBP   ( góc nội tiếp )
⇒ BIP = IBP ⇒ PBI cân tại đỉnh P.DoPB = PC => PCI cân tại đỉnh P
Ta có : =
PIC PQC + QCI
= KCI + KCP
PCI
Do   PCI cân tại P nên PIC = PCI
⇒ PQC + QCI = KCI + KCP
Nhưng do PQC = KCP (góc nội tiếp)
⇒ QCI = KCI ⇒ CI là phân giác của góc KCQ ⇒ điều phải chứng minh
2)( 1,5 điểm )

Cách thứ nhất: Goi O là tâm đường tròn ngoại tiếp ABC .
PC 2 = PM PO
Ta có :  .
PC 2 = PK .PQ
 ⇒ PM .PO = PK .PQ
⇒   Tú giác KMOQ nội tiếp
⇒  AKQ = QOH
1
Do AKQ = QOA ⇒ QOH = AOH ⇒ OH là phân giác của tam giác cân OAQ
2
⇒ OH ⊥ AQ ⇒ AQ / / BC vì cùng vuông góc với PH
Cách thứ hai:
Liên hệ tài liệu word môn toán: 039.373.2038 TÀI LIỆU TOÁN HỌC
Website:tailieumontoan.com

MC MA
MKC ∼MBA = > =
KC BA
MB MA
MKB ∼MCA = > =
KB CA
KB CA
Chia (1) cho (2) vế theo vế => =
KC BA
Mặt khác, theo kết quả của câu 1) ta có
KB KC KB QB
= ⇒ =
QB QC KC QC
AC QB
Tì (3) và (4) => =
AB QC
= > ABC ∼QBC = > ACB ==⇒
QBC AQB AQ / / BC
Câu IV: ( 1 điểm )
Ta chứng minh rằng với x ≥ 2 thì
x |n x |n −1 + x |n − 2 +… + x + 1

(
−1 ∣( x∣−1) x |n −1 + x |n − 2 +… + x + 1
Thực vậy, ta có | x |n = )
Do x ≥ 2 ⇒ x − 1 ≥ 1 ⇒ x |n −1 ≥ x |n −1 + | x |n − 2 +… + x + 1
⇒ x |n x |n −1 + x |n − 2 +… + x + 1
Do ai nhận các giá trị 0, ±1 nên ta có
x |n −1 + x |n − 2 +… + x + 1 ≥ a1 x n −1 + a2 x n − 2 + … + an x + an
≥ a1 x n −1 + a2 x n − 2 + … + an −1 + an
Từ (1) và (2) ⇒> với x ≥ 2 thì
=
a0 x n x |n a1 x n −1 + a2 x n − 2 + … + an −1 x + an
Chú ý rằng nếu a > b thì a + b và a cùng dương hoặc cùng âm ( thật vậy,
a ( a + b ) = a 2 + ab ≥ a 2 − ab = a ( a − b ) > 0
Do đó từ (3) với x ≥ 2 , ta có:
a0 x n + a1 x n −1 + a2 x n − 2 + … + an −1 x + an và a0 x n cùng dương hoặc cùng âm ( a0 x n ≠ 0 ) Do vậy
phương trình đã cho không có nghiệm x mà x ≥ 2
Vậy, nếu phương trình có nghiệm x0 thì x0 < 2 .

ĐẠI HỌC QUỐC GIA HÀ NỘI ĐỀ THI TUYỂN SINH LỚP 10


TRƯỜNG ĐẠI HỌC KHOA HỌC TỰ NHIÊN HỆ THPT CHUYÊN NĂM 2007

Liên hệ tài liệu word môn toán: 039.373.2038 TÀI LIỆU TOÁN HỌC
Website:tailieumontoan.com

MÔN : TOÁN (Vòng 1)


Thời gian làm bài: 150 phút (không kể thời gian phát đề)

Câu I (3 điểm)
1 (1,5 điểm) Phương trình đã cho tương đương với:
2x + 1 ( )
2x − 1 − 1 − x = ( )
2 x − 1 − 1 0  Ñieàu kieän ≥
1
2
  

(
⇔   2x − 1 − 1 )( 2x + 1 − x =0 )
a) 2 x − 1 − 1 = 0
→  1x=
b) 2 x + 1 − x = 0
→  1x= −
(loại)
Vậy nghiệm của phương trình là: x = 1
2 (1,5 điểm) Phương trình thứ 2 của hệ có dạng
( x + y )3 − 3 xy ( x + y ) + x + y =4   ⇔ ( x + y )3 + ( x + y ) − 10 =0

(
đặt: x + y = t  ⇒  t 3 + t − 10 = 0 ⇔  ( t − 2 ) t 2 + 2t + 5 = 0 )
x + y = 2
⇒ t = 2  vôùi   ⇒x= y= 1
 xy = 1
Câu II (3 điểm)
1 (1,5 điểm)
 x1 + x2 = 4
  
 x1 x2 = 1
x12 + x22 = ( x1 + x2 ) − 2 x1 x2 = 14
2

x13 + x23 = ( x1 + x2 ) − 3 x1 x2 ( x1 + x2 ) = 52
3

( )( )
x15 + x25 = x13 + x23 x12 + x22 − ( x1 x2 ) ( x1 + x2 ) =
⇒  724
2

2 (1,5 điểm)
4a + a + b= ( 4 + 2 ) + ( a + 1) + ( b + 2007 ) − 2010
a

4a + 2 = ( 4 − 1) + 3 = ( 4 − 1) ( 4 + … + 4 + 1) + 3 3
a a −1
P
mặt khác:
4a + 2 laø soá chaün → 4a + 2  6
4a + a + b là tổng của các số hạng đều chia hết cho 6 . Vậy ta có điều phải chứng minh.
Câu III (3 điểm)

Liên hệ tài liệu word môn toán: 039.373.2038 TÀI LIỆU TOÁN HỌC
Website:tailieumontoan.com

1 (1,5 điểm) Xét tứ giác CEFD có:


 chắn cung
CEF
 có đỉnh D nằm trong đường tròn nên
BDF
 1   

BDF =  sđ  AM + BF 
2  
1    1
( )

=  sđ  MB+ BF= 
sđ MBF

2   2
=
Từ (1) và (2) → CEF 
BDF
 + CDF
→ CEF  = 180 → tứ giác CEFD nội tiếp.
 = AEM
2 (1,5 điểm) vì MAB  (chắn hai cung bằng nhau) nên theo tiêu chuẩn nhận biết của
góc giữa tia tiếp tuyến và dây cung ta có MA tiếp xúc với đường tròn ngoại tiếp
ACE → MA ⊥ AO1 → nếu kéo dài AO1 cắt đường tròn ( O ) tại N thì MN là đường
kính của đường tròn ( O ) . Do M cố định nên N cố định.
Tương tự MB tiếp xúc với đường tròn ngoại tiếp BDF nên BO 2 phải đi qua N. Từ đó ta có
điều phải chứng minh.
Câu IV (1 điểm)
a b c
Ta có:   + + =
ab + a + 1 bc + b + 1 ca + c + 1
a ab abc
 = + + 2
ab + a + 1 abc + ab + a a bc + abc + ab
a ab 1
=
 1 + + =
ab + a + 1 ab + a + 1 ab + a + 1
Liên hệ tài liệu word môn toán: 039.373.2038 TÀI LIỆU TOÁN HỌC
Website:tailieumontoan.com

Áp dụng bất đẳng thức Bunhia Cốp ski với a, b, c, x, y, z > 0


Ta có:
x y z 2
( x + y + z ) 2  =∃ a + b + c θ ≤
a b c
 x2 y 2 z 2 
 ≤ ( a + b + c )  + + 
 a b c 
x 2 y 2 z 2 ( x + y + z )2
 ⇒ + + ≥
a b c a+b+c
Ta có:
a b c
+ + =
(ab + a + 1) 2
(bc + c + 1) 2
(ca + c + 1) 2
a ab + a + 1 b c
+ +
a a abc + b + 1 aca + c + 1
a b c 1
 ≥ + + =
ab + a + 1 bc + b + 1 ca + c + 1 a + b + c

ĐẠI HỌC QUỐC GIA HÀ NỘI ĐỀ THI TUYỂN SINH LỚP 10


TRƯỜNG ĐẠI HỌC KHOA HỌC TỰ NHIÊN HỆ THPT CHUYÊN NĂM 2005

MÔN : TOÁN (Vòng 1)


Thời gian làm bài: 150 phút (không kể thời gian phát đề)

 x= + y + xy 3 =S + P 3 x + y = S
Bài 1. Hệ đã cho tương đương với  ⇔  với   
 ( x + y ) −=  S −=  xy = P
2 2
2 xy 2 2P 2

S = 2

P = 1
Hệ (*) có nghiệm là   ⇒ x = y =1
  
 S = −4
 
  P = 7
Vậy nghiệm của hệ đã cho là: x= y= 1
Bài 2. Phương trình đã cho tương đương với

Liên hệ tài liệu word môn toán: 039.373.2038 TÀI LIỆU TOÁN HỌC
Website:tailieumontoan.com

11 − x − 4 x + 3 − 2 3 − 2 x =0
( ) (
⇔  x + 3 − 4 x + 3 + 4 + 3 − 2 x − 2 3 − 2 x + 1 =0 )
⇔  ( x + 3 − 2) 2 + ( 3 − 2 x − 1) 2 =0
 x + 3 − 2 = 0
⇔  ⇔  1
x=
 3 − 2 x − 1 =0

Bài 3. Phương trình đã cho có dạng x 2 + 17  y 2 + 2 xy + 3 ( x + y )  =


1740

Chú ý rằng với số x nguyên, x có thể có dạng như sau:


x 17 k ± r   với r = 0,1, 2,3, 4,5, 6, 7,8
=

Từ đó suy ra x 2 có các dạng tương ứng sau:


x 2 = 17 k
=x 2 17 k + 1
=x 2 17 k + 4
=
x 2 17 k + 9
=
x 2 17 k + 16
=
x 2 17 k + 8
=x 2 17 k + 2
=
x 2 17 k + 15
=
x 2 17 k + 13
Nhận thấy rằng vế phải 1740 khi chia cho 17 có số dư là 6. Trong khi đó vế trái khi chia cho 17
trong mọi trường hợp đều không có số dư là 6 . Vậy phương trình đã cho không có nghiệm
nguyên.
Bài 4.
1 (Xem hình 1)
Cách 1: Ta có OM / / O ′D ( OM và O ′D cùng vuông góc với CD )

′ = OO
⇒ MOO  
′D < IO 
′D =
IO 
′M OO ′M ⇒ OM O ′M

Cách 2: Gọi M ′ là giao điểm của OA và O ′D . Ta xét hình bình hành MOM ′O ′ . Ký hiệu S là
diện tích hình bình hành đó, ta có
OM AB
= =
S AB.O ′M CD.OM ⇒ =
O ′M CD
Vì I thuộc đoạn AB , nên AB = AI + IB = AI + ID = AI + IC + CD > CD hay
Liên hệ tài liệu word môn toán: 039.373.2038 TÀI LIỆU TOÁN HỌC
Website:tailieumontoan.com

AB
>1
CD
Từ (1) và (2) suy ra đpcm.
2 (Xem hình 2)
Cách 1 Tứ giác ACBE nội tiếp và IA = IC , nên IB = IE . Mặt khác ta có IB = ID , do đó
= ID
IB = IE và  BED vuông tại B suy ra
BD ⊥ BE
Tứ giác ADBF nội tiếp và IB = ID , nên
= IF   
IA ⇒ AF / / BD
Từ (1) và ( 2 ) suy ra AF ⊥ BE .

Cách 2: Ta có
 
= FDB
FAB 
= IDB 
= 
= IBD ABD ⇒ AF / / BD
 và IO là phân giác DIA
Vì IO ′ là phân giác BID  , nên IO ⊥ IO ′ .

Lại có AC ⊥ IO , do vậy AC / / IO ′ . Ta có

=
ABE   ⇒ BE / / AC / / O ′I
ACE = BAC
Từ (1) và ( 2 ) suy ra BD ⊥ EB ⇒ AF ⊥ BE .

1 1
Bài 5. Điều kiện đã cho có thể viết lại là xy 2 + x 2 +y 2 =
3
z z
1
Biểu thức P có dạng P =
1
4
+ x4 + y 4
z
1
Đặt = t , ta thu được bài toán sau:
z
Với x, y, t > 0 thoả mãn xy 2 + yt 2 + tx 2 =
3 , tìm giá trị lớn nhất của biểu thức
1
P=
x + y4 + t 4
4

Ta có

Liên hệ tài liệu word môn toán: 039.373.2038 TÀI LIỆU TOÁN HỌC
Website:tailieumontoan.com

 x 4 + y 4 + y 4 + 14 xy 2
 4 4 4
 y + t + t + 14 yt
2

 t 4 + x 4 + x 4 + 14tx 2

( ) (
⇒ x 4 + y 4 + t 4 + 34 xy 2 + yt 2 + tx 2 =
 3 12 )
⇒ x + y + t4 ≥ 3
   4 4

1 1
 ⇒ 4 
x + y +t
4 4
3

1
Vậy P = đạt được khi x= y= z= 1 .
3

ĐẠI HỌC QUỐC GIA HÀ NỘI ĐỀ THI TUYỂN SINH LỚP 10


TRƯỜNG ĐẠI HỌC KHOA HỌC TỰ NHIÊN HỆ THPT CHUYÊN NĂM 2005

MÔN : TOÁN (Vòng 2)


Thời gian làm bài: 150 phút (không kể thời gian phát đề)

Bài 1. Giải phương trình 2 − x + 2 + x + 4 − x2 =


2.
Điều kiện: −2x2 .
t2 − 4
Đặt t = 2 − x + 2 + x ⇒ 4 − x2 = .
2
Phương trình đã cho có dạng
t 2 + 2t − 8 =0
 t1 = −4  loai 
⇔ 
t2 = 2 ⇔ 2 − x + 2 + x = 2 ⇔ 4 + 2 4 − x = 4 ⇔ x = ±2
2

1 nên phương trình thứ hai của hệ có dạng:


Bài 2. Do x3 + y 3 − xy 2 =
4x4 + y 4 = ( 4 x + y ) ( x3 + y 3 − xy 2 )
(
⇒   xy 3 y 2 − 4 xy + x 2 =
0 )
a) x =0 ⇒   y =1 ⇒ hệ có nghiệm ( 0,1) . 2.31. Đáp án tuyển sinh lớp 10 năm 2005 (cho thí sinh
chuyên toán và chuyên tin)129

Liên hệ tài liệu word môn toán: 039.373.2038 TÀI LIỆU TOÁN HỌC
Website:tailieumontoan.com

b) y =0 ⇒   x =1 ⇒ hệ có nghiệm (1, 0 ) .
c) 3 y 2 − 4 xy + x 2 =
0
2
 y  y
Với x ≠ 0 chia cả hai vế của phương trình   ()
*
cho x ta nhận được: 3   − 4   + 1 =
2

x x
0

 y 1
Đặt   = t   
⇒ 3t 2 = 4t + 1 = 0  
⇒ t1 = 1, t2 =
x 3
 y
Với   =1    
⇒ x =y ⇒ x =y =1
x
 y 1 1 3
Với   = ⇒ x =3 y ⇒ 25 y 3 =1 ⇒ y = 3 ⇒ x =3
x 3 25 25
Vậy hệ có tất cả các nghiệm là:
    3
    x= 3
=
 x 0= x 1=x 1  25
 ; ; ;  
=
 y 1= y 0=y 1 y = 1
    3
25

Bài 3.
1 ( )
Từ ( x + y ) 2 2 x 2 + y 2 = 2 ⇒ x + y ≤ 2 .
1
Dấu bằng xảy ra khi x= y= .
2
Ta lại có: ( x + y ) 2 =x 2 + y 2 + 2 xy =1 + 2 xy1  
⇒ x + y1
Dấu bằng xảy ra khi x = 0 hoặc y = 0
2.
= 1 + 2x + 1 + 2 y

P 2 =2 + 2 ( x + y ) + 2 1 + 2 ( x + y ) + 4 xy
 

Do x + y 2 và 4 xy2 x 2 + y 2 =
2( )
⇒ P 22 + 2 2 + 2 1 + 2 2 + 2
  

⇒ P 2 + 2 2 + 2 3 + 2 2
  

Vậy Pmax = 2 + 2 2 + 2 3 + 2 2
1
Đạt được khi x= y=
2

Liên hệ tài liệu word môn toán: 039.373.2038 TÀI LIỆU TOÁN HỌC
Website:tailieumontoan.com

Mặt khác, do x + y1 và 4 xy0


Ta có:
P 22 + 2 + 2 1 + 2 + 0
⇒ P 4 + 2 3

=
Vậy Pmin 4+2 3
Đạt được khi x = 0 hoặc y = 0
Bài 4.
1 Lấy điểm P ′ khác phía với điểm P đối với đường thẳng Ab sao cho  BPP ′ vuông cân
(vuông tai B ) (xem hình 1 ).

Ta có ∆BPC = ∆BP ′A  (c.g.c) ⇒ BP ′A = 1350
= 
Do BP ′P 45
=0 
⇒ PP ′A 900
=
Theo Pitago: PA 2
′P 2 PC 2 + 2 PB 2
AP '2 + P=
2 Trước hết ta chứng minh nhận xét sau (xem hình 2)
Cho hình chữ nhật ABCD, I là điểm nằm trong hình chữ nhật. Qua I kẻ các đường thẳng
MN , PQ tương ứng song song với AB, AD . Gọi diện tích hình chũ̃ nhật IPBN là S1 , diện tích
hình chữ nhật IQDM là S 2 .
Khi đó, S1 = S 2 khi và chỉ khi I thuộc đường cheo AC .
Thật vậy, giả sử I thuộc đường chéo AC , chú ý rằng đường chéo của hình chữ nhật chia hình
chữ nhật thành hai phần có diện tích bằng nhau nên dễ dàng suy ra S1 = S 2 .
Ngược lại, giả sử S1 = S 2 khi đó do
S1 = S 2
⇒ IN .IP =IM .IQ
IN IQ NC
⇒ = =
IM IP MA
=
Suy ra hai tam giác vuông MAI , NIC dồng dạng với nhau ⇒ MIA .
$ NIC
Do M , I , N thẳng hàng suy raA, I , C thẳng hàng.
Bây giờ ta chứng minh bài toán (xem hình 3)
Dễ thấy NBMQ là hình chữ nhật.
Qua P và Q kẻ các đường thẳng song song với các cạnh của hình vuông.
Do P thuộc đường chéo AM của hình chữ nhật ABMR nên S BLPK = S PIRS .
Do P thuộc đường chéo CN của hình chữ nhật NBCH nên S BLPK = S PTHF .
Từ đó suy ra: S PIRS = S PTHF .
Liên hệ tài liệu word môn toán: 039.373.2038 TÀI LIỆU TOÁN HỌC
Website:tailieumontoan.com

Do hai hình chữ nhật này có phần chung là hình chữ nhật PIQF nên S FQRS = SQITH .
Theo nhận xét đã chứng minh, suy ra Q thuộc đường chéo PD của hình chữ nhật SPTD , tức đi
qua điểm D .
Bài 5.
1. Các đỉnh của ( H ) chia đường tròn ngoại tiếp nó thành 14 cung bằng nhau, mỗi cung có số
360
đo là α = . Các dây nối hai đỉnh của ( H ) chắn các 2.31. Đáp án tuyển sinh lớp 10 năm
14
2005 (cho thí sinh chuyên toán và chuyên tin)131
cung nhỏ có số đo là α , 2α ,3α , …, 7α do vậy độ dài các dây đó chỉ nhận 7 giá trị khác nhau.
Lấy 6 đỉnh của ( H ) thì số dây nối hai đỉnh trong 6 đỉnh đó là ( 6 × 5 ) : 2 = 15 . Vì 15 dây này có
các độ dài nhận không quá 7 giá trị khác nhau nên phải có ba dây cùng độ dài. Trong ba dây đó
luôn có hai dây không chung đầu mút (vì nếu hai dây bất kỳ trong ba dây đó đều chung đầu mút
thì ba dây bằng nhau đó tạo thành một tam giác đều, do đó số đỉnh của ( H ) chia hết cho 3, trái
với giả thiết). Dễ thấy hai dây bằng nhau của một đường tròn không chung đầu mút thì 4 đầu mút
của chúng là 4 đỉnh của một hình thang (cân). Từ đó suy ra trong 6 đỉnh bất kỳ của ( H ) luôn có
4 đỉnh là các đỉnh của một hình thang.
2. Phân tích 13860 thành nhân tử nguyên tố ta được 13860 = 2.2.3.3.5 .7.11 vì m ⋅ n =13860 nên
m phải là ước số của 13860 tức là tích của một số nhân tử trong 7 nhân tử trên, còn n là tích của
các nhân tử còn lại.
m
Nếu m có chứa nhân tử 2 (hoặc 3 ) thì nó phải chứa 22 (hoặc 32 ) vì ngược lại thì không tối
n
giản.
Do đó nếu ta ký hiệu=
a1 22=
, a2 32=
, a3 5,=
a4 7,=
a5 11 thì m là tích của một số nhân tử
trong số a1 , a2 , a3 , a4 , a5 , còn n là tích các nhân tử còn lại.
Vì vậy, chỉ có các trường hợp sau
1. Có 1 phân số có tử số là 1 (mẫu số là 13860 ).
2. Có 5 phân số có tử số là 1 trong 5 nhân tử a1 , a2 , a3 , a4 , a5 .
3. Có 10 phân số có tử số là tích của hai nhân tử trong số a1 , a2 , a3 , a4 , a5 .
4. Có 10 phân số có tử số là tích của ba nhân tử trong số a1 , a2 , a3 , a4 , a5 (mẫu là tích của
hai nhân tử).
5. Có 5 phân số có tử số là tích của 4 nhân tử.
13860
6. Có 1 phân số có tử số là tích của cả 5 nhân tử (tức là số ).
1

Liên hệ tài liệu word môn toán: 039.373.2038 TÀI LIỆU TOÁN HỌC
Website:tailieumontoan.com

m
Vậy số phân số tối giản thoả mãn m ⋅ n =13860 là 1 + 5 + 10 + 10 + 5 + 1 =32 . Các phân số
n
32
trên được chia thành từng cặp nghịch đảo nhau và khác 1 nên phân số lớn hơn 1 là = 16 .
2

ĐẠI HỌC QUỐC GIA HÀ NỘI ĐỀ THI TUYỂN SINH LỚP 10


TRƯỜNG ĐẠI HỌC KHOA HỌC TỰ NHIÊN HỆ THPT CHUYÊN NĂM 2004

MÔN : TOÁN (Vòng 1)


Thời gian làm bài: 150 phút (không kể thời gian phát đề)

Bài 1.
1) Cách 1: Phương trình tương đương với
( x + 1 − 1)( x − 1 − 1) =0
x = 0
• Giải x + 1 =1 ⇔ x + 1 =±1 ⇔ 
 x = −2
x = 0
• Giải x − 1 =1 ⇔ x − 1 =±1 ⇔  Đáp số: x = 0;  2
x= ±
x = 2
Cách 2:
x = 0
• Xét trường  2 x ta có =
hợp  1  x x2 ⇔  (loại)
x = 2
x = 0
• x − ta có  2
Xét trường hợp  1  − x = x2 ⇔ 
 x = −2
• Xét trường hợp
  − 1 < x < 1 ta có 2 = 2 − x 2 ⇔ x = 0
Đáp số: x = 0;  2
x= ±

Liên hệ tài liệu word môn toán: 039.373.2038 TÀI LIỆU TOÁN HỌC
Website:tailieumontoan.com

( x + 2 y + 2 )( x − y ) =
7
2) Hệ đã cho tương đương với  3
x + y + x − y =
3
8
Từ (1) , do x, y ∈  ta thu được các trường hợp sau:

 x + 2 y=
+2 7  x +=2y 5 y = 2
a)  ⇔ 6 ⇒
  ⇒ 3y =   (thoả mãn)
 x − y =−1  x − y =−1  x = 1
 x + 2 y + 2 =−7  x + 2 y =−9
b)  ⇔ ⇒ y=
 3 −10
= x − y 1 = x − y 1
10
⇒y=− ∉ ( loại )
3
 x + 2 y + 2 =1  x + 2 y =−1  y =2
c)  ⇔ ⇒ 3 y =6 ⇒ 
 x − y =−7  x − y =−7  x =−5
(loại vì không thoả mãn (2))
 x + 2 y + 2 =−1  x + 2 y =−3
d)  ⇔ ⇒ y=
 3 −10
= x − y 7 = x − y 7
10
⇒y= − ∉    (loại)
3
Đáp số:= x 1,= y 2
Bài 2. Ta có đẳng thức
a102 + b102= (a
101
) (
+ b101 ( a + b ) − ab a100 + b100 )
⇔  1 = a + b − ab
⇔   ( a − 1)( b − 1) =
0
 a = 1 ⇒ 1 + b100 = 1 + b101 ⇔ b = 1

b = 1 ⇒ 1 + a = 1 + a ⇔ a = 1
100 101

Thu được P = 2 .
Bài 3. Gọi H , I , P lần lượt là chân các đường vuông góc, phân giác, trung tuyến hạ từ B .
1
Vì 32 + 42 =
52 suy ra tam giác vuông tại B và S = ⋅3⋅ 4 = 6
2
1
Vì AP =CP ⇒ SCBP = S =3 (  cm )
2
S AI 3
Vì ABI = =⇒ 4 S ABI = 3S BCI
S BCI IC 4
Ta có:

Liên hệ tài liệu word môn toán: 039.373.2038 TÀI LIỆU TOÁN HỌC
Website:tailieumontoan.com

3 7
6 = S ABI + S BCI = S ABI = S BCI + S BCI = SCBI
4 4
⇒   cm
SCBI =
24
7
2
( )
⇒ S PBI = SCBI − SCPB =
24
7
− 3=
3
7
 cm 2 ( )
2
S ABH  3  54
⇒ =   ⋅= 6
S 5 25
 54  24 72
S HBI = SCBH − SCIB = 6 −  −
 25  7
=
175
 cm 2 ( )
Đáp số:

S ABH =
54
25
(
 cm 2 )
S HBI =
72
175
(
 cm 2 )
3
( )
S IBP =  cm 2
7
S PCB = 3 (  cm )2

Bài 4. Tứ giác BMHN là tứ giác nội tiếp suy ra



= CBH
CHN 
=  AHQ
Vì ABCN là tứ giác nội tiếp
 = CAD
CBD 
= 
Từ (1) và (2) suy ra: CAD AHQ → AQ = HQ

Mặt khác
1
HQ = AD → AQ = QD
2
Tương tự suy ra:
CP = DP
Từ (3) và (4) suy ra: QP / / AC
 = QHA
Góc PQN  nên tứ giác PQMN là tứ giác nội tiếp suy ra bốn điểm P, Q, M , N cùng nằm trên
một đường tròn.
Bài 5. Ta có
1  x10 y10 
+ 2  2 x y
2 2

2  y2 x 

(
1 16
4
)
x + y16 x 4 y 4

Liên hệ tài liệu word môn toán: 039.373.2038 TÀI LIỆU TOÁN HỌC
Website:tailieumontoan.com

Suy ra:
1  x10 y10  1 16
( 5
) ( )
2
+ 2  + x + y +  1 + 2x y
16 2 2

2  y2 x  4 2
5
(
⇒ Q ≥ = Qmin    Khi  x 2 = y 2 = 1
2
)

ĐẠI HỌC QUỐC GIA HÀ NỘI ĐỀ THI TUYỂN SINH LỚP 10


TRƯỜNG ĐẠI HỌC KHOA HỌC TỰ NHIÊN HỆ THPT CHUYÊN NĂM 2004

MÔN : TOÁN (Vòng 2)


Thời gian làm bài: 150 phút (không kể thời gian phát đề)

Bài 1. x + 3 + x −1 =2
Điều kiện: x ≥ 1
Cách 1: Dễ thấy x = 1 là một nghiệm của (1). Với x > 1 ta có
x + 3 + x −1 > 1+ 3 + 1−1 =2
 
Vậy x = 1 là nghiệm duy nhất

Cách 2: Đặt u = x + 3,  1
v = x − ta có u > 0, v ≥ 0 và

u=+v 2 u=
+v 2 =
u 2
 2 ⇔ ⇔ ⇔x=
1
u = −v u=
−v 2 =
2
4 v 0
Cách 3: Với điều kiện x ≥ 1 thì (1) tương đương với
2x + 2 + 2 x2 + 2 x − 3 =4
x 2 + 2 x − 3 =1 − x
Do vế trái không âm còn vế phải không dương ( x ≥ 1) nên (2) suy ra

 x 2 + 2 x − 3 =0
 ⇒x=
 1
1 − x = 0

Liên hệ tài liệu word môn toán: 039.373.2038 TÀI LIỆU TOÁN HỌC
Website:tailieumontoan.com

(
( x + y ) x 2 + y 2 =
 15 )
Bài 2. 
(
( x − y ) x − y =
2 2
3 )
Cách 1. Hệ tương đương với

( )
( x + y ) x 2 + y 2 = 15 ( )
( x + y ) x 2 + y 2 = 5 ( x + y ) ( x − y ) 2
⇔
 x 2 + y 2 = 5( x − y ) 2 ( 3)
⇔

 ( x =
+ y ) ( x − y)2 3 ( x=+ y ) ( x − y)2 3  ( x + y ) ( x − y ) = 3 ( 4)
2

Chú ý: x + y ≠ 0
 y = 2x
( 3) ⇔ 2 x 2 − 5 xy + 2 y 2 =0 ⇔ ( y − 2 x )( 2 y − x ) =0 ⇔ 
x = 2y

Thay y = 2 x vào (4) ta được 3 x3 =3 ⇒ x =1 ⇒ y =2


Thay x = 2 y vào (4) ta được 3 y 3 =3 ⇒ y =1 ⇒ x =2
Vậy hệ có hai nghiệm là=
x 1,=
y 2 và=
x 2,=
y 1
Cách 2: Hệ (1), (2) tương đương với
 (
( x + y ) 2 x + 2 y =
2 2
30 )
 2
(
( x + y ) x − 2 xy + y =
2
3 )
Trừ (3) cho (4) ta được hệ tương đương với


(
( x + y ) 2 x + 2 y =
2 2
30

)2 x 2 + 2 y 2 =

10
 ( x + y )3 =
27  x+ y = 3

2 x + 2(3=
− x) 2 10  x 2 −=
3x + 2 0
2
⇔  ⇔
 y= 3− x  y= 3− x
=
 x 1,=
y 2
⇔ 
=
 x 2,=y 1
Bài 3. Cách 1.

P=
(x 3
) (
+ y3 − x2 + y 2 ) = x ( x − 1) + y ( y − 1)
2 2

 =
x2
+
y2

( x − 1)( y − 1) ( x − 1)( y − 1) y −1 x −1
xy

 2
y −1 x −1
x2 y2
Dấu "=" đạt được khi và chỉ khi = .
y −1 x −1

Liên hệ tài liệu word môn toán: 039.373.2038 TÀI LIỆU TOÁN HỌC
Website:tailieumontoan.com

( x − 1) + 1 x
Ta có x − 1= ( x − 1) .1 ≤ = , dấu "=" đạt được khi x − 1 =1 hay x = 2 . Tương tự
2 2
y
y −1 ≤ , dấu "=" đạt được khi và chỉ khi y = 2 . Do đó
2
2 xy
P≥ =
8
x y

2 2
Dấu "=" đạt được khi và chỉ khi
 x2 y2
 =
⇒x= y=
 y − 1 x − 1  2
 x 2,=
=
 y 2
Vậy P đạt giá trị nhỏ nhất bằng 8 đạt được khi và chỉ khi x= y= 2
Cách 2. Theo trên ta có
x2 y2
=P +
y −1 x −1
Đặt x − 1= a, y − 1= b thì a, b > 0 và x =a + 1, y =b + 1 , do đó
(a + 1) 2 (b + 1) 2 4a 4b a b
P= + ≥ + = 4 +  ≥ 8
b a b a b a
Do đó P đạt giá trị nhỏ nhất bằng 8 , đạt được khi và chỉ khi a= b= 1 hay x= y= 2
Bài 4. 1) Nếu M là tâm O của hình vuông thì hiển nhiên M thoả mãn giả thiết. Ta chứng minh
điều ngược lại.
 
= MCB
Cách 1: Từ giả thiết ta có MBA = MDC = MAD  do đó
=
MAB = = MDA suy ra MA
MBC MCD = MB = MC = MD và
 
= BMC
AMB = CMD
= DMA
= 90 hay M là tâm của hình vuông.
Cách 2: Giả sử M ≠ 0 , khi đó M thuộc một trong bốn miền tam giác MAB, MBC , MCD, MDA .
Không mất tổng quát, giả sử M thuộc miền tam giác OAD . Do M ≠ 0 nên một trong hai góc
 , MDA
MAD  bé hơn 45 .

 < 45 thì MAB


Nếu MAD  > 45 ≥ MDA
 , trái với giả thiết.
 < 45 thì MAB
Nếu MDA  ≥ 45 > MDA
 , trái với giả thiết. Suy ra nếu M ≠ 0 thì M không thoả
mãn giả thiết bài toán.
AN AC
2) Do  ANM và  ABC vuông cân nên = = 2 suy ra  AOB ∼ ANC (góc A
AO AB
OB AO 1
chung) suy ra = = không đổi.
CN AN 2
Liên hệ tài liệu word môn toán: 039.373.2038 TÀI LIỆU TOÁN HỌC
Website:tailieumontoan.com

3) Do ( S 2 ) đi qua tâm O của ( S1 ) và đi qua điểm C nằm ngoài ( S1 ) nên ( S 2 ) và ( S1 )


cắt nhau nên các tiếp tuyên là tiếp tuyến chung ngoài. Giả sử hai tiếp tuyến chung đó là
PP ′ và QQ ′ ( P ′, Q ′ ∈ ( S1 ) ) . Tia OO ′ cắt ( S1 ) tại T . Gọi tâm của ( S 2 ) là O ′ ta có
O ′P / / OP ′ (cùng vuông góc với PP ′ ) nên
′ = OPO
POP ′
Do NO ⊥ OC nên O nằm trên đường tròn ( S 2 ) suy ra O ′PO cân ở O ′ nên ta có
 ′ ≡ PP
=′ POO
OPO  ′ = POT
′O và từ (1) ta có POP  từ đó ta có  POP
=′  POT ( c − g − c ) suy
 = PP
ra PTO   = 90 hay P, T , Q thẳng hàng và PQ là tiếp tuyến
′O = 90 . Tương tự ta có QTO
của ( S1 ) tại T .
n
Bài 5. Do 2 là số vô tỷ nên với mọi số nguyên dương n thì là số vô tỷ.
2
Ta có
 n + 1  n  n + 1  n  n + 1  n  1
xn = − < − < − − 1 = + 1 < 2 ⇒ 0 ≤ xn ≤ 1
 2   2 2  2 2  2  2
Vì vậy x0 , x1 , x2 , …, x199 chỉ nhận một trong hai giá trị 0 hoặc 1. Suy ra các số khác 0 trong chúng
chỉ nhận giá trị 1. Do đó số các số khác 0 bằng
 1   0   2   1   200  199 
x0 + x1 + … + x199  =  − + −  + +  − 
 2  2  2  2  2  2
 200  
= 100
=   2 
 2
Vì 141 < 100 2 < 142 nên [100 2 = 144 .
Vậy trong hai trăm số x0 , x1 , …, x199 đã cho có đúng 141 số khác 0 .
n +1 n 1 n n +1
Chú ý: Có thể thấy rằng − = < 1 nên giữa hai số và có không quá một số
2 2 2 2 2
 n + 1  n 
nguyên, do vậy 0 ≤ =
xn  −  ≤1
 2   2

Liên hệ tài liệu word môn toán: 039.373.2038 TÀI LIỆU TOÁN HỌC
Website:tailieumontoan.com

ĐẠI HỌC QUỐC GIA HÀ NỘI ĐỀ THI TUYỂN SINH LỚP 10


TRƯỜNG ĐẠI HỌC KHOA HỌC TỰ NHIÊN HỆ THPT CHUYÊN NĂM 2003

MÔN : TOÁN (Vòng 1)


Thời gian làm bài: 150 phút (không kể thời gian phát đề)

Bài 1. Điều kiện để phương trình có nghĩa: x − 2


[Chú ý rằng: x 2 + 7 x + 10 = ( x + 5 )( x + 2 ) 
Cách 1:
Nhân hai vế của phương trình với ( x + 5 + x + 2 > 0) ta được phương trình tương đương

1 + x 2 + 7 x + 10 = x+5 + x+2
 x+5 =  x =−4
( )( )
1
⇔   x + 5 −1 x + 2 −1 = 0 ⇔  ⇔   (loai) 
 x + 2 =
1  x = −1
Đáp số: x = −1
Cách 2:
Đặt ( ⇒ u v0, u 2 − v 2 = 3 và u , v thoả mãn hệ phương trình
x + 5 = u , x + 2 = v  0,

 u 2 − v 2 3
= = u 2 − v2 3
 ⇔ 
( u − v )(1 + uv ) =3 ( u − v )(1 + uv ) =u − v
2 2

  ⇔ ( u − v )(1 + uv − u − v ) =0
u = v
⇔ ( u − v )( u − 1)( v − 1) = 0 1
  
⇔ u =
 v = 1
Liên hệ tài liệu word môn toán: 039.373.2038 TÀI LIỆU TOÁN HỌC
Website:tailieumontoan.com

Với u = 1 tức x + 5 =⇔
1 x =−4 (loại).
Với v = 1 tức x + 2 =⇔
1 x=−1
Đáp số: x = −1
Bài 2.
Cách 1:
2 x3 + 3=x2 y 5 8 x 3 + 12=
x 2 y 20 (2 x +=
y )3 27
 3 ⇔  ⇔ 
 y + 6 xy
=  y + 6 xy=  y + 6 xy
=
2 3 2 3 2
7 7 7
   1
2 x + y = 3 2 x= 3 − y
⇔ 3
=
⇔
x (3 − y )
 3 2
 y + 6 xy =  y + 3(3 − y ) y =
2 2
7 7
2 y − 9 y + 7 =
3 2
  0
Giải (2) ta được
y =1 ⇒ x =1 và
7 ± 105 5  105
=y =⇒x
4 8
Cách 2: Nhân hai vế của phương trình thứ nhất với 7 và hai vế của phương trình thứ hai với 5 rồi
trừ cho nhau, ta được hệ phương trình tương đương
2 x3 + 3x 2 y =
5
 3
14 x − 5 y + 21x y − 30 xy =
3 2 2
0
Trong (2) đặt y = tx (Chú ý rằng hệ không có nghiệm với x = 0 ), ta được phương trình:

( )
⇔ ( t − ) 5t 2 + 35t + 14 =0
5t 3 + 30t 2 − 21t − 14 =0  1
Giải ra ta được
−35 ± 3 105
= =
t 1  và  t = 0 t1,2
1
Với t = 1 tức y = x , thay vào (1) ta được nghiệm x= y= 1

(
Với t = t1 tức y = t1 x , thay vào (1) ta được phương trình i = 1, 2 )
5 5 50
( 2 + 3ti ) x3 = 5 ⇔ x3 = ⇔x=3 =3 = xi
2 + 3ti 2 + 3ti −85 ± 9 105
Từ đó suy ra ti , xi
Chú ý: Đến khi thu được giá trị t1,2 ta có thể viết ngắn gọn

5 5
=xi =
3 yi ti=
,  3 , 
i 1, 2
2 + 3ti 2 + 3ti

Liên hệ tài liệu word môn toán: 039.373.2038 TÀI LIỆU TOÁN HỌC
Website:tailieumontoan.com

hoặc có thể tính toán và rút gọn tiếp tục


Bài 3.
2 y 2 x + x + y + 1 = x 2 + 2 y 2 + xy
⇔   2 y 2 ( x − 1) − x ( x − 1) − y ( x − 1) + 1 =
0
Do phương trình không có nghiệm với x = 1 nên (1) tương đương với
1
2 y2 − x − y + =0
x −1
Phương trình có nghiệm x, y nguyên
1 x = 2
⇒   1 ∈  ⇒ x − ∈ {1, −1} ⇒  
x = 0
x −1 
Thay x = 2 vào phương trình ( 2 ) :

y = 1
2
2y − y − 1 = 0 ⇒ 
 y = − 1   loaïi 
 2
Thay x = 0 vào phương trình ( 2 ) :

y = 1
2 y − y − 1 = 0 ⇒  
2
 y = − 1   loaïi 
 2
Vậy phương trình có hai nghiệm nguyên ( x, y ) = {( 2,1) ; ( 0,1)}
Bài 4.
1 Hạ AA′ và BB ′ vuông góc với MN . Gọi H là trung điểm của MN suy ra OH ⊥ MN .
Trong hình thang AA′B ′B ta có:
1 R 3 R
OH = ( AA′ + BB ′ ) = ⇒ MH = ⇒ MN = R   và OMN dều.
2 2 2
2) 
AMB = ANB =900 ⇒ KMI  =KNI
 =900 ⇒ M , N , I , K cùng nằm trên một đường tròn đường
kính IK .
1
Do = 90 , KAN
ANK 
= MAN = = 30 nên 
MON AKN = 60 .
2
Gọi trung điểm IK là O ′
 = MN R
⇒ MO ′N ==
2 MON 1200 ⇒ MN MO
= ′ 3 ⇒ MO ′ =
3 3
R
Vậy bán kính đường tròn qua M , N , I , K là .
3
Liên hệ tài liệu word môn toán: 039.373.2038 TÀI LIỆU TOÁN HỌC
Website:tailieumontoan.com

3 Do I là trực tâm  ABK nên KI ⊥ AB (giả sử tại P ).


Do O, O ′ nằm trên đường trung trực của MN nên O, H , O ′ thẳng hàng.

MOO ′  có  OMO 0 
′ = 90=
(
MOO ′ 30
=  
, MO ′O 60 )
2R
⇒ OO ′ = 2 MO ′ =
3
 KAB có AB không đổi nên nó có diện tích lớn nhất khi và chỉ khi KP lớn nhất.
R 2R
Do KPKO= ′ + OO ′ = + 3R
3 3
Dấu = đạt được khi P ≡ O ⇔ KAB cân ở K ⇔ KAB đều (vì 
AKB = 600 nên diện tích tam )
AB 2 3
giác KAB đạt max bằng = R 2 3 đạt được khi và chỉ khi  KAB đều.
4
Chú ý: Chúng ta có thể chứng minh tính chất: Trong các tam giác KAB có AB cố định và

AKB = 60 , tam giác cân ở K (tức tam giác đều) có diện tích lớn nhất. Từ đó suy ra S đạt KAB
2
max bằng R 3
Bài 5.
Cách 1: Ta có
x 2 + 12 x=
,  daáu   đñaït  =
⇔x 1
2
y + 12 y= ,  daáu   đñaït  =
⇔y 1
2
z + 12 z= ,  daáu   đñaït  =
⇔z 1
( )
2 x 2 + y 2 + z2 2 ( xy + yz + zx ) ,  daáu =  đñaït  ⇔ x = y = z.

2.27. Đáp án tuyển sinh lớp 10 năm 2003(cho thí sinh chuyên toán và chuyên tin)115
Cộng lại ta được
( )
3 x 2 + y 2 + z 2 + 32 ( x + y + z + xy + yz + zx ) ,   dấu = đạt  1
⇔x=y=z=
Mà theo giả thiết thì x + y + z + xy + yz + zx =
6 , dẳng thức này thoả mãn khi x= y= z= 1 nên
ta có
( )
3 x 2 + y 2 + z 2 + 32.6  hay  3,
x 2 + y 2 + z 2  
dấu " =" ⇔ x = y = z =1
Cách 2: Ta có
( )
3 x 2 + y 2 + z 2 + 3( x + y + z ) 2

⇒  3 x 2 + y 2 + z 2 x + y + z
Ngoài ra: x 2 + y 2 + z 2xy + yz + zx . Cộng lại ta được

Liên hệ tài liệu word môn toán: 039.373.2038 TÀI LIỆU TOÁN HỌC
Website:tailieumontoan.com

3 x 2 + y 2 + z 2 + x 2 + y 2 + z 26

Đặt t ta được t0 và t 2 + 3t6 ⇔ t + 2 3 (t −


x2 + y 2 + z 2 = ( ) 3)0

t 2 + 3t6 ⇔ t + 2 ( 3 )( t − 3 ) 0

⇔ t= 3 ⇔ x 2 + y 2 + z 23

ĐẠI HỌC QUỐC GIA HÀ NỘI ĐỀ THI TUYỂN SINH LỚP 10


TRƯỜNG ĐẠI HỌC KHOA HỌC TỰ NHIÊN HỆ THPT CHUYÊN NĂM 2003

MÔN : TOÁN (Vòng 2)


Thời gian làm bài: 150 phút (không kể thời gian phát đề)

Bài 1.
x 4 + 2mx 2 + 4 =0
Đặt x 2 = t ta được phương trình
t 2 + 2mt + 4 =0
Phương trình (1) có 4 nghiệm phân biệt ⇔ phương trình (2) có 2 nghệm dương
′= m 2 − 4 > 0  m > 2
 
t1 ≠ t2 ⇔ t1 + t2 = −2m > 0 ⇔   m < −2 ⇔ m < −2
t ⋅ t = 4 > 0 m < 0
1 2 
Khi đó (1) có 4 nghiệm là x1,2 =
± t1 ; x3,4 =
± t2 và

( )
x14 + x24 + x34 + x44 = 2 t12 + t22 = 2 ( t1 + t2 ) − 4t1t2 = 8m 2 − 16
2

Do đó x14 + x24 + x34 + x44 = 32 ⇔ 8m 2 − 16 = 32 ⇔ m 2 = 6 ⇔ m = − 6 (vì m < −2 )

Vậy: m = − 6
Bài 2.
2 x 2 + xy − y 2 − 5 x + y + 2 =
0
 2
x + y + x + y − 4 =
2
0
Cách 1:
(1) ⇔ y 2 − ( x + 1) y − 2 x 2 + 5 x − 2 =0
x + 1 ± ( x + 1) 2 − 4 −2 x 2 + 5 x − 2 2 x − 1 ( )
=⇔y = 
2 − x + 2

Liên hệ tài liệu word môn toán: 039.373.2038 TÀI LIỆU TOÁN HỌC
Website:tailieumontoan.com

Vậy hệ đã cho tương đương với


 = y 2x − 1
 2  4 13
  x + y + x + y − 4 = x = − ,y =

2
0
 ⇔ 5 5
 y =− x + 2 

 2  x= y= 1
  x + y + x + y − 4 =
2
0
Cách 2:
(1) ⇔ y 2 − ( x + 1) y − 2 x 2 + 5 x − 2 =
  0
⇔ y − ( x + 1) y + ( x − 2 )(1 − 2 x ) =
  2
0
⇔ ( y + x − 2 )( y − 2 x + 1) =
  0
Vậy hệ tương đương với
 y + x − 2 = 0
 2  x= y= 1
  x + y + x + y − 4 =
2
0 
 ⇔ 4 13
 y − 2 x + 1 =0 x = − ,y =

 2  5 5
  x + y + x + y − 4 =
2
0
Bài 3.
Cách 1: Ta tìm nghiệm thoả mãn x  y
Khi đó
xy xy y 2   và x=
2 2
y x 2 xy + y 23 y 2
Dễ thấy, nếu phương trình có nghiệm với x = 0 thì y = 0 và ngược lại.
Với y ≠ 0 thì từ phương trình (1) ⇒ x 23 ⇒ x ∈ {−1,1}
Với x = 1 thay vào phương trình đã cho ta được y = −1 , còn
Với x = −1 ta được y = −1
Do vai trò của x, y trong phương trình đã cho đối xứng nên trường hợp x  y ta cũng chỉ thu
được ba nghiệm trên.
Vậy phương trình có ba nghiệm nguyên là:
x= 0, y = x =y =
0;  1, −1;  
x=−1, y =
1
Cách 2: Ta chứng minh phương trình không có nghiệm với x 2, y  2.
Thật vậy, với x 2 và y 2 ta có

 x 2 y 24 x 2
 2 2 ⇒ x2 y 2  2 ( )
 x2 + y 2 = x2 + y 2 + x2 + y 2 ≥
 x y 4 y
2

x 2 + y 2 + xy > x 2 + y 2 + xy
 2

Liên hệ tài liệu word môn toán: 039.373.2038 TÀI LIỆU TOÁN HỌC
Website:tailieumontoan.com

• Trường hợp x ± 2 hoặc y = ±2 phương trình không có nghiệm nguyên


• Thử với các trường hợp= x 1 và x = −1 ta được ba nghiệm
x 0,=
x=
0, y = x =y =
0;  1, −1;  
x=−1, y =
1
= p . Theo tính chất của đường tròn
= AN
Bài 4. 1) Gọi nửa chu vi tam giác ABC là p thì AM
nội tiếp dễ thấy rằng CD= p − AB .
Ngoài ra, BP= BM= p − AB . Vậy BP = CD .
Chú ý: Có thể chứng minh như sau: Giả sử P thuộc đoạn BD . Khi đó:
FM = FB + BM = BD + BP = 2 BP + PD
Tương tự
EN = EC + CN = CD + CP = 2CD + PD
mà: FM = EN nên BP = CD .
Trường hợp D thuộc đoạn BP , chứng minh tương tự.
= 
2  BMI có BMI AMN =   ⇒ BI = BM = BP = CD = CE
ANM = BIM
Do hai đoạn BI , CE song song, bằng nhau và ngược chiều nên BICE là hình bình hành.
 =
Tương tự: CNK có CNK  ⇒ CK = CN = CP = BD = BF .
AMN = CKN
Do hai đoạn CK , BF song song, bằng nhau và ngược chiều nên BKCF là hình bình hành. 3)
Cách 1: Kẻ đường phân giác góc PBI cắt PO ′ tại S thì  BPS =  BIS
 = BPS
⇒ BIS  = 900   và  SI = SP
Xét  BPS và CDO , ta có
 1=  1  
=
OCD ACB = CBI OP 
2 2

BP= CD  ⇒ BPS= CDO ⇒ OD= SP
  
= BPS
CDO = 90 

Từ đó suy ra: CPS = BDO( chú ý: CP = BD)

= 1
 = DBO
 ⇒ PCS
1
ABC = BCK
2 2
⇒ SK = SP
   và    = SPC
SKC  = 900

Từ (1) và (2) suy ra, đường tròn tâm S , bán kính SI đi qua I , K , P và tiếp xúc với BI , CK và
BC tại I , K , P tương ứng.
Cách 2: Gọi H là trung điểm của BC và S là điểm đối xứng với O qua H . Dễ thấy các cặp
điểm sau đây đối xứng với nhau qua H : B và C ; P và D; I và E ; F và K .
Do đó: SI = SK (vì chúng lần lượt bằng OE
= SP = =
, SK OS , SP OD)

Liên hệ tài liệu word môn toán: 039.373.2038 TÀI LIỆU TOÁN HỌC
Website:tailieumontoan.com

 CEO
=
BIS  
= 90 ,   BPS  
= 90 ,  , CKS
= CDO 
= BFO
= 900
Suy ra đường tròn tâm S bán kính SP đi qua P, I , K và tiếp xúc với BC , BI , CK (lần lượt tại
P, I , K ).
Chú ý: Chúng ta có thể lý luận như sau:
Gọi giao điểm của BI và CK là T thì ABTC là hình bình hành và trung điểm H của BC là
giao điểm của AT và BC . Qua phép đối xứng tâm H tam giác ABC biến thành TCB , các
điểm D, E , F biến thành P, I , K tương ứng suy ra đường tròn qua D, E , F biến thành đường
tròn đi qua P, I , K .
Do đường tròn qua D, E , F nội tiếp  ABC nên đường tròn qua P, I , K nội tiếp TCB , tức tiếp
xúc với BC , BI , CK tại P, I , K tương ứng.
Bài 5. Đặt y= 3 − x , bài toán đã cho trở thành:
Tìm giá trị nhỏ nhất của biểu thức P = x 4 + y 4 + 6 x 2 y 2 , trong đó x, y là các số thực thay đổi và
thoả mãn hệ thức:
x + y = 3
 2
 x + y 5
2

Từ các hệ thức trên ta có:


 x 2 + y 2 + 2 xy =
9

 x + y 5
2 2

( ) (
⇒ x 2 + y 2 + 4 x 2 + y 2 + 2 xy 5 + 4.9 =
   41 )
(
⇒ x2 + y 2
  5 ) + 4 ( 2 xy )41
Ta có

( ) ( ) ( 2 xy )
2
16 x 2 + y 2 + 25(2 xy ) 2 40 x 2 + y 2

( 5 ( 2 xy )
Dấu = đạt được ⇔ 4 x 2 + y 2 = )
( )
2
Cộng hai vế bất đẳng thức thu được với 25 x 2 + y 2 + 16(2 xy ) 2 ta thu được:

(
41  x 2 + y 2 ) ( )
+ (2 xy ) 2   5 x 2 + y 2 + 4 ( 2 xy )  41
2 2

 

(x )
2
⇔ 2
+ y2 + (2 xy ) 2 41 hay x 4 + y 4 + 6 x 2 y 2412
Dấu = đạt được khi và chỉ khi

Liên hệ tài liệu word môn toán: 039.373.2038 TÀI LIỆU TOÁN HỌC
Website:tailieumontoan.com

x + y = 3
 2 =
 x 1,=
y 2
 x= +y ⇔
2
5
 =
 x 2,=y 1
 (
4 x 2
+ )
y 2
=
5 ( 2 xy )
Vậy P đạt giá trị nhỏ nhất bằng 41 , đạt được khi và chỉ khi x = 1 hoặc x = 2

ĐẠI HỌC QUỐC GIA HÀ NỘI ĐỀ THI TUYỂN SINH LỚP 10


TRƯỜNG ĐẠI HỌC KHOA HỌC TỰ NHIÊN HỆ THPT CHUYÊN NĂM 2002

MÔN : TOÁN (Vòng 1)


Thời gian làm bài: 150 phút (không kể thời gian phát đề)

Bài 1.
1 Điều kiện: 0 x5 . Phương trình đã cho tương đương với

8 + t + 5=
− t 5  =
x t( )
⇔ 13 + 2 ( 8 + t )( 5 − t ) =
25

⇔ 40 − 3t − t 2 =
6
2
⇔ t + 3t − 4 = 0
t1 =1 ⇒ x =1
t2 = −4  loaïi 
Vậy phương trình có một nghiệm x = 1 Cách khác

Đặt   8 + x = u , 5 − x = v
u + v = 5
⇔ 2 2
 ⇒   Ñaùp soá. 
u + v = 13
2 Hệ đã cho tương đương với
= x + y + xy 7 = x + y + xy 7
 2 ⇔
x + y + x + =y + xy 17 ( x + y ) − xy +=x + y 17
2 2

 S+P= 7
 ⇔  2
S + S − P = 17

Liên hệ tài liệu word môn toán: 039.373.2038 TÀI LIỆU TOÁN HỌC
Website:tailieumontoan.com

với x + = = P.
y S , xy
Từ (1) và ( 2 )
⇒ S 2 + 2 S − 24 =
   0
⇒ S1 =
   4, S 2 =
−6
=  S 4= x 1 = x 3
Với  ⇒   và  
=  P 3=y 3 = y 1
 S = −6
Với    hệ này vô nghiệm
 P = 13
(1;3)
Vậy hệ đã cho có hai nghiệm là ( x, y ) = 
( 3;1)
Bài 2 .
∆= (a + b + c) 2 − 4 ( ab + bc + ca )
∆= a 2 + b 2 + c 2 − 2ab − 2bc − 2ca
a + b > c c ( a + b ) > c 2
 
b + c > a ⇔ a ( b + c ) > a ⇒ 2 ( ab + bc + ca ) > a + b + c ⇒< 0
2 2 2 2
Tù̀  
 b ( c + a ) > b 2
c + a > b 
Cách khác:
∆ = a 2 + b 2 + c 2 − ab − ab − bc − bc − ca − ca
∆ = a ( a − b − c ) + b (b − c − a ) + c ( c − a − b ) < 0

Bài 3. Giả sử n 2 + 2002 là số chính phương


⇒ n 2 + 2002 =
   (m )
m 2   nguyên 
⇒ m −n =
 2002 2 2

⇒ ( m − n )( m + n ) =
 2002
Chú ý rằng m và n phải cùng chẵn hoặc cùng lẻ
⇒  ( m − n ) 2  và ( m + n ) 2

Suy ra đẳng thức (*) không thể xảy ra vì vế trái chia hết cho 4 nhưng vế phải không chia hết cho 4.

Vậy không có số nguyên n nào để n 2 + 2002 là số chính phương.


Bài 4.

( a + b + c )  + +   , gấu "=" đạt được khi


1 1 1
Trước hết chú ý rằng với a, b, c > 0 ta luôn có:  9
a b c
và chỉ khi a= b= c .

Liên hệ tài liệu word môn toán: 039.373.2038 TÀI LIỆU TOÁN HỌC
Website:tailieumontoan.com

a b a c  b c
 +  +  +  +  +   hiển nhiên đúng do
Thật vậy bất đẳng thức tương đương với  6
b a c a c b
x y
+ 2 với x, y > 0 , dấu "=" đạt được khi x = y .
y x
Áp dụng với 1 + xy = a, 1 
+ yz = b, 1 
+ zx = c
Ta có: ( 3 + xy + yz + zx ) .P9 . Dấu "=" đạt được khi và chỉ khi 1 + xy =+
1 yz =+
1 zx hay
x= y= z
9 1 9 3
⇒ P   =
3 + xy + yz + zx 3 + x + y + z
2 2 2
3+3 2
3
Vậy Pmin = đạt được khi và chỉ khi x= y= z= 1 .
2
 = 450
Bài 5. Từ giả thiết ta suy ra MAN
1 
Tứ giác ADNQ có QDN 
= QAN
= 450 . Suy ra tứ giác ADNQ là tứ giác nội tiếp.
Trong tứ giác nội tiếp ADNQ có   =90 suy ra điểm Q nhìn đoạn MN dưới
AND =90 ⇒ NQA
một góc vuông.
=
Hoàn toàn tương tự, tứ giác ABMP nội tiếp ⇒ MPA 900 suy ra điểm P nhìn đoạn MN dưới
một góc vuông. Vậy năm điểm P, Q, M , C , N nằm trên đường tròn đường kính MN .
2 Trên cạnh CB kéo dài về phía B lấy điểm K sao cho BK = DN .
=
Dễ thấy góc MAK 45 ⇒ AMN =  AMK do cạnh AM chung, AN = AK và
= MAK
MAN = 45 , suy ra đường cao AH của  AMN phải bằng đường cao AB của  AMK .
Như vậy, khoảng cách từ A đến MN bằng độ dài đoạn AB nên đường tròn tâm A bán kính
AB tiếp xúc với MN .
3 Do  AQN và  APM vuông tại Q và P nên:
AQ AP 1
= =
AN AM 2
S APQ
AQ ⋅ AP 1 1 1
⇒ = = ⋅ =
S AMN AN ⋅ AM 2 2 2
1
⇒ S APQ = ⋅S
2  AMN
S
⇒ S APQ= SMNPQ ⇒ 1= 1  khoâng ñoåi. 
S2

Liên hệ tài liệu word môn toán: 039.373.2038 TÀI LIỆU TOÁN HỌC
Website:tailieumontoan.com

ĐẠI HỌC QUỐC GIA HÀ NỘI ĐỀ THI TUYỂN SINH LỚP 10


TRƯỜNG ĐẠI HỌC KHOA HỌC TỰ NHIÊN HỆ THPT CHUYÊN NĂM 2002

MÔN : TOÁN (Vòng 2)


Thời gian làm bài: 150 phút (không kể thời gian phát đề)

Bài 1. 1) Phương trình đã cho tương đương với

( x − 1)( x − 2 ) + x+3 = x−2 + ( x − 1)( x + 3)


⇔ x−2 ( )
x −1 −1 − x + 3 ( )
x −1 −1 =0

⇔ (
  x −1 −1 )( x−2 − x + 3) =0

a) x −1 −1 = 0 ⇔ x −1 = 1 ⇔ x = 2
 b)  x−2 − x+3 = 0 ⇔ x−2 = x + 3  voâ nghieäm. 
Vậy phương trình chỉ có một nghiệm là x = 2
2) Phương trình đã cho có dạng: ( x + 1)( y + 1) =
10
Vậy có hai khả năng sau đây:
x + 1 =1 x + 1 =10
a)    hoặc  ⇒ có hai nghiệm ( 0,9 ) , ( 9, 0 )
  
y +1 = 10 y +1 = 1
 x + 1 =−1  x + 1 =−10
b)    hoặc 
    ⇒ có hai nghiệm ( −2, −11) , ( −11, −2 )
 y + 1 =−10  y + 1 =−1
Tương tự cho các trường hợp:
 x + 1 =2  x + 1 =5  x + 1 =−2  x + 1 =−5
 
;  
;  

 y + 1 =5 
 y + 1 =2  y + 1 =−5  y + 1 =−2
⇒ có thêm bốn nghiệm nữa là: (1, 4 ) , ( 4,1) , ( −3, −6 ) , ( −6, −3) 2.25.

Liên hệ tài liệu word môn toán: 039.373.2038 TÀI LIỆU TOÁN HỌC
Website:tailieumontoan.com

1 nên phương trình thứ hai của hệ có dạng:


Bài 2. Do x 2 + y 2 + xy =

(
x3 + y 3 = ( x + 3 y ) x 2 + y 2 + xy )
⇔ x3 + y 3 = x3 + xy 2 + x 2 y + 3 x 2 y + 3 y 3 + 3 xy 2
⇔ 2 y 3 + 4 xy 2 + 4 x 2 y = 0
⇔ ( )
2 y y 2 + 2 xy + 2 x 2 =
0

 x = 1  x = −1
a) y =0 ⇒ x =±1 ⇒ hệ có hai nghiệm   
= y 0= y 0
b) y 2 + 2 xy + 2 x 2 =0 ⇔ ( x + y ) 2 + x 2 =0
Dấu = xẩy ra khi và chỉ khi x= y= 0 nên trong trường hợp này hệ vô nghiệm.
Vậy hệ dã cho có hai nghiệm là=
x 1,=
y 0 và x =
−1, y =
0
Bài 3. Mười số đã cho viết thành một hàng là: a1 , a2 , …, a10
Xét mười tổng: a1 + 1, a2 + 2, … a10 + 10 Khi đó:
S = ( a1 + 1) + ( a2 + 2 ) +  + ( a10 + 10 )
S = (1 + 2 +  + 10 ) + (1 + 2 +  + 10 ) = 110
Chú ý rằng tổng S có tận cùng bằng 0 .
Giả sử rằng 10 tổng trên không có hai tổng nào có tận cùng giống nhau. Khi đó chữ số tận cùng
của chúng là: 0,1, 2,3, 4,5, 6, 7,8,9 .
Do 0 + 1 + 2 +  + 9 =45 nên chữ số tận cùng của tổng S = ( a1 + 1) +  + ( a10 + 10 ) là 5 ⇒ vô
lý ⇒ điều phải chứng minh.
Bài 4.
8a 18b 32c
+  2 P =
+
b+c−a a+c−b a+b−c
a+b+c  a+b+c  a+b+c 
= 4
  − 1 + 9  − 1 + 16  − 1
b+c−a   a+c−b  a+b−c 
 4 9 16 
= (a + b + c)
 29 + + −
b+c−a a+c−b a+b−c
Chú ý rằng:
 2 3
 b+c−a + a+c−b +
 b+c−a a+c−b
4 
+
  a +b−c  = 2+3+ 4 = 9
a+b−c 
Áp dụng bất đẳng thức Bunhiacôpski ta có:

Liên hệ tài liệu word môn toán: 039.373.2038 TÀI LIỆU TOÁN HỌC
Website:tailieumontoan.com

 2 3 4
=92   b+c−a + a+c−b +
 b+c−a a+c−b a+b−c
 4 9 16 

 [ + +  (b + c − a ) + ( a + c − b ) +
b+c−a a+c−b a+b−c
 4 9 16  
=  + +  ( a + b + c )
b+c−a a+c−b a+b−c 
Từ đó suy ra 2 P81 − 29 = 52 ⇒ P26 .
Dấu bằng đạt được khi:
2 3 4 1
= = =  ( vôùi  k > 0)
b+c−a a+c−b a+b−c k
  7k
b + c − a =  a=
2k 2
 
⇒ a + c − b = 3k ⇒ b = 3k
a + b − c =4k  5k
 a =
  2
Cụ thể hơn, có thể chon:=
a 7,=
b 6,=
c 5.
Chú ý: Có thể dễ dàng chứng minh bất đẳng thức:
( a1b1 + a2 b2 + a3b3 )
2
(
 a12 + a22 + a32 )( b
1
2
+ b22 + b32 )
a1 a2 a3
Dấu = xảy ra khi và chỉ khi: = =
b1 b2 b3
Cách giải khác:
b + c − a =2x a= y + z
 
Đặt  
c + a − b = x y, z > 0 và
thì  ,
2y    b= z + x
a + b − c = 
 2z c= x + y
 y+z z+x x+ y
⇒ 2P = 
 4  + 9  + 16  
 x   y   z 
 y x  z x  z y
=
 4 + 9  +  4 + 16  +  9 + 16 
 x y  x z  y z
Đặt
 y x  z x  z y
A=
 4 + 9 ; B =
 4 + 16  ; C =
 9 + 16 
 x y  x z  y z
Áp dụng bất đẳng thức α + β 2 αβ với α , β > 0 .
Ta có A12, B16, C24 ⇒ 2 P12 + 16 + 24 = 52 ⇒ P26

Liên hệ tài liệu word môn toán: 039.373.2038 TÀI LIỆU TOÁN HỌC
Website:tailieumontoan.com

y 3
x = 2

z 3
Dấu bằng đạt được khi:  = 2   
⇔ y = x; z = 2 x
x 2
z 4
y = 3

Chẳng hạn lấy x = 2 ⇒ y = 3, z = 4 ⇒ a = 7, b = 6, c = 5
Tóm lại Pmin = 26 .
Bài 5.
1 Tam giác vuông AIB ′ = tam giác vuông AIC ′ (vì AI chung và
IB ′ = AIB ′ = 
IC ′ ) ⇒  AIC ′ ⇒   sd   
MB ′ = sd  MC ′ .

Trong  A′B ′C ′A′M là đường phân giác trong của B ′A′C ′ .


Tương tự B ′N , C ′P là các đường phân giác trong của  A′B ′C ′ suy ra ba đường A′M , B ′N , C ′P
dồng quy.

=  + CBD
= Bˆ Aˆ
2 Trong  BID có: IBD IBC + .
2 2
ˆ
= B̂ + A .
 là góc ngoài của  IAB nên DIB
Mặt khác, DIB
2 2
Vậy  BID cân tại D . Hạ DE ⊥ BI và IQ ⊥ BC .
ˆ
= C ⇒ tam giác vuông IDE dồng dạng với tam giác vuông ICQ . Suy ra
= ICQ
Do IDE
2
ID IE 2 IE IB IB ⋅ IC
== = ⇒ =
2.IQ = (
2r  dpcm )
IC IQ 2 IQ 2 IQ ID
Cách giải khác: Như lý luận ở cách giải trên, các  DBI , DCI đều là tam giác cân tại D .
Xét đường tròn tâm D đi qua 3 điểm B, I , C . Đường tròn này cắt ID kéo dài tại K .
Từ tam giác vuông IBQ dồng dạng với tam giác vuông IKC và tam giác vuông IQC dồng dạng
với tam giác vuông IBK , dễ dàng suy ra đpcm.

Liên hệ tài liệu word môn toán: 039.373.2038 TÀI LIỆU TOÁN HỌC
Website:tailieumontoan.com

ĐẠI HỌC QUỐC GIA HÀ NỘI ĐỀ THI TUYỂN SINH LỚP 10


TRƯỜNG ĐẠI HỌC KHOA HỌC TỰ NHIÊN HỆ THPT CHUYÊN NĂM 2001

MÔN : TOÁN (Vòng 1)


Thời gian làm bài: 150 phút (không kể thời gian phát đề)

y2 − 1
Bài 1. Ta=
có x 2 ( y ≠ ) ( y = −2 không phải là nghiệm)
 2
y+2
3 3
x2 = y − 2 + ⇒ y−2+ ∈
y+2 y+2
Vậy y + 2 =±1;   y + 2 =±3 - y + 2 =1 → y =−1 → x 2 =0 → x =0 ta có nghiệm ( x = 0, y = −1)

• y + 2 =−1 → y =−3 → x 2 =−8 (loại)


• ( x 0,=
y + 2 = 3 → y =1 → x 2 = 0 → x = 0 ta có nghiệm = y 1)
• y + 2 =−3 → y =−5 → x =−8 (loại)
( x 0,=
Đáp số: = y 1) ;=
( x 0,=
y −1)

  1
 1  x − 3
 x −  và  x0 
Bài 2. 1) Điều kiện để phương trình có nghĩa  3 ⇔  x1
 x0  và  x1 x = 0
 
 

Ta có x = 0 là nghiệm. Chia hai vế cho x 2 ≠ 0 ta thu được


1 1
3+ − 1− =2
x x

Liên hệ tài liệu word môn toán: 039.373.2038 TÀI LIỆU TOÁN HỌC
Website:tailieumontoan.com

1 1 u − v = 2
Đặt u =+
3 0, v =1 − 0 ta thu được  2
u + v =
2
x x 4
Ta có u= 2 + v suy ra (2 + v) 2 + v 2 =4 ↔ 2v 2 + 4v =0
v = 0 1
 ↔ v = 1− = 0 ↔ x = 1
( i )
 v = −2  loaï x
Đáp số: x = 0 hoăc x = 1
2 Hệ phương trình được viết lại:
 x = 1
 y ( x − 1) + x 2 − 3 x + 2 =0 ( x − 1)( y + x − 2 ) =0  2
⇔  x + y =
2
 2
 x +y =
2 2
2  x +y =
2 2
2 ⇔
 x + y = 2
   2
  x + y =
2
2
=  x 1= x 1
Giải (I)    và  
y = 1  y = −1
Giải (II) thu được x 2 + (2 − x) 2 = 2 ↔ 2 x 2 − 4 x + 2 = 0 ↔ x = 1  và  y = 1
( x 1,=
Đáp số: = y 1) ;  1,
(x =
= y −1)
Bài 3. a) Ta có
1 a a2 a2 a 3
OH = OM = ⇒ MH  = − =
2 4 4 16 4
a 2 a 15
FH  = a2 − =
16 4
Suy ra
3 + 15
= MF ⋅a
4
1 3 + 15
=
FF
 ' =MF ⋅a
2 8
EM  = ME1 = HE1 − MH = FH − MH = FH − MH
15 − 3
⇒   
EM = ⋅a
4
1 15 − 3
Suy ra: EE ′=
= EM ⋅a
2 8
15
Ta có tổng hai đáy hình thang EE ′ + FF ′ = ⋅a
4

Liên hệ tài liệu word môn toán: 039.373.2038 TÀI LIỆU TOÁN HỌC
Website:tailieumontoan.com

Ta có
3 3 15 − 3
ME ′ = EM ⋅
= ⋅ ⋅a
2 2 4
3 3 3 + 15
=′ FM ⋅ =
MF ⋅ ⋅a
2 2 4
3 15 45
Vậy đường cao hình thang vuông là: h = ME ′ + MF ′ = ⋅ ⋅a = ⋅a
2 2 4
1 15 45 15 3 2
Diện tích hình thang vuông là: S = ⋅ ⋅a⋅ ⋅a = ⋅a
2 4 4 32
a
Suy ra EF tiếp xúc với vòng tròn tâm O bán kính .
2
Bài 4. Nhân hai vế của phương trình (1) với xyz ta thu được
2 xyz + x 2 z + x 2 y + y 2 x + y 2 z + z 2 y + z 2 x =
0
⇔ ( xyz + x z ) + ( xyz + y z ) + ( x
2 2 2
)
y + y2 x + z2 ( y + x) =
0
⇔ xz ( y + x ) + yz ( x + y ) + yx ( x + y ) + z 2 ( y + x ) =
0
⇔ ( )
( x + y ) xz + yz + yx + z 2 =
 0
⇔ ( x + y )( y + z )( z + x ) =
 0
1 1
Không mất tổng quát giả sử: x+ y =0→  
x3 + y 3 = 0 →  + =0
x y
1
Suy ra: z 3 =1 →  z =1 →   =1
z
1 1 1
Vậy: + + =1
x y z
xyz 1
Bài 5. Ta có: M =
2 xy 2 yz 2 zx 8
1
Vậy M max = khi x= y= z .
8

Liên hệ tài liệu word môn toán: 039.373.2038 TÀI LIỆU TOÁN HỌC
Website:tailieumontoan.com

ĐẠI HỌC QUỐC GIA HÀ NỘI ĐỀ THI TUYỂN SINH LỚP 10


TRƯỜNG ĐẠI HỌC KHOA HỌC TỰ NHIÊN HỆ THPT CHUYÊN NĂM 2001

MÔN : TOÁN (Vòng 2)


Thời gian làm bài: 150 phút (không kể thời gian phát đề)

Bài 1.
1 =
Từ giả thiết ta có c f ( 0 ) ∈  , còn a, b không nhất thiết phải nguyên, chẳng hạn với
1 1 1 x ( x + 1)
a= b= , c ∈  ta có f ( x=
) x2 + x + =
c + c ∈  với mọi x ∈ 
2 2 2 2
2 Giả sử ( x, y ) là nghiệm không âm của phương trình x 2 = y 2 + y + 1 . Khi đó

x2 > y 2
Mặt khác do y ≥ 0 nên y + 1 ≤ 4 y 2 + 4 y + 1= (2 y + 1) 2
Hay y + 1 ≤ 2 y + 1 ⇒ y 2 + y + 1 ≤ y 2 + 2 y + 1 = ( y + 1) 2
Dấu "=" đạt được khi và chỉ khi y = 0 . Từ (1) và (2) suy ra
y 2 < x 2 = y 2 + y + 1 ≤ ( y + 1) 2 ⇒ x 2 = ( y + 1) 2
(vì y 2 và ( y + 1) 2 là hai số chính phương liên tiếp) ta có:
y 2 + y + 1 = ( y + 1) 2 ⇒ y = 0
Với y = 0 ta được nghiệm là=
x 1,=
y 0.
Vậy phương trình đã cho có đúng một nghiệm nguyên là=
x 1,=
y 0
Bài 2.

Liên hệ tài liệu word môn toán: 039.373.2038 TÀI LIỆU TOÁN HỌC
Website:tailieumontoan.com

4 x + 1 = x 2 − 5 x + 14
( ) (
⇒ x 2 − 6 x + 9 + x + 1 − 4 x + 1 + 4 =0
   )
⇒ x − 3) 2 + ( x + 1 − 2) 2 =
 ( 0
⇒ x−3=
 [  0
x +1 − 2 =0
Bài 3.
ax + by = 3
ax 2 + by 2 =
 5
 3
ax + by =
3
9
ax + by =
4 4
17
Nhân (2) với x + y ta có ax3 + by 3 + xy ( ax + by ) = 5 ( x + y ) .
Từ đây và (1) , ( 3) ta có 9 + 3 xy = 5 ( x + y )

( )
Nhân (3) với x + y ta có ax 4 + by 4 + xy ax 2 + by 2 = 9 ( x + y ) .

Từ đây và ( 2 ) , ( 4 ) ta có 17 + 5 xy = 9 ( x + y )

9 + 3 xy = 5 ( x + y )
Vậy x, y thoả mãn hệ 
17 + 5 xy = 9 ( x + y )
 x +=
y 3 =x 1, =
y 2
Giải hệ này ta được  ⇒
= xy 2 =  x 2,=y 1

a + 2b =3
a + 4b =
 5
Với= y 2 hệ đã cho có dạng 
x 1,= ⇒ a =b =
 1
a + 8b =9
a + 16b =17
Trường hợp= y 1 , tương tự ta cũng thu được a= b= 1 .
x 2,=
Vậy S n = 1 + 2n do đó
ax n + by n =
A =ax5 + by 5 =1 + 25 =33
B= ax 2001 + by 2001 =
1 + 22001
Bài 4. Kéo dài MO cắt d 2 tại P thì OAM =OBP suy ra hai đường cao tương ứng của chúng
= OA . Vậy đường tròn ( O ) đường kính AB tiếp xúc với MN tại
= OB
bằng nhau, tức là OH
H (và tiếp xúc d1 , d 2 tại A, B) . Do đó

Liên hệ tài liệu word môn toán: 039.373.2038 TÀI LIỆU TOÁN HỌC
Website:tailieumontoan.com

= =
AM HM ,   BN HN
MH MA MI
⇒ = = ⇒ HI / / BN
HN NB IB

hay đường thẳng ( HIK ) / / d 2 và d1 suy ra ( HIK ) ⊥ AB và HKB 
= MEB
=  (vì EMHB là
NHB
tứ giác nội tiếp). Mà
 = NBH
NHB  = KHB
 ⇒ HKB
 = KHB

AB
hay  BHK cân ở B nên AB là đường trung trực của HK ( AB ⊥ HK ) suy ra OK
= OH
=
2
hay K luôn chạy trên đường tròn ( O ) đường kính AB .
Bài 5. Nhận xét: Nếu đồng tiền được đổi mặt một số lẻ lần thì mặt đỏ ngửa lên phía trên.
Đánh số các đồng tiền theo chiều kim đồng hồ lần lượt là a1 , a2 , …, a2001 .
Cách 1: Ta đổi mặt theo thứ tự sau (mỗi lần 5 dồng tiền viết trên một dòng).
𝑎𝑎1 𝑎𝑎2 𝑎𝑎3 𝑎𝑎4 𝑎𝑎5
𝑎𝑎2 𝑎𝑎3 𝑎𝑎4 𝑎𝑎5 𝑎𝑎6
𝑎𝑎3 𝑎𝑎4 𝑎𝑎5 𝑎𝑎6 𝑎𝑎7
𝑎𝑎4 𝑎𝑎5 𝑎𝑎6 𝑎𝑎7 𝑎𝑎8
𝑎𝑎5 𝑎𝑎6 𝑎𝑎7 𝑎𝑎8 𝑎𝑎9
𝑎𝑎6 𝑎𝑎7 𝑎𝑎8 𝑎𝑎9 𝑎𝑎10
… … … …
𝑎𝑎1997 𝑎𝑎1998 𝑎𝑎1999 𝑎𝑎2000 𝑎𝑎2001
𝑎𝑎1998 𝑎𝑎1999 𝑎𝑎2000 𝑎𝑎2001 𝑎𝑎1
𝑎𝑎1999 𝑎𝑎2000 𝑎𝑎2001 𝑎𝑎1 𝑎𝑎2
𝑎𝑎2000 𝑎𝑎2001 𝑎𝑎1 𝑎𝑎2 𝑎𝑎3
𝑎𝑎2001 𝑎𝑎1 𝑎𝑎2 𝑎𝑎3 𝑎𝑎4
Qua lược đồ trên ta thấy sau 2001 lần đổi mặt, mỗi đồng tiền đều được đổi mặt 5 lần, do đó tất cả
các mặt đỏ đều ngửa lên phía trên
Cách 2: Lần lượt đổi mặt các đồng tiền theo chiều kim đồng hồ như sau:
Lần 1: a1 , a2 , a3 , a4 , a5
Lần 2: a6 , a7 , a8 , a9 , a10
Lần 3: a11 , a12 , a13 , a14 , a15
Lần 2001: a1997 , a1998 , a1999 , a2000 , a2001
Như thế sau 2001 lần đổi mặt thì mỗi đồng tiền được đổi mặt đúng 5 lần. Do đó tất cả các mặt đỏ
đều ngữa lên phía trên.

Liên hệ tài liệu word môn toán: 039.373.2038 TÀI LIỆU TOÁN HỌC
Website:tailieumontoan.com

ĐẠI HỌC QUỐC GIA HÀ NỘI ĐỀ THI TUYỂN SINH LỚP 10


TRƯỜNG ĐẠI HỌC KHOA HỌC TỰ NHIÊN HỆ THPT CHUYÊN NĂM 2000

MÔN : TOÁN (Vòng 1)


Thời gian làm bài: 150 phút (không kể thời gian phát đề)

Bài 1.
1 Ta có
 1 1 1  1 1 
=  −  +  −  + + 
 1S − 
 2  2 3  1999 2000 
1 1999
⇔   1 S = − =
2000 2000
1 x
2 Đặt u =
x + ,   v =ta có
y y
2
1  1  2x
x + 2 = x +  −
2
= u 2 − 2v
y  y y
u=+v 3 =u 2, = v 1
Thay vào hệ đã cho ta nhận được:  2 ⇒ u 2 + u − 6 = 0  
   ⇔ 
u − v =3 u =−3, v =6
 1
x + y =
2  1
 x + = 2 x = 1
a) Giải  ⇔ y ⇔
x =1 x = y x = 1
 y 

Liên hệ tài liệu word môn toán: 039.373.2038 TÀI LIỆU TOÁN HỌC
Website:tailieumontoan.com

b) Giải
 1
x + y =−3  1
 6 y + = −3
 ⇔ y ⇔   Voâ nghieäm
x = 6  x = 6y
 y 

x = 1
Đáp số: 
y =1
Bài 2.
1 Viết lại phương trình dưới dạng
x − 1 + x 3 + x 2 + x + 1 =1 + ( x − 1) ( x3 + x 2 + x + 1)
Điều kiện: x1 ta có


 (1)   ( x −1 −1 )( )
x3 + x 2 + x + 1 − 1 =0

a) Giải x − 1 = 1  2
⇔ x=
b) Giải x 3 + x 2 + x + 1 = 1  0 
⇔ x = (loại)
Đáp số: x = 2
2 Cách giải thû́ nhât: Phương trình đã cho viết lại như sau:
11  7
x 2 − 4ax + 4a 2 =− x 2 + x − 7 =( 2 − x )  x − 
2  2

( 2 − x )  x − 
7
⇔   ( x − 2a ) 2 =
 2
 7 7
 suy ra  ( 2 − x )  x −  0 ⇔ 2x
 2 2
7
2x0
2
Vậy chỉ có thể= x0 3 . Thay vào phương trình ta nhận được:
x0 2,=
a) (2 − 2a ) 2 =0  
⇒ a =1
19
Thử lại với a = 1 ta có phương trình 2 x 2 −x + 11 =
0
2
Phương trình này có nghiệm nguyên x = 2 .
1 3 1
b) (3 − 2a ) 2 =  ⇒ a = ±
2 2 2 2

Liên hệ tài liệu word môn toán: 039.373.2038 TÀI LIỆU TOÁN HỌC
Website:tailieumontoan.com

3 1  23  33
Thử lại với a= + ta có phương trình 2 x 2 −  + 2  x + +3 2 =
0
2 2 2  2  2
có nghiệm nguyên x = 3 .
3 1  23  33
Với a= − thu được phương trình 2 x 2 −  − 2  x + −3 2 =
0
2 2 2  2  2
có nghiệm nguyên x = 3
3 1
Vậy a= 1, a= ± là các giá trị của a cần tìm.
2 2 2
11x
Cách thứ 2: Phương trình được viết dưới dạng 4a 2 − 4 xa + 2 x 2 − +7 =0
2
Suy ra
7
−4 x 2 + 22 x − 280 ⇔  2x
 ′=
2
Vậy nghiệm nguyên x0 chỉ có thể nhận các giá trị= x0 3 . Thay vào phương trình ta nhận
x0 2,=
3 1
được a= 1, a= ±
2 2 2
Bài 3.
1 Đường tròn tiếp xúc với AD tại P và BC tại Q . Vì AB / / DC ⇒ Aˆ + Dˆ =
1800
Ta có
  + ODA
=  − OAD
AOD  180 
( )
Aˆ + Bˆ
=80 −
 1 =900
2
⇒ AOD vuông tại O .
Tương tự  BOC vuông tại O .
= r=
Xét hai tam giác vuông AOD và BOC ta có: PA.PD 2
QB.QC
BE DF
= =
Vì AE PA =
, DF PD , BE QB= (
, CF QC ta suy ra: AE.DF= BE.CF     dpcm 
⇒ = )
AE CF
2 Ta có
r
S ABCD= ( AB + AD + DC + BC )
2
Vì AB + AD= DC + BC ⇒ S = r ( AB + CD )
Từ tính chất

Liên hệ tài liệu word môn toán: 039.373.2038 TÀI LIỆU TOÁN HỌC
Website:tailieumontoan.com

BE DF 1
= ⇒   
CF = ⋅ CD =CQ
AE CF 3
2a
BQ =  EB=
3
1 2a
Ta suy ra: b = CD + ⇒ CD =3b − 2a
3 3
⇒ AB + CD = 3b − a ⇒ S ABCD = r ( 3b − a )
Ta có

2a 3b − 2a 2a ( 3b − 2a )
t 2 = BQ ⋅ CQ = ⋅ ⇒r=
3 3 3
Suy ra
2a ( 3b − 2a )
=
S ABCD ⋅ ( 3b − a )
3
Bài 4. Cách 1: Bất đẳng thức được viết lại dưới dạng
4 x2 y 2 x4 + y 4
=M + 2 2 3
(x )
2
2
+ y2 x y

(x + y ) (x ) + (x )
2 2 2
4 x2 y 2
2 2
4 x2 y 2
2
+ y2 2
+ y2
M = + = +
( ) (x )
2
x2 + y 2 2 x2 y 2 2
+ y2
2
2 x2 y 2 2 x2 y 2

)
2
x2 + y 2
M 2 + 2 + 1 =3
2 x2 y 2
Dấu đẳng thức xảy ra khi x = y .
x2 y2 x2 + y 2 x2 + y 2
Cách 2: Bất đẳng thức được viết lại: N = 4 ⋅ + + 5
x2 + y 2 x2 + y 2 x2 y2
x2 y2
Đặt a
= = , b , ta có a, b > 0, a + b =
1
x2 + y 2 x2 + y 2
Từ đó
1 1 1 1
n= 4ab + + = 8ab + + − 4ab
a b a b
a+b
2

N 3 8 − 4 
3
 =
5
 2 
Dấu đẳng thức xảy ra khi và chỉ khi a = b ⇔ x = y .

Liên hệ tài liệu word môn toán: 039.373.2038 TÀI LIỆU TOÁN HỌC
Website:tailieumontoan.com

ĐẠI HỌC QUỐC GIA HÀ NỘI ĐỀ THI TUYỂN SINH LỚP 10


TRƯỜNG ĐẠI HỌC KHOA HỌC TỰ NHIÊN HỆ THPT CHUYÊN NĂM 2000

MÔN : TOÁN (Vòng 2)


Thời gian làm bài: 150 phút (không kể thời gian phát đề)

Bài 1.
1.
y ( x − 1) = x 2 + 2

x2 + 2 3
Dễ thấy x = 1 không thể thoả mãn (1) , do đó (1) tương đương với y = ⇒ y = x +1+
x −1 x −1
3
Với x ∈  thì y ∈  suy ra ∈  hay z − 1 là một trong các số: ±1, ±3
x −1
x −1 = 1 → x = 2 → y = 6
x − 1 =−1 → x =0 → y =−2
x −1 = 3 → x = 4 → y = 6
x − 1 =−3 → x =−2 → y =−2
Vậy phương trình (1) có bốn nghiệm nguyên ( x, y ) là ( 2;6 ) , ( 0; −2 ) , ( 4;6 ) , ( −2; −2 )
2.

Liên hệ tài liệu word môn toán: 039.373.2038 TÀI LIỆU TOÁN HỌC
Website:tailieumontoan.com

−1 ≤ x + y ≤1
−1 ≤ xy + x + y ≤ 1
Ta chứng minh x ≤ 2 .
Giả sử ngược lại x > 2 khi đó có hai khả năng
(a) x > 2 . Từ (1) ta có y ≤ 1 − x < −1 suy ra xy < −2
(b) x < −2 . Từ (1) ta có y ≥ −1 − x > 1 suy ra xy < −2
Vậy nếu x > 2 thì xy < −2 và do x + y ≤ 1 nên ta có xy + x + y < −1 trái với (2) nên giả thiết
x > 2 là sai suy ra x ≤ 2 . Lý luận tương tự ta có y ≤ 2
Bài 2.
1 Phương trình đã cho viết lại dưới dạng
4 1 5
− x + x − − 2x − =0
x x x
1 5 4
Đặt u = x − , v = 2 x − thì u , v ≥ 0 và u 2 − v 2 = − x . Do đó (1) có dạng
x x x
u 2 − v 2 + u − v = 0 ⇒ ( u − v )( u + v + 1) = 0 ⇒ u = v   vì  
( u, v ≥ 0 )
1 5 1 5
Từ đó ta có x− = 2x − ⇒ x − = 2x − ≥ 0
x x x x
1 5
Phương trình x − = 2 x − có nghiệm là x = ±2 .
x x
Từ (2) suy ra chỉ có x = 2 là nghiệm của phương trình đã cho.
2 f ( x ) = ax 2 + bx + c . Gọi tập số hữu tỷ là  ta có
f (1) = a + b + c ∈ 
f ( 4 )= 16a + 4b + c ∈ 
f ( 9 )= 81a + 9b + c ∈ 
Trừ (2) cho (1) ta có 15a + 3b ∈  ⇒ 5a + b ∈ 
Trừ (3) cho (2) ta có 65a − 5b ∈  ⇒ 13a − b ∈ 
Cộng (4) và (5) ta được 18a ∈  ⇒ a ∈  . Từ (3) suy ra b ∈  và từ (1) ta có c ∈  . Vậy
a , b, c ∈ 
Bài 3.
1 Vẽ vòng tròn ( O ) đường kính AC . Do Bˆ ≥ 90 , Dˆ ≥ 90 nên B và D nằm bên trong
hoặc trên ( O ) . Kéo dài BD cắt ( O ) tại M , N thì BD ≤ MN ≤ AC

Liên hệ tài liệu word môn toán: 039.373.2038 TÀI LIỆU TOÁN HỌC
Website:tailieumontoan.com

2 Vì A, B, C là ba đỉnh của một tam giác nên B không thuộc đường thẳng AC . Theo giả
thiết  ABC phải thoả mãn đồng thời bốn điều kiện sau:
 Aˆ ≤ Bˆ

 Aˆ ≤ Cˆ
 
 Bˆ ≤ 90
ˆ 
C ≤ 90
Ta lần lượt tìn tập hợp các điểm B thoả mãn mỗi điều kiện trên a) Aˆ ≤ Bˆ suy ra BC < AC hay
B nằm bên trong hoặc trên đường tròn ( C ) tâm C , bán kính CA . b) Aˆ ≤ Bˆ suy ra BC < BA
hay B thuộc nửa mặt phẳng chứa C có bờ là đường trung trực  của AC ( B có thể thuộc ) .
c) Bˆ ≤ 90 hay B nằm bên ngoài hoặc trên đường tròn ( O ) đường kính AC .

d) Cˆ ≤ 90 hay B thuộc nửa mặt phẳng chứa A , có bờ là đường thẳng d đi qua C , vuông góc
với AC ( B có thể thuộc d nhưng B ≠ C ) .
Vậy tập hợp các điểm B thoả mãn giả thiết bài toán là phần không bị gạch trên hình vẽ (kể cả
biên trừ điểm C ).
Bài 4. Gọi 6 điểm đã cho là A, B, C , D, E , F . Ta tô màu các đoạn thẳng nối các cặp điểm trong
chúng như sau:
Trong mỗi tam giác có 3 đỉnh là 3 trong 6 điểm đã cho, ta tô cạnh bé nhất của nó màu đổ. Sau
khi đã tô hết các cạnh như vậy bằng màu đỏ, các đoạn thẳng còn lại ta tô màu xanh (như vậy
đoạn được tô xanh không phải là cạnh bé nhất của một tam giác nào cả). Ta chứng minh tồn tại 3
điểm trong 6 điểm đã cho mà tam giác tạo bởi ba điểm đó có ba cạnh cùng màu.
Thật vậy, vì 5 đoạn AB, AC , AD, AE , AF được tô bởi hai màu xanh, đỏ nên trong chúng phải có
3 đoạn cùng màu. Không mất tổng quát giả sử đó là AB, AC , AD . Khi đó nếu một trong 3 đoạn
BC , CD, DB , chẳng hạn BC , cùng màu với các đoạn AB, AC , AD , thì  ABC có ba cạnh cùng
màu. Còn nếu cả ba đoạn BC , CD, DB đều khác màu với các đoạn AB, AC , AD thì chúng cùng
màu với nhau. Suy ra  BCD có ba cạnh cùng màu.
Vậy luôn có ba điểm trong 6 điểm đã cho có tạo thành tam giác có ba cạnh cùng màu.
Giả sử  ABC có ba cạnh cùng màu. Khi đó do cạnh bé nhất của nó có màu đỏ nên cả ba cạnh
cùng màu đỏ. Do cạnh lớn nhất của  ABC có màu đổ nên nó cũng là cạnh bé nhất của một tam
giác khác.

Liên hệ tài liệu word môn toán: 039.373.2038 TÀI LIỆU TOÁN HỌC
Website:tailieumontoan.com

ĐẠI HỌC QUỐC GIA HÀ NỘI ĐỀ THI TUYỂN SINH LỚP 10


TRƯỜNG ĐẠI HỌC KHOA HỌC TỰ NHIÊN HỆ THPT CHUYÊN NĂM 1999

MÔN : TOÁN (Vòng 1)


Thời gian làm bài: 150 phút (không kể thời gian phát đề)

Bài 1.
a + b + c =0
 2
a + b + c =
2 2
14
Bình phương hai vế của (1) ta có
ab + bc + ca =
−7
Bình phương hai vế của (3) ta có
a 2 b 2 + b 2 c 2 + c 2 a 2 + 2abc ( a + b + c ) =49 → a 2 b 2 + b 2 c 2 + c 2 a 2 =49
Bình phương hai vế của (2) ta được
( )
a 4 + b 4 + c 4 + 2 a 2 b 2 + b 2 c 2 + c 2 a 2 = 196 → a 4 + b 4 + c 4 = 98
Vậy biểu thức cần tính
P =1 + a 4 + b 4 + c 4 =99
Bài 2. 1. Giải phương trình: x + 3 − 7 − x= 2x − 8
Phương trình tương đương với

Liên hệ tài liệu word môn toán: 039.373.2038 TÀI LIỆU TOÁN HỌC
Website:tailieumontoan.com

4 ≤ x ≤ 7

 x + 3= 2x − 8 + 7 − x
Phương trình (2) tương đương với
x + 3 = x − 1 + 2 −2 x 2 + 22 x − 56
→ 2 = −2 x 2 + 22 x − 56
→ 4=−2 x 2 + 22 x − 56
→ x 2 − 11x + 30 =
0
→ x1 = x2 =
5,  6
 x1 = 5
Cả hai nghiệm đều thoả mãn (1). Vậy phương trình đã cho có hai nghiệm 
 x2 = 6
2 Hệ đã cho có dạng
 1  1 9
 x + +y+  =
 y  x 2

 x + 1  1 9
  y +  =
 y  x 2
1 1
Đặt x + u và y + =
= v ta có:
y x
 9 u = 3
u + v =  3
2⇔ u =
  3  ⇔  hoaëc   2
uv = 9 v = v = 3
  2 
2
Vậy hệ đã cho có bốn nghiệm là
   1 x = 1
=
 x 1=x 2 x = 
 
;  ; 2;  1
=
 y 2= y 1  y = 1  y = 2
  
Bài 3. Ta có
n 2 + 9n − 2 20
= n−2+
n + 11 n + 11
muốn n 2 + 9n − 2 n + 11 thì n + 11 phải là ước số của 20 suy ra n = 9 là duy nhất
Bài 4.1) và 2) Gọi trung điểm của OI là O ′ thì O ′E ′ là đường trung bình của tam giác IOE do
1 1
đó O
= ′E ′ = OE R( R là bán kính của đường tròn ( C ) .
2 2
R
Tương tự: O ′F ′ = O ′M ′ = O ′N ′ = .
2
Liên hệ tài liệu word môn toán: 039.373.2038 TÀI LIỆU TOÁN HỌC
Website:tailieumontoan.com

R
Vậy M ′E ′N ′F ′ luôn là tứ giác nội tiếp đường tròn có bán kính R ′ = .
2
1 1
3 ′ = S M ′E ′N ′F ′
Nếu MN ′ ⊥ EF thì S= = M ′N ′.E ′F ′ MN .EF ′ Hạ OK ⊥ MN , OH ⊥ EF
2 8
ta có
MN 2 + EF 2 = 4 MK 2 + 4 EH 2
( ) ( ) (
  =4 R 2 − OK 2 + 4 R 2 − OH 2 =8 R 2 − 4 OK 2 + OH 2 )
=
  8 R − 4OI
2 2

không đổi (vì I cố định). Do đó S ′ =


1
8
MN .EF ≤
1
16
( )
MN 2 + EF 2 =
1
4
(
2 R 2 − OI 2 )
Dấu "=" đạt được khi và chỉ khi MN = EF hay OK = OH hay hình chũ̃ nhật OKIH là hình
vuông khi và chỉ khi MN và EF lập với OI các góc bằng 45 .

Vậy S ′ = S M ′N ′E ′F ′ đạt max bằng


1
4
( )
2 R 2 − OI 2 , đạt được khi và chỉ khi MN và EF lập với OI

các góc bằng 45 .
Bài 5. Từ giả thiết x > 0, y > 0, x + y = 1 ta có
( x + y)2 1 1
xy ≤ =→ x 2 y 2 ≤
4 4 16
1
Dấu "=" đạt được khi và chỉ khi x= y=
2
1
Ta có: P = x 2 y 2 + 2 2 + 2
x y
1 1
Cách 1: Trước hết chú ý rằng nếu a ≥ b ≥ 1 thì a + ≥ b + . Dấu "=" đạt được khi và chỉ khi
a b
a =b.
Thật vậy:
 1  1  ab − 1 
S =  a +  −  b +  = (a − b)  ≥0
 a  b  ab 
Dấu "=" đạt được khi và chỉ khi a = b .
1
Coi =
a 2 2
≥=
b 16 ta có
x y
1 1 289
=
P 2 2
+ x 2 y 2 + 2 ≥ 16 + +=2
x y 16 16

Liên hệ tài liệu word môn toán: 039.373.2038 TÀI LIỆU TOÁN HỌC
Website:tailieumontoan.com

1 1 1
Dấu "=" đạt được khi và chỉ khi 2 2
= 16 hay xy = hay x= y= . Vậy P đạt min bằng
x y 4 2
289 1
, đạt được khi x= y=
16 2
Cách 2:
1 1
P = x2 y 2 + 2 2
+ 2 = 162 x 2 y 2 + 2 2 + 2 − 255 x 2 y 2
x y x y
1 1 1 1
mà 162 x 2 y 2 + 2 2
≥ 32 , dấu "=" đạt được khi 16xy = hay xy = hay x= y= .
x y xy 4 2
1 255 1
Còn do x 2 y 2 ≤ nên −255 x 2 y 2 ≥ − , dấu "=" đạt được khi x= y= .
16 16 2
255 289 1
Vậy P ≥ 32 + 2 − = , dấu "=" đạt được khi và chỉ khi x= y= suy ra P đạt min bằng
16 16 2
289 1
đạt được khi và chỉ khi x= y=
16 2
ĐẠI HỌC QUỐC GIA HÀ NỘI ĐỀ THI TUYỂN SINH LỚP 10
TRƯỜNG ĐẠI HỌC KHOA HỌC TỰ NHIÊN HỆ THPT CHUYÊN NĂM 1998

MÔN : TOÁN (Vòng 1)


Thời gian làm bài: 150 phút (không kể thời gian phát đề)

Bài 1.
1 Giải phương trình
2 − x2 + x2 + 8 =4
Điều kiện: x 22 . Bình phương hai vế, rut gọn và chuyển vế ta được (1) tương đương với
 x2 = 1
− x 4 − 6 x 2 + 16 =3 ⇒ − x 4 − 6 x 2 + 16 =9 ⇒  2
 x = −7
Nghiệm của phương trình là: x = ±1
2 Giải hệ phương trình
 x 2 + xy + y 2 = 7
 4
x + x y + y =
2 2 4
21
 x2 + y 2 =
u
Cách 1: Đặt  thì hệ (1) , ( 2 ) có dạng
 xy = v

Liên hệ tài liệu word môn toán: 039.373.2038 TÀI LIỆU TOÁN HỌC
Website:tailieumontoan.com

u=+v 7 u=
+v 7 =
u 5
 2 ⇒ ⇒
u =
−v 21 u=
−v 3 =
2
v 2
Từ đó ta có hệ
 x 2 +=
y 2 5 ( x + =
y)2 9  x + y =±3
 ⇒ ⇒
=  xy 2=  xy 2  xy = 2
Giải hai hệ
x + y =3 x + y =−3
   và  
= xy 2=  xy 2
Ta thu được nghiệm là
 x =1  x =
2 x =
−1  x =
−2
 ,  
,  ,
 y =
2  y =
1 y =
−2  y =
−1
Cách 2: (2) tương đương với

(x ) ( )( )
2
2
+ y2 − x 2 y 2 = 21 ⇒ x 2 + y 2 + xy x 2 + y 2 − xy = 21
Do đó hệ tương đương với
 x + xy += y 2 7  x 2 +=
2
y2 5
 2 ⇒ 
 x − xy =
+ y2 3 =  xy 2
Sau đó giải tiếp tục như cách 1 .
Bài 2.
a 3 − 3ab 2 =
19
 3
b − 3a b =
2
98
Bình phương hai vế của các đẳng thức trên và cộng lại ta được

( )
3
a 6 + 3a 4 b 2 + 3a 2 b 4 + b6 =9965 ⇒ a 2 + b 2 =9965 ⇒ a 2 + b 2 = 3 9965

Bài 3. Do a, b, c ∈ [ 0,1] nên

(1 − a )(1 − b )(1 − c ) 0  ⇒ 1 − a − b − c + ab + bc + ca − abc0


⇒ a + b + c − ab − bc − ca1 − abc1
 
Do a, b, c ∈ [ 0,1] nên b 2b, c 3c . Từ đó ta có
a + b 2 + c 3 − ab − bc − caa + b + c − ab − bc − ca1
Bài 4.
1 a) Do AI và BI vuông góc nên I chạy trên đường tròn cố định đường kính AB .
b) Kẻ đường kính BC . Dễ thấy:

Liên hệ tài liệu word môn toán: 039.373.2038 TÀI LIỆU TOÁN HỌC
Website:tailieumontoan.com

• Nếu M thuộc nửa đường tròn đường kính BC không chứa A thì I thuộc đoạn thẳng
AM và N thuộc cung lớn AB . - Nếu M thuộc cung nhỏ AC thì góc ∠ BAM tù, do
đó I nằm trên MA kéo dài về phía A . Khi đó điểm N có thể thuộc cung nhỏ AB ( H2 )
, hoặc cung lớn AB ( H3) .
Trong cả ba trường hợp trên đều dễ chứng minh được rằng tổng số đo hai cung nhỏ AB và MN
luôn bằng 180 , tức là ∠ MON + ∠ AOB = 180 . Gọi trung điểm AB là H ta có:
1 1
∠ NOJ + ∠ AOH = ∠ MON + ∠ AOB= 90
2 2

∠ OAH + ∠ AOH =
90 ⇒ ∠ NOJ =
∠ OAH
⇒ NOJ =
OAH
= R, ∠ NOJ
= OA
(Hai tam giác vuông có cạnh huyền ON = ∠ OAH )
AB
= AH
OJ =
2
AB
Vậy J chạy trên đường tròn tâm O bán kính R ′ = .
2
2 Kẻ đường kính EF ⊥ AB với E , F lần lượt là các điểm chính giữa cung lớn và cung nhỏ
AB . Kéo dài AM một đoạn MD = MB thì MA + MB = MA + MD = AD .
Vì MAB có AB cố định nên chu vi MAB lớn nhất khi và chỉ khi MA + MB lớn nhất khi và
chỉ khi AD lớn nhất.
Vì MF là phân giác ∠ AMB còn EM ⊥ FM nên đường thẳng EM chứa phân giác ∠ BMD
(kề bù với ∠ AMB . Vì MBD cân ở M nên EM là đường trung trực của BD . Do đó
ED = EB = EA ⇒ ADEA + ED = EA + EB
dấu "=" đạt được khi và chỉ khi E thuộc đoạn AD hay M ≡ E .
Vậy chu vi tam giác MAB đạt giá trị lớn nhất khi và chỉ khi M ≡ E ( E là trung điểm của cung
lớn AB) .
Bài 5.
1 Giả sử
n + 26 =a3

n − 11 =
b3
Do n + 26 > n − 11 nên a > b . Trừ (1) cho (2) ta được
( )
37 = a 3 − b3 ⇒ ( a − b ) a 2 + ab + b 2 = 37

Vì 37 là số nguyên tố còn hiển nhiên 0 < a − b < a 2 + ab + b 2 nên ta có:

Liên hệ tài liệu word môn toán: 039.373.2038 TÀI LIỆU TOÁN HỌC
Website:tailieumontoan.com

a − b =
1
 2
a + ab + b =
2
37
Giải ra ta được= b 3 (loại nghiệm âm). Thay vào (1) (hoặc (2)) ta được n = 38 .
a 4,=
Thử lại ta thấy n = 38 thoả mãn (1) , ( 2 ) với a = 4 và b = 3 .
2 Từ điều kiện
x 2 + y 2 + z 2 =1 ⇒ x 21, y 21, z 21
Do đó
x 2 ( y − z ) 2 ( y − z ) 2 = y 2 + z 2 − 2 yz
Dấu "=" đạt được khi và chỉ khi y = z hoặc x 2 = 1
Tương tự
y 2 ( z − x) 2 z 2 + x 2 − 2 zx
Dấu "=" đạt được khi và chỉ khi z = x hoặc y 2 = 1
z 2 ( x − y ) 2 x 2 + y 2 − 2 xy
Dấu "=" đạt được khi và chỉ khi x = y hoặc z 2 = 1
Cộng ba bất đẳng thức thu được ta có:
x 2 ( y − z ) 2 + y 2 ( z − x) 2 + z 2 ( x − y ) 2
(
 2 x 2 + y 2 + z 2 − 2 ( xy + yz + zx ) )
  =2 − 2 ( xy + yz + zx )
1 2
P = xy + yz + zx +  x ( y − z ) 2 + y 2 ( z − x) 2 + z 2 ( x − y ) 2  1
2
Dấu "=" đạt được khi và chỉ khi
= y z=
 hoaëc  x 2 1

=  hoaëc  y 2 1
 z x=
= x y=
 hoaëc  z2 1

Nhưng với x 2 = 1 thì do x 2 + y 2 + z 2 =1 ⇒ y =z =0 , do đó điều kiện (2), (3) không thể thoả
mãn.
Tương tự, trường hợp y 2 = 1 hoặc z 2 = 1 cũng bị loại.
Vậy P = 1 khi và chỉ khi
 x= y= z 1
 2   ⇒ x =y =z =±
x + y + z =
2 2
1 3

Liên hệ tài liệu word môn toán: 039.373.2038 TÀI LIỆU TOÁN HỌC
Website:tailieumontoan.com

1
Vậy P đạt max bằng 1 , đạt được khi và chỉ khi x = y = z = ± .
3

ĐẠI HỌC QUỐC GIA HÀ NỘI ĐỀ THI TUYỂN SINH LỚP 10


TRƯỜNG ĐẠI HỌC KHOA HỌC TỰ NHIÊN HỆ THPT CHUYÊN NĂM 1998

MÔN : TOÁN (Vòng 2)


Thời gian làm bài: 150 phút (không kể thời gian phát đề)

Bài 1.
1 Giải hệ phương trình
 x + x 2 + x3 + x 4 =y + y 2 + y 3 + y 4
 2
x + y =
2
1

(
Ta có (1) tương đương với ( x − y ) 1 + x + y + x 2 + xy + y 2 + x 2 + y 2 ) ( x + y ) =
0 . Do đó hệ

  x − y = 0
   2
( x − y )  2 + 2 ( x + y ) + xy  = x + y =
2

(1) và ( 2 ) tương đương  2 2 


0  1

 2 + 2 ( x + y ) + xy =
x + y = 1
 2
0

  x + y =
2
 1
1
a) Giải hệ ( 3) , (4) ta được hai nghiệm là x = y = ± .
2
x + y = u 2 + 2u + v =0 v =−2u − 2
b) Đặt  thì hệ ( 5 ) , ( 6 ) có dạng  2 ⇒ 2
 xy = v u − 2v = 1 u + 2 ( 2u + 2 ) =
1

Liên hệ tài liệu word môn toán: 039.373.2038 TÀI LIỆU TOÁN HỌC
Website:tailieumontoan.com

u = −1 u = −3
Giải ra ta được  hoặc   Với u = 0 ta được hệ
−1, v =
v = 0 v = 4
x + y = −1  x = 0, y = −1
 ⇒
 xy = 0 x = −1, y = 0

x + y = −3
Với u = 4 ta được hệ 
−3, v = Hệ này vô nghiệm
 xy = 4
 1 1   1 1 
Vậy hệ đã cho có 4 nghiệm:  , , − ,−  , ( 0, −1) , ( −1, 0 ) .
 2 2  2 2
2 Xét phương trình
1− x + 1+ x = 1− a + 1+ a

Điều kiện: −1x1 . Khi đó: ( 1 − x + 1 + x ) 2 = 2 + 2 1 − x 2 4 ⇒ 1 − x + 1 + x2


Đẳng thức có được khi x = 0 .
1 − a + 1 + a  1 − a + 1 + a =2

Đẳng thức có được khi (1 − a )(1 + a ) 0 ⇒ −1a1 .

Do đó (1) có nghiệm khi và chỉ khi 1 − a + 1 + a = 2 ⇒ −1a1 . Khi đó nghiệme của phương
trình là x = 0 .
Chú ý: Bình phương hai vế của (1) và rút gọn ta được (1) tương đương với
1 − x2 = a2 + 1 − a2

Do 1 − x 2 1 dấu "=" đạt được khi và chỉ khi x = 0 còn a 2 +∣1 − a 2  a 2 + 1 − a 2∣=1 , dấu "="

( )
đạt được khi và chỉ khi a 2 1 − a 2  0 ⇒ a 21 .

Do đó ( 2 ) có nghiệm khi và chỉ khi a 21 hay −1a1 . Khi đó nghiệm của (2) (hay của (1) )
là: x = 0 .
Bài 2. Tìm nghiệm nguyên
19 x3 − 98 y 2 =
1998
(1) tương đương với
(
19 x3 − 100 ) ( )
= 98 y 2 + 1 ⇒ 19 x3 − = (
2 98 y 2 + 20 ) ( )
( ) ( )
Ta có 98 y 2 + 20 chia hết cho 7 (với y nguyên). Ta chứng minh 19 x3 − 2 không chia hết cho
7 và như vậy (2) không có nghiệm nguyên. 2.18. Đáp án tuyển sinh lớp 10 năm 1998(cho thí
sinh chuyên toán và chuyên tin)89
Với ∀x ∈  ta có x = 7 k + i, k ∈  còn i nhận các giá trị từ 0 đến 6 .
Liên hệ tài liệu word môn toán: 039.373.2038 TÀI LIỆU TOÁN HỌC
Website:tailieumontoan.com

x3 − 2 = (7 k + i )3 − 2 = (7 k )3 + 3(7 k ) 2 i + 3.7 ki 2 + i 3 − 2 = 7 m + i 3 − 2, m ∈ 
Dễ kiểm tra được rằng: Với i nhận các giá trị từ 0 đến 6 thì i 3 − 2 không chia hết cho 7 suy ra
x3 − 2 không chia hết cho 7 . Vì 19 và 7 nguyên tố cùng nhau nên 19 ( x3 − 2 ) cũng không chia
hết cho 7. Vậy (1) không có nghiệm nguyên.
Bài 3.
1 Do 0 < a < b nên bất đẳng thức
a+b+c
> 3 ⇒ a + b + c > 3 ( b − a ) ⇒ 4a + c > 2b
b−a
b2 b2 b2
Vì phương trình ax + bx + c =
2
0 vô nghiệm nên c > ⇒ 4a + c > 4a + 2 4a ⋅ =
2b
4a 4a 4a
a+b+c
Từ đó suy ra (1) đúng và như vậy > 3.
b−a
2 Ta có
x2 y2 z2 x2 y2 z2
P= + +  + + =1 
x 2 + 2 yz y 2 + 2 zx z 2 + 2 xy x 2 + y 2 + z 2 y 2 + z 2 + x 2 z 2 + x 2 + y 2
 
Dấu "=" đạt được khi và chỉ khi 2 yz =
y 2 + z 2 , 2 zx =
z 2 + x 2 , 2 xy =x2 + y 2 ⇒ x = y = z .
Vậy P đạt min bằng 1 , đạt được khi và chỉ khi x= y= z .
Bài 4. Ta điền vào các ô vuông các số theo quy tắc sau:
Điền vào ô ( m, n ) số f ( m, n ) = i nếu n − m = 5k + i trong đó k ∈  còn i là một trong 5 số:
i ( mod5 ) . Khi đó:
0,1, 2,3, 4 , tức là n − m =
a) Trên bảng thu được, trong 5 ô liên tiếp của một hàng hay một cột đều có 5 số khác nhau là
0,1, 2,3, 4 .
b) Trên mỗi hàng có 2000 ô nên trên bảng thu được số ô chứa số 0 bằng số ô chứa số 1 , bằng số
ô chứa số 2 , bằng số ô chứa số 3 , bằng số ô chứa số 4.
c) Vì q − p = ( q + 1) − ( p + 1) = ( q + 2 ) − ( p + 2 ) = ( q + 3) − ( p + 3) = ( q + 4 ) − ( p + 4 ) , nên 5 ô
tô lần thứ nhất chứa cùng một số, giả sử đó là số i. Từ lần thứ hai trở đi mỗi lần tô 5 ô chứa 5 số
khác nhau và sau một lần tô bất kỳ (kể từ lần thứ hai) số ô được tô chứa số i nhiều hơn các ô
được tô chứa mỗi số còn lại 5 ô. Do đó không thể sau một lần tô nào ta có thể tô được hết các ô
vuông con của bảng đã cho (theo b)).
Bài 5. Gọi bán kính của các đường tròn ε1 , ε 2 , ε 3 là R và các tâm của chúng là O1 , O2 , O3 tương
ứng thì O1O2 O3 là tam giác đều có cạnh bằng 2R . Gọi tâm của ε là O thì do
OO1= OO2= OO3= R + r nên O là trọng tâm (trực tâm...) của O1O2 O3 .

Liên hệ tài liệu word môn toán: 039.373.2038 TÀI LIỆU TOÁN HỌC
Website:tailieumontoan.com

Do
= đó OO1
2
=
3
O2 O3
2
3 O2 O3
3
hay R + r=
2R
3
⇒ R=
3
2− 3
r= (3 + 2 3 ) r
Gọi các tiếp điểm của ε 2 , ε 3 với cạnh BC là M , N tương ứng thì O2 M và O3 N vuông góc với
BC và O2 M= O3 N= R ⇒ O2 MNO3 là hình chũ̃ nhật và =
MN O=
2 O3 2R .

= R 3 (  BMO2 và CNO3 vuông góc và có ∠=


= CN
Dễ thấy BM O2 BM ∠=
O3 CN 30 . )
( )(
BC = BM + MN + NC = 2 3R + 2 R = 2 3 + 2 3 + 2 3 r = 18 + 10 3 r ) ( )
(
Vậy cạnh của tam giác đều ABC là 18 + 10 3 r . )

ĐẠI HỌC QUỐC GIA HÀ NỘI ĐỀ THI TUYỂN SINH LỚP 10


TRƯỜNG ĐẠI HỌC KHOA HỌC TỰ NHIÊN HỆ THPT CHUYÊN NĂM 1997

MÔN : TOÁN (Vòng 1)


Thời gian làm bài: 150 phút (không kể thời gian phát đề)

Bài 1.

=x
3

=
(
10 + 6 3 3 − 1 ) 3
(10 + 6 3 ) ( 3 − 1)3

6+2 5 − 5 ( 5 + 1) 2 − 5


3
(
10 + 6 3 6 3 − 10
= 3=
)(
8 2
)
1
⇒ x3 − 4 x + 1 =
   1

( )
1997
⇒ P = x3 − 4 x + 1
  = 1
Bài 2.
x+3 + x+8 =5 x
Điều kiện: x0 . Bình phương hai vế của (1) và giản ước ta được (1) tương đương với.
4 ( x + 3)( x + 8 )= (23 x − 11) 2

2 x + 3 ⋅ x + 8= 23 x − 11 ⇒  11
x ≥
 23

Liên hệ tài liệu word môn toán: 039.373.2038 TÀI LIỆU TOÁN HỌC
Website:tailieumontoan.com

1
Giải (2) ta được hai nghiệm x = 1 và x = (loại vì không thỏa mãn (3). Vậy phương trình có
21
một nghiệm là x = 1 .
Bài 3.
 2 xy = x + y + 1  4 xy = 2 x + 2 y + 2
 
2 yz = y + z + 7 ⇒ 4 yz = 2 y + 2 z + 14
 2 xz = z + x + 2  4 xz = 2 z + 2 x + 4
 
 ( 2 x=− 1)( 2 y − 1) 3  ( 2 x=
− 1)( 2 y − 1) 3
 
⇒ ( 2 y − 1)( 2 z − 1) = 15 ⇒  ( 2 y − 1)( 2 z − 1) = 15
 ( 2 z − 1)(=2 x − 1) 5 
 [( 2 x − 1)( 2 y − 1)( 2=z − 1)]2 152

(3) tương đương với ( 2 x − 1)( 2 y − 1)( 2 z − 1) =±15

2 x − 1 =1

Nếu ( 2 x − 1)( 2 y − 1)( 2 z − 1) =
15 thì từ (1) và ( 2 ) ta suy ra 2 y − 1 =3
2 z − 1 =
 5
Do đó hệ có nghiệm là:=
x 1,=
y 2,=
z 3
2 x − 1 =−1

Nếu ( 2 x − 1)( 2 y − 1)( 2 z − 1) =−15 thì từ (1) và ( 2 ) ta suy ra 2 y − 1 =−3
2 z − 1 =−5

Do đó hệ có nghiệm là: x =
0, y =
−1, z =
−2
(1, 2,3)
Vậy hệ đã cho có 2 nghiệm là: ( x, y, z ) = 
( 0, −1, −2 )
Bài 4.
• 16 là số chính phương.
Với n = 0 thì 2n + 15 =
• 17 không phải là số chính phương.
Với n = 1 thì 2n + 15 =
• Với n2 thì 2n chia hết cho 4 , như vậy 2n + 15 chia cho 4 dư 3 .
Dễ thấy mọi số chính phương m dều chia hết cho 4 hoặc chia cho 4 dư 1. Thật vậy, nếu số chính
phương m là bình phương m là bình phương của số chã̃n thì rõ ràng m : 4 . Còn nếu m là bình
phương của số lẻ tức m = (2k + 1) 2 = 4k 2 + 4k + 1( k ∈  ) thì m chia cho 4 dư 1 .

Do đó với n2 số 2n + 15 không thể là số chính phương (vì 2n + 15 chia cho 4 dư 3).
Vậy chỉ với n = 0 thì 2n + 15 là số chính phương.
Bài 5.

Liên hệ tài liệu word môn toán: 039.373.2038 TÀI LIỆU TOÁN HỌC
Website:tailieumontoan.com

1 Gọi các tiếp điểm của ( O, R ) và ( O ′, R ′ ) với cạnh BC lần lượt là M , N thì OM và O ′N
vuông góc với BC . Do ABC dều nên ∠ OBM = ∠ O ′CN = 30 . Do đó
BM = R 3, CN = R ′ 3 ⇒ MN = BC − BM − CN = 1 − 3 ( R + R ′ )
Trong hình thang vuông OMNO ′ ta luôn có
MN OO ′  ⇒ 1 − 3 ( R + R ′ ) R + R ′ ⇒ ( )
3 + 1 ( R + R ′ ) 1

1 3 −1
 ⇒ R + R ′ =
3 +1 2
3 −1
Dấu đẳng thức có được khi và chỉ khi OO ′  MN ⇒ OM = O ′N hay R
= R=′ .
4
3 −1 3 −1
Vậy R + R ′ = R=′
. Dấu đẳng thức có được khi R .
2 4
2 Gọi S và S ′ là diện tích các hình tròn ( O, R ) và ( O ′, R ′ ) ta có
2
π π  3 −1 2− 3
(
S + S=′ π R + R2 '2
) 2 ( R + R′) 2
 
2 2
=
4
π

3 −1
= R=′
Các bất đẳng thức trở thành đẳng thức khi và chỉ khi R .
4
2− 3 3 −1
Vậy S + S ′ đạt min bằng = R=′
π , đạt được khi R .
4 4

Liên hệ tài liệu word môn toán: 039.373.2038 TÀI LIỆU TOÁN HỌC
Website:tailieumontoan.com

ĐẠI HỌC QUỐC GIA HÀ NỘI ĐỀ THI TUYỂN SINH LỚP 10


TRƯỜNG ĐẠI HỌC KHOA HỌC TỰ NHIÊN HỆ THPT CHUYÊN NĂM 1997

MÔN : TOÁN (Vòng 2)


Thời gian làm bài: 150 phút (không kể thời gian phát đề)

Bài 1. Giải hệ phương trình


 y 3 + y 2 x + 3x − 6 y =
0
 2
 x + xy = 3
Hệ đã cho tương đương với
 3  y 2 ( x + y ) + 3x − 6 y =
0
 y + y 2
x + 3 x − 6 y =
0 
 ⇒ 
x( x + y) =
3
 3  x+ y =
  x
 23
 y x + 3 x − 6 y =0 3 y 2 + 3 x 2 − 6 xy = 0

 ⇒  ⇒ 3
 3  x+ y =
x+ y =  x
 x
(3) tương đương với 3( x − y ) 2 = 0 ⇒ x = y . Thay vào (4) ta được:
3 3 3
2 x = ⇒ x2 = ⇒ x =±
x 2 2
3 3
Vậy nghiệm của hệ là: x= y= và x = y = −
2 2
Bài 2.
Liên hệ tài liệu word môn toán: 039.373.2038 TÀI LIỆU TOÁN HỌC
Website:tailieumontoan.com

1 Với y chẵn thì 1992 x1993 + 1993 y1994 là số chẵn ( x ∈  ) . Do đó phương trình
1992 x1993 + 1993 y1994 =
1995 không có nghiệm nguyên với y chẵn.
2 = 2k + 1 , khi đó
Với y lẻ thì y 997 là số lẻ, giả sử y1997

( ) ( )
2
1993 y=
1994
1993 y 997 = 1993(2k +=
1) 2 1993 4k 2 + 4k + 1

( )
= 4 1993 k 2 + k  + 1993

Nên 1993y1994 chia 4 dư 1 suy ra 1992 x1993 + 1993 y1994 chia 4 dư 1 (vì 1992:4). Trong khi đó
1995 chia cho 4 dư 3 nên phương trình đã cho không có nghiệm nguyên với y lẻ.
Vậy phương trình đã cho không có nghiệm x, y nguyên.
Bài 3. Giả sử 1997 viết được dưới dạng tổng của n hợp số a1 , a2 , …, an :
1997 = a1 + a2 +  + an , nhưng không viết được dưới dạng tổng của n + 1 hợp số. Khi đó dễ
thấy:
1 Mỗi hợp số ai ( i= 1, …, n ) không viết được dưới dạng tổng của hai hợp số.
2 Tổng của hai hợp số ai , ak không viết được dưới dạng tổng của ba hợp số.
Do 1997 là lẻ nên trong số a1 , …, an phải có một hợp số là lẻ và nếu ai là hợp số lẻ thì nó phải là
9 . Thật vậy: 1,3,5, 7,11,13 không phải là hợp số còn nếu ai15 thì ai =9 + ( ai − 9 ) , trong đó 9
là hợp số còn ai − 9 là số chẵn 6 nên nó cũng là hợp số, như vậy ai viết được dưới dạng tổng
của hai hợp số, trái với giả thiết thứ nhất.
Ngoài ra không thể có quá một hợp số bằng 9 , vì nếu có hai trường hợp số bằng 9 thì tổng của
chúng bằng 18 là tổng của ba hợp số (18 = 6 + 6 + 6 = 4 + 6 + 8 = ), trái với giả thiết thứ hai.
Vậyt trong số a1 , a2 , …, an có đúng một hợp số lẻ và số đó là 9 . Không mất tổng quát, giả sử
a1 = 9 . Khi đó
1997 =9 + a1 + a2 +  + an ⇒ a1 + a2 +  + an =1988
trong đó a1 , a2 , …, an là các hợp số chẵn.
Dễ thấy hợp số chẵn chỉ có thể là 4 hoặc 6 . Vì nếu ai là hợp số chẵn 8 thì ai =4 + ( ai − 4 ) ,
trong đó 4 và ai − 4 (là số chẵn 4 ) đều là hợp số, trái với nhận xét thứ nhất.
Số hợp số bằng 6 không quá 1 , vì nếu có 2 hợp số bằng 6 thì tổng 2 hợp số đó bằng
12 = 4 + 4 + 4 là tổng của 3 hợp số, trái với nhận xét thứ hai.
Nhưng nếu có đúng một hợp số bằng 6 còn các hợp số còn lại bằng 4 thì tổng a2 + a3 +  + an
()
không chia hết cho 4 mà 1988 : 4 , trái với giả thiết  * . Do đó không có hợp số bằng 6 .

Vậy a2 + a3 +  + an đều bằng 4 và từ (*) ta có:

Liên hệ tài liệu word môn toán: 039.373.2038 TÀI LIỆU TOÁN HỌC
Website:tailieumontoan.com

 ( n − 1) =
4 1988 ⇒ =
n 498
 
 1997
 = 9 + 4
+4 
+ .... +4 
 
 498 soá 4 
Bài 4. Dễ dàng chứng minh được hai kết luận sau:
1 1 1
a) Với x, y, z > 0 ta có ( x + y + z )  + +  9 , dấu "=" đạt được khi và chỉ khi x= y= z .
x y z
b) Trong ABC với các chiều cao ha , hb , hc và bán kính đường tròn nội tiếp bằng r ta có:
1 1 1 1
+ + =
ha hb hc r
Chứng minh.
x y x y x y
a) Trước tiên dễ thấy + 2 2 , dấu "=" đạt được khi = ⇒ x = y ( x, y > 0) . Áp
=
y x y x y x
dụng ta có:
1 1 1  x y  y z  z x 
( x + y + z) + +  = 3  +   +   + 
x y z  y x  z y  x z 
 3 + 2 + 2 + 2 =9
dấu "=" đạt được khi và chỉ khi x= y= z .
aha
b) Từ công thức tính diện tích tam giác S= = p ⋅ r ( p là nửa chi vi tam giác) ta có:
2
1 a
=
ha 2 pr

Tương tự
1 b 1 c
= = ,
hb 2 pr hc 2 pr
1 1 1 a+b+c 1
 ⇒ + =+ =
ha hb hc 2 pr r
1 1 1
Bây giờ ta xét M = + +
ha + 2hb hb + 2hc hc + 2ha
Ta có

Liên hệ tài liệu word môn toán: 039.373.2038 TÀI LIỆU TOÁN HỌC
Website:tailieumontoan.com

 1 2  1 1 1 
(ha
+ 2hb )  +  = ( ha + hb + hb )  + +  9
 ha hb   ha hb hb 
1 1 1 2
⇒   +    daáu "=" đñaït ñöôïc ⇒ ha hb
=
ha + 2hb 9  ha hb 

1 1 1 2
Tương tự   +    daáu"=" đñaït ñöôïc ⇒ hb =
hc
hb + 2hc 9  hb hc 

1 1 1 2
  +    daáu "=" đñaït ñöôïc ⇒ hc =
ha
hc + 2ha 9  hc ha 
Cộng ba bất đẳng thức cuối ta được
1 1 1 1 1 1 1
M   + +  = ⋅ =   vì  ( r = 1)
3  ha hb hc  3 r 3
dấu "=" đạt được khi và chỉ khi ha = hb = hc ⇒ a = b = c suy ra ABC đều.
1
Vậy M đạt max bằng , đạt được khi và chỉ khi ABC đều, (khi đó h= a h=
b h=
c 3) .
3
Bài 5. Vì 16 điểm được tô bằng 3 màu nên phải có ít nhất 6 điểm được tô cùng một màu. Giả sử
6 điểm đó là A, B, C , D, E , F 4 . Xét 5 đoạn thẳng AB, AC , AD, AE , AF . Vì 5 đoạn đó được tô bởi
hai màu tím và nâu, nên có ít nhất 3 đoạn được tô cùng màu, giả sử đó là ba đoạn AB, AD, AF
và chúng được tô bằng màu tím. Khi đó nếu một trong 3 đoạn BD, DF , FB cũng được tô bằng
màu tím, chẳng hạn đoạn BD , thì ta được ABD có ba đỉnh cùng màu và ba cạnh cùng màu
tím. (Nếu DF hoặc FB được tô màu tím ta lý luận tương tự).
Nếu cả 3 đoạn BD, DF , FB đều không được tô màu tím thì cả 3 đoạn phải được tô màu nâu và
khi đó BDF có ba đỉnh cùng màu và ba cạnh cùng màu nâu.
Vậy ta luôn tìm được một tam giác thoả mãn điều kiện bài toán.

Liên hệ tài liệu word môn toán: 039.373.2038 TÀI LIỆU TOÁN HỌC
Website:tailieumontoan.com

ĐẠI HỌC QUỐC GIA HÀ NỘI ĐỀ THI TUYỂN SINH LỚP 10


TRƯỜNG ĐẠI HỌC KHOA HỌC TỰ NHIÊN HỆ THPT CHUYÊN NĂM 1996

MÔN : TOÁN (Vòng 1)


Thời gian làm bài: 150 phút (không kể thời gian phát đề)

Bài 1.
2
 1   3 1 
6 3 2
 1  6 1  x+   −x + 3 
 x +  −  x + 6  − 2  x    x 
P   =    
x x
= 3 3
 1 1  1  3 1 
x+  + x + 3 x+  +x + 3 
3

 x x  x  x 
3
 1  1   1 1
=  x +  −  x 3 + 3  = 3  x +  6 x ⋅ = 6
 
 x  x   x x
1
Suy ra P6 . Dấu "=" đạt được khi và chỉ khi x = ⇔ x = 1( x > 0) . Vậy P đạt giá trị bé nhất
x
bằng 6 , đạt được khi x = 1 .
Bài 2.
 1 1
 + 2− 2
=
 x y 1 1
   Ñieàu kieän:  x , y ≥
 1 1 2 2
+ 2− =2
 y x

Liên hệ tài liệu word môn toán: 039.373.2038 TÀI LIỆU TOÁN HỌC
Website:tailieumontoan.com

1 1
Đặt
= u = ,v khi đó hệ có dạng
x y

u + 2 − v 2 =2  2 − v 2 =2 − u
 
 ⇒
v + 2 − u =2  2 − u 2 =2 − v
2

Điều kiện: u ≤ 2, v ≤ 2 khi đó hệ tương đương với


2 − v 2 =4 − 4u + u2 u + v
2 2
− 4u + 2 =0
 2 2
⇒  2 2
2 − u =4 − 4v + v u + v − 4v + 2 =0
 4 ( u − v ) =
0  u=v
⇒  2 ⇒ 2
2
u + v − 4v + 2 = 0  2u − 4u + 2 =0
⇒ u=
v=1  (thoûa maõn u ≤ 2, v ≤ 2)
Từ đó ta có: x= y= 1 là nghiệm của hệ đã cho.
Bài 3.
n 3 + 5n = n 3 − n + 6 n = ( n − 1) n ( n + 1) + 6n
Do 6n     6 còn ( n − 1) n ( n + 1) là ba số nguyên liên tiếp nên trong chúng có một số chia hết cho 2
và một số chia hết cho 3, do vậy ( n − 1) n ( n + 1)    6 . (Chú ý rằng 2 và 3 nguyên tố cùng nhau).

Vậy n3 + 5n     6 với mọi n nguyên dương.


Bài 4.
a3 a3
+ ab2 ⋅ ab =
2a 2
b b
a3
Dấu "=" đạt được khi và chỉ khi = ab ⇒ a 2 = b 2 ⇒ a = b(a > 0, b > 0) . Vậy
b
a3
+ ab2a2 ,   daáu "=" đñaït ñöôïc ⇔ a =b
b
Tương tự
b3
+ bc2b2 ,   daáu "=" đñaït ñöôïc  ⇔ b = c
c
c3
+ ca2c 2 ,   daáu "=" đñaït ñöôïc ⇔ c =a
a
Ngoài ra
( )
2 a2 + b2 + c 2 2 ( ab + bc + ca )
 daáu "=" ñaït ñöôïc khi vaø chæ khi a= b= c

Liên hệ tài liệu word môn toán: 039.373.2038 TÀI LIỆU TOÁN HỌC
Website:tailieumontoan.com

Cộng 4 bất đẳng thức cuối và giản ước ta được


a3 b3 c3
+ + ab + bc + ca,   daáu "=" đñaït ñöôïc ⇔ a = b = c
b c a
Chú ý: Bất đẳng thức (1) chứng minh dễ dàng bằng cách chuyển vế và đưa về tổng các bình
phương.
Bài 5.
1 Đặt S = MN 2 + NP 2 + PQ 2 + QM 2 ta có
S  = BM 2 + BN 2 + CN 2 + CP 2 + DP 2 + DQ 2 + AQ 2 + AM 2
 = ( AM 2
) ( ) ( ) (
+ BM 2 + BN 2 + CN 2 + CP 2 + DP 2 + DQ 2 + AQ 2 )
(a) Do
( AM + BM ) 2 a 2
AM 2 + BM 2 = , dấu "=" đạt được ⇔ AM =
BM
2 2
a2 a2 a2
Tương tự BN 2 + CN 2 , CP 2 + DP 2 , DQ 2 + AQ 2 , các dấu "=" tương ứng đạt được
2 2 2
=
khi và chỉ khi =
BN CN , CP DP, DQ = AQ . Do đó từ (1) ta có S2a 2 , dấu "=" đạt được khi
và chỉ khi M , N , P, Q là trung điểm các cạnh tương ứng chứa chúng.
(b)
AM 2 + BM 2( AM + BM )2 =a2 ,   daáu "=" đñaït ñöôïc 
 ⇔ AM = 0 hoaëc BM =⇔ 0 M   töông öùng A  hoaëc B.
Tương tự
BN 2 + CN 2a 2 , dấu "=" đạt được khi N trùng với B hoặc C
CP 2 + DP 2a 2 , dấu "=" đạt được khi P trùng với C hoặc D
DQ 2 + AQ 2a 2 , dấu "=" đạt được khi Q trùng với D hoặc A
Do đó tù̀ (1) ta có S4a 2 , dấu "=" đạt được khi M , N , P, Q trùng với một trong hai đầu mút của
các đoạn thẳng chứa chúng.
2 Nếu N , P, Q thuộc các cạnh BC , CD, DA tương ứng mà MNPQ là hình vuông thì
∠ AMQ + ∠ BMN = =
90 
nên ∠ AMQ ∠= BNM (∠ AQM ∠ BMN ) suy ra
 AMQ=  BMN ⇒ AM = BN . Tương tự ta có BN
= CP = AM . Do đó nếu
= DQ
= CP
MNPQ là hình vuông thì BN = DQ= AM
Ngược lại dễ thấy nếu BN
= CP
= DQ
= AM thì bốn tam giác vuông AMQ, BNM , CPN , DQP
= NP
bằng nhau suy ra MN = PQ = QA , ngoài ra ∠ AMQ + ∠ BMN =⇒
90 ∠ QMN =
90 và
do đó MNPQ là hình vuông.

Liên hệ tài liệu word môn toán: 039.373.2038 TÀI LIỆU TOÁN HỌC
Website:tailieumontoan.com

= CP
Vậy: MNPQ là hình vuông khi và chỉ khi BN = DQ
= AM .

ĐẠI HỌC QUỐC GIA HÀ NỘI ĐỀ THI TUYỂN SINH LỚP 10


TRƯỜNG ĐẠI HỌC KHOA HỌC TỰ NHIÊN HỆ THPT CHUYÊN NĂM 1996

MÔN : TOÁN (Vòng 2)


Thời gian làm bài: 150 phút (không kể thời gian phát đề)

Bài 1.
x − 1 + 1)3 + 2 x − 1 = 2 − x
⇒ x − 1 + 1)3 + ( x − 1) + 2 x − 1 + 1 − 2 =0

⇒ x − 1 + 1)3 + ( x − 1 + 1) 2 − 2 =0
Đặt x −1 +1 =t thì phương trình có dạng
t3 + t2 − 2 =0
(
⇒  ( t − 1) t 2 + 2t + 2 =
0)
⇒ t=
1
Hay x −1 +1 = 1 ⇒ x = 1
Vậy x = 1 là nghiệm của phương trình đã cho.
Bài 2.
x − y = 1

y − z = 1

 z − x =
1

Liên hệ tài liệu word môn toán: 039.373.2038 TÀI LIỆU TOÁN HỌC
Website:tailieumontoan.com

 Ñieàu kieän: x , y, z > 0
Nếu ( x, y, z ) là nghiệm thì x= y= z . Thật vậy giả sử x là số bé nhất trong ba số x, y, z . Khi đó
do xy nên từ (1) và ( 2 ) ta có yz . Khi đó từ (2) và (3) suy ra zx . Vậy x= y= z . (Trường
hợp y hoặc z là số bé nhất chứng minh tương tự).
Từ nhận xét trên ta suy ra hệ (1) , ( 2 ) , ( 3) tương đương với hệ

 x= y= z

=
x x +1
Giải (5) ta được
1+ 5 3+ 5
=
x ⇒
= x
2 2
Vậy nghiệm của hệ đã cho là x= y= z= 3 + 5 2.
Bài 3. Từ giả thiết x + y =201 và x, y nguyên dương ta có 1x, y200 và=
y 201 − x

( ) ( )
P = x x 2 + y + y y 2 + x = x3 + y 3 + 2 xy = ( x + y )3 − 3 xy ( x + y ) + 2 xy

Thay x + y =201 ta được=


P 2013 − 601xy .
Ta chứng minh 200xy100.101 .
Thật vậy,
xy − 200 = x ( 201 − x ) − 200 = 200 x + x − x 2 − 200 =
 = x ( 200 − x ) + ( x − 200 ) = ( x − 1)( 200 − x ) 0,   Vì 1
(   x200 )
Vậy xy200 , dấu "=" đạt được khi và chỉ khi
= =
 x 1  và   y 200
= =
 x 200  và   y 1
Do đó P2013 − 601.200 =
8000401 , dấu "=" đạt được khi và chỉ khi
= x 1,=
y 200
= =
 x 200, y 1
Suy ra P đạt giá trị lớn nhất bằng 8000401.
Tiếp tục ta có
xy 100.101 − x ( 201 −=
100.101 −= x ) 100.101 − 101x − 100 x +=x2
= 101(100 − x ) − x (100 − x )= (100 − x )(101 − x ) 0 ⇒ xy100.101
Dấu "=" đạt được khi
 x 101,
= = y 100
 ⇒ P  đñaït giaù trò beù nhaát baèng 2050501
 x 100,
= = y 101
Liên hệ tài liệu word môn toán: 039.373.2038 TÀI LIỆU TOÁN HỌC
Website:tailieumontoan.com

Chú ý: Có thể giải bài toán bằng cách xét khoảng tăng, giảm của hàm số
f ( x) =xy =x ( 201) = − x 2 + 201x trên [1; 200] . Từ đó suy ra giá trị lớn nhất và bé nhất của
f ( x ) với x nguyên, x ∈ [1; 200] và giá trị lớn nhất, bé nhất của P .

Bài 4. Vẽ đường tròn tâm O , đường kính BC = 2 R . Do khoảng cách từ O đến ( d ) bé hơn
BC
R= nên đường tròn đó cắt ( d ) tại hai điểm A1 , A2 và đường tròn ngoại tiếp A1 BC và
2
BC
A2 BC có bán kính bằng R = .
2
1 Trên ( d ) ta lấy điểm A ≠ A1 và A ≠ A2 . Khi đó A không thuộc đường tròn đường kính
BC nên ∠ BAC ≠ 90 suy ra BC là một dây không phải là đường kính của đường tròn
( O ′, R ′ ) ngoại tiếp ABC ⇒ 2 R ′BC =
2 R hay R ′ > R .
BC
Vậy bán kính đường tròn ngoại tiếp ABC đạt giá trị bé nhất bằng , đạt được khi A trùng
2
với A1 hoặc A2 .
Chú ý: Có thể giải câu (1) cách khác như sau:
Do tâm I của đường tròn ngoại tiếp ABC chạy trên đường trung trực của BC nên bán kính
BC
của đường tròn đó là IBOB = , dấu "=" đạt được khi và chỉ khi I ≡ O . Do đó đường tròn
2
có bán kính bé nhất là đường tròn đường kính BC và điểm A phải tìm là một trong hai điểm
A1 , A2 nói trên.
2 Giả sử ha , hb , hc là độ dài các đường cao hạ từ A, B, C dến BC , CA, AB tương ứng và h
là khoảng cách giữa ( d ) và BC thì ha = h không đổi nên diện tích ABC có diện tích
1
S= BC.h không đổi. Ta có AB ⋅ AC2 S , dấu "=" đạt được khi và chỉ khi
2
∠ BAC = 90 . Do đó
2S 2S
ha ⋅ hb ⋅ hc =h ⋅ ⋅ h.2 S = BC ⋅ h 2
AC AB
Dấu "=" đạt được khi ∠ BAC = 90 nghĩa là A là giao điểm của ( d ) với đường tròn đường kính
BC tức là A trùng với A1 hoặc A2 .
Vậy ha ⋅ hb ⋅ hc đạt giá trị lớn nhất bằng BC.h 2 , đạt được khi A trùng với A1 hoặc A2 .
Bài 5. (Dành cho chuyên toán)
Trước tiên ra chứng minh rằng: Với mọi số x1 , x2 , y1 , y2 ta có

Liên hệ tài liệu word môn toán: 039.373.2038 TÀI LIỆU TOÁN HỌC
Website:tailieumontoan.com

x12 + x22 + y12 + y22  ( x1 + y1 ) + ( x2 + y2 )


2 2

Thật vậy bình phương hai vế và rút gon ra được (1) tương đương với

(x2
1 + x22 )( y 2
1 )
+ y22 x1 y1 + x2 y2

Nếu x1 y1 + x2 y2 < 0 thì (2) hiển nhiên đúng. Với x1 y1 + x2 y20 thì lại bình phương hai vế và rút
gon ta được (2) tương đương với
)
x12 y22 + x22 y12 2 x1 y1 x2 y2 ⇒ ( x1 y2 − x2 y1 ) 0
2

Bất đẳng thức cuối cùng luôn đúng, vậy (1) đúng.
Áp dụng hai lần bất đẳng thức (1) ta được

x12 + x22 + y12 + y22 + z12 + z22  ( x1 + y1 + z1 ) + ( x2 + y2 + z2 )


2 2

3
Trở lại bài toán, với x, y, z > 0 và x + y + z , áp dụng (3) ta có
2
1 1 1
 P = x2 + 2
+ y2 + 2 + z2 + 2
x y z
2
1 1 1
  ( x + y + z ) +  + +  2

x y z
2
1 1 1
= [4 ( x + y + z )] +  + +  − 15( x + y + z ) 2
2

x y z
1 1 1
 ( x + y + z )  + +  − 15( x + y + z ) 2 =Q
 8
x y z
1 1 1 3
Do x, y, z > 0 nên ( x + y + z )  + +  9 và do 0 < x + y + z ≥ suy ra
x y z 2
135
−15( x + y + z ) 2  −
4
Do đó
135 135 3
Q 72 − = = 17
4 4 2
3 1
Vậy P 17 . Có thể thấy dấu "=" đạt được khi và chỉ khi x= y= z =
2 3
Bài 5. (Dành cho chuyên tin)
Ta chứng minh rằng không thể chuyển được tất cả các viên bi vào một hình quạt được.
Ta tô màu đen 7 hình quạt như hình vẽ và gọi 7 hình quạt còn lại là hình quạt trắng.

Liên hệ tài liệu word môn toán: 039.373.2038 TÀI LIỆU TOÁN HỌC
Website:tailieumontoan.com

Ở thời điểm ban đầu số bi ở các hình quạt đen và số bi ở các hình quạt trắng là số lẻ (cùng bằng
7). Dễ thấy qua một bước biến đổi số bi ở các hình quạt trắng hoặc giữ nguyên, hoặc tăng thêm
hai hoặc giảm đi hai viên. Do đó, sau mỗi bước biến đổi bất kỳ số bi ở các hình quạt trắng và số
bi ở các hình quạt đen đều là số lẻ. Vì vậy không thể chuyển được cả 14 viên bi vào cùng một
hình quạt sau một bước nào cả.

ĐẠI HỌC QUỐC GIA HÀ NỘI ĐỀ THI TUYỂN SINH LỚP 10


TRƯỜNG ĐẠI HỌC KHOA HỌC TỰ NHIÊN HỆ THPT CHUYÊN NĂM 1995

MÔN : TOÁN (Vòng 1)


Thời gian làm bài: 150 phút (không kể thời gian phát đề)

Bài 1.

2 x 2 − y 2 =1 2 x − y =
2 2
1
 ⇒ 
 xy + x =
2
2 2
(
 xy + x= 2 2 x − y
2 2
)
Từ (2) suy ra

 y=x
2 y + xy − 3 x =0 ⇒ ( y − x )( 2 y + 3 x ) =0 ⇒ 
2 2
y = − 3 x
 2

Với y = x thay vào (1) ra được phương trình: x 2 = 1

Do đó hệ có nghiệm là x = y = ±1

3 x2
Với y = − x thay vào (1) ta được phương trình: − =
1 (vô nghiệm)
2 2

Liên hệ tài liệu word môn toán: 039.373.2038 TÀI LIỆU TOÁN HỌC
Website:tailieumontoan.com

Vậy hệ đã cho có hai nghiệm: x= y= 1 và x = y = −1 .

Bài 2.

1− x + 4 + x =3

Điều kiện: −4x1 . Khi đó (2.4) tương đương với

1 − x + 4 + x + 2 (1 − x )( 4 + x ) =9

⇒ 4 − 3x − x 2 =
2
⇒ 4 − 3x − x =
2
4
 x=0
⇒ x 2 + 3 x =0 ⇒ 
 x = −3

Cả hai nghiệm đều thoả mãn điều kiện.

Vậy phương trình (2.4) có hai nghiệm: x = 0 và x = −3 .

Bài 3.

a + 1 b + 1 a 2 + b 2 + a + b ( a + b) + ( a + b )
2

=+ = −2
b a ab ab

a +1 b +1
Do + ∈  nên
b a

( a + b) 2 + ( a + b )
∈  ⇒ ( a + b) 2 + ( a + b ) =
ka
ab

với k ∈  . Nếu d > 0 là ước số chung của a, b thì


= =
a md , b nd , (n, m ∈
) ⇒ a + b = ( m + n ) d , ab = mnd 2 . Do đó (1) có dạng

  ( m + n) 2 d 2 + ( m + n ) d =
kmnd 2
⇒ m + n=  kmn − (m + n) 2  d= ld ( l ∈  )
⇒ + b ld 2d 2 ⇒ d  a + b
a=

Bài 4. Gọi diện tích của hai hình chữ nhật là S thì ta có:

Liên hệ tài liệu word môn toán: 039.373.2038 TÀI LIỆU TOÁN HỌC
Website:tailieumontoan.com

S S
ab = cd = S ⇒ b = , d =  và 
a c
S  S S S  S
a + b − ( c + d ) = a + −  c +  = a − c −  −  = ( a − c ) 1 − >0
a  c c a  ac

S S
(vì a > c > d nên a − c > 0 và < =
1 ).
ac dc

Vậy a + b > c + d và chu vi hình chữ nhật thứ nhất lớn hơn chu vi hình chữ nhật thứ hai.

Bài 5.

1 Dễ thấy  ABE ∼ AEC nên ta có:


AE AC
= ⇒ AE 2 =AB ⋅ AC
AB AE
⇒ AF = AE = AB ⋅ AC  khoâng ñoåi

Vậy E , F luôn chạy trên đường tròn cố định tâm A , bán kính AB ⋅ AC .

2 Giả sử O ∉ BC và đường thẩng OI cắt cung BC không chứa F tại M . Khi đó do


A, E , F , O, I cùng thuộc đường tròn tâm AO nên trong mọi trường hợp ta có
1
EOM ∠ EFI ≡ ∠ EFE ′ = ∠ EOE ′
∠=
2

Vậy OM là đường phân giác của góc ∠ EOE ′ suy ra OM ⊥ EE ′ hay OI ⊥ EE ′ . Mà OI ⊥ BC


nên EE ′  BC ≡ AB .

Trường hợp O ∈ BC khi đó O ≡ I thì ∠ FEE ′ = 90 mà FE ⊥ BC nên EE ′  BC .

3 Giả sử O ≠ I . Gọi giao điểm của BC và EF là P thì đường tròn ngoại tiếp ONI là
đường tròn đường kính OP ∠= (
PNO ∠= )
PIO 90 . Dễ thấy
AB ⋅ AC
AP ⋅ AI = AN ⋅ AO = AE 2 = AB ⋅ AC ⇒ AP = không đổi, mà P thuộc tia AB
AI
cố định nên P cố định.
Gọi trung điểm của PI là K thì K cố định và tâm O ′ của đường tròn đường kính OP (tức
đường tròn ngoại tiếp ONI ) luôn nằm trên đường thẳng d cố định vuông góc với BC ở K .

Liên hệ tài liệu word môn toán: 039.373.2038 TÀI LIỆU TOÁN HỌC
Website:tailieumontoan.com

ĐẠI HỌC QUỐC GIA HÀ NỘI ĐỀ THI TUYỂN SINH LỚP 10


TRƯỜNG ĐẠI HỌC KHOA HỌC TỰ NHIÊN HỆ THPT CHUYÊN NĂM 1995

MÔN : TOÁN (Vòng 2)


Thời gian làm bài: 150 phút (không kể thời gian phát đề)

Bài 1.

(x + x2 + 3 )( y + y2 + 3 =
3)
Ta có:

(x + )(
x2 + 3 − x + x2 + 3 =
3 )
(y+ y2 + 3 )( − y + y2 + 3) =
3

Nhân (2) với (3) và chia cho (1) ta được:

(−x + )(
x 2 + 3 − y + y 2 + 3 =3 )
(1) ⇒ xy + x y 2 + 3 + y x2 + 3 + x2 + 3 ⋅ y 2 + 3 =3
( 4 ) ⇒ xy − x y 2 + 3 − y x2 + 3 + x2 + 3 ⋅ y 2 + 3 =3
Trừ ( 5 ) cho (6) ta được

x y 2 + 3 + y x2 + 3 =0
Liên hệ tài liệu word môn toán: 039.373.2038 TÀI LIỆU TOÁN HỌC
Website:tailieumontoan.com

Suy ra x, y trái dấu hoặc cùng bằng 0 và

x y 2 + 3 =− y x 2 + 3 ( ) (
⇒ x 2 y 2 + 3 =y 2 x 2 + 3
  )
  ⇒ 3x =3 y ⇒ x =y ⇒ x =
2 2
−y
(vì x, y trái dấu hoặc cùng bằng 0 ) nên E = x + y = 0
Bài 2.
 x + xy=+ y 1 ( x + 1)( y= + 1) 2
 
 y + yz + z = 3 ⇒ ( y + 1)( z + 1) = 4
 z + zx
= +x 7 ( z + 1)( x=+ 1) 8
 
( x + 1)( y + 1) 2
= =( x + 1)( y + 1) 2
 
( y + 1=)( z + 1) 4 ⇒ ( y + 1=
  )( z + 1) 4
[( x + 1)( y + 1)( z + 1)]2 =
64 ( x + 1)( y + 1)( z + 1) =
±8
 
Với ( x + 1)( y + 1)( z + 1) =
8 ta có:
x + 1 = 2, y + 1 = 1, z + 1 = 4 ⇒ x = 1, y = 0, z = 3
Với ( x + 1)( y + 1)( z + 1) =−8 ta có:
x + 1 =−2, y + 1 =−1, z + 1 =−4 ⇒ x =−3, y =−2, z =−5
Vậy hệ có hai nghiệm
 =x 1,= y 0,=
z 3
x =−3, y =−2, z =
−5

Chú ý: Có thể giải bằng cách nhân (1) với (3) rồi chia cho (2) ta được ( x + 1) 2 =4 ⇒ x + 1 =±2
từ đây dễ dàng tìm được các nghiệm.
1 suy ra với 0x, y1 ta có
Bài 3. Từ giả thiết x, y0, x 2 + y 2 =
 x 3x 2
 3 2  1 ⇒ x 3 + y 3x 2 + y 2 =
 y y
(Dễ thấy dấu "=" đạt được khi và chỉ khi=
x 1,=
y 0 hoặc=
x 0,=
y 1 ) Ta có:

( )
3
1=x2 + y 2 =+
x6 3x 4 y 2 + 3x 2 y 4 + y 6

( )
2
2 x3 + y 3 =2 x 6 + 4 x3 y 3 + 2 y 6
Trừ hai đẳng thức cuối ta được

Liên hệ tài liệu word môn toán: 039.373.2038 TÀI LIỆU TOÁN HỌC
Website:tailieumontoan.com

( )
2
2 x3 + y 3 − 1 = x6 + y 6 + 4 x3 y 3 − 3x 4 y 2 − 3x 2 y 4

( ) (
− 3 x 2 y 2 ( x − y ) 2 = ( x − y ) 2  x 2 + xy + y 2 ) − 3x 2 y 2 
2 2
= x3 − y 3
 
 
0  
( x − y ) 2  x 4 + y 4 + 2 x 3 y + 2 xy 3   
 = (
vì x 4 + y 4 + 2 x 3 y + 2 xy 3 > 0 )
( x + y)2  ≤ 2 ( x 2
)
+ y2 ≤ 2 ⇒ x + y ≤ 2

(x ) ( ) ( )
2
≤ ( x + y ) x3 + y 3
2
1  = 2
+ y2 = x x3 + y y 3
1 1
 ⇒ x 3 + y 3 ≥ ≥
x+ y 2
Bài 4. Đặt a = a1 a2 a3 thì b1b2 b3 = 2a . Khi đó

(10 )
2
A = 10
a1 a2 a3 b1b2 b3 a1 a2 a=
3
6
a + 2 ⋅ 103 a + a= 3
+ 1 a= 10012 ⋅ a
=
 7 2
⋅ 112.132 ⋅ a
Vì A viết được dưới dạng A = p12 p22 p32 p42 , trong đó p1 , p2 , p3 , p4 là bốn số nguyên tố khác
nhau, nên ba trong bốn số p1 , p2 , p3 phải là 7,11,13 còn số thứ tư có bình phương bằng a . Do
đó a là bình phương của một số nguyên tố khác 7,11,13 .
1 1000
Chú ý rằng: 100 ≤ a = b< < 500 suy ra a = 17 2 hoặc a = 192 . Vậy có hai số thoả mãn
2 2
điều kiện bài toán là
= =
A 289.578.289,   và  A 361.722.361
Bài 5. Gọi diện tích tam giác IAN , ICN , IBM , IDM lần lượt là S1 , S 2 , S3 , S 4 . Do BM = DM nên
AN S1 S1 S 4 IA.IN ID ⋅ IM IA ID
S3 = S 4 ⇒ = = ⋅ = ⋅ = ⋅
NC S 2 S3 S 2 IB ⋅ IM IC.IN IC IB
Do
ID IA
 IAC ∼ IDB ⇒ =
IB IC
Vậy
AN IA2
=
NC IC 2

Liên hệ tài liệu word môn toán: 039.373.2038 TÀI LIỆU TOÁN HỌC
Website:tailieumontoan.com

ĐẠI HỌC QUỐC GIA HÀ NỘI ĐỀ THI TUYỂN SINH LỚP 10


TRƯỜNG ĐẠI HỌC KHOA HỌC TỰ NHIÊN HỆ THPT CHUYÊN NĂM 1994

MÔN : TOÁN (Vòng 1)


Thời gian làm bài: 150 phút (không kể thời gian phát đề)

Bài 1.
1.
x 4 − 2 x3 − 6 x 2 + 16 x − 8 =0
Phân tích vế trái thành các nhân tử ta được (1)
2
( )
( x − 2) 2 x 2 + 2 x − 2 = 0 ⇔ x = 
 −1 ± 3
Vậy phương trình có ba nghiệm: x =1,  1
x =− + 3 , và x = −1 − 3
2.
x 2 + 2 x +=
4 3 x3 + 4 x
x2 + 4 + 2x − 3 (x 2
)
+4 x =0

Điều kiện:  0
x
Đặt: x 2 += x v thì phương trình có dạng:
4 u,   =

Liên hệ tài liệu word môn toán: 039.373.2038 TÀI LIỆU TOÁN HỌC
Website:tailieumontoan.com

u 2 + 2v 2 − 3uv = 0
( u − v )( u − 2v ) =
  0
v
u=
 2v
Với u = v ta được x2 + 4= x ⇔ x 2 + 4= x vô nghiệm.
Với u = 2v ta được x2 + 4 = 2 x ⇔ x2 + 4 = 4x ⇔ x = 2 .
Vậy phương trình có nghiệm duy nhất: x = 2 .
Bài 2.
=
A xy + 2 zt
(
A2 = xy + 4 zt + 4 xyzt =xy + 4 zt + 2 2 yz ⋅ xt ≤ )

 xy + 4 zt + 2 yz + 2 xt =
9
Từ giả thiết suy ra A3 .
Dấu "=" đạt chẳng hạn khi x= y= z= t = 1 , (thoả mãn xy + 4 zt + 2 yz + 2 xt =
9 ).
Vậy A đạt giá trị lớn nhất bằng 3 .

Bài 3.
 xy − 3 zt = ( )  ⇒  x 2 y 2 − 6 xyzt + 9 z 2 t 2 =
1 1 1
  2 2
 xz + yt =  x z + 2 xyzt + y t =
2 2
2 4
Nhân (4) với 3 và cộng từng vế với (3) ta có
x 2 y 2 + 9 z 2 t 2 + 3x 2 z 2 + 3 y 2 t 2 =
13
Vi x, y, z , t là các số nguyên mà tổng các hệ số của vế trái là 16 nên nếu x, y, z , t thoả mãn hệ đã
cho thì phải có một số bằng 0 .
Nếu x = 0 hoặc y = 0 thì từ (1) ta có: −3 zt =
1 , vô lý.
Nếu z = 0 hệ có dạng
 xy = 1

 yt = 2
Hệ này có hai nghiệm nguyên là: x= y= 1, t= 2 và x ==
y −1, t =
−2 Nếu t = 0 hệ có dạng
 xy = 1

 xz = 2
Hệ này có hai nghiệm nguyên là: x= y= 1, z= 2 và x == −2 Vậy hệ đã cho có 4
y −1, z =
nghiệm nguyên là
x = y = 1, z = 0, t = 2; x = y = −1, z = 0, t = −2
x ==
y 1, t =
0, z =
2; x ==
y −1, t =
0, z =
−2
Liên hệ tài liệu word môn toán: 039.373.2038 TÀI LIỆU TOÁN HỌC
Website:tailieumontoan.com

Bài 4. Gọi tâm đường tròn đã cho là O và trung điểm của AB là I thì OI ⊥ AB và
= BI
AI = AD = DC . Đặt CD = x .
Do AH  OB (cùng  BC ), nên ∠ OBI = ∠ BAH . Từ đó suy ra
AI OA
OBI ∼ BAH  ⇒ =
AH AB
AI . AB 2 x 2
 ⇒ AH= =
OA R
Mặt khác do đường tròn ( O ) tiếp xúc với BC tại B nên

( )
CD ⋅ CA = BC 2 = 4 BH 2 = 4 AB 2 − AH 2 = 16 x 2 − 4 AH 2
⇒ x 2 = 16 x 2 − 4 AH 2
 2
7 2
 ⇒ AH 2 = x
2
4x4 7 2 7 2 R 7
Từ (2.1) và ( 2.2 ) ta suy ra 2= x ⇒ x 2= R ⇒ x=
R 2 8 2 2
R 7
Vậy AD= x=
2 2
Từ (2.1) và (2.3) suy ra
7 49 2 28 2 49 2 7 2
AH = R ⇒ HB 2 =AB 2 − AH 2 =4x2 − R = R − R = R
4 16 8 16 16
Do HE ⋅ HA =
HB 2 nên ta có
HB 2 R 3
HE = = ⇒ AE =AH − HE = R
HA 4 2
Bài 5. Do BC > AC nên ∠ BAC > ∠ ABC . Trong nửa mặt phẳng có bờ là AB và chứa C kẻ
tia Bx sao cho ∠ ABx = ∠ BAC.Bx cắt đường thẳng MN tại P thì M nằm giữa N và P (vì
∠ ABP > ∠ ABM ). Khi đó ABPN là hình thang cân nên ∠ APN = ∠ BNP . Xét AMP ta có:
∠ AMP > ∠ ANM > ∠ BNM ≡ ∠ BNP = ∠ APN ≡ ∠ APM .
Do đó: AM < AP =
BN .

Liên hệ tài liệu word môn toán: 039.373.2038 TÀI LIỆU TOÁN HỌC
Website:tailieumontoan.com

ĐẠI HỌC QUỐC GIA HÀ NỘI ĐỀ THI TUYỂN SINH LỚP 10


TRƯỜNG ĐẠI HỌC KHOA HỌC TỰ NHIÊN HỆ THPT CHUYÊN NĂM 1994

MÔN : TOÁN (Vòng 2)


Thời gian làm bài: 150 phút (không kể thời gian phát đề)

Bài 1.
( x + y )( y + z ) =
4 xy 2 z

( y + z )( z + x ) =
4 yz 2 x
( z + x )( x + y ) =
4 zx 2 y

Rõ ràng x= y= z= 0 là một nghiệm của hệ. Ngược lai, dễ thấy nếu ( x, y, z ) là nghiệm của hệ
mà môt trong ba số x, y, z bằng 0 thì hai số kia cũng bằng 0.
Ta tìm nghiệm thoả mãn x ≠ 0, y ≠ 0, z ≠ 0 .
Ta chứng minh nếu ( x, y, z ) là nghiệm mà x ≠ 0, y ≠ 0, z ≠ 0 thì x= y= z . Thật vậy, nếu
( x, y, z ) thoả mãn (1) , ( 2 ) , ( 3) thì x + y ≠ 0, y + z ≠ 0, z + x ≠ 0 , do đó chia (1) cho (2) ta được
x+ y y
= ⇒ xz + yz = yz + xy ⇒ x ( y − z ) = 0 ⇒ y = z
z+x z
Tương tự, chia (2) cho (3) ta thu được z = x .
Vậy với điều kiện x ≠ 0, y ≠ 0, z ≠ 0 hệ (1) , ( 2 ) , ( 3) tương đương với

Liên hệ tài liệu word môn toán: 039.373.2038 TÀI LIỆU TOÁN HỌC
Website:tailieumontoan.com

 x= y= z
 2 ⇒ x =y =z =
 1 ±
4 x = 4 x
4

Vậy hệ đã cho có ba nghiệm là


 ( 0, 0, 0 )
( x, y, z ) =  (1,1,1)
( −1, −1, −1)

Bài 2.
( )
12 x 2 + 6 xy + 3 y 2= 28 ( x + y ) ⇒ 3 4 x 2 + 2 xy + y 2 = 28 ( x + y )
3k với k ∈  . Từ (1) suy ra
Do 3 và 28 nguyên tố cùng nhau nên x + y :3 hay x + y =
3 x 2 + ( x + y )2 = 28k ⇒ 3 x 2 + 9k 2 = 28k ⇒ k : 3  hay  k = 3n ( k ∈  )
⇒ x 2 + 3k 2 = 28n   mà  k = 3n ⇒ x 2 + 27n2 = 28n ⇒ x 2 = n ( 28 − 27n ) 0
 28  28
⇒ n  − n  0 ⇒ 0n= ⇒ n 0  hoaë
= c n 1
 27  27
x = 0
Với n = 0 ⇒ k = 0 ⇒  ⇒ x = y =0
 
x + y =
0
 x2 = 1 =x 1;= y 8
Với n =1 ⇒ k =3 ⇒  ⇒
x + y = 9 x =−1; y =
10
Vậy phương trình đã cho có ba nghiệm nguyên là
x= y= 0; x =
1, y =
8  và  x =
−1, y =
10
n n ! (đọc là n giai thừa). Ta có
= 1.2.3…=
Bài 3. Ký hiệu A
n ( n + 1)
B =1 + 2 + 3 +  + n = ( n3)
 
2
Với n = 3 thì rõ ràng A= B= 6 suy ra A : B .
Ta xét n4 . Khi đó có hai khả năng sau:
a) n + 1 là số nguyên tố. Ta chứng minh A không chia hết cho B . Thật vậy, nếu A : B thì
n ( n + 1)
n! = k ⇒ 2 ( n − 1) ! = k ( n + 1)
2
Điều này vô lý vì n + 1 là số nguyên tố nên ( n + 1) và các số 1, 2, …, n − 1 là nguyên tố cùng
nhau.
b) n + 1 là hợp số. Khi đó
n + 1 = p ⋅ q  ( p, q ∈ , p, q2 )

Liên hệ tài liệu word môn toán: 039.373.2038 TÀI LIỆU TOÁN HỌC
Website:tailieumontoan.com

n +1
Suy ra n + 12 p hay p .
2
n +1
Do n > 3 ta có 2n > n + 3 suy ra 2n − 2 > n + 1 hay < n − 1 nên p < n − 1 . Tương tự
2
q < n −1.
Do đó nếu n + 1 có thể viết được dưới dạng (1) với p ≠ q thì p, q là các số tự nhiên nhỏ hơn
n − 1 nên trong tích ( n − 1) ! =
1.2.3 . . ( n − 1) có hai thừa số là p và q suy ra

( n − 1)!: ( p − q ) = n + 1 ⇒ n !: n ( n − 1) ⇒ A : B

Nếu n + 1 có dạng (1) với p = q tức n + 1 =p 2 ( p2 ) và p là hợp số thì n + 1 cũng có dạng (1)
với p ≠ q do đó A : B .
Ta xét trường hợp n + 1 =p 2 với p số nguyên tố. Khi đó, do n + 15 nên p3 suy ra p 29
hay n8 . Ta chứng minh

 n −1
2

p2 = n + 1 <  
 2 
Ta có ( 2 ) tương đương với

( )
4 n + 4 n 2 − 2 n + 1 ⇒ n 2 − 6 n − 3 0 ⇒ n 2 − 8n + ( 2 n − 3 ) > 0
n −1
Bất đẳng thức này đúng vì n > 8 , do vậy (2) đúng. Từ (2) suy ra p < . Khi đó
2
( n − 1)!= 1.2… p ⋅ ( p + 1…( n − 1) . Do ( n − 1) > 2 p nên 2.10. Đáp án tuyển sinh lớp 10 năm
1994(cho thí sinh chuyên toán và chuyên tin)65
tích ( p + 1) …( n − 1) có nhiều hơn p thừa số do đó có một thừa số chia hết cho p nên ta có

( n − 1)!: p 2 = n + 1 ⇒ n !: n ( n + 1) ⇒ A : B
Kết hợp với trường hợp n = 3 ta có kết luận:
Với n + 1 là số nguyên tố thì A không chia hết cho B .
Với n + 1 là hợp số thì A : B .
Bài 4. Ta chứng minh rằng với x, y1 ta có
1 1 1
+ 
1 + x 1 + y 1 + xy
Ta có (1) tương đương với

Liên hệ tài liệu word môn toán: 039.373.2038 TÀI LIỆU TOÁN HỌC
Website:tailieumontoan.com

( ) ( )
  (1 + y ) 1 + xy + (1 + x ) 1 + xy − 2 (1 + x )(1 + y ) 0

⇒ 1 + xy + y + y xy + 1 + xy + x + x xy − 2 − 2 x − 2 y − 2 xy0
⇒ x xy + y xy − 2 xy − x − y + 2 xy0
⇒ xy ( x − y ) 2 − ( x − y ) 2 0
⇒ ( )
xy − 1 ( x − y ) 2 0
 

Bất đẳng thức này đúng vì x, y1 hay (1) đúng (có thể thấy rằng dấu "=" đạt được khi và chỉ khi
x = y hoặc xy = 1 ).
Áp dụng với a, b, c1 ta có
1 3 1 1 2  1 1 
+ = + +  
 2 + 
1+ a 1+ b 1+ a 1+ b 1+ b  1 + ab 1 + b 
4 4
  =
1 + b ab 1 + ab
4 3

Vậy
1 3 4
+ 
1 + a 1 + b 1 + 4 ab3
Tương tự
1 3 4
+ 
1 + b 1 + c 1 + 4 bc 3

1 3 4
+ 
1 + c 1 + a 1 + 4 ca 3
Cộng (2), (3), (4) rồi chia cho 4 ta được bất đẳng thức phải chứng minh.
Bài 5.
1 Giả sử ∠ BAC = 20 . Trên các cạnh AB, AC lấy các điểm D, K tương ứng sao cho
= KC
AD = AC > BC ) . Ta chứng minhAD
= BC (chú ý AB = DK
= KC
Phía trong ABC dựng tam giác đều BCI thì A, I nằm trên trung trực của BC suy ra AI là
phân giác của góc ∠ BAC . Khi đó ∠ ACB = 80 ⇒ ∠ ACI = 20 =∠ CAD , mà
= BC
AD = CI nên dễ thấy ACID là hình thang cân (đáy là AC và ID ), từ đó ta có AC 
ID ⇒ ∠ DIA = ∠ IAC = ∠ IAD = 10 ⇒ ADI cân ở D . Suy ra = = CK nên CIDK là
ID AD
hình bình hành suy ra DK = BC .
= DK
Vậy AD = KC
= CB .

Liên hệ tài liệu word môn toán: 039.373.2038 TÀI LIỆU TOÁN HỌC
Website:tailieumontoan.com

2 Ngược lại, giả sử tồn tại các điểm D và K trên các cạnh AB, AC tương ứng sao cho
= DK
AD = KC = CB . Kẻ đoạn thẳng CI song song, cùng chiều và bằng KD thì
CKDI là hình bình hành và là hình thoi. Do = = AD và
CI DK
∠=DAC ∠= AKD ∠ ACI nên ACID là hình thang cân.
Vì CD là phân giác của ∠ ACI ( CKDI là hình thoi) nên dễ thấy AI là phân giác của góc
∠ DAC từ đó ta có AI là trung trực của BC suy ra IB
= IC = BC hay IBC dều. Đặt
∠ BAC = x , ta có
∠ ABI =∠ ACI =∠ BAC =x ⇒ ∠ IBC + ∠ ICB + 3x =180
hay
120 + 3 x =180 ⇒ ∠ BAC =x =20

ĐẠI HỌC QUỐC GIA HÀ NỘI ĐỀ THI TUYỂN SINH LỚP 10


TRƯỜNG ĐẠI HỌC KHOA HỌC TỰ NHIÊN HỆ THPT CHUYÊN NĂM 1993

MÔN : TOÁN
Thời gian làm bài: 150 phút (không kể thời gian phát đề)

Bài 1.

1 1
1 Giải phương trình x + x+ + x+ =2
2 4
1
Điều kiện: x −
2

Liên hệ tài liệu word môn toán: 039.373.2038 TÀI LIỆU TOÁN HỌC
Website:tailieumontoan.com

2
 1 1 1 1
(1)  ⇒ x +  x + +  = 2 ⇒ x + x + + = 2
 4 2  4 2

2
 1 1 1 1
 ⇒  x + +  = 2 ⇒ x + = 2 −
 4 2  4 2

1 1
 ⇒ x + = 2 + − 2
4 4
 1
 ⇒ x = 2 − 2   thoûa man ñieàu kieän x − 
 2

2
 x 3 + 2 xy 2 + 12 y =
Giải hệ phương trình  2 ⇒ 
2
(
0  x + 2 xy + 8 y + x y =
3 2 2
0 )
8 y + x =
2
12 8 y + x =
2 2
12
Ta có (1) tương đương với x 3 + x 2 y + 2 xy 2 + 8 y 3 =
0
Dễ thấy hệ không có nghiệm với y = 0 , vì nếu y = 0 thì từ (3) suy ra x = 0 không thoả mãn (2).
3 2
x x x
Với y ≠ 0 tương đương với   +   + 2 + 8 =0
 y  y y

Đặt
x
y
( )
= t thì (4) có dạng t 3 + t 2 + 2t + 8 = 0 ⇒ ( t + 2 ) t 2 − t + 4 = 0 ⇒ t = −2

x
Từ đó =−2 ⇒ x =−2 y . Thay vào (2) ta được 12 y 2 =12 ⇒ y =±1 ⇒ x =2
y
Vạỹ hẹ̃ đã cho có hai nghiẹ̄ m x = 2, y = −1 và x =
−2, y =
1

a+b+c+d 
4

Bài 2. Trước hết ta chứng minh rằng: Với a, b, c, d0 thì abcd  
 4 
Dắu "=" đạt được khi và chỉ khi a= b= c= d .
a+b
2

Ta có với a, b0 thì ab   , dá́ u "=" đạt được khi và chỉ khi a = b . Do đó với
 2 
a, b, c, d0 thì

a+b c+d  a+b c+d 


2 2 2

abcd      =   ⋅ 
 2   2   2 2 
2
 a + b + c + d  2  a+b+c+d 
4

     = 
 4    4 
a+b+c+d 
4

 ⇒ abcd   
 4 

Liên hệ tài liệu word môn toán: 039.373.2038 TÀI LIỆU TOÁN HỌC
Website:tailieumontoan.com

Nếu trong a, b, c, d có một số bằng 0 thì dáu " "=" đạt được khi và chỉ khi a= b= c= d= 0 .
Nếu cả 4 số a, b, c, d đều dương thì dá́ u "=" đạt được khi và chỉ khỉ
a = b

c = d  ⇒ a = b = c = d
a + b = c + d

a+b+c+d 
4

Tóm lại abcd   dá́ u "=" đạt được khi và chỉ khi a= b= c= d . Xét
 4 
A x 2 y ( 4 − 
= x − y ) với x0, y0, x + y6
a) Giá trị lớn nhất:
Với x + y4 thì A0
4
x x 
x x  2 + 2 + y+4− x− y
Vởi x + y < 4 ta có A = 4 ⋅ ⋅ ⋅ y ( 4 − x − y ) 4   =4
2 2  4

 
x
Dấu "=" đạt được khi và chỉ khi = y = 4 − x − y ⇒ x = 2, y = 1 .
2
Vặy A đạt giá trị lớn nhắt bằng 4 đạt được khỉ= x 2,=y 1 (thoả mãn điều kiẹ̄ n (1)).
b) Giá trị bé nhấtt.
Với x + y4 thì A0 .
Với 4 < x + y6 ta có
4
x x 
 2 + 2 + y+ x+ y−4  2( x + y) − 4 
4
A x x
− = ⋅ ⋅ y ( x + y − 4 )    = 
4 2 2  4   4 
 
 2.6 − 4 
4


 16  =
 4 
x
Vì x + y6 nên A − 64 . Dá́ u "=" đạt được khi và chỉ khi = y = x + y − 4 và
2
x + y = 6 ⇒ x = 4, y = 2 .
Vậy A đạt giá trị bé nhất bằng −64 , đạt được khì=
x 4,=
y 2.
Bài 3. Kẻ đường trung trực của AB cất AC ở O1 , cắt BD ơ O2 thì O1 , O2 là tām các đường
tròn ngọị tiếp ∆ABD và ∆ABC suy ra R = O1 A, r = O2 B .

Liên hệ tài liệu word môn toán: 039.373.2038 TÀI LIỆU TOÁN HỌC
Website:tailieumontoan.com

O A AI AB ⋅ AI a2 1 AC 2
∆AIO1 ∼ ∆AOB ⇒ 1= ⇒=
R O=
1 A = ⇒ =
AB AO AO AC R2 a4
Tương tư:
1 BD 2 1 1 AC 2 + BD 2 4 AB 2 4
2
= 4
⇒ 2
+ 2
= 4
= 4
= 2
r a R r a a a
Bài 4. Giả sử trên đường tròn ( O ) chiều đi từ A → B → C → A là ngược chiều kim đồng hồ và
giả sử quay ABC mọṭ góc 90 thuạ̄n chiều kim đồng hồ quanh ( O ) ta thu được A1 B1C1 . Khỉ
đó A1 , B1 , C1 thuộc các cung nhở AC , AB, BC tương ứng. Do đó cạnh A1 B1 phải cấ các cạnh
AB, AC , giả sử lần lượt tại M , N . Tương tự ta có cạnh A1C1 cắt các cạnh AC , BC lằn lượt tại
P, Q và cạnh B1C1 cất các cạnh BC , BA làn lượt tại T , K , suy ra phằn chung của hai hình tam
giác ABC và A1 B1C1 là lục giác MNPQTK . Gọi diẹ̄ n tích lục giác đó là S thì
S = S ABC − S AMN − S BKT − SCPQ
Ta có:
BC 2 3 3 3R 2
=
S ABC =
4 4
Khỉ quay mọ̄t gốc 90 thì OA1 ⊥ OA, OB1 ⊥ OB, OC1 ⊥ OC mà BC ⊥ OA nên OA1  BC , tương
tự: OB1CA, OC1 AB .
Gọi giao điểm của OA1 với AC là E . Do các cung AA1 , BB1 , CC1 có số đo bằng 90 nēn
∠ AMN = 90 mà ∠ MAN = 60 nēn suy ra AN = 2 AM .
2
Dễ thấy NEA1 cân ở E (các góc ở đáy bằng 30 ). Do OE  BC còn AO bằng trung tuyến
3
AA′ của ABC nên
2 2 BC R
=
AE =
AC =
R, OE =
3 3 3 3
R
⇒ EN =EA1 =OA1 − OE =R −
3
 R 
⇒ AN = AE − EN =
2
3
R −R −

=
3
( )
3 −1 R

AM .MN 1 AN AN 3 3
⇒ S AMN=
= = AN=
2

2 2 2 2 8
3(4 − 2 3 2 2 3 − 3 2
= = R R
8 4

Liên hệ tài liệu word môn toán: 039.373.2038 TÀI LIỆU TOÁN HỌC
Website:tailieumontoan.com

2 3 −3 2
Tương tự ta có S=
BKT S=
CPQ R .
4
Vậy
9−3 3 2
S= R
4
Bài 5.
1 1 1 1 1 1 ab + bc + ca + a + b + c
A= + + + + + =
a b c ab bc ca abc
Ta chứng minh ba số a, b, c cùng chẵn hoặc cùng lẻ.
Nếu abc là lẻ thì mổi số a, b, c đều lẻ.
Nếu abc là chẵn thì một trong ba số phải chẵn, chẳng hạn a chẵn. Vi tử số ở (1) chia hết cho
abc nên tử số phải chẵn suy ra bc + b + c chẩn hay ( b + 1)( c + 1) − 1 chẫn ⇒ ( b + 1)( c + 1) lẻ.
Vậy b + 1 và c + 1 là lẻ hay b, c chẩn. Vậy a, b, c cùng chẫn hoặc cùng lẻ. Vì a, b, c đôi một
khác nhau nên ta có thể giả sử rằng a < b < c . Khi đó, a2 , vì nếu a3 thì b5, c7 , do đó
1 1 1 1 1 1
A + + + + + <1
3 5 7 15 35 21
Suy ra A không nguyên.
a) Với a = 2 thì b4, c6 và
1 1 1 1 1 1 1 1 1 1 1
A=
+ + + + +  + + + + +
2 b c 2b 2c bc 2 4 6 8 12
1 1 28
+ +
 2 = <
16 24 24
⇒ A=  .
Với b6 thì c8 khi đó
1 1 1 1 1 1 46
A + + + + + = <1
2 6 8 12 16 48 48
⇒ A không nguyên
1 1 1 1 1 1
Do đó b = 4 . Vậy a = 2 thì b = 4 suy ra 1 = + + + + + .
2 4 c 8 4c 2 c
Từ đây ta thu được c = 14 .
Vậy với a = 2 ta được nghiệm là=
a 2,=
b 4,=
c 14 .
Chú ý : Từ điều kiện a = 2 và A = 1 ta được phương trình

Liên hệ tài liệu word môn toán: 039.373.2038 TÀI LIỆU TOÁN HỌC
Website:tailieumontoan.com

1 1 1 1 1 1 2b + 2c + b + c + 2 1
1= + + + + + ⇒ =
2 b c 2b 2c bc 2bc 2
=
b−3 1 = b 4
⇒ 3b + 3c + 2 = bc ⇒ ( b − 3)( c − 3) = 11 ⇒  ⇒
=
c − 3 11 = c 14
Do đó ta cũng tìm được nghiệm trên.
b) a = 1 , khi đó b3, c5 và
2 2 1 32
A =1 + + + = <3
b c bc 15
2 2 1
mà A > 1 ⇒ A = 2 ⇒ + + =1
b c bc
2 2 1
Khi đó nếu b5 thì + + < 1 , do đó b chỉ có thể là 3 . Với b = 3 ta được
b c bc
2 2 1
+ + =1 ⇒ c = 7 .
3 c 3c
Vậy với a = 1 ta được nghiệm là =a 1,= b 3,= c 7.
Tóm lại, với giả thiết a < b < c ta có hai nghiệm là ( 2, 4,14 ) và (1,3, 7 ) . Thay đổi vai trò̀ a, b, c
ta thu được 12 nghiệm là các cách sấp thứ tự của ba số 2, 4,14 và ba số 1,3, 7 .

ĐẠI HỌC QUỐC GIA HÀ NỘI ĐỀ THI TUYỂN SINH LỚP 10


TRƯỜNG ĐẠI HỌC KHOA HỌC TỰ NHIÊN HỆ THPT CHUYÊN NĂM 1992

MÔN : TOÁN (Vòng 1)


Thời gian làm bài: 150 phút (không kể thời gian phát đề)

Bài 1.
1.
x + 2 + 3 2x − 5 + x − 2 − 3 2x − 5 =2 2
5
Điều kiện: x
2
Nhân hai vế với 2 ta có (1) tương đương với

Liên hệ tài liệu word môn toán: 039.373.2038 TÀI LIỆU TOÁN HỌC
Website:tailieumontoan.com

( 2 x − 5) + 6 2x − 5 + 9 + ( 2 x − 5) − 2 2x − 5 + 1 =4

⇒ ( 2 x − 5 + 3) 2 + ( 2 x − 5 − 1) 2 =4
⇒ 2x − 5 + 3 + 2x − 5 − 1 =4

Với x3 thì (2) có dạng


2x − 5 + 3 + 2x − 5 − 1 = 4 ⇒ 2x − 5 = 1
⇒ ( a maõn x3)
2 x − 5 = 1 ⇒ x = 3  thoû
5
Với x < 3 thì (2) có dạng 2 x − 5 + 3 + 1 − 2 x − 5 =4 luôn thoả mãn.
2
5
Vậy nghiệm của (1) là x3 .
2
2.
 xy 2 − 2 y + 3 x 2 =0  xy 2 − 2 y =
−3 x 2
 2 ⇒  2
 y + x y + 2 x = x y + 2x = − y2
2
0
Dễ thấy= y 0 là một nghiệm của hệ và ngược lại nếu ( x, y ) là nghiệm mà̀ x hoặc y
x 0,=
bằng 0 thì số kia cũng bằng 0 . Ta tìm nghiệm thoả mãn x ≠ 0, y ≠ 0 .
Với điều kiện x ≠ 0, y ≠ 0 , chia (1) cho y 2 và ( 2 ) cho x 2 ta được hệ (1), (2) tương đương với

 2 x2
x − = −3 2
 y y

y + 2 = y2
− 2
 x x
Nhân hai phương trình với nhau ta được hệ ( 3) , ( 4 ) tương đương với

 2 x2
x − = −3 2  xy 2 − 2 y =
−3x 2
 y y 
 ⇒ 4
 x − 2   y + 2  =  xy −
   3  xy
 y  x
Ta có (5) tương đương với
 −1
( xy ) 2 − 3 xy − 4 = 0 ⇒ xy = 
4
1
Thay xy = −1 hay y = − vào (1) ta được
x

Liên hệ tài liệu word môn toán: 039.373.2038 TÀI LIỆU TOÁN HỌC
Website:tailieumontoan.com

1 2
+ =−3 x 2 ⇒ x3 =−1 ⇒ x =−1 ⇒ y =1
x x
4
Thay xy = 4 hay y = vào (1) ta được
x
16 8 8 2 [
− =−3 x 2 ⇒ x3 =− ⇒ x =− 3 ⇒ y =−2  3
x x 3 3 3
2
Vậy hệ đã cho có ba nghiệm là x =
y=0; x =
−1, y =
1 và x = − ,y =
−2 3 3
3
3
Bài 2.
x 2 − mnx + m + n =0
Với m = 0 phương trình (1) có dạng x 2 + n =0 . Nếu phương trình có nghiệm nguyên a với
n0 nào đó thì a 2 + n =0 ⇒ n =−a 2 < 0 do đó n = 0 và nghiệm là x= a= 0 . Lý luận tương
tự đối với trường hợp n = 0 .
Như vậy nếu một trong hai số m, n bằng 0 thì (1) có nghiệm nguyên suy ra số còn lại cũng bằng
0 và nghiệm là x = 0 . Ta xét trường hợp m, n1 . Khi đó giả sử a, b là các nghiệm của (1), theo
định lý Viét ta có
a + b =mn

ab= m + n
Do m, n1 nên từ đây suy ra a, b > 0 ⇒ a, b1 ⇒ ( a − 1)( b − 1) 0 hay
ab − a − b + 10  
⇒ m + n − mn + 10 ⇒ mn − m − n + 12
⇒ ( m − 1)( n − 1) 2
 
1 Với m = 1 thì (1) có dạng
x 2 − nx + 1 + n =0
x2 + 1 2
x 2 + 1 = n ( x − 1) ⇒ n = ⇒ n = x +1+
x −1 x −1
Nếu phương trình có nghiệm nguyên x thì x − 1 là ước của 2 , do đó x − 1 ∈ {1, −1, 2, −2} .

Với x − 1 =1 tức x = 2 thì n = 5 , phương trình (2) có dạng x 2 − 5 x + 6 = 0 (có hai nghiệm
=
x 2,= x 3 ).
Với x − 1 =−1 tức x = 0 thì n = −1 (loại).
Với x − 1 =2 tức x = 3 thì n = 5 , phương trình=
x 2,=
x 3.
Với x − 1 =−2 tức x = −1 thì n = −1 (loại).
Vậy với m = 1 thì phương trình (1) có nghiệm nguyên khi n = 5 .

Liên hệ tài liệu word môn toán: 039.373.2038 TÀI LIỆU TOÁN HỌC
Website:tailieumontoan.com

Lý luận tương tự ta có với n = 1 thì phương trình (1) có nghiệm nguyên khi m = 5 . Ta xét
trường hợp m, n2 .
2 Với m = 2 thì từ (3) ta có n = 2 hoặc n = 3 .
Khi n = 2 thì (1) có dạng x 2 − 4 x + 4 =0 có nghiệm nguyên x = 2 .
Khi n = 3 thì (1) có dạng x 2 − 6 x + 5 =0 có nghiệm nguyên là x = 1 và x = 5 .
Như vậy với m = 2 thì (1) có nghiệm nguyên khi n = 2 hoặc n = 3 .
Lý luận tương tự ta có với n = 2 thì (1) có nghiệm nguyên khi m = 2 hoặc m = 3 .
3 Với m, n3 đẳng thức (3) không thoả mãn.
Tóm lại: Phương trình (1) có nghiệm nguyên (với m, n không âm) khi ( m, n ) là một trong các
cặp số sau: ( 0, 0 ) ; (1,5 ) ; ( 5,1) ; ( 2, 2 ) ; ( 2,3) ; ( 3, 2 ) .
Bài 3.
S MNP =−
S S ABM − S BCN − SCAP
Qua A′ kẻ đường thẳng song song với BB ′ cắt CB ′ tại I , ta có:
AM AB ′ AB ′ B ′C BC
= = ⋅
 3 =⋅ = 3.3 =9
MA′ B ′I B ′C B ′I BA′
AM AM 9 S AM 9
 = = ⇒ ABM = =
AA′ AM + MA′ 10 S ABA′ AA′ 10
S 9 3
S ABA′
 mà    =⇒ S ABM = S ABA′ = S
3 10 10
Tương tự: Qua B ′ kẻ đường thẳng song song với CC ′ cắt AC ′ tại H , ta có
BN BC ′ BC ′ C ′A CA
= = ⋅
  =1⋅ =
4
NB ′ C ′K C ′A C ′K CB ′
BN 4 S BN 4
 =⇒ BCN = =
BB ′ 5 S BCB′ BB ′ 5
S 4 S
S BCB′
 mà    =⇒ S BCN =S BCB′ =
4 5 5
Hoàn toàn tương tự ta tính được
2  3 S 2  S
SCAP = S ⇒ S MNP = S −  S + + S  =
5  10 5 5  10
Bài 4. Trên cạnh BC lấy điểm M sao cho ∠=
BDM ∠ ADC ⇒ ∠ CDM = ∠ ADB . Dễ thấy
BM DH
 BDM ∽ ADC ⇒ = (tỷ số hai đường cao tương ứng của hai tam giác đồng dạng
AC DI
bằng tỷ số đồng dạng), suy ra
BM AC
=
DH DI
Liên hệ tài liệu word môn toán: 039.373.2038 TÀI LIỆU TOÁN HỌC
Website:tailieumontoan.com

Tương tự
CM DH
CDM ∼ ADB  ⇒ =
AB DK
CM AB
 ⇒ =
DH DK
Cộng (1) và ( 2 ) ta được
BC AC AB
= +
DM DI DK
Bài 5. Giả sử m, n nguyên dương và
2m + 1: n
 ⇒ ( 2m + 1)( 2n + 1) mn
2n + 1 m
⇒ 2m + 2n + 1 mn ⇒ 2m + 2n +=   k ∈ *
1 kmn ( )
Suy ra: k , m, n đều lẻ. Vì
m1, n1 ⇒ ( m − 1)( n − 1) 0 ⇒ m + nmn + 1
2m + 2n + 12mn + 35mn
Do đó tù̀ (1) suy ra k5 .
1 Với k = 5 ⇒ 2m + 2n + 1 = 5mn , từ (2) ta suy ra 3mn =3 ⇒ m = n = 1 . Rõ ràng
m= n= 1 thoả mãn 2m + 1 n và 2n + 1: m .
2 Với k = 3 , giả sử mn , ta có 3mn = 2m + 2n + 15m ⇒ 3n
5 ⇒ n = 1 ⇒ 3m = 2m + 3 ⇒ m = 3 . Rõ ràng= n 1,=m 3 thoả mãn 2m + 1: n và
2n + 1: m . Với mn ta được=
n 3,=
m 1 thoả mãn.
3 Với k = 1 , giả sử mn , ta có mn = 2m + 2n + 15m ⇒ n5 ⇒ n = 1 hoặc n = 3 hoặc
n = 5.
Với n = 1 đẳng thức mn = 2m + 2n + 1 trở thành =
m 2m + 3 vô lý
Với n = 3 ta có 3m= 2m + 6 + 1 ⇒ m= 7 . Rõ ràng= m 7 thoả mãn điều kiện
n 3,=
2m + 1: n; 2n + 1: m .
11
Với n = 5 ta có 5m= 2m + 10 + 1 ⇒ m= (loại).
3
Với mn lý luận tương tự ta được = n 7 thoả mãn điều kiện 2m + 1 n; 2n + 1 m
m 3,=
Tóm lại: Có 5 cặp số ( m, n ) thoả mãn yêu cầu bài toán là

(1,1) ; (1,3) ; ( 3,1) ; ( 3, 7 ) ; ( 7,3)

Liên hệ tài liệu word môn toán: 039.373.2038 TÀI LIỆU TOÁN HỌC
Website:tailieumontoan.com

ĐẠI HỌC QUỐC GIA HÀ NỘI ĐỀ THI TUYỂN SINH LỚP 10


TRƯỜNG ĐẠI HỌC KHOA HỌC TỰ NHIÊN HỆ THPT CHUYÊN NĂM 1992

MÔN : TOÁN (Vòng 2)


Thời gian làm bài: 150 phút (không kể thời gian phát đề)

Bài 1.

( ) + (n )
2
1. P =n 4 + 2n3 + 2n 2 + n + 7 = n 2 + n 2
+n +7

Do n nguyên nên n 2 + n0 . Đặt =


k n 2 + n ta có P = k 2 + k + 7 > k 2 . Ta có:
(
P < (k + 3) 2    vì (k + 3) 2 − P = 5k + 2 > 0 )
Do đó k 2 < P < (k + 3) 2 ⇒ P = (k + 1) 2 hoặc p= (k + 2) 2 .
Với P= (k + 1) 2 tức k 2 + k + 7 = k 2 + 2k + 1 ta thu được k= 6 ⇒ n 2 + n = 6 ⇒ n = 2 và
n = −3 .
Với P= (k + 2) 2 tức k 2 + k + 7 = k 2 + 4k + 4 ta thu được k = 1 ⇒ n 2 + n =1 ⇒ n = 2 không có
nghiệm nguyên.
Vậy P là số chính phương khi n = 2 và n = −3 (khi đó P = 49 ).
Liên hệ tài liệu word môn toán: 039.373.2038 TÀI LIỆU TOÁN HỌC
Website:tailieumontoan.com

1 1 1
1. A= + 2 + 2
a + 2bc b + 2ca c + 2ab
2

Đặt x =+
a 2 2bc, y =+ c 2 2ab thì x, y, z > 0 và x + y + z = (a + b + c) 2 1 , do
b 2 2ac, z =+
1 1 1
( x + y + z) A = ( x + y + z) + +  9 mà x + y + z1 nên A9 .
x y z
Bài 2. Với n ∈  ta ký hiệu tổng các chữ số của n là S ( n ) . Ta có

( 2 )= ( 2 ) =
1945 3.1945
=
N 9 3
85835 < 105835
Nên N có không quá 5835= ( N ) 5835.9 52515 suy ra a có không quá 5 chữ
chũ số mà a S=
= ( a ) 5.9 45 . Trong các số tự nhiên từ 0 đến 45 thì số có tổng các chữ số lớn nhất là
số, b S=
39 và tổng các chữ số của nó là 12. Suy ra S ( b ) 12 .
Ta biết rằng với mọi n ∈  thì S ( n ) ≡ n ( mod9 ) , do đó S ( b ) ≡ b =S ( a ) ≡ a =S ( N )( mod9 ) .
Mà N = 85835 suy ra N ≡ −1( mod9 ) hay S ( b ) ≡ −1( mod9 ) . Do S ( b ) 12 nên S ( b ) = 8 .
Vậy tổng các chữ số của b là 8 .
Bài 3. Giả sử K nằm ngoài đoạn BC về phía C (trường hợp AB > AC ) , khi đó dễ thấy
∠ ACK nhọn và ∠ ACB tù suy ra cung AB không chứa C lớn hơn cung ACE . Trên cung AB
không chứa C lấy điểm E sao cho cung BE bằng cung AC , khi đó ACBE là hình thang cân.
2.7. Đáp án tuyển sinh lớp 10 năm 1992(cho thí sinh chuyên toán và chuyên tin)55
Ta có ∠ AEB + ∠ EAB = ∠ EAC + ∠ EAB = 2 ( ∠ EAB + ∠ BAD ) = 2∠ EAD = 90 suy ra
∠ ABE = 90 nên AE là đường kính của đường tròn ( O ) ngoại tiếp ABC . Do đó
AB 2 + AC 2 = AB 2 + EB 2 = AE 2 = 4 R 2
Chú ý: Có thể chứng minh bằng cách lấy điểm M trên cung AB không chứa C sao cho cung
AM bằng cung AC , sau đó chứng minh ∠ BAM = 90 , từ đó suy ra đẳng thức
AB 2 + AC 2 = 4R2
Bài 4. Gọi các đường thẳng đã cho là d1 , d 2 , …, d1992 và giao điểm của đường thẳng di , d k là Aik
hoặc Aki .
a) Xét đường thẳng di bất kỳ trong 1992 đường thẳng đã cho. Do không có 3 đường thẳng nào
đồng quy nên các giao điểm Akl của các cặp đường thẳng d k , dl ( k ≠ i, l ≠ i ) đều nằm ngoài di .
Do số giao điểm đó là hữu hạn nên có 1 giao điểm gần nó nhất, giả sử đó là Akl . Ta chứng minh
tam giác Akl Aki Ali là tam giác xanh. Thật vậy nếu tam giác đó bị đường thẳng d m nào đó trong
1989 đường thẳng còn lại cắt thì d m phải cắt một trong hai đoạn Akl Aki hoặc Akl Ali , giả sử d m
cắt đoạn Akl Aki tại Akm thì Akm gần di hơn Akl , trái với giả thiết Akl là điểm gần di nhất. Như

Liên hệ tài liệu word môn toán: 039.373.2038 TÀI LIỆU TOÁN HỌC
Website:tailieumontoan.com

vậy với mỗi đường thẳng di luôn tồn tại một tam giác xanh có cạnh nằm trên nó. Trên mỗi di ta
chọn một cạnh của một tam giác xanh thì ta thu được 1992 cạnh khác nhau của các tam giác
xanh. Từ đó suy ra số tam giác xanh không ít hơn 1992 : 3 = 664
b) Xét đường thẳng di trong số 1992 đường thẳng đã cho. Nếu trong mỗi nửa mặt phẳng có bờ là di
đều có các giao điểm của các cặp đường thẳng còn lại thì trong mỗi nửa mặt phẳng ta lấy giao điểm
gần di nhất và lý luận như câu a) ta được hai tam giác xanh nằm về hai phía của di . Hai tam giác đó
có hai cạnh nằm trên di và hai cạnh đó là khác nhau (không có ba đường nào đồng quy).
Ta chứng minh rằng số đường thẳng mà các giao điểm của các cặp đường thẳng còn lại nằm về
cùng một phía của nó không vượt quá 2. Thật vậy, giả sử có 3 đường thẳng như vậy, chẳng hạn
đó là di , d k , dl . Khi đó xét đường thẳng d n khác, d n cắt di , d k , dl tại 3 điểm phân biệt
Ani , Ank , Anl . Trong 3 điểm đó có 1 điểm nằm giữa hai điểm kia, giả sử Ank . Khi đó hai giao điểm
Ani và Anl nằm về hai phía của d k trái với giả thiết.
Vậy có ít nhất 1990 đường thẳng mà về hai phía của mỗi đường đều có các giao điểm của các
đường thẳng còn lại. Theo lý luận ở trên thì có hai tam giác xanh nằm về hai phía của mỗi đường
thẳng đó và có hai cạnh khác nhau nằm trên nó. Trong hai đường thẳng còn lại, trên mỗi đường
thẳng có ít nhất một cạnh của một tam giác xanh. Như vậy số cạnh khác nhau của các tam giác
xanh không ít hơn 1990 × 2 +=2 3982 = 1327.3 + 1 . Suy ra số tam giác xanh không ít hơn
1327 + 1 = 1328 .
ĐẠI HỌC QUỐC GIA HÀ NỘI ĐỀ THI TUYỂN SINH LỚP 10
TRƯỜNG ĐẠI HỌC KHOA HỌC TỰ NHIÊN HỆ THPT CHUYÊN NĂM 1991

MÔN : TOÁN (Vòng 1)


Thời gian làm bài: 150 phút (không kể thời gian phát đề)

Bài 1.
1.
a+x + a−x
= b
a+x − a−x
Để các căn có nghĩa ta phải có −a ≤ x ≤ a . Do vế phải dương nên a + x − a − x > 0 suy ra
x > 0.
Vậy điều kiện đối với x là: 0 < x ≤ a . Với điều kiện đó (1) tương đương với
2a + 2 a 2 − x 2 2a
= b⇒ −1 = b
2a − 2 a − x
2 2
1 − a2 − x2
2a a ( b − 1)
=a − a 2 − x 2 ⇒ a 2 − x 2 =
b +1 b +1

Liên hệ tài liệu word môn toán: 039.373.2038 TÀI LIỆU TOÁN HỌC
Website:tailieumontoan.com

Do đó
Nếu b < 1 thì (2) vô nghiệm do đó (1) vô nghiệm
Nều b ≥ 1 thì (2) tương đương với
a 2 (b − 1) 2 4b 2a
a2 − x2 = ⇒ x2 =a2 ⇒x=± b
(b + 1) 2
(b + 1) 2
b +1
2a
Loại nghiệm âm ta được nghiệm x = b thoả mãn điều kiện 0 < x ≤ a ( vì b + 1 ≥ 2 b ) .
b +1
2a b
Vậy (1) có nghiệm khi và chỉ khi b ≥ 1 và nghiệm đó là x =
b +1
2.
x 2 + ax + b + 1 =0
Giả sử x1 , x2 là hai nghiệm nguyên của (1). Do b ≠ −1 nên x1 ≠ 0 và x2 ≠ 0 . Theo định lý Viét
ta có
 x1 + x2 =−a

 x1 x2= b + 1
→ a 2 + b 2 = ( x1 + x2 ) + ( x1 x2 − 1) = x12 + x22 + x12 x22 + 1
2 2
 
( )(
 =x12 + 1 x22 + 1 )
Do x1 , x2 là các số nguyên khác 0 nên x12 + 1 và x22 + 1 là các số nguyên không bé hơn 2 và như
vậy a 2 + b 2 là hợp số.
Bài 2.
a 3 x + a 2 y + az =
1
 3
b x + b y + bz =
2
1
c3 x + c 2 y + cz =1

Nhân (1) với b và (2) với a rồi trừ từng vế cho nhau, sau đó chia cho a − b ≠ 0 ta được phương
trình
ab ( a + b ) x + aby =
−1
Tương tự, nhân (1) với c và (3) với a rồi trừ từng vế cho nhau, sau đó chia cho a − c ≠ 0 ta
được
ac ( a + c ) x + acy =
−1
Khi đó hệ đã cho tương đương với

Liên hệ tài liệu word môn toán: 039.373.2038 TÀI LIỆU TOÁN HỌC
Website:tailieumontoan.com

a 3 x + a 2 y + az =1

ab ( a + b ) x + aby =−1
ac ( a + c ) x + acy =−1

Nhân (4) với c , (5) với b rồi trừ từng vế cho nhau, sau đó chia cho b − c ≠ 0 ta được abcx = 1
1 1 a+b+c 1 a+b+c
hay x = . Thay x = vào ( 4 ) thu được y = − . Thay x = ,y = −
abc abc abc abc abc
ab + bc + ca
vào (1) thu được z = . Vậy nghiệm của hệ đã cho là
abc
1 a+b+c ab + bc + ca
x= y=
, − z=

abc abc abc
Bài 3 Dễ thấy 7 x chia 4 dư 3 nếu x lẻ và dư 1 nếu x chẵn. Phương trình đã cho tương đương
với
7x − 1 =3.2 y
Nếu x lẻ thì 7 x − 1 chia 4 dư 2 còn với y ≥ 2 thì 3.2 y : 4 do đó y chỉ có thể là 1 . Với y = 1 ta
được nghiệm là=
x 1,=
y 1.
Nếu x chẵn tức là x = 2 z ( z nguyên dương) phương trình (1) có dạng
(7 z
)( )
+ 1 7z − 1 =
3.2 y

( )( )
Vì 2,3 là các số nguyên tố, nên (2) là dạng phân tích của 7 z + 1 7 z − 1 thành tích các thừa số
nguyên tố.
Do 7 z + 1 chia 3 dư 2 nên 7 z + 1 = 2n ⋅ ( 3) với n là số nguyên dương nào đó. Từ đó ta có
7 z − 1 = 2n − 2 suy ra (2) có dạng
( ) ( )
2n 2n − 2 = 3.2 y ⇒ 2n +1 2n −1 − 1 = 3.2 y

Do đó 2n −1 − 1 không chia hết cho 2 nên 2n −1 − 1 =3 hay n = 3 . Thay vào (3) ta thu được z = 1
suy ra x = 2 . Thay x = 2 vào (1) ta được nghiệm là= x 2,= y 4.
Vậy phương trình đã cho có hai nghiệm nguyên dương là=
x 1,=
y 1 và=
x 2,=
y 4
Bài 4.
1 Gọi các giao điểm của EF với AB, CD tương ứng là I , K . Qua F kẻ đường thẳng song
song với AB và CD cắt AD ở M , cắt BC ở N .
Ta có
MF AM BN FN
= = = ⇒ MF = FN
DC AD BC DC
Do AB / / MN , CD / / MN nên ba đường thẳng đồng quy EC , ED, EK cắt các đường thẳng
AB, MN , DC thành các đoạn thẳng tỷ lệ
Liên hệ tài liệu word môn toán: 039.373.2038 TÀI LIỆU TOÁN HỌC
Website:tailieumontoan.com

IA KD FM
= = = 1
IB KC FN

Suy ra EF đi qua các trung điểm I , K của AB và CD


2.
S IJK = S NJC ⇒ S ICK = S ICN ⇒ KN / / IC

Liên hệ tài liệu word môn toán: 039.373.2038 TÀI LIỆU TOÁN HỌC
Website:tailieumontoan.com

Khi đó theo (1) thì AJ đi qua trung diểm E của KN suy ra S AKJ = S ANJ mà S AKP = SCJN nên
S APJ = S AIC ⇒ PJ = JC ⇒ S BPJ = S BJC
Do S BIM = S IJK nên từ đây ta có S BIKP = SCJIM . Tương tự ta chứng minh được SCJIM = S AKJN . Vậy
diện tích của ba tứ giác không gạch chéo bằng nhau.
Bài 5. Trên nửa đường tròn đường kính AB ta lấy 1991 điểm khác nhau và khác
A, B : A1 , A2 , A3 , …, A1991 . Giả sử Ai , Aj , Ak là ba điểm bất kỳ trong chúng, khi đó Ai , Aj , Ak
không thẳng hàng. Giả sử trên nửa đường tròn đã cho AJ nằm giữa Ai và Ak (tức Aj thuộc
cung nhỏ Aj Ak ) khi đó

Ai Aj Ak Và ∆Ai Aj Ak là tam giác tù

Liên hệ tài liệu word môn toán: 039.373.2038 TÀI LIỆU TOÁN HỌC
Website:tailieumontoan.com

Vậy trên mặt phẳng tồn tại 1991 điểm mà ba điểm bất kỳ trong chúng là ba đỉnh của một tam
giác tù.

Liên hệ tài liệu word môn toán: 039.373.2038 TÀI LIỆU TOÁN HỌC
Website:tailieumontoan.com

ĐẠI HỌC QUỐC GIA HÀ NỘI ĐỀ THI TUYỂN SINH LỚP 10


TRƯỜNG ĐẠI HỌC KHOA HỌC TỰ NHIÊN HỆ THPT CHUYÊN NĂM 1991

MÔN : TOÁN (Vòng 2)


Thời gian làm bài: 150 phút (không kể thời gian phát đề)

Bài 1.
1 Rút gọn
A =3
 2 3 − 4 2 6 44 + 16 6
=
  3
2 3 − 4 2 6 (2 3 + 4 2) 2

=
  (
3 2 3 −4 2
)(
2 3+4 2 =
− 3 20 )
P = ( x − y )5 + ( y − z )5 + ( z − x )5
Đặt a =x − y,   z − x =− ( a + b ) Khi đó
b =y − z ,  
𝑃𝑃 = 𝑎𝑎5 + 𝑏𝑏 5 − (𝑎𝑎 + 𝑏𝑏)5 = (𝑎𝑎 + 𝑏𝑏)[𝑎𝑎4 − 𝑎𝑎3 𝑏𝑏 + 𝑎𝑎2 𝑏𝑏 2 − 𝑎𝑎𝑏𝑏 3 + 𝑏𝑏 4 − (𝑎𝑎 + 𝑏𝑏)4 ]
= (𝑎𝑎 + 𝑏𝑏)(𝑎𝑎4 − 𝑎𝑎3 𝑏𝑏 + 𝑎𝑎2 𝑏𝑏 2 − 𝑎𝑎𝑏𝑏 3 + 𝑏𝑏 4 − 𝑎𝑎4 − 4𝑎𝑎3 𝑏𝑏 − 6𝑎𝑎2 𝑏𝑏 2 − 4𝑎𝑎𝑏𝑏 3 − 𝑏𝑏 4 )
= (𝑎𝑎 + 𝑏𝑏)[−5(𝑎𝑎3 𝑏𝑏 + 𝑎𝑎2 𝑏𝑏 2 + 𝑎𝑎𝑏𝑏 3 )] = −5(𝑎𝑎 + 𝑏𝑏)𝑎𝑎𝑎𝑎(𝑎𝑎2 + 𝑎𝑎𝑎𝑎 + 𝑏𝑏 2 )
= 5(𝑥𝑥 − 𝑦𝑦)(𝑦𝑦 − 𝑧𝑧)(𝑧𝑧 − 𝑥𝑥)[(𝑥𝑥 − 𝑦𝑦)2 + (𝑥𝑥 − 𝑦𝑦)(𝑦𝑦 − 𝑧𝑧) + (𝑦𝑦 − 𝑧𝑧)2 ]
= 5(𝑥𝑥 − 𝑦𝑦)(𝑦𝑦 − 𝑧𝑧)(𝑧𝑧 − 𝑥𝑥)(𝑥𝑥 2 + 𝑦𝑦 2 + 𝑧𝑧 2 − 𝑥𝑥𝑥𝑥 − 𝑦𝑦𝑦𝑦 − 𝑧𝑧𝑧𝑧)
Bài 2.
1.

a + b + c =0

α + β + γ = 0
α β γ
 + + = 0
a b c
Từ phương trình 1 và 2 ta có c =− (a + b) ,   γ =− (α + β ) . Thay vào phương trình 3 ta được
α β γ
+ + =
 0
a b c
→ (α b + β a )( a + b ) + (α + β ) ab =
 0
→ α b 2 + β a 2 + 2ab (α + β ) =
  0
→ α b + β a − 2abγ =
  2 2
0
Tương tự
β c 2 + γ b 2 − 2bcα =
0
γ a + α c − 2ca β =
2 2
0
Liên hệ tài liệu word môn toán: 039.373.2038 TÀI LIỆU TOÁN HỌC
Website:tailieumontoan.com

Cộng (1), (2), (3) ta được


( β + γ ) a 2 + (γ + α ) b2 + (α + β ) c 2 − 2 ( bcα + ca β + abγ ) =
  0
→ ( )
− α a 2 + β b 2 + γ c 2 = ( bcα + ca β + abγ )=
 2
α β γ 
 = 2abc  + +=  0
a b c
Vậy A= α a 2 + β + β b 2 + γ c 2= 0
2 0 ≤ a, b, c, d ≤ 1 . Đặt P = a + b + c + d − ab − bc − cd − da
a) Ta có
a − ab = a (1 − b ) ≥ 0  daáu "=" đñaït ñöôïc  ⇔ a = 0  hoaëc  b = 1
b − bc ≥ 0  daáu "=" đñaït ñöôï=
c ⇔ b 0 hoaë
= c c 1
c − cd ≥ 0  daáu "=" đñaït ñöôï
= c ⇔ c 0  hoaë
= c  d 1
d − da ≥ 0  daáu "=" đñaït ñöôï
= c  ⇔ d 0  hoaë
= c  a 1
Cộng bốn bất đẳng thức trên ta được P ≥ 0
Giả sử dấu "=" đạt được, khi đó cả bốn bất đẳng thức ở (1), (2), (3), (4) phải là đẳng thức. Từ (1)
ta có a = 0 hoặc b = 1 .
Nếu a = 0 thì từ (4) ta có d = 0 , do đó từ ( 3) ta có c = 0 và từ ( 2 ) suy ra b = 0 . Vậy
a= b= c= d= 0 Nếu b = 1 thì từ (2) ta có c = 1 , từ (3) ta có d = 1 và từ (4) suy ra a = 1 . Vậy
a= b= c= d= 1
Ngược lại dễ thấy với a= b= c= d= 0 hoặc a= b= c= d= 1 thì p = 0
Tóm lại: P ≥ 0, P =
0 khi và chỉ khi a= b= c= d= 0 hoặc a= b= c= d= 1
b) Từ (1 − a )( a − b ) ≥ 0 → 1 − a − b + ab ≥ 0 → a + b − ab ≤ 1 .
Vậy
a + b − ab ≤ 1,   daáu "=" đñaït ñöôï=
c ⇔ a 1 hoaë
= c b 1
Tương tự
b + c − bc ≤ 1,  daáu "=" đñaït ñöôï
= c  ⇔ b 1 =
 hoaëc  c 1
c + d − cd ≤ 1,  daáu "=" đñaït ñöôï
= c  ⇔ c 1 =
 hoaëc  d 1
d + a − da ≤ 1,  daáu "=" đñaït ñöôï
= c  ⇔ d 1 =
 hoaëc  a 1
Mặt khác
( a + c )( b + d ) ≥ 0  → ab + bc + cd + da ≥ 0
 → − ( ab + bc + cd + da ) ≤ 0
 Daáu "=" đñaït ñöôïc  = ⇔ a + c 0  hoaë=c  b + d 0
 → a = c = 0  hoaëc  b = d = 0

Liên hệ tài liệu word môn toán: 039.373.2038 TÀI LIỆU TOÁN HỌC
Website:tailieumontoan.com

Cộng các bất đẳng thức ở ( 5 ) , ( 6 ) , ( 7 ) , ( 8 ) và ( 9 ) ta được 2 p ≤ 4 hay p ≤ 2


Nếu dấu "=" đạt được thì ở cả 5 bất đẳng thức đều có dấu "=", như vậy, từ (9) ta có a= c= 0
hoặc b= d= 0 .
Với a= c= 0 thì từ ( 5 ) , ( 7 ) suy ra b= d= 1 .
Với b= d= 0 thì từ ( 5 ) , ( 7 ) suy ra a= c= 1
Ngược lại, với a= c= 0, b= d= 1 hoặc b= d= 0, a= c= 1 thì P = 2 .
2 khi và chỉ khi a= c= 0, b= d= 1 hoặc b= d= 0, a= c= 1
Tóm lại P ≤ 2, P =
Chú ý: Có thể giải cách khác như sau:
P = ( a + c )(1 − b − d ) + b + d ) →
P = ( b + d )(1 − a − c ) + a + c) →
2 P = ( a + c )( 2 − b − d ) + ( b + d )( 2 − a − c )
1
≤ (a + c + 2 − b − d ) 2 + (b + d + 2 − a − c) 2 
 
4
2.5. Đáp án tuyển sinh lớp 10 năm 1991(cho thí sinh chuyên toán và chuyên tin)47
b + d + − a − c ≥ 0 nên từ đây ta có.
Do a + c + 2 − b − d ≥ 0,  2
1
2 P ≤ [( a + c + 2 − b − d ) + ( b + d + 2 − a − c )]2 = P≤
4  hay  2
4
Dấu "=" đạt được khi và chỉ khi
 a + c + 2 − b − d = 0  a = c = 0,  1
b= d =
 →
b + d + 2 − a − c = 0  b = d = 0, a = c = 1
(vì 0 ≤ a, b, c, d ≤ 1)
d < k . Đặt an= a + nd và
Bài 3. Giả sử k là số tự nhiên thoả mãn a < 10k ,  10
an a d
=
xn = +n
1991 1991 1991
a d
Do < 10k và > 0 nên tồn tại số tự nhiên m mà
1991 1991
xm −1 ≤ 10k < xm
Mặt khác
d 10k 10k 1992.10k
xm =xm −1 + < xm −1 + ≤ 10k + =
1991 1991 1991 1991
Từ (1) và ( 2 ) ta có

1992.10k
10k < xm < → 1991.10k < 1991.xm < 1992.10k
1991
Liên hệ tài liệu word môn toán: 039.373.2038 TÀI LIỆU TOÁN HỌC
Website:tailieumontoan.com

Hay
1991.10k < am < 1992.10k
Do đó am= a + md là số có bốn chữ số đầu tiên là 1991 .
Bài 4. Gọi 100 người dự hội thảo là a1 , a2 , a3 , …, a99 , a100 . Giả sử a1 quen biết với 67 người là
a2 , a3 , …, a68 .
Khi đó không kể a1 và 32 người là a69 , a70 , …, a100 thì a2 quen biết với ít nhất 34 người trong số
a3 , a4 , …, a68 . Không mất tổng quát, giả sử a2 quen biết với a3 , a4 , …, a36 . Tương tự, không kể
a1 , a2 và 64 người là a37 , a38 , …, a100 thì a3 quen biết với ít nhất một người trong số
a4 , a5 , …, a36 , chẳng hạn a3 quen biết với a4 .
Từ đó ta có 4 người là a1 , a2 , a3 , a4 đôi một quen biết nhau.
Bài 5. 1) Cùng phía với hình vuông đối với CD dựng tam giác đều M ′CD . Khi đó ADM cân
ở D và BCM cân ở C nên ta có
 ′ = 90 − 60 = 30 → DAM
ADM ′ = BCM ′ = CBM ′ = 75
Mặt khác, dễ thấy M ′ nằm cùng phía với hình vuông ABCD dối với AB . Theo giả thiết thì M
nằm trong hình vuông và = BAM
ABM 
= 15 suy ra M ′ ≡ M hay MCD dều

Chú ý: Có thể chứng minh bằng phản chứng như sau:


Giả sử MCD không dều. Dễ thấy  AMD = BMC (c-g-c) suy
= =
ra MC MD  MDC
, MCD .
Có hai khả năng

= MCD
a) MDC  < CD và 
 < 60 suy ra MD  = 75 nên
ADM > 30 . Mà DAM
 < 750 → AD < MD < CD vô lý (Vì ABCD là hình vuông).
DMA
  > 60 , lý luận tương tự dẫn đến điều vô lý, do đó giả thiết MCD không đều là
= MCD
b) MDC
sai.

Liên hệ tài liệu word môn toán: 039.373.2038 TÀI LIỆU TOÁN HỌC
Website:tailieumontoan.com

2 =
Trong hình vuông ABCD dựng bốn điểm M , N , P, Q thoả mãn MAB
= NBC
MBA = NCB = PCD
= PDC = QDA 
= QAD = 15 thì theo chứng minh trên, bốn
tam giác MCD, NDA, PAB và QBC là đều.

Ta chứng minh tập hợp 8 điểm { A, B, C , D, M , N , P, Q} thoả mãn điều kiện bài toán.
8.7
Có tất cả là = 28 đoạn thẳng nối hai trong tám điểm trên. Ta chia chúng thành 6 nhóm sau:
2
a) 4 cạnh AB, BC , CD, DA của hình vuông ABCD
b) 2 đường chéo AC , BD của hình vuông ABCD
c) 4 cạnh MN , NP, PQ, QM của hình vuông MNPQ (dễ thấy MNPQ là hình vuông)
d) 2 đường chéo MP, NQ của hình vuông MNPQ e) 8 đoạn MA, MB, NB, NC , PC , PD, QD, QA
f) 8 đoạn MC , MD, ND, NA, PA, PB, QB, QC
Ta chứng minh các đoạn ở nhóm e) và f ) thoả mãn điều kiện bài toán.
(Việc chứng minh các đoạn ở các nhóm còn lại thoả mãn điều kiện bài toán đơn giản hơn, bạn
đọc tự chứng minh)
Do AMD cân ở D và  = MDQ
ADQ 
= 15 nên DQ là trung trực của AM hay trung trực của
AM đi qua hai điểm D, Q , đối với các đoạn khác ở nhóm e) chứng minh tương tự.
Do MCD dều còn MQD cân ở Q nên trung trực của MD đi qua hai điểm C , Q . Đối với các
đoạn khác ở nhóm f ) chứng minh tương tự.

Liên hệ tài liệu word môn toán: 039.373.2038 TÀI LIỆU TOÁN HỌC
Website:tailieumontoan.com

ĐẠI HỌC QUỐC GIA HÀ NỘI ĐỀ THI TUYỂN SINH LỚP 10


TRƯỜNG ĐẠI HỌC KHOA HỌC TỰ NHIÊN HỆ THPT CHUYÊN NĂM 1989

MÔN : TOÁN (Vòng 1)


Thời gian làm bài: 150 phút (không kể thời gian phát đề)

Bài 1. Gọi tập hợp các số chính phương là  .P ( x ) = ax 2 + bx + c Ta có


P ( 0 ) = c ∈  → c = c12 ,   c1 ∈ 
P (1) = a + b + c ∈ 
a + b ∈  2a= a1 ∈ 
P ( −1) = a − b + c ∈  →  →
a − b ∈  2a= b1 ∈ 
P ( 4 ) = 16a + 4b + c12 = k 2 , k ∈  → 8a1 + 2b1 = k 2 − c12
Do 8a1 + 2b1 chẵn nên k 2 − c12 chẵn hay k và c1 cùng tính chẵn, lẻ nên
b1
k 2 − c12 : 4 → b1  2 → b = ∈
2
Do a − b ∈  nên từ đây ta có a ∈ 
P ( 2 ) = 4a + 2b + c12 = t 2 ,  
t ∈  → 4a + 2b = t 2 − c12 → t 2 − c12 là chẵn
suy ra t và c1 cùng tính chẵn, lẻ nên ta có t 2 − c12 : 4 → b là chẵn
Vậy a, b, c ∈  và b chẵn.
Bài 2. Đặt
P = a 2 + ab + b 2 − a − 3b + 1989
 3
→ 4 P  2 ( )
= a 2 − ab + b 2 + 3 a 2 + b 2 + 4 + 2ab − 4a − 4b + 4.1989 − 12
= a − b) + 3(a + b − 2) + 4.1986 ≥ 4.1986
 ( 2 2

Suy ra P ≥ 1986 , dấu "=" đạt được khi và chỉ khi


a − b = 0
 →a =b =
 1
a + b − 2 =0
Vậy P đạt giá trị bé nhất bằng 1986, đạt được khi a= b= 1
Bài 3. Gọi 52 số nguyên dương bất kỳ đã cho là a1 , a2 , …, a52 . Mỗi số ai đều có dạng
ai 100bi + ci , trong đó bi , ci ∈  và 0 ≤ ci ≤ 99,  1,52
= i= . ( )
Nếu trong số c1 , c2 , …, c52 có hai số bằng nhau, giả sử ci = ck → ai − ak = 100 ( bi − bk )100 .
Nếu tất cả c1 , c2 , …, c52 đôi một khác nhau thì có ít nhất 51 số khác 50 , giả sử đó là c1 , c2 , …, c51 .
Khi đó ta đặt =
di 100 − ci thì d1 , d 2 , …, d51 là các số nguyên khác nhau và 1 ≤ di ≤ 100 . Như

Liên hệ tài liệu word môn toán: 039.373.2038 TÀI LIỆU TOÁN HỌC
Website:tailieumontoan.com

vậy 102 số c1 , c2 , …, c52 , d1 , d 2 , …, d51 chỉ nhận không quá 101 giá trị (từ 0 dến 100) và do đó có
2 số trong chúng bằng nhau. Do các số c1 , c2 , …, c51 khác nhau và d1 , d 2 , …, d51 khác nhau nên
hai số bằng nhau là ci và d k nào đó suy ra c=
i d=
k 100 , ở đây i ≠ k vì
100 − ck → ci + ck =
ak 100 ( bi + bk ) + 100 :100 .
ci ≠ 50 → ai + =
Bài 4 Kéo dài BE , CF các đoạn EI = BE và FK = CF . Khi đó  ABI , ACK cân ở A và
= CAK
BAK = 30 .
1  = 150 thì B, A, K thẳng hàng, C , A, K thẳng hàng và
Nếu BAC
BK =BA =AK =IA + AC =IC .
1 1
Do E , M , F là các trung điểm của IB, BC , CK nên EM
= = IC = MF ⇒MEF cân ở
BK
2 2
M
2 Gọi giao điểm của IC và BK là O thì trong mọi trường hợp ta đều có A, B, O, I cùng
nằm trên một đường tròn và góc giữa hai tia BK , CI bằng 150 . Từ đó ta có
= MFE
MEF = 15 .
Bài 5. Giả sử theo thứ tự 9 bạn học sinh là a1 , a2 , a3 , a4 , a5 , a6 , a7 , a8 , a9 . Ta chứng minh bài toán
bằng phản chứng.
Giả sử ngược lại: Không có bạn nào đứng cách đều hai bạn cùng lớp (1).
Không mất tổng quát giả sử a5 là học sinh lớp A , khi đó a4 và a6 không thể cùng thuộc lớp A .
Vì vậy có hai khả năng sau:
1 a4 và a6 cùng thuộc lớp B . Khi đó do a4 cách đều a2 và a6 , còn a6 cách đều a4 và a8
nên a2 và a8 thuộc lớp A suy ra a5 đứng cách đều hai bạn cùng lớp là a2 và a8 , trái với
giả thiết (1) .
2 a4 và a6 thuộc hai lớp khác nhau, không mất tổng quát giả sử a4 thuộc lớp A còn a6
thuộc lớp B . Do a4 cách đều a3 và a5 , nên a3 thuộc lớp B . Do a6 cách đều a3 và a9
nên a9 thuộc lớp A . Do a5 cách đều a1 và a9 nên a1 thuộc lớp B . Do a2 cách đều
a1 , a3 nên a2 thuộc lớp A . Do a5 cách đều a2 , a8 nên a8 thuộc lớp B . Do a6 , a8 thuộc
lớp B nên a7 thuộc lớp A . Như vậy a7 đứng cách đều hai bạn cùng lớp A là a5 và a9 ,
trái với giả thiết (1) .
Vậy cả hai khả năng a) và b) đều dẫn đến vô lý nên điều giả sử (1) là sai.

Liên hệ tài liệu word môn toán: 039.373.2038 TÀI LIỆU TOÁN HỌC

You might also like